Π‘Π»ΠΎΠΆΠ΅Π½ΠΈΠ΅ числа ΠΈ ΠΌΠ°Ρ‚Ρ€ΠΈΡ†Ρ‹: ДСйствия с ΠΌΠ°Ρ‚Ρ€ΠΈΡ†Π°ΠΌΠΈ

Π‘ΠΎΠ΄Π΅Ρ€ΠΆΠ°Π½ΠΈΠ΅

ДСйствия с ΠΌΠ°Ρ‚Ρ€ΠΈΡ†Π°ΠΌΠΈ

ΠœΠ°Ρ‚Ρ€ΠΈΡ†Ρ‹ ΠΈ дСйствия с Π½ΠΈΠΌΠΈ, ΠΎΠΏΡ€Π΅Π΄Π΅Π»ΠΈΡ‚Π΅Π»ΠΈ

Π‘Π»ΠΎΠΆΠ΅Π½ΠΈΠ΅ ΠΌΠ°Ρ‚Ρ€ΠΈΡ†

Π‘Π»ΠΎΠΆΠ΅Π½ΠΈΠ΅ ΠΎΠΏΡ€Π΅Π΄Π΅Π»Π΅Π½ΠΎ для ΠΌΠ°Ρ‚Ρ€ΠΈΡ† ΠΎΠ΄Π½ΠΎΠ³ΠΎ Ρ‚ΠΈΠΏΠ°, Ρ‚.Π΅. для ΠΌΠ°Ρ‚Ρ€ΠΈΡ†, Ρƒ ΠΊΠΎΡ‚ΠΎΡ€Ρ‹Ρ… число строк ΠΈ столбцов совпадаСт. Π‘ΡƒΠΌΠΌΠ° ΠΌΠ°Ρ‚Ρ€ΠΈΡ† \(A=\{A_{ik}\}\) ΠΈ \(B=\{B_{ik}\}\), ΠΌΠ°Ρ‚Ρ€ΠΈΡ†Π° \(A+B\), опрСдСляСтся ΡΠ»Π΅Π΄ΡƒΡŽΡ‰ΠΈΠΌ ΠΎΠ±Ρ€Π°Π·ΠΎΠΌ: \((A+B)_{ik}=A_{ik}+B_{ik}\), \(1 \leq i \leq m, 1 \leq k \leq n\). Π˜Π½Ρ‹ΠΌΠΈ словами: ΡΠΊΠ»Π°Π΄Ρ‹Π²Π°ΡŽΡ‚ΡΡ элСмСнты ΠΌΠ°Ρ‚Ρ€ΠΈΡ† \(A\) ΠΈ \(B\), стоящиС Π½Π° ΠΎΠ΄ΠΈΠ½Π°ΠΊΠΎΠ²ΠΎΠΌ мСстС (Ρ‚.Π΅. Π½Π° пСрСсСчСнии ΠΎΠ΄ΠΈΠ½Π°ΠΊΠΎΠ²Ρ‹Ρ… строк ΠΈ столбцов) ΠΈ Π·Π°ΠΏΠΈΡΡ‹Π²Π°ΡŽΡ‚ΡΡ Π² Ρ‚ΠΎ ΠΆΠ΅ мСсто.

ΠŸΡ€ΠΈΠΌΠ΅Ρ€. ΠŸΡƒΡΡ‚ΡŒ \[ A=\left( \begin{array}{ccc} 1 &4 & -1 \\ 3 & -6 & 7 \end{array} \right) , \] \[ B=\left( \begin{array}{ccc} 2 &1 & 0 \\ 1 & 3 & 4 \end{array} \right) , \] Ρ‚ΠΎΠ³Π΄Π° \[ A+B=\left( \begin{array}{ccc} 3 & 5 & -1 \\ 4 & -3 & 11 \end{array} \right) .

\]

Π£ΠΌΠ½ΠΎΠΆΠ΅Π½ΠΈΠ΅ ΠΌΠ°Ρ‚Ρ€ΠΈΡ†Ρ‹ Π½Π° число

ΠŸΡƒΡΡ‚ΡŒ \(A=\{a_{ik}\}\) — ΠΌΠ°Ρ‚Ρ€ΠΈΡ†Π° Ρ‚ΠΈΠΏΠ° \((m,n)\), \(\lambda\) — ΠΏΡ€ΠΎΠΈΠ·Π²ΠΎΠ»ΡŒΠ½ΠΎΠ΅ число. Π’ΠΎΠ³Π΄Π° ΠΌΠ°Ρ‚Ρ€ΠΈΡ†Π° \(\{\lambda a_{ik}\}\) называСтся ΠΏΡ€ΠΎΠΈΠ·Π²Π΅Π΄Π΅Π½ΠΈΠ΅ΠΌ числа \(\lambda \) Π½Π° ΠΌΠ°Ρ‚Ρ€ΠΈΡ†Ρƒ \(A\) ΠΈ обозначаСтся \(\lambda \cdot A\).

ΠŸΡ€ΠΈΠΌΠ΅Ρ€. ΠŸΡƒΡΡ‚ΡŒ \[ A=\left( \begin{array}{ccc} 1 &4 & -1 \\ 7 & 5 & 2 \\ 3 & -6 & 7 \end{array} \right) , \] Ρ‚ΠΎΠ³Π΄Π° \[ 5A=\left( \begin{array}{ccc} 5 &20 & -5 \\ 35 & 25 & 10 \\ 15 & -30 & 35 \end{array} \right) . \]

Π—Π°ΠΌΠ΅Ρ‡Π°Π½ΠΈΠ΅.

Как ΠΈ Π² ΠΎΠ±Ρ‹Ρ‡Π½ΠΎΠΉ, Π² ΠΌΠ°Ρ‚Ρ€ΠΈΡ‡Π½ΠΎΠΉ Π°Ρ€ΠΈΡ„ΠΌΠ΅Ρ‚ΠΈΠΊΠ΅ Π·Π½Π°ΠΊ умноТСния ΠΈΠ½ΠΎΠ³Π΄Π° Π½Π΅ ΡƒΠΊΠ°Π·Ρ‹Π²Π°ΡŽΡ‚, Ρ‚Π°ΠΊ Ρ‡Ρ‚ΠΎ выраТСния \(c\cdot A\) ΠΈ \(cA\) Ρ€Π°Π²Π½ΠΎΠΏΡ€Π°Π²Π½Ρ‹.

ΠŸΡƒΡΡ‚ΡŒ \[ A=\left( \begin{array}{cc} 2 & 3 \\ 4 & 5 \end{array} \right), B=\left( \begin{array}{cc} 1 & -2 \\ 3 & 4 \end{array} \right). \]

Π’Ρ‹Ρ‡ΠΈΡΠ»ΠΈΡ‚ΡŒ \(3A-2B\). T\) (это Π²Ρ‹Ρ€Π°ΠΆΠ΅Π½ΠΈΠ΅ Π·Π°Π½ΠΈΠΌΠ°Π΅Ρ‚ мСньшС мСста).

Π­Π»Π΅ΠΌΠ΅Π½Ρ‚Π°Ρ€Π½Ρ‹Π΅ свойства ΠΎΠΏΠ΅Ρ€Π°Ρ†ΠΈΠΉ с ΠΌΠ°Ρ‚Ρ€ΠΈΡ†Π°ΠΌΠΈ

Π’Π²Π΅Π΄Π΅Π½Π½Ρ‹Π΅ ΠΎΠΏΠ΅Ρ€Π°Ρ†ΠΈΠΈ ΠΎΠ±Π»Π°Π΄Π°ΡŽΡ‚ ΠΌΠ½ΠΎΠ³ΠΈΠΌΠΈ СстСствСнными арифмСтичСскими свойствами. ΠŸΠ΅Ρ€Π΅Ρ‡ΠΈΡΠ»ΠΈΠΌ ряд ΠΈΠ· Π½ΠΈΡ….

1. Для Π»ΡŽΠ±Ρ‹Ρ… ΠΌΠ°Ρ‚Ρ€ΠΈΡ† \(A,B,C\) ΠΎΠ΄Π½ΠΎΠ³ΠΎ Ρ‚ΠΈΠΏΠ° \((A+B)+C=A+(B+C)\)(Π°ΡΡΠΎΡ†ΠΈΠ°Ρ‚ΠΈΠ²Π½ΠΎΡΡ‚ΡŒ слоТСния).

2. Для Π»ΡŽΠ±Ρ‹Ρ… ΠΌΠ°Ρ‚Ρ€ΠΈΡ† \(A,B\) ΠΎΠ΄Π½ΠΎΠ³ΠΎ Ρ‚ΠΈΠΏΠ° \(A+B=B+A\) (ΠΊΠΎΠΌΠΌΡƒΡ‚Π°Ρ‚ΠΈΠ²Π½ΠΎΡΡ‚ΡŒ слоТСния).

3. ΠŸΡƒΡΡ‚ΡŒ \((m,n)\)-ΠΌΠ°Ρ‚Ρ€ΠΈΡ†Π° \(O\) состоит ΠΈΠ· Π½ΡƒΠ»Π΅ΠΉ. Вакая ΠΌΠ°Ρ‚Ρ€ΠΈΡ†Π° ΠΈΠ³Ρ€Π°Π΅Ρ‚ Ρ€ΠΎΠ»ΡŒ нуля ΠΏΡ€ΠΈ слоТСнии ΠΌΠ°Ρ‚Ρ€ΠΈΡ† Ρ‚ΠΈΠΏΠ° \((m,n)\), \(A+O=A\), \(0\cdot A=O\) для любой ΠΌΠ°Ρ‚Ρ€ΠΈΡ†Ρ‹ \(A\) Ρ‚ΠΎΠ³ΠΎ ΠΆΠ΅ Ρ‚ΠΈΠΏΠ°.

4. Для Π»ΡŽΠ±Ρ‹Ρ… чисСл \(c_1,c_2\) ΠΈ любой ΠΌΠ°Ρ‚Ρ€ΠΈΡ†Ρ‹ \(A\) Π²Π΅Ρ€Π½ΠΎ \((c_1+c_2)A=c_1A+c_2A\).

5. Для Π»ΡŽΠ±Ρ‹Ρ… ΠΌΠ°Ρ‚Ρ€ΠΈΡ† \(A,B\) ΠΎΠ΄Π½ΠΎΠ³ΠΎ Ρ‚ΠΈΠΏΠ° ΠΈ любого числа \(c\) Π²Π΅Ρ€Π½ΠΎ \(c(A+B)=cA+cB\).

6. Для Π»ΡŽΠ±Ρ‹Ρ… чисСл \(c_1,c_2\) ΠΈ любой ΠΌΠ°Ρ‚Ρ€ΠΈΡ†Ρ‹ \(A\) Π²Π΅Ρ€Π½ΠΎ \((c_1c_2)A=c_1(c_2A)\).

7. Для любой ΠΌΠ°Ρ‚Ρ€ΠΈΡ†Ρ‹ \(A\) Π²Π΅Ρ€Π½ΠΎ \(1\cdot A=A\).

8. Для Π»ΡŽΠ±Ρ‹Ρ… ΠΌΠ°Ρ‚Ρ€ΠΈΡ† \(A,B\) ΠΎΠ΄Π½ΠΎΠ³ΠΎ Ρ‚ΠΈΠΏΠ° \((A+B)^T=A^T+B^T\). na_{im}b_{mk}. \] Π’Π°ΠΊΠΈΠΌ ΠΎΠ±Ρ€Π°Π·ΠΎΠΌ слСдуСт Π²Ρ‹Ρ‡ΠΈΡΠ»ΠΈΡ‚ΡŒ всС \(mp\) элСмСнтов ΠΌΠ°Ρ‚Ρ€ΠΈΡ†Ρ‹ \(C\). Π•Ρ‰Π΅ Ρ€Π°Π· ΠΏΠΎΠ΄Ρ‡Π΅Ρ€ΠΊΠ½Π΅ΠΌ, Ρ‡Ρ‚ΠΎ для Ρ‚ΠΎΠ³ΠΎ, Ρ‡Ρ‚ΠΎΠ±Ρ‹ ΠΌΠΎΠΆΠ½ΠΎ Π±Ρ‹Π»ΠΎ ΠΏΠ΅Ρ€Π΅ΠΌΠ½ΠΎΠΆΠ°Ρ‚ΡŒ ΠΌΠ°Ρ‚Ρ€ΠΈΡ†Ρ‹ \(A\) ΠΈ \(B\), ΠΈΡ… Ρ‚ΠΈΠΏΡ‹ Π΄ΠΎΠ»ΠΆΠ½Ρ‹ Π±Ρ‹Ρ‚ΡŒ согласованы!

ΠŸΡ€ΠΈΠΌΠ΅Ρ€. ΠŸΡƒΡΡ‚ΡŒ \[ A=\left( \begin{array}{ccc} 1 &4 & -1 \\ 3 & -6 & 7 \end{array} \right) , B=\left( \begin{array}{cc} 2 &1 \\ 1 & 3 \\ -3 &5 \end{array} \right) . \]

Π’ Π΄Π°Π½Π½ΠΎΠΌ случаС ΠΌΠ°Ρ‚Ρ€ΠΈΡ†Π° \(A\) ΠΈΠΌΠ΅Π΅Ρ‚ Ρ‚ΠΈΠΏ (2,3), ΠΌΠ°Ρ‚Ρ€ΠΈΡ†Π° \(B\) ΠΈΠΌΠ΅Π΅Ρ‚ Ρ‚ΠΈΠΏ (3,2), Ρ‚Π°ΠΊ Ρ‡Ρ‚ΠΎ Ρ‚ΠΈΠΏΡ‹ ΠΌΠ°Ρ‚Ρ€ΠΈΡ† согласнованы ΠΈ Π² Ρ€Π΅Π·ΡƒΠ»ΡŒΡ‚Π°Ρ‚Π΅ умноТСния \(A\) Π½Π° \(B\) ΠΏΠΎΠ»ΡƒΡ‡ΠΈΠΌ ΠΌΠ°Ρ‚Ρ€ΠΈΡ†Ρƒ Ρ‚ΠΈΠΏΠ° \((2,2)\). ΠŸΠΎΠ»ΡƒΡ‡Π°Π΅ΠΌ: \[ AB=\left ( \begin{array}{cc} 1\cdot 2 +4 \cdot 1+(-1)\cdot (-3) & 1\cdot 1 +4 \cdot 3+(-1)\cdot 5\\ 3\cdot 2 +(-6) \cdot 1+7\cdot (-3) &3\cdot 1 +(-6) \cdot 3+7\cdot 5 \end{array} \right )= \left( \begin{array}{cc} 9 & 8\\ -21 & 20 \end{array} \right). T\).

6. Для ΠΊΠ²Π°Π΄Ρ€Π°Ρ‚Π½Ρ‹Ρ… ΠΌΠ°Ρ‚Ρ€ΠΈΡ† \(A,B\) ΠΎΠ΄Π½ΠΎΠ³ΠΎ Ρ‚ΠΈΠΏΠ° \(det(AB)=detA \cdot detB\).

7. Рассмотрим ΠΊΠ²Π°Π΄Ρ€Π°Ρ‚Π½ΡƒΡŽ ΠΌΠ°Ρ‚Ρ€ΠΈΡ†Ρƒ порядка \(n\), \(E=diag\{1,1,1,…,1\}\). Вакая ΠΌΠ°Ρ‚Ρ€ΠΈΡ†Π° ΠΈΠ³Ρ€Π°Π΅Ρ‚ Π²Ρ‹Π΄Π΅Π»Π΅Π½Π½ΡƒΡŽ Ρ€ΠΎΠ»ΡŒ Π² ΡƒΠΌΠ½ΠΎΠΆΠ΅Π½ΠΈΠΈ ΠΌΠ°Ρ‚Ρ€ΠΈΡ†: для Π»ΡŽΠ±Ρ‹Ρ… ΠΌΠ°Ρ‚Ρ€ΠΈΡ† \(A,B\) ΠΈΠΌΠ΅Π΅ΠΌ \(EA=A\), \(BE=B\). ΠœΠ°Ρ‚Ρ€ΠΈΡ†Π° \(E\) называСтся Π΅Π΄ΠΈΠ½ΠΈΡ‡Π½ΠΎΠΉ ΠΌΠ°Ρ‚Ρ€ΠΈΡ†Π΅ΠΉ порядка \(n\). Богласно описанным Π²Ρ‹ΡˆΠ΅ Ρ€Π΅Π·ΡƒΠ»ΡŒΡ‚Π°Ρ‚Π°ΠΌ, \(detE=1\).

1. Π£ΠΌΠ½ΠΎΠΆΠΈΡ‚ΡŒ ΠΌΠ°Ρ‚Ρ€ΠΈΡ†Ρ‹:

Π°) \[ \left( \begin{array}{cc} 2 & 1 \\ 3 & 4 \end{array} \right)\cdot \left( \begin{array}{cc} 1 & -1 \\ 2 & 1 \end{array} \right). \]

Π±) \[ \left( \begin{array}{ccc} 3 & 1 & 1 \\ 2 & 1 & 2 \\ 1 & 2 & 1 \end{array} \right)\cdot \left( \begin{array}{ccc} 1 &1 & -1 \\ 2 & -1 & 1 \\ -1 & 2 & 1 \end{array} \right). \]

2. Π’Ρ‹Ρ‡ΠΈΡΠ»ΠΈΡ‚ΡŒ \[ \left( \begin{array}{cc} 3 & 2 \\ -4 & -2 \end{array} \right)^5. {-1}=\frac{1}{ad-bc}\left( \begin{array}{cc} d & -b \\ -c & a \end{array} \right). \]

Π’Π°ΠΊΠΈΠΌ ΠΎΠ±Ρ€Π°Π·ΠΎΠΌ, для ΠΌΠ°Ρ‚Ρ€ΠΈΡ†Ρ‹ порядка 2 Ρ„ΠΎΡ€ΠΌΡƒΠ»Ρ‹ для ΠΎΠ±Ρ€Π°Ρ‚Π½ΠΎΠΉ ΠΌΠ°Ρ‚Ρ€ΠΈΡ†Ρ‹ достаточно простыС. Для Π±ΠΎΠ»ΡŒΡˆΠΈΡ… порядков Ρ„ΠΎΡ€ΠΌΡƒΠ»Ρ‹ становятся сущСствСнно Π±ΠΎΠ»Π΅Π΅ Π³Ρ€ΠΎΠΌΠΎΠ·Π΄ΠΊΠΈΠΌΠΈ.

Найти ΠΎΠ±Ρ€Π°Ρ‚Π½ΡƒΡŽ ΠΌΠ°Ρ‚Ρ€ΠΈΡ†Ρƒ для ΠΌΠ°Ρ‚Ρ€ΠΈΡ†Ρ‹

1. \[ A=\left( \begin{array}{ccc} 2 &2 & 3 \\ 1 & -1 & 0 \\ -1 & 2 & 1 \end{array} \right). \]

2. \[ A=\left( \begin{array}{ccc} 2 &-1 & 0 \\ 0 & 2 & -1 \\ -1 & -1 & 1 \end{array} \right). \]

3. \[ A=\left( \begin{array}{ccc} 1 &1 & 1 \\ 1 & 2 & 2 \\ 2 & 3 & 4 \end{array} \right). \]

ΠœΠ°Ρ‚Ρ€ΠΈΡ‡Π½Ρ‹Π΅ уравнСния

ΠœΠ°Ρ‚Ρ€ΠΈΡ‡Π½Ρ‹ΠΌΠΈ уравнСниями Π½Π°Π·Ρ‹Π²Π°ΡŽΡ‚ΡΡ уравнСния Π²ΠΈΠ΄Π° \[ AX=G, \quad \quad(12)\] \[ XB=G, \quad \quad(13)\] \[ AXB=G, \quad \quad(14)\] Π³Π΄Π΅ ΠΌΠ°Ρ‚Ρ€ΠΈΡ†Ρ‹ \(A,B,G\) Π·Π°Π΄Π°Π½Ρ‹ ΠΈ трСбуСтся ΠΏΠΎΡΡ‚Ρ€ΠΎΠΈΡ‚ΡŒ ΠΌΠ°Ρ‚Ρ€ΠΈΡ†Ρƒ \(X\). {-1}. \]

1. Найти Ρ€Π΅ΡˆΠ΅Π½ΠΈΠ΅ ΠΌΠ°Ρ‚Ρ€ΠΈΡ‡Π½ΠΎΠ³ΠΎ уравнСния (12), Ссли \[ A=\left( \begin{array}{cc} 2 & 6 \\ -9 & 3 \end{array} \right) , G=\left( \begin{array}{cc} -26 & -50 \\ 27 & -15 \end{array} \right) . \]

2. Найти Ρ€Π΅ΡˆΠ΅Π½ΠΈΠ΅ ΠΌΠ°Ρ‚Ρ€ΠΈΡ‡Π½ΠΎΠ³ΠΎ уравнСния (12), Ссли \[ A=\left( \begin{array}{cc} 8 & -7 \\ -5 & 4 \end{array} \right) , G=\left( \begin{array}{cc} 25 & -34 \\ -16 & 22 \end{array} \right) . \]

3. Найти Ρ€Π΅ΡˆΠ΅Π½ΠΈΠ΅ ΠΌΠ°Ρ‚Ρ€ΠΈΡ‡Π½ΠΎΠ³ΠΎ уравнСния (13), Ссли \[ B=\left( \begin{array}{cc} -8 & -5 \\ -9 & 5 \end{array} \right) , G=\left( \begin{array}{cc} -20 & 30 \\ -19 & 20 \end{array} \right) . \]

4. Найти Ρ€Π΅ΡˆΠ΅Π½ΠΈΠ΅ ΠΌΠ°Ρ‚Ρ€ΠΈΡ‡Π½ΠΎΠ³ΠΎ уравнСния (13), Ссли \[ B=\left( \begin{array}{cc} 9 & 8 \\ -3 & 7 \end{array} \right) , G=\left( \begin{array}{cc} -72 & 23 \\ 0 & 58 \end{array} \right) . \]

5. Найти Ρ€Π΅ΡˆΠ΅Π½ΠΈΠ΅ ΠΌΠ°Ρ‚Ρ€ΠΈΡ‡Π½ΠΎΠ³ΠΎ уравнСния (14), Ссли \[ A=\left( \begin{array}{cc} 4 & 2 \\ 3 & -4 \end{array} \right) , B=\left( \begin{array}{cc} -1 & 2 \\ -2 & -1 \end{array} \right) , G=\left( \begin{array}{cc} 20 & -50 \\ 26 & 23 \end{array} \right) . \]

6. Найти Ρ€Π΅ΡˆΠ΅Π½ΠΈΠ΅ ΠΌΠ°Ρ‚Ρ€ΠΈΡ‡Π½ΠΎΠ³ΠΎ уравнСния (14), Ссли \[ A=\left( \begin{array}{cc} -4 & -2 \\ -3 & 3 \end{array} \right) , B=\left( \begin{array}{cc} 3 & 4 \\ 4 & 3 \end{array} \right) , G=\left( \begin{array}{cc} 132 & 134 \\ 18 & 24 \end{array} \right) . \]

Π‘Π»ΠΎΠΆΠ΅Π½ΠΈΠ΅ ΠΈ Π²Ρ‹Ρ‡ΠΈΡ‚Π°Π½ΠΈΠ΅ ΠΌΠ°Ρ‚Ρ€ΠΈΡ†, ΡƒΠΌΠ½ΠΎΠΆΠ΅Π½ΠΈΠ΅ ΠΌΠ°Ρ‚Ρ€ΠΈΡ† Π½Π° число Π² EXCEL. ΠŸΡ€ΠΈΠΌΠ΅Ρ€Ρ‹ ΠΈ описаниС

Π’ этой ΡΡ‚Π°Ρ‚ΡŒΠ΅ рассмотрСны ΠΎΠΏΠ΅Ρ€Π°Ρ†ΠΈΠΈ слоТСния ΠΈ вычитания Π½Π°Π΄ ΠΌΠ°Ρ‚Ρ€ΠΈΡ†Π°ΠΌΠΈ ΠΎΠ΄Π½ΠΎΠ³ΠΎ порядка, Π° Ρ‚Π°ΠΊΠΆΠ΅ ΠΎΠΏΠ΅Ρ€Π°Ρ†ΠΈΠΈ умноТСния ΠΌΠ°Ρ‚Ρ€ΠΈΡ†Ρ‹ Π½Π° число. ΠŸΡ€ΠΈΠΌΠ΅Ρ€Ρ‹ Ρ€Π΅ΡˆΠ΅Π½Ρ‹ Π² MS EXCEL.

ΠžΠΏΠ΅Ρ€Π°Ρ†ΠΈΡ слоТСния ΠΎΠΏΡ€Π΅Π΄Π΅Π»Π΅Π½Π° Ρ‚ΠΎΠ»ΡŒΠΊΠΎ для ΠΌΠ°Ρ‚Ρ€ΠΈΡ† ΠΎΠ΄Π½ΠΎΠ³ΠΎ порядка. Π’.Π΅. нСльзя Π³ΠΎΠ²ΠΎΡ€ΠΈΡ‚ΡŒ ΠΎ слоТСнии ΠΌΠ°Ρ‚Ρ€ΠΈΡ† Ρ€Π°Π·Π½ΠΎΠΉ размСрности. Π’Π°ΠΊΠΆΠ΅ Π½Π΅ΠΎΠΏΡ€Π΅Π΄Π΅Π»Π΅Π½Π° опСрация слоТСния ΠΌΠ°Ρ‚Ρ€ΠΈΡ†Ρ‹ ΠΈ числа. Напротив, опСрация умноТСния ΠΌΠ°Ρ‚Ρ€ΠΈΡ†Ρ‹ Π½Π° число ΠΎΠΏΡ€Π΅Π΄Π΅Π»Π΅Π½Π°.

Π‘ΡƒΠΌΠΌΠ° Π΄Π²ΡƒΡ… ΠΌΠ°Ρ‚Ρ€ΠΈΡ† А ΠΈ Π’ — это ΠΌΠ°Ρ‚Ρ€ΠΈΡ†Π°, элСмСнты ΠΊΠΎΡ‚ΠΎΡ€ΠΎΠΉ Ρ€Π°Π²Π½Ρ‹ суммС ΡΠΎΠΎΡ‚Π²Π΅Ρ‚ΡΡ‚Π²ΡƒΡŽΡ‰ΠΈΡ… элСмСнтов ΠΌΠ°Ρ‚Ρ€ΠΈΡ† А ΠΈ Π’.

Π’ MS EXCEL ΠΎΠΏΠ΅Ρ€Π°Ρ†ΠΈΡŽ слоТСния ΠΌΠ°Ρ‚Ρ€ΠΈΡ† Ρ€Π΅Π°Π»ΠΈΠ·ΠΎΠ²Π°Ρ‚ΡŒ элСмСнтарно.

ΠŸΠΎΠΌΠ΅ΡΡ‚ΠΈΠ² ΠΌΠ°Ρ‚Ρ€ΠΈΡ†Ρƒ А размСрности 2Ρ…2 Π² Π΄ΠΈΠ°ΠΏΠ°Π·ΠΎΠ½ А8:Π’9 , Π°Β ΠΌΠ°Ρ‚Ρ€ΠΈΡ†Ρƒ B Ρ‚ΠΎΠΉ ΠΆΠ΅ размСрности 2Ρ…2 Π² Π΄ΠΈΠ°ΠΏΠ°Π·ΠΎΠ½ D8:E9,

Π² ячСйкС J8 Π²Π²Π΅Π΄Π΅ΠΌ Ρ„ΠΎΡ€ΠΌΡƒΠ»Ρƒ =A8+D8 . Π‘ΠΊΠΎΠΏΠΈΡ€ΠΎΠ²Π°Π² Ρ„ΠΎΡ€ΠΌΡƒΠ»Ρƒ Π² Π½ΡƒΠΆΠ½Ρ‹Π΅ ячСйки, Π½Π°ΠΏΡ€ΠΈΠΌΠ΅Ρ€, с ΠΏΠΎΠΌΠΎΡ‰ΡŒΡŽ ΠœΠ°Ρ€ΠΊΠ΅Ρ€Π° заполнСния , ΠΏΠΎΠ»ΡƒΡ‡ΠΈΠΌ ΠΌΠ°Ρ‚Ρ€ΠΈΡ†Ρƒ А+Π’.

Аналогичного Ρ€Π΅Π·ΡƒΠ»ΡŒΡ‚Π°Ρ‚Π° ΠΌΠΎΠΆΠ½ΠΎ Π΄ΠΎΠ±ΠΈΡ‚ΡŒΡΡ с использованиСм Ρ„ΠΎΡ€ΠΌΡƒΠ»Ρ‹ массива . Π’Ρ‹Π΄Π΅Π»ΠΈΠ² Π΄ΠΈΠ°ΠΏΠ°Π·ΠΎΠ½ G 8:H9 Π² Π‘Ρ‚Ρ€ΠΎΠΊΠ΅ Ρ„ΠΎΡ€ΠΌΡƒΠ» Π²Π²Π΅Π΄ΠΈΡ‚Π΅ Ρ„ΠΎΡ€ΠΌΡƒΠ»Ρƒ =A8:B9+D8:E9 ΠΈ Π½Π°ΠΆΠΌΠΈΡ‚Π΅ CTRL+SHIFT+ENTER . ΠŸΡ€Π΅ΠΈΠΌΡƒΡ‰Π΅ΡΡ‚Π²ΠΎ Ρ„ΠΎΡ€ΠΌΡƒΠ»Ρ‹ массива состоит Π² Ρ‚ΠΎΠΌ, Ρ‡Ρ‚ΠΎ Π½Π΅Π²ΠΎΠ·ΠΌΠΎΠΆΠ½ΠΎ ΡƒΠ΄Π°Π»ΠΈΡ‚ΡŒ ΠΎΡ‚Π΄Π΅Π»ΡŒΠ½Ρ‹Π΅ элСмСнты ΠΌΠ°Ρ‚Ρ€ΠΈΡ†Ρ‹ А+Π’ (появится ΠΎΠΊΠ½ΠΎ НСвозмоТно ΡƒΠ΄Π°Π»ΠΈΡ‚ΡŒ Ρ‡Π°ΡΡ‚ΡŒ массива ).

ΠŸΠΎΠ½ΡΡ‚Π½ΠΎ, Ρ‡Ρ‚ΠΎ опСрация вычитания ΠΌΠ°Ρ‚Ρ€ΠΈΡ† Π² MS EXCEL рСализуСтся Π°Π½Π°Π»ΠΎΠ³ΠΈΡ‡Π½ΠΎ (см. Ρ„Π°ΠΉΠ» ΠΏΡ€ΠΈΠΌΠ΅Ρ€Π° ).

Π£ΠΌΠ½ΠΎΠΆΠ΅Π½ΠΈΠ΅ ΠΌΠ°Ρ‚Ρ€ΠΈΡ† Π½Π° число

ΠžΠΏΠ΅Ρ€Π°Ρ†ΠΈΡ умноТСния ΠΌΠ°Ρ‚Ρ€ΠΈΡ†Ρ‹ Π½Π° число ΠΎΠΏΡ€Π΅Π΄Π΅Π»Π΅Π½Π° для ΠΌΠ°Ρ‚Ρ€ΠΈΡ† любого порядка.

ΠŸΡ€ΠΎΠΈΠ·Π²Π΅Π΄Π΅Π½ΠΈΠ΅ ΠΌΠ°Ρ‚Ρ€ΠΈΡ†Ρ‹ А ΠΈ числа k — это ΠΌΠ°Ρ‚Ρ€ΠΈΡ†Π°, элСмСнты ΠΊΠΎΡ‚ΠΎΡ€ΠΎΠΉ ΠΏΠΎΠ»ΡƒΡ‡Π°ΡŽΡ‚ΡΡ ΡƒΠΌΠ½ΠΎΠΆΠ΅Π½ΠΈΠ΅ΠΌ всСх элСмСнтов исходной ΠΌΠ°Ρ‚Ρ€ΠΈΡ†Ρ‹ А Π½Π° число k.

Π’ MS EXCEL это Ρ€Π΅Π°Π»ΠΈΠ·ΠΎΠ²Π°Π½ΠΎ с ΠΏΠΎΠΌΠΎΡ‰ΡŒΡŽ Ρ„ΠΎΡ€ΠΌΡƒΠ»Ρ‹ =A21*$D$21 (прСдполагаСтся, Ρ‡Ρ‚ΠΎ ΠΌΠ°Ρ‚Ρ€ΠΈΡ†Π° находится Π² Π΄ΠΈΠ°ΠΏΠ°Π·ΠΎΠ½Π΅ А21:Π’23 , Π° число Π² ячСйкС D21 ). ΠžΠ±Ρ€Π°Ρ‚ΠΈΡ‚Π΅ Π²Π½ΠΈΠΌΠ°Π½ΠΈΠ΅, Ρ‡Ρ‚ΠΎ Π² Ρ„ΠΎΡ€ΠΌΡƒΠ»Π΅ использована Π°Π±ΡΠΎΠ»ΡŽΡ‚Π½Π°Ρ адрСсация Π½Π° ячСйку с числом. Π­Ρ‚ΠΎ позволяСт ΠΏΡ€ΠΈ ΠΊΠΎΠΏΠΈΡ€ΠΎΠ²Π°Π½ΠΈΠΈ Ρ„ΠΎΡ€ΠΌΡƒΠ»Ρ‹ (для отобраТСния всСх элСмСнтов ΠΌΠ°Ρ‚Ρ€ΠΈΡ†Ρ‹ k*А) ΡΡΡ‹Π»Π°Ρ‚ΡŒΡΡ Π½Π° ΠΎΠ΄Π½Ρƒ ΠΈ Ρ‚Ρƒ ΠΆΠ΅ ячСйку с числом.

ΠœΠ°Ρ‚Π΅ΠΌΠ°Ρ‚ΠΈΠΊΠ° ΠΎΠ½Π»Π°ΠΉΠ½

РСшСниС ΠΌΠ°Ρ‚Π΅ΠΌΠ°Ρ‚ΠΈΠΊΠΈ ΠΎΠ½Π»Π°ΠΉΠ½

Math34.biz – это соврСмСнный способ Ρ€Π΅ΡˆΠ΅Π½ΠΈΡ ΠΌΠ°Ρ‚Π΅ΠΌΠ°Ρ‚ΠΈΠΊΠΈ, Π² Ρ‚ΠΎΠΌ числС для сравнСния ΡΠ°ΠΌΠΎΡΡ‚ΠΎΡΡ‚Π΅Π»ΡŒΠ½Ρ‹Ρ… Ρ€Π΅ΡˆΠ΅Π½ΠΈΠΉ с ΠΌΠ°ΡˆΠΈΠ½Π½Ρ‹ΠΌΠΈ вычислСниями.

ПользованиС сСрвисом ΡƒΠ΄ΠΎΠ±Π½ΠΎ ΠΈ понятно ΠΊΠ°ΠΆΠ΄ΠΎΠΌΡƒ Ρ‡Π΅Π»ΠΎΠ²Π΅ΠΊΡƒ, ΠΏΠΎΠΏΠ°Π²ΡˆΠ΅ΠΌΡƒ Π½Π° сайт Π²ΠΏΠ΅Ρ€Π²Ρ‹Π΅. Π‘Ρ€Π°Π·Ρƒ Π²Ρ‹Π±ΠΈΡ€Π°Π΅Ρ‚Π΅ Π½ΡƒΠΆΠ½Ρ‹ΠΉ ΠΊΠ°Π»ΡŒΠΊΡƒΠ»ΡΡ‚ΠΎΡ€, Π²Π²ΠΎΠ΄ΠΈΡ‚Π΅ Π½Π΅ΠΎΠ±Ρ…ΠΎΠ΄ΠΈΠΌΡ‹Π΅ Π΄Π°Π½Π½Ρ‹Π΅ ΠΏΠΎ вашСй Π·Π°Π΄Π°Ρ‡Π΅ ΠΈ Π½Π°ΠΆΠΈΠΌΠ°Π΅Ρ‚Π΅ ΠΊΠ½ΠΎΠΏΠΊΡƒ «РСшСниС». Π—Π° считанныС сСкунды ΠΎΡ‚Π²Π΅Ρ‚ Π³ΠΎΡ‚ΠΎΠ².

Π§Ρ‚ΠΎΠ±Ρ‹ Π½Π΅ Π²ΠΎΠ·Π½ΠΈΠΊΠ°Π»ΠΎ трудностСй с Π²Π²ΠΎΠ΄ΠΎΠΌ Π΄Π°Π½Π½Ρ‹Ρ…, ΠΌΡ‹ ΠΏΠΎΠ΄Π³ΠΎΡ‚ΠΎΠ²ΠΈΠ»ΠΈ ΡΠΏΠ΅Ρ†ΠΈΠ°Π»ΡŒΠ½ΡƒΡŽ ΡΡ‚Π°Ρ‚ΡŒΡŽ Как Π²Π²ΠΎΠ΄ΠΈΡ‚ΡŒ Π΄Π°Π½Π½Ρ‹Π΅? Помимо ΠΏΡ€Π°Π²ΠΈΠ» написания Ρ„ΠΎΡ€ΠΌΡƒΠ» ΠΈ чисСл, Π² Π½Π΅ΠΉ Π²Ρ‹ ΠΌΠΎΠΆΠ΅Ρ‚Π΅ ΡƒΠ²ΠΈΠ΄Π΅Ρ‚ΡŒ, ΠΊΠ°ΠΊ ΠΏΡ€Π°Π²ΠΈΠ»ΡŒΠ½ΠΎ вводятся Ρ€Π°Π·Π»ΠΈΡ‡Π½Ρ‹Π΅ константы ΠΈ матСматичСскиС Ρ„ΡƒΠ½ΠΊΡ†ΠΈΠΈ.

О ΠΊΠ°Π»ΡŒΠΊΡƒΠ»ΡΡ‚ΠΎΡ€Π°Ρ…

По ΠΌΠ΅Ρ€Π΅ возмоТности Π΄ΠΎΠ±Π°Π²Π»ΡΡŽΡ‚ΡΡ Π½ΠΎΠ²Ρ‹Π΅ матСматичСскиС ΠΊΠ°Π»ΡŒΠΊΡƒΠ»ΡΡ‚ΠΎΡ€Ρ‹. На сСгодняшний дСнь ΠΈΡ… Π±ΠΎΠ»Π΅Π΅ 85.

Если Π½Π΅ ΡƒΠ΄Π°Π»ΠΎΡΡŒ Π½Π°ΠΉΡ‚ΠΈ Π½ΡƒΠΆΠ½Ρ‹ΠΉ ΠΊΠ°Π»ΡŒΠΊΡƒΠ»ΡΡ‚ΠΎΡ€, ΠΊΠΎΡ‚ΠΎΡ€Ρ‹ΠΌ ΠΌΠΎΠΆΠ΅Ρ‚ Π±Ρ‹Ρ‚ΡŒ Ρ€Π΅ΡˆΠ΅Π½Π° ваша матСматичСская Π·Π°Π΄Π°Ρ‡Π°, ΠΈΠ»ΠΈ Π΅ΡΡ‚ΡŒ ΠΏΡ€Π΅Π΄Π»ΠΎΠΆΠ΅Π½ΠΈΠ΅ ΠΏΠΎ ΡƒΠ»ΡƒΡ‡ΡˆΠ΅Π½ΠΈΡŽ ΠΈΠΌΠ΅ΡŽΡ‰Π΅Π³ΠΎΡΡ ΠΊΠ°Π»ΡŒΠΊΡƒΠ»ΡΡ‚ΠΎΡ€Π°, поТалуйста, сообщитС ΠΎΠ± этом Π½Π° ΠΏΠΎΡ‡Ρ‚Ρƒ info@math34. biz

ΠŸΡ€Π΅ΠΈΠΌΡƒΡ‰Π΅ΡΡ‚Π²Π°

1. БСсплатно
РСшСниС ΠΌΠ°Ρ‚Π΅ΠΌΠ°Ρ‚ΠΈΠΊΠΈ ΠΎΠ½Π»Π°ΠΉΠ½ Π½Π΅ Π±ΡƒΠ΄Π΅Ρ‚ Π²Π°ΠΌ ΡΡ‚ΠΎΠΈΡ‚ΡŒ Π½ΠΈ ΠΊΠΎΠΏΠ΅ΠΉΠΊΠΈ. Наш сСрвис Π°Π±ΡΠΎΠ»ΡŽΡ‚Π½ΠΎ бСсплатный ΠΈ доступСн Π»ΡŽΠ±ΠΎΠΌΡƒ ΠΏΠΎΠ»ΡŒΠ·ΠΎΠ²Π°Ρ‚Π΅Π»ΡŽ ΠΈΠ½Ρ‚Π΅Ρ€Π½Π΅Ρ‚Π°.

2. Π‘Π΅Π· рСгистрации
Для пользования ΠΊΠ°Π»ΡŒΠΊΡƒΠ»ΡΡ‚ΠΎΡ€Π°ΠΌΠΈ Π½Π΅ трСбуСтся рСгистрации Π½Π° сайтС, отнимая врСмя Π½Π° Π·Π°ΠΏΠΎΠ»Π½Π΅Π½ΠΈΠ΅ ΠΏΠΎΡ‡Ρ‚ΠΎΠ²Ρ‹Ρ… ящиков ΠΈ Π΄Ρ€ΡƒΠ³ΠΈΡ… Π»ΠΈΡ‡Π½Ρ‹Ρ… Π΄Π°Π½Π½Ρ‹Ρ….

3. ΠŸΠΎΠ΄Ρ€ΠΎΠ±Π½Ρ‹Π΅ Ρ€Π΅ΡˆΠ΅Π½ΠΈΡ
На ΠΌΠ½ΠΎΠ³ΠΈΠ΅ Π·Π°Π΄Π°Ρ‡ΠΈ Π²Ρ‹ ΠΏΠΎΠ»ΡƒΡ‡ΠΈΡ‚Π΅ ΠΏΠΎΡˆΠ°Π³ΠΎΠ²Ρ‹ΠΉ Ρ€Π°Π·Π²Π΅Ρ€Π½ΡƒΡ‚Ρ‹ΠΉ ΠΎΡ‚Π²Π΅Ρ‚, Ρ‡Ρ‚ΠΎ позволяСт ΠΏΠΎΠ½ΡΡ‚ΡŒ, ΠΊΠ°ΠΊΠΈΠΌ ΠΎΠ±Ρ€Π°Π·ΠΎΠΌ Π±Ρ‹Π»ΠΎ ΠΏΠΎΠ»ΡƒΡ‡Π΅Π½ΠΎ Ρ€Π΅ΡˆΠ΅Π½ΠΈΠ΅ Π·Π°Π΄Π°Ρ‡ΠΈ.

4. Π Π°Π·Π½Ρ‹Π΅ способы Ρ€Π΅ΡˆΠ΅Π½ΠΈΡ Π·Π°Π΄Π°Ρ‡
Для популярных ΠΊΠ°Π»ΡŒΠΊΡƒΠ»ΡΡ‚ΠΎΡ€ΠΎΠ² доступны Ρ€Π°Π·Π½Ρ‹Π΅ ΠΌΠ΅Ρ‚ΠΎΠ΄Ρ‹ Ρ€Π΅ΡˆΠ΅Π½ΠΈΡ Π·Π°Π΄Π°Ρ‡, Ссли ΠΎΠ½ΠΈ ΠΏΡ€ΠΈΠΌΠ΅Π½ΠΈΠΌΡ‹, Ρ‡Ρ‚ΠΎ позволяСт, Π²ΠΎ-ΠΏΠ΅Ρ€Π²Ρ‹Ρ…, Π»ΡƒΡ‡ΡˆΠ΅ ΠΏΠΎΠ½ΡΡ‚ΡŒ, ΠΊΠ°ΠΊ Ρ€Π΅ΡˆΠ°Π΅Ρ‚ΡΡ Π·Π°Π΄Π°Ρ‡Π° извСстным Π²Π°ΠΌ способом, Π°, Π²ΠΎ-Π²Ρ‚ΠΎΡ€Ρ‹Ρ…, Π½Π°ΡƒΡ‡ΠΈΡ‚ΡŒΡΡ Ρ€Π΅ΡˆΠ°Ρ‚ΡŒ Ρ‚Ρƒ ΠΆΠ΅ ΡΠ°ΠΌΡƒΡŽ Π·Π°Π΄Π°Ρ‡Ρƒ Π°Π»ΡŒΡ‚Π΅Ρ€Π½Π°Ρ‚ΠΈΠ²Π½Ρ‹ΠΌΠΈ ΠΌΠ΅Ρ‚ΠΎΠ΄Π°ΠΌΠΈ.

5. Π’ΠΎΡ‡Π½ΠΎΡΡ‚ΡŒ вычислСний
Π’ ΠΏΠΎΠ»ΡƒΡ‡Π΅Π½Π½ΠΎΠΌ ΠΎΡ‚Π²Π΅Ρ‚Π΅ Π½Π΅ приходится ΡΠΎΠΌΠ½Π΅Π²Π°Ρ‚ΡŒΡΡ, вСдь мощная систСма расчСта обСспСчиваСт Π²Ρ‹ΡΠΎΠΊΡƒΡŽ Ρ‚ΠΎΡ‡Π½ΠΎΡΡ‚ΡŒ ΠΏΡ€ΠΈ Ρ€Π΅ΡˆΠ΅Π½ΠΈΠΈ матСматичСских Π·Π°Π΄Π°Ρ‡ ΠΎΠ½Π»Π°ΠΉΠ½.

Однако, ΠΌΡ‹ Π½Π΅ ΠΈΡΠΊΠ»ΡŽΡ‡Π°Π΅ΠΌ Π²ΠΎΠ·ΠΌΠΎΠΆΠ½ΠΎΡΡ‚ΡŒ ΠΊΠ°ΠΊΠΈΡ…-Π»ΠΈΠ±ΠΎ ошибок, вСдь извСстно, Ρ‡Ρ‚ΠΎ Π°Π»Π³ΠΎΡ€ΠΈΡ‚ΠΌΡ‹ ΠΏΠΈΡˆΡƒΡ‚ΡΡ хотя ΠΈ ΠΎΡ‡Π΅Π½ΡŒ ΡƒΠΌΠ½Ρ‹ΠΌΠΈ, Π½ΠΎ всё ΠΆΠ΅ людьми. Π’ случаС обнаруТСния ошибки, поТалуйста, Π½Π΅ ΠΏΠΎΠ»Π΅Π½ΠΈΡ‚Π΅ΡΡŒ ΠΈ сообщитС Π½Π°ΠΌ ΠΎ Π½Π΅ΠΉ.

ΡƒΠΌΠ½ΠΎΠΆΠ΅Π½ΠΈΠ΅ Π½Π° число, слоТСниС, Π²Ρ‹Ρ‡ΠΈΡ‚Π°Π½ΠΈΠ΅, ΡƒΠΌΠ½ΠΎΠΆΠ΅Π½ΠΈΠ΅ ΠΌΠ°Ρ‚Ρ€ΠΈΡ†. Бвойства ΠΎΠΏΠ΅Ρ€Π°Ρ†ΠΈΠΉ Π½Π°Π΄ ΠΌΠ°Ρ‚Ρ€ΠΈΡ†Π°ΠΌΠΈ.

Π‘Π»ΠΎΠΆΠ΅Π½ΠΈΠ΅ ΠΌΠ°Ρ‚Ρ€ΠΈΡ†:

Π’Ρ‹Ρ‡ΠΈΡ‚Π°Π½ΠΈΠ΅ ΠΈ слоТСниС ΠΌΠ°Ρ‚Ρ€ΠΈΡ† сводится ΠΊ ΡΠΎΠΎΡ‚Π²Π΅Ρ‚ΡΡ‚Π²ΡƒΡŽΡ‰ΠΈΠΌ опСрациям Π½Π°Π΄ ΠΈΡ… элСмСнтами. ΠžΠΏΠ΅Ρ€Π°Ρ†ΠΈΡ слоТСния ΠΌΠ°Ρ‚Ρ€ΠΈΡ† вводится Ρ‚ΠΎΠ»ΡŒΠΊΠΎ для ΠΌΠ°Ρ‚Ρ€ΠΈΡ† ΠΎΠ΄ΠΈΠ½Π°ΠΊΠΎΠ²ΠΎΠ³ΠΎ Ρ€Π°Π·ΠΌΠ΅Ρ€Π°, Ρ‚. Π΅. для ΠΌΠ°Ρ‚Ρ€ΠΈΡ†, Ρƒ ΠΊΠΎΡ‚ΠΎΡ€Ρ‹Ρ… число строк ΠΈ столбцов соотвСтствСнно Ρ€Π°Π²Π½ΠΎ. Π‘ΡƒΠΌΠΌΠΎΠΉ ΠΌΠ°Ρ‚Ρ€ΠΈΡ† А ΠΈ Π’, называСтся ΠΌΠ°Ρ‚Ρ€ΠΈΡ†Π° Π‘, элСмСнты ΠΊΠΎΡ‚ΠΎΡ€ΠΎΠΉ Ρ€Π°Π²Π½Ρ‹ суммС ΡΠΎΠΎΡ‚Π²Π΅Ρ‚ΡΡ‚Π²ΡƒΡŽΡ‰ΠΈΡ… элСмСнтов. Π‘ = А + Π’ cij = aij + bij Аналогично опрСдСляСтся Ρ€Π°Π·Π½ΠΎΡΡ‚ΡŒ ΠΌΠ°Ρ‚Ρ€ΠΈΡ†.

Π£ΠΌΠ½ΠΎΠΆΠ΅Π½ΠΈΠ΅ ΠΌΠ°Ρ‚Ρ€ΠΈΡ†Ρ‹ Π½Π° число:

ΠžΠΏΠ΅Ρ€Π°Ρ†ΠΈΡ умноТСния (дСлСния) ΠΌΠ°Ρ‚Ρ€ΠΈΡ†Ρ‹ любого Ρ€Π°Π·ΠΌΠ΅Ρ€Π° Π½Π° ΠΏΡ€ΠΎΠΈΠ·Π²ΠΎΠ»ΡŒΠ½ΠΎΠ΅ число сводится ΠΊ ΡƒΠΌΠ½ΠΎΠΆΠ΅Π½ΠΈΡŽ (дСлСнию) ΠΊΠ°ΠΆΠ΄ΠΎΠ³ΠΎ элСмСнта

ΠΌΠ°Ρ‚Ρ€ΠΈΡ†Ρ‹ Π½Π° это число. ΠŸΡ€ΠΎΠΈΠ·Π²Π΅Π΄Π΅Π½ΠΈΠ΅ΠΌ ΠΌΠ°Ρ‚Ρ€ΠΈΡ†Ρ‹ А Π½Π° число k называСтся ΠΌΠ°Ρ‚Ρ€ΠΈΡ†Π° Π’, такая Ρ‡Ρ‚ΠΎ

bij = k Γ— aij. Π’ = k Γ— A bij = k Γ— aij. ΠœΠ°Ρ‚Ρ€ΠΈΡ†Π°Β Β  — А = (-1) Γ— А   называСтся ΠΏΡ€ΠΎΡ‚ΠΈΠ²ΠΎΠΏΠΎΠ»ΠΎΠΆΠ½ΠΎΠΉ ΠΌΠ°Ρ‚Ρ€ΠΈΡ†Π΅ А.

Бвойства слоТСния ΠΌΠ°Ρ‚Ρ€ΠΈΡ† ΠΈ умноТСния ΠΌΠ°Ρ‚Ρ€ΠΈΡ†Ρ‹ Π½Π° число:

ΠžΠΏΠ΅Ρ€Π°Ρ†ΠΈΠΈ слоТСния ΠΌΠ°Ρ‚Ρ€ΠΈΡ† ΠΈ умноТСния ΠΌΠ°Ρ‚Ρ€ΠΈΡ†Ρ‹ Π½Π° число ΠΎΠ±Π»Π°Π΄Π°ΡŽΡ‚ ΡΠ»Π΅Π΄ΡƒΡŽΡ‰ΠΈΠΌΠΈ свойствами: 1. А + Π’ = Π’ + А; 2. А + (Π’ + Π‘) = (А + Π’) + Π‘; 3. А + 0 = А; 4. А — А = 0; 5. 1 Γ— А = А; 6. Ξ± Γ— (А + Π’) = αА + Ξ±Π’; 7. (Ξ± + Ξ²) Γ— А = αА + βА; 8. Ξ± Γ— (βА) = (Ξ±Ξ²) Γ— А; , Π³Π΄Π΅ А, Π’ ΠΈ Π‘ — ΠΌΠ°Ρ‚Ρ€ΠΈΡ†Ρ‹, Ξ± ΠΈ Ξ² — числа.

Π£ΠΌΠ½ΠΎΠΆΠ΅Π½ΠΈΠ΅ ΠΌΠ°Ρ‚Ρ€ΠΈΡ† (ΠŸΡ€ΠΎΠΈΠ·Π²Π΅Π΄Π΅Π½ΠΈΠ΅ ΠΌΠ°Ρ‚Ρ€ΠΈΡ†):

ΠžΠΏΠ΅Ρ€Π°Ρ†ΠΈΡ умноТСния Π΄Π²ΡƒΡ… ΠΌΠ°Ρ‚Ρ€ΠΈΡ† вводится Ρ‚ΠΎΠ»ΡŒΠΊΠΎ для случая, ΠΊΠΎΠ³Π΄Π° число столбцов ΠΏΠ΅Ρ€Π²ΠΎΠΉ

ΠΌΠ°Ρ‚Ρ€ΠΈΡ†Ρ‹ Ρ€Π°Π²Π½ΠΎ числу строк Π²Ρ‚ΠΎΡ€ΠΎΠΉ ΠΌΠ°Ρ‚Ρ€ΠΈΡ†Ρ‹. ΠŸΡ€ΠΎΠΈΠ·Π²Π΅Π΄Π΅Π½ΠΈΠ΅ΠΌ ΠΌΠ°Ρ‚Ρ€ΠΈΡ†Ρ‹ АmΓ—n Π½Π° ΠΌΠ°Ρ‚Ρ€ΠΈΡ†Ρƒ Π’nΓ—p, называСтся ΠΌΠ°Ρ‚Ρ€ΠΈΡ†Π° Π‘mΓ—p такая, Ρ‡Ρ‚ΠΎ сik = ai1 Γ— b1k + ai2 Γ— b2k + . .. + ain Γ— bnk, Ρ‚. Π΅. Π½Π°Ρ…ΠΎΠ΄ΠΈΡ‚ΡŒΡΡ сумма ΠΏΡ€ΠΎΠΈΠ·Π²Π΅Π΄Π΅Π½ΠΈΠΉ элСмСнтов i — ΠΎΠΉ строки ΠΌΠ°Ρ‚Ρ€ΠΈΡ†Ρ‹ А Π½Π° ΡΠΎΠΎΡ‚Π²Π΅Ρ‚ΡΡ‚Π²ΡƒΡŽΡ‰ΠΈΠ΅ элСмСнты j — ΠΎΠ³ΠΎ столбца ΠΌΠ°Ρ‚Ρ€ΠΈΡ†Ρ‹ Π’. Если ΠΌΠ°Ρ‚Ρ€ΠΈΡ†Ρ‹ А ΠΈ Π’ ΠΊΠ²Π°Π΄Ρ€Π°Ρ‚Π½Ρ‹Π΅ ΠΎΠ΄Π½ΠΎΠ³ΠΎ Ρ€Π°Π·ΠΌΠ΅Ρ€Π°, Ρ‚ΠΎ произвСдСния АВ ΠΈ ВА всСгда ΡΡƒΡ‰Π΅ΡΡ‚Π²ΡƒΡŽΡ‚. Π›Π΅Π³ΠΊΠΎ ΠΏΠΎΠΊΠ°Π·Π°Ρ‚ΡŒ, Ρ‡Ρ‚ΠΎ А Γ— Π• = Π• Γ— А = А, Π³Π΄Π΅ А квадратная ΠΌΠ°Ρ‚Ρ€ΠΈΡ†Π°, Π• — Сдиничная ΠΌΠ°Ρ‚Ρ€ΠΈΡ†Π° Ρ‚ΠΎΠ³ΠΎ ΠΆΠ΅ Ρ€Π°Π·ΠΌΠ΅Ρ€Π°.

Бвойства умноТСния ΠΌΠ°Ρ‚Ρ€ΠΈΡ†:

Π£ΠΌΠ½ΠΎΠΆΠ΅Π½ΠΈΠ΅ ΠΌΠ°Ρ‚Ρ€ΠΈΡ† Π½Π΅ ΠΊΠΎΠΌΠΌΡƒΡ‚Π°Ρ‚ΠΈΠ²Π½ΠΎ, Ρ‚.Π΅. АВ β‰  ВА Π΄Π°ΠΆΠ΅ Ссли ΠΎΠΏΡ€Π΅Π΄Π΅Π»Π΅Π½Ρ‹ ΠΎΠ±Π° произвСдСния. Однако, Ссли для ΠΊΠ°ΠΊΠΈΡ… — Π»ΠΈΠ±ΠΎ

ΠΌΠ°Ρ‚Ρ€ΠΈΡ† ΡΠΎΠΎΡ‚Π½ΠΎΡˆΠ΅Π½ΠΈΠ΅ АВ=ВА выполняСтся, Ρ‚ΠΎ Ρ‚Π°ΠΊΠΈΠ΅ ΠΌΠ°Ρ‚Ρ€ΠΈΡ†Ρ‹ Π½Π°Π·Ρ‹Π²Π°ΡŽΡ‚ΡΡ пСрСстановочными. Π‘Π°ΠΌΡ‹ΠΌ Ρ…Π°Ρ€Π°ΠΊΡ‚Π΅Ρ€Π½Ρ‹ΠΌ ΠΏΡ€ΠΈΠΌΠ΅Ρ€ΠΎΠΌ ΠΌΠΎΠΆΠ΅Ρ‚ ΡΠ»ΡƒΠΆΠΈΡ‚ΡŒ Сдиничная ΠΌΠ°Ρ‚Ρ€ΠΈΡ†Π°, которая являСтся пСрСстановочной с любой Π΄Ρ€ΡƒΠ³ΠΎΠΉ ΠΌΠ°Ρ‚Ρ€ΠΈΡ†Π΅ΠΉ Ρ‚ΠΎΠ³ΠΎ ΠΆΠ΅ Ρ€Π°Π·ΠΌΠ΅Ρ€Π°. ΠŸΠ΅Ρ€Π΅ΡΡ‚Π°Π½ΠΎΠ²ΠΎΡ‡Π½Ρ‹ΠΌΠΈ ΠΌΠΎΠ³ΡƒΡ‚ Π±Ρ‹Ρ‚ΡŒ Ρ‚ΠΎΠ»ΡŒΠΊΠΎ ΠΊΠ²Π°Π΄Ρ€Π°Ρ‚Π½Ρ‹Π΅ ΠΌΠ°Ρ‚Ρ€ΠΈΡ†Ρ‹ ΠΎΠ΄Π½ΠΎΠ³ΠΎ ΠΈ Ρ‚ΠΎΠ³ΠΎ ΠΆΠ΅ порядка. А Γ— Π• = Π• Γ— А = А

Π£ΠΌΠ½ΠΎΠΆΠ΅Π½ΠΈΠ΅ ΠΌΠ°Ρ‚Ρ€ΠΈΡ† ΠΎΠ±Π»Π°Π΄Π°Π΅Ρ‚ ΡΠ»Π΅Π΄ΡƒΡŽΡ‰ΠΈΠΌΠΈ свойствами: 1. А Γ— (Π’ Γ— Π‘) = (А Γ— Π’) Γ— Π‘; 2. А Γ— (Π’ + Π‘) = АВ + АБ; 3. (А + Π’) Γ— Π‘ = АБ + Π’Π‘; 4. Ξ± Γ— (АВ) = (αА) Γ— Π’; 5. А Γ— 0 = 0; 0 Γ— А = 0; 6. (АВ)Π’ = ВВАВ; 7. (АВБ)Π’ = БВВВАВ; 8. (А + Π’)Π’ = АВ + Π’Π’;

2. ΠžΠΏΡ€Π΅Π΄Π΅Π»ΠΈΡ‚Π΅Π»ΠΈ 2-Π³ΠΎ ΠΈ 3-Π³ΠΎ порядков. Бвойства ΠΎΠΏΡ€Π΅Π΄Π΅Π»ΠΈΡ‚Π΅Π»Π΅ΠΉ.

ΠžΠΏΡ€Π΅Π΄Π΅Π»ΠΈΡ‚Π΅Π»Π΅ΠΌ ΠΌΠ°Ρ‚Ρ€ΠΈΡ†Ρ‹ Π²Ρ‚ΠΎΡ€ΠΎΠ³ΠΎ порядка, ΠΈΠ»ΠΈ ΠΎΠΏΡ€Π΅Π΄Π΅Π»ΠΈΡ‚Π΅Π»Π΅ΠΌ Π²Ρ‚ΠΎΡ€ΠΎΠ³ΠΎ порядка, называСтся число, ΠΊΠΎΡ‚ΠΎΡ€ΠΎΠ΅ вычисляСтся ΠΏΠΎ Ρ„ΠΎΡ€ΠΌΡƒΠ»Π΅:

ΠžΠΏΡ€Π΅Π΄Π΅Π»ΠΈΡ‚Π΅Π»Π΅ΠΌ ΠΌΠ°Ρ‚Ρ€ΠΈΡ†Ρ‹ Ρ‚Ρ€Π΅Ρ‚ΡŒΠ΅Π³ΠΎ порядка, ΠΈΠ»ΠΈ ΠΎΠΏΡ€Π΅Π΄Π΅Π»ΠΈΡ‚Π΅Π»Π΅ΠΌ Ρ‚Ρ€Π΅Ρ‚ΡŒΠ΅Π³ΠΎ порядка, называСтся число, ΠΊΠΎΡ‚ΠΎΡ€ΠΎΠ΅ вычисляСтся ΠΏΠΎ Ρ„ΠΎΡ€ΠΌΡƒΠ»Π΅:

Π­Ρ‚ΠΎ число прСдставляСт Π°Π»Π³Π΅Π±Ρ€Π°ΠΈΡ‡Π΅ΡΠΊΡƒΡŽ сумму, ΡΠΎΡΡ‚ΠΎΡΡ‰ΡƒΡŽ ΠΈΠ· ΡˆΠ΅ΡΡ‚ΠΈ слагаСмых. Π’ ΠΊΠ°ΠΆΠ΄ΠΎΠ΅ слагаСмоС Π²Ρ…ΠΎΠ΄ΠΈΡ‚ Ρ€ΠΎΠ²Π½ΠΎ ΠΏΠΎ ΠΎΠ΄Π½ΠΎΠΌΡƒ элСмСнту ΠΈΠ· ΠΊΠ°ΠΆΠ΄ΠΎΠΉ строки ΠΈ ΠΊΠ°ΠΆΠ΄ΠΎΠ³ΠΎ столбца ΠΌΠ°Ρ‚Ρ€ΠΈΡ†Ρ‹. КаТдоС слагаСмоС состоит ΠΈΠ· произвСдСния Ρ‚Ρ€Π΅Ρ… сомноТитСлСй.

Π—Π½Π°ΠΊΠΈ, с ΠΊΠΎΡ‚ΠΎΡ€Ρ‹ΠΌΠΈ Ρ‡Π»Π΅Π½Ρ‹ опрСдСлитСля ΠΌΠ°Ρ‚Ρ€ΠΈΡ†Ρ‹ входят Π² Ρ„ΠΎΡ€ΠΌΡƒΠ»Ρƒ нахоТдСния опрСдСлитСля ΠΌΠ°Ρ‚Ρ€ΠΈΡ†Ρ‹ Ρ‚Ρ€Π΅Ρ‚ΡŒΠ΅Π³ΠΎ порядка ΠΌΠΎΠΆΠ½ΠΎ ΠΎΠΏΡ€Π΅Π΄Π΅Π»ΠΈΡ‚ΡŒ, ΠΏΠΎΠ»ΡŒΠ·ΡƒΡΡΡŒ ΠΏΡ€ΠΈΠ²Π΅Π΄Π΅Π½Π½ΠΎΠΉ схСмой, которая называСтся ΠΏΡ€Π°Π²ΠΈΠ»ΠΎΠΌ Ρ‚Ρ€Π΅ΡƒΠ³ΠΎΠ»ΡŒΠ½ΠΈΠΊΠΎΠ² ΠΈΠ»ΠΈ ΠΏΡ€Π°Π²ΠΈΠ»ΠΎΠΌ Барруса. ΠŸΠ΅Ρ€Π²Ρ‹Π΅ Ρ‚Ρ€ΠΈ слагаСмыС бСрутся со Π·Π½Π°ΠΊΠΎΠΌ плюс ΠΈ ΠΎΠΏΡ€Π΅Π΄Π΅Π»ΡΡŽΡ‚ΡΡ ΠΈΠ· Π»Π΅Π²ΠΎΠ³ΠΎ рисунка, Π° ΠΏΠΎΡΠ»Π΅Π΄ΡƒΡŽΡ‰ΠΈΠ΅ Ρ‚Ρ€ΠΈ слагаСмыС бСрутся со Π·Π½Π°ΠΊΠΎΠΌ минус ΠΈ ΠΎΠΏΡ€Π΅Π΄Π΅Π»ΡΡŽΡ‚ΡΡ ΠΈΠ· ΠΏΡ€Π°Π²ΠΎΠ³ΠΎ рисунка.

ΠžΠΏΡ€Π΅Π΄Π΅Π»ΠΈΡ‚ΡŒ количСство слагаСмых, для нахоТдСния опрСдСлитСля ΠΌΠ°Ρ‚Ρ€ΠΈΡ†Ρ‹, Π² алгСбраичСской суммС, ΠΌΠΎΠΆΠ½ΠΎ вычислив Ρ„Π°ΠΊΡ‚ΠΎΡ€ΠΈΠ°Π»: 2! = 1 Γ— 2 = 2 3! = 1 Γ— 2 Γ— 3 = 6

Бвойства ΠΎΠΏΡ€Π΅Π΄Π΅Π»ΠΈΡ‚Π΅Π»Π΅ΠΉ ΠΌΠ°Ρ‚Ρ€ΠΈΡ†

Бвойства ΠΎΠΏΡ€Π΅Π΄Π΅Π»ΠΈΡ‚Π΅Π»Π΅ΠΉ ΠΌΠ°Ρ‚Ρ€ΠΈΡ†:

Бвойство β„– 1:

ΠžΠΏΡ€Π΅Π΄Π΅Π»ΠΈΡ‚Π΅Π»ΡŒ ΠΌΠ°Ρ‚Ρ€ΠΈΡ†Ρ‹ Π½Π΅ измСнится, Ссли Π΅Π³ΠΎ строки Π·Π°ΠΌΠ΅Π½ΠΈΡ‚ΡŒ столбцами, ΠΏΡ€ΠΈΡ‡Π΅ΠΌ ΠΊΠ°ΠΆΠ΄ΡƒΡŽ строку столбцом с Ρ‚Π΅ΠΌ ΠΆΠ΅ Π½ΠΎΠΌΠ΅Ρ€ΠΎΠΌ, ΠΈ Π½Π°ΠΎΠ±ΠΎΡ€ΠΎΡ‚ (ВранспонированиС). |А| = |А|Π’

БлСдствиС:

Π‘Ρ‚ΠΎΠ»Π±Ρ†Ρ‹ ΠΈ строки опрСдСлитСля ΠΌΠ°Ρ‚Ρ€ΠΈΡ†Ρ‹ Ρ€Π°Π²Π½ΠΎΠΏΡ€Π°Π²Π½Ρ‹, ΡΠ»Π΅Π΄ΠΎΠ²Π°Ρ‚Π΅Π»ΡŒΠ½ΠΎ, свойства присущиС строкам Π²Ρ‹ΠΏΠΎΠ»Π½ΡΡŽΡ‚ΡΡ ΠΈ для столбцов.

Бвойство β„– 2:

ΠŸΡ€ΠΈ пСрСстановкС 2-Ρ… строк ΠΈΠ»ΠΈ столбцов ΠΎΠΏΡ€Π΅Π΄Π΅Π»ΠΈΡ‚Π΅Π»ΡŒ ΠΌΠ°Ρ‚Ρ€ΠΈΡ†Ρ‹ ΠΈΠ·ΠΌΠ΅Π½ΠΈΡ‚ Π·Π½Π°ΠΊ Π½Π° ΠΏΡ€ΠΎΡ‚ΠΈΠ²ΠΎΠΏΠΎΠ»ΠΎΠΆΠ½Ρ‹ΠΉ, сохраняя Π°Π±ΡΠΎΠ»ΡŽΡ‚Π½ΡƒΡŽ Π²Π΅Π»ΠΈΡ‡ΠΈΠ½Ρƒ, Ρ‚.Π΅.:

Бвойство β„– 3:

ΠžΠΏΡ€Π΅Π΄Π΅Π»ΠΈΡ‚Π΅Π»ΡŒ ΠΌΠ°Ρ‚Ρ€ΠΈΡ†Ρ‹, ΠΈΠΌΠ΅ΡŽΡ‰ΠΈΠΉ Π΄Π²Π° ΠΎΠ΄ΠΈΠ½Π°ΠΊΠΎΠ²Ρ‹Ρ… ряда, Ρ€Π°Π²Π΅Π½ Π½ΡƒΠ»ΡŽ.

Бвойство β„– 4:

ΠžΠ±Ρ‰ΠΈΠΉ ΠΌΠ½ΠΎΠΆΠΈΡ‚Π΅Π»ΡŒ элСмСнтов ΠΊΠ°ΠΊΠΎΠ³ΠΎ-Π»ΠΈΠ±ΠΎ ряда опрСдСлитСля ΠΌΠ°Ρ‚Ρ€ΠΈΡ†Ρ‹ ΠΌΠΎΠΆΠ½ΠΎ вынСсти Π·Π° Π·Π½Π°ΠΊ опрСдСлитСля.

БлСдствия ΠΈΠ· свойств β„– 3 ΠΈ β„– 4:

Если всС элСмСнты Π½Π΅ΠΊΠΎΡ‚ΠΎΡ€ΠΎΠ³ΠΎ ряда (строки ΠΈΠ»ΠΈ столбца) ΠΏΡ€ΠΎΠΏΠΎΡ€Ρ†ΠΈΠΎΠ½Π°Π»ΡŒΠ½Ρ‹ ΡΠΎΠΎΡ‚Π²Π΅Ρ‚ΡΡ‚Π²ΡƒΡŽΡ‰ΠΈΠΌ элСмСнтам ΠΏΠ°Ρ€Π°Π»Π»Π΅Π»ΡŒΠ½ΠΎΠ³ΠΎ ряда, Ρ‚ΠΎ Ρ‚Π°ΠΊΠΎΠΉ ΠΎΠΏΡ€Π΅Π΄Π΅Π»ΠΈΡ‚Π΅Π»ΡŒ ΠΌΠ°Ρ‚Ρ€ΠΈΡ†Ρ‹ Ρ€Π°Π²Π΅Π½ Π½ΡƒΠ»ΡŽ.

Бвойство β„– 5:

Если всС элСмСнты какой–либо строки ΠΈΠ»ΠΈ столбца опрСдСлитСля ΠΌΠ°Ρ‚Ρ€ΠΈΡ†Ρ‹ Ρ€Π°Π²Π½Ρ‹ Π½ΡƒΠ»ΡŽ, Ρ‚ΠΎ сам ΠΎΠΏΡ€Π΅Π΄Π΅Π»ΠΈΡ‚Π΅Π»ΡŒ ΠΌΠ°Ρ‚Ρ€ΠΈΡ†Ρ‹ Ρ€Π°Π²Π΅Π½ Π½ΡƒΠ»ΡŽ.

Бвойство β„– 6:

Если всС элСмСнты какой–либо строки ΠΈΠ»ΠΈ столбца опрСдСлитСля прСдставлСны Π² Π²ΠΈΠ΄Π΅ суммы 2-Ρ… слагаСмых, Ρ‚ΠΎ ΠΎΠΏΡ€Π΅Π΄Π΅Π»ΠΈΡ‚Π΅Π»ΡŒ ΠΌΠ°Ρ‚Ρ€ΠΈΡ†Ρ‹ ΠΌΠΎΠΆΠ½ΠΎ ΠΏΡ€Π΅Π΄ΡΡ‚Π°Π²ΠΈΡ‚ΡŒ Π² Π²ΠΈΠ΄Π΅ суммы 2-Ρ… ΠΎΠΏΡ€Π΅Π΄Π΅Π»ΠΈΡ‚Π΅Π»Π΅ΠΉ ΠΏΠΎ Ρ„ΠΎΡ€ΠΌΡƒΠ»Π΅:

Бвойство β„– 7:

Если ΠΊ какой–либо строкС (ΠΈΠ»ΠΈ столбцу) опрСдСлитСля ΠΏΡ€ΠΈΠ±Π°Π²ΠΈΡ‚ΡŒ ΡΠΎΠΎΡ‚Π²Π΅Ρ‚ΡΡ‚Π²ΡƒΡŽΡ‰ΠΈΠ΅ элСмСнты Π΄Ρ€ΡƒΠ³ΠΎΠΉ строки (ΠΈΠ»ΠΈ столбца), ΡƒΠΌΠ½ΠΎΠΆΠ΅Π½Π½Ρ‹Π΅ Π½Π° ΠΎΠ΄Π½ΠΎ ΠΈ Ρ‚ΠΎΠΆΠ΅ число, Ρ‚ΠΎ ΠΎΠΏΡ€Π΅Π΄Π΅Π»ΠΈΡ‚Π΅Π»ΡŒ ΠΌΠ°Ρ‚Ρ€ΠΈΡ†Ρ‹ Π½Π΅ ΠΈΠ·ΠΌΠ΅Π½ΠΈΡ‚ своСй Π²Π΅Π»ΠΈΡ‡ΠΈΠ½Ρ‹.

ΠŸΡ€ΠΈΠΌΠ΅Ρ€ примСнСния свойств для вычислСния опрСдСлитСля ΠΌΠ°Ρ‚Ρ€ΠΈΡ†Ρ‹:

ΠšΠ°Π»ΡŒΠΊΡƒΠ»ΡΡ‚ΠΎΡ€ ΠΎΠ½Π»Π°ΠΉΠ½ — ΠžΠΏΠ΅Ρ€Π°Ρ†ΠΈΠΈ Π½Π°Π΄ ΠΌΠ°Ρ‚Ρ€ΠΈΡ†Π°ΠΌΠΈ: слоТСниС, Π²Ρ‹Ρ‡ΠΈΡ‚Π°Π½ΠΈΠ΅, ΡƒΠΌΠ½ΠΎΠΆΠ΅Π½ΠΈΠ΅, Π΄Π΅Π»Π΅Π½ΠΈΠ΅, Π²ΠΎΠ·Π²Π΅Π΄Π΅Π½ΠΈΠ΅ Π² ΡΡ‚Π΅ΠΏΠ΅Π½ΡŒ. ВычислСниС опрСдСлитСля ΠΌΠ°Ρ‚Ρ€ΠΈΡ†Ρ‹, ΠΎΠ±Ρ€Π°Ρ‚Π½ΠΎΠΉ ΠΌΠ°Ρ‚Ρ€ΠΈΡ†Ρ‹, Ρ€Π°Π½Π³Π° ΠΌΠ°Ρ‚Ρ€ΠΈΡ†Ρ‹

ΠžΠ±Π½Π°Ρ€ΡƒΠΆΠ΅Π½ΠΎ Ρ‡Ρ‚ΠΎ Π½Π΅ Π·Π°Π³Ρ€ΡƒΠ·ΠΈΠ»ΠΈΡΡŒ Π½Π΅ΠΊΠΎΡ‚ΠΎΡ€Ρ‹Π΅ скрипты, Π½Π΅ΠΎΠ±Ρ…ΠΎΠ΄ΠΈΠΌΡ‹Π΅ для Ρ€Π΅ΡˆΠ΅Π½ΠΈΡ этой Π·Π°Π΄Π°Ρ‡ΠΈ, ΠΈ ΠΏΡ€ΠΎΠ³Ρ€Π°ΠΌΠΌΠ° ΠΌΠΎΠΆΠ΅Ρ‚ Π½Π΅ Ρ€Π°Π±ΠΎΡ‚Π°Ρ‚ΡŒ.
Π’ΠΎΠ·ΠΌΠΎΠΆΠ½ΠΎ Ρƒ вас Π²ΠΊΠ»ΡŽΡ‡Π΅Π½ AdBlock.
Π’ этом случаС ΠΎΡ‚ΠΊΠ»ΡŽΡ‡ΠΈΡ‚Π΅ Π΅Π³ΠΎ ΠΈ ΠΎΠ±Π½ΠΎΠ²ΠΈΡ‚Π΅ страницу.

ΠœΠ°Ρ‚Ρ€ΠΈΡ†Ρ‹ ΠΈ ΠΎΠΏΠ΅Ρ€Π°Ρ†ΠΈΠΈ Π½Π°Π΄ Π½ΠΈΠΌΠΈ

Π’ΠΈΠ΄Ρ‹ ΠΌΠ°Ρ‚Ρ€ΠΈΡ†

ΠžΠΏΡ€Π΅Π΄Π΅Π»Π΅Π½ΠΈΠ΅ 1.
ΠœΠ°Ρ‚Ρ€ΠΈΡ†Π΅ΠΉ Ρ€Π°Π·ΠΌΠ΅Ρ€Π° \(m \times n \) Π½Π°Π·Ρ‹Π²Π°ΡŽΡ‚ ΠΏΡ€ΡΠΌΠΎΡƒΠ³ΠΎΠ»ΡŒΠ½ΡƒΡŽ Ρ‡ΠΈΡΠ»ΠΎΠ²ΡƒΡŽ Ρ‚Π°Π±Π»ΠΈΡ†Ρƒ, ΡΠΎΡΡ‚ΠΎΡΡ‰ΡƒΡŽ ΠΈΠ· \(mn\) чисСл, ΠΊΠΎΡ‚ΠΎΡ€Ρ‹Π΅ располоТСны Π² \(m\) строках ΠΈ \(n\) столбцах. Π‘ΠΎΡΡ‚Π°Π²Π»ΡΡŽΡ‰ΠΈΠ΅ ΠΌΠ°Ρ‚Ρ€ΠΈΡ†Ρƒ числа Π½Π°Π·Ρ‹Π²Π°ΡŽΡ‚ элСмСнтами этой ΠΌΠ°Ρ‚Ρ€ΠΈΡ†Ρ‹.

Как ΠΏΡ€Π°Π²ΠΈΠ»ΠΎ, ΠΈΡ… ΠΎΠ±ΠΎΠ·Π½Π°Ρ‡Π°ΡŽΡ‚ строчной Π±ΡƒΠΊΠ²ΠΎΠΉ с двумя индСксами, Π½Π°ΠΏΡ€ΠΈΠΌΠ΅Ρ€ \(a_{ij}\), Π³Π΄Π΅ \(i\) β€” Π½ΠΎΠΌΠ΅Ρ€ строки ( \(i=\overline{1,m} \) ), \(j\) β€” Π½ΠΎΠΌΠ΅Ρ€ столбца ( \(j=\overline{1,n} \) ), Π² ΠΊΠΎΡ‚ΠΎΡ€Ρ‹Ρ… располоТСн этот элСмСнт.

ΠœΠ°Ρ‚Ρ€ΠΈΡ†Ρƒ Π·Π°ΠΏΠΈΡΡ‹Π²Π°ΡŽΡ‚ Ρ‚Π°ΠΊ:
$$ \begin{pmatrix} a_{11} & a_{12} & \cdots & a_{1n} \\ a_{21} & a_{22} & \cdots & a_{2n} \\ \vdots & \vdots & \ddots & \vdots \\ a_{m1} & a_{m2} & \cdots & a_{mn} \\ \end{pmatrix} $$

Если ΠΏΠΎ тСксту ясно, Π² ΠΊΠ°ΠΊΠΈΡ… ΠΏΡ€Π΅Π΄Π΅Π»Π°Ρ… ΠΈΠ·ΠΌΠ΅Π½ΡΡŽΡ‚ΡΡ индСксы \(i\) ΠΈ \(j\), Ρ‚ΠΎ сокращённо ΠΌΠ°Ρ‚Ρ€ΠΈΡ†Ρƒ ΠΌΠΎΠΆΠ½ΠΎ Π·Π°ΠΏΠΈΡΠ°Ρ‚ΡŒ Ρ‚Π°ΠΊ: \( \left(a_{ij} \right) \). ΠœΠ°Ρ‚Ρ€ΠΈΡ†Ρƒ ΠΊΠ°ΠΊ Π΅Π΄ΠΈΠ½Ρ‹ΠΉ ΠΎΠ±ΡŠΠ΅ΠΊΡ‚ ΠΎΠ±ΠΎΠ·Π½Π°Ρ‡Π°ΡŽΡ‚ прописной Π±ΡƒΠΊΠ²ΠΎΠΉ: \(A\), \(B\) ΠΈ Ρ‚.Π΄.

Π­Π»Π΅ΠΌΠ΅Π½Ρ‚Π°ΠΌΠΈ ΠΌΠ°Ρ‚Ρ€ΠΈΡ† ΠΌΠΎΠ³ΡƒΡ‚ Π±Ρ‹Ρ‚ΡŒ Π½Π΅ Ρ‚ΠΎΠ»ΡŒΠΊΠΎ Π΄Π΅ΠΉΡΡ‚Π²ΠΈΡ‚Π΅Π»ΡŒΠ½Ρ‹Π΅ числа, Π½ΠΎ ΠΈ комплСксныС, ΠΈ Π΄Π°ΠΆΠ΅ Π΄Ρ€ΡƒΠ³ΠΈΠ΅ матСматичСскиС ΠΎΠ±ΡŠΠ΅ΠΊΡ‚Ρ‹. НапримСр, элСмСнтами ΠΌΠ°Ρ‚Ρ€ΠΈΡ† ΠΌΠΎΠ³ΡƒΡ‚ Π±Ρ‹Ρ‚ΡŒ ΠΌΠ½ΠΎΠ³ΠΎΡ‡Π»Π΅Π½Ρ‹ ΠΈΠ»ΠΈ ΠΌΠ°Ρ‚Ρ€ΠΈΡ†Ρ‹.

ΠœΠ°Ρ‚Ρ€ΠΈΡ†Ρƒ Π½Π°Π·Ρ‹Π²Π°ΡŽΡ‚ ΠΌΠ°Ρ‚Ρ€ΠΈΡ†Π΅ΠΉ-строкой, Ссли ΠΌΠ°Ρ‚Ρ€ΠΈΡ†Π° ΠΈΠΌΠ΅Π΅Ρ‚ Ρ€Π°Π·ΠΌΠ΅Ρ€ \(1 \times n \), Ρ‚.Π΅. Ссли Ρƒ ΠΌΠ°Ρ‚Ρ€ΠΈΡ†Ρ‹ всСго ΠΎΠ΄Π½Π° строка. Число элСмСнтов Π² ΠΌΠ°Ρ‚Ρ€ΠΈΡ†Π΅-строкС Π½Π°Π·Ρ‹Π²Π°ΡŽΡ‚ Π΅Ρ‘ Π΄Π»ΠΈΠ½ΠΎΠΉ.
\(A=(a_1, \; …,\; a_n) \)

ΠœΠ°Ρ‚Ρ€ΠΈΡ†Ρƒ Π½Π°Π·Ρ‹Π²Π°ΡŽΡ‚ ΠΌΠ°Ρ‚Ρ€ΠΈΡ†Π΅ΠΉ-столбцом, Ссли ΠΌΠ°Ρ‚Ρ€ΠΈΡ†Π° ΠΈΠΌΠ΅Π΅Ρ‚ Ρ€Π°Π·ΠΌΠ΅Ρ€ \(m \times 1 \), Ρ‚.Π΅. Ссли Ρƒ ΠΌΠ°Ρ‚Ρ€ΠΈΡ†Ρ‹ ΠΎΠ΄ΠΈΠ½ столбСц. Число элСмСнтов Π² ΠΌΠ°Ρ‚Ρ€ΠΈΡ†Π΅-столбцС Π½Π°Π·Ρ‹Π²Π°ΡŽΡ‚ Π΅Ρ‘ высотой.
\(A = \begin{pmatrix} a_{1} \\ \vdots \\ a_{m} \\ \end{pmatrix} \)

ΠœΠ°Ρ‚Ρ€ΠΈΡ†Ρƒ Π½Π°Π·Ρ‹Π²Π°ΡŽΡ‚ ΠΊΠ²Π°Π΄Ρ€Π°Ρ‚Π½ΠΎΠΉ порядка \(n\), Ссли \( m=n\), Ρ‚.Π΅. ΠΊΠΎΠ³Π΄Π° ΠΌΠ°Ρ‚Ρ€ΠΈΡ†Π° ΠΈΠΌΠ΅Π΅Ρ‚ ΡΡ‚ΠΎΠ»ΡŒΠΊΠΎ ΠΆΠ΅ столбцов, сколько ΠΈ строк :
$$ \begin{pmatrix} a_{11} & a_{12} & \cdots & a_{1n} \\ a_{21} & a_{22} & \cdots & a_{2n} \\ \vdots & \vdots & \ddots & \vdots \\ a_{n1} & a_{n2} & \cdots & a_{nn} \\ \end{pmatrix} $$ Π° Ссли \(m \neq n \) β€” ΠΏΡ€ΡΠΌΠΎΡƒΠ³ΠΎΠ»ΡŒΠ½ΠΎΠΉ.

Π£ ΠΊΠ²Π°Π΄Ρ€Π°Ρ‚Π½Ρ‹Ρ… ΠΌΠ°Ρ‚Ρ€ΠΈΡ† Π²Ρ‹Π΄Π΅Π»ΡΡŽΡ‚ ΠΏΠΎΡΠ»Π΅Π΄ΠΎΠ²Π°Ρ‚Π΅Π»ΡŒΠ½ΠΎΡΡ‚ΠΈ элСмСнтов \( a_{11}, \; a_{22}, \; …, \; a_{nn} \) β€” Π³Π»Π°Π²Π½ΡƒΡŽ диагональ, ΠΈ \( a_{n1}, \; a_{n-1,2}, \; …, \; a_{1n} \) β€” ΠΏΠΎΠ±ΠΎΡ‡Π½ΡƒΡŽ диагональ. Π­Π»Π΅ΠΌΠ΅Π½Ρ‚Ρ‹ Π³Π»Π°Π²Π½ΠΎΠΉ Π΄ΠΈΠ°Π³ΠΎΠ½Π°Π»ΠΈ Π½Π°Π·Ρ‹Π²Π°ΡŽΡ‚ Π΄ΠΈΠ°Π³ΠΎΠ½Π°Π»ΡŒΠ½Ρ‹ΠΌΠΈ. ΠŸΠΎΠ½ΡΡ‚ΠΈΡ диагонального элСмСнта ΠΈ Π³Π»Π°Π²Π½ΠΎΠΉ Π΄ΠΈΠ°Π³ΠΎΠ½Π°Π»ΠΈ Ρ€Π°ΡΠΏΡ€ΠΎΡΡ‚Ρ€Π°Π½ΡΡŽΡ‚ ΠΈ Π½Π° ΠΏΡ€ΡΠΌΠΎΡƒΠ³ΠΎΠ»ΡŒΠ½Ρ‹Π΅ ΠΌΠ°Ρ‚Ρ€ΠΈΡ†Ρ‹.

Если Π² ΠΊΠ²Π°Π΄Ρ€Π°Ρ‚Π½ΠΎΠΉ ΠΌΠ°Ρ‚Ρ€ΠΈΡ†Π΅ порядка \(n\) всС элСмСнты, стоящиС Π²Π½Π΅ Π³Π»Π°Π²Π½ΠΎΠΉ Π΄ΠΈΠ°Π³ΠΎΠ½Π°Π»ΠΈ, Ρ€Π°Π²Π½Ρ‹ Π½ΡƒΠ»ΡŽ, Ρ‚.Π΅. Ссли ΠΌΠ°Ρ‚Ρ€ΠΈΡ†Π° ΠΈΠΌΠ΅Π΅Ρ‚ Π²ΠΈΠ΄
$$ \begin{pmatrix} a_{11} & 0 & \cdots & 0 \\ 0 & a_{22} & \cdots & 0 \\ \vdots & \vdots & \ddots & \vdots \\ 0 & 0 & \cdots & a_{nn} \\ \end{pmatrix} $$,
Ρ‚ΠΎ Π΅Ρ‘ Π½Π°Π·Ρ‹Π²Π°ΡŽΡ‚ диагональной ΠΊ ΠΎΠ±ΠΎΠ·Π½Π°Ρ‡Π°ΡŽΡ‚ \( \text{diag} (a_{11}, \; …, \; a_{nn} ) \).
Если Π² диагональной ΠΌΠ°Ρ‚Ρ€ΠΈΡ†Π΅ порядка \(n\) Π½Π° Π΄ΠΈΠ°Π³ΠΎΠ½Π°Π»ΠΈ стоят Π΅Π΄ΠΈΠ½ΠΈΡ†Ρ‹, Ρ‚ΠΎ Π΅Ρ‘ Π½Π°Π·Ρ‹Π²Π°ΡŽΡ‚ Π΅Π΄ΠΈΠ½ΠΈΡ‡Π½ΠΎΠΉ ΠΈ ΠΎΠ±ΠΎΠ·Π½Π°Ρ‡Π°ΡŽΡ‚ ΠΎΠ±Ρ‹Ρ‡Π½ΠΎ \(E\) :
$$ E = \begin{pmatrix} 1 & 0 & \cdots & 0 \\ 0 & 1 & \cdots & 0 \\ \vdots & \vdots & \ddots & \vdots \\ 0 & 0 & \cdots & 1 \\ \end{pmatrix} $$,

ΠœΠ°Ρ‚Ρ€ΠΈΡ†Ρƒ Ρ€Π°Π·ΠΌΠ΅Ρ€Π° \(m \times n \), всС элСмСнты ΠΊΠΎΡ‚ΠΎΡ€ΠΎΠΉ Ρ€Π°Π²Π½Ρ‹ Π½ΡƒΠ»ΡŽ, Π½Π°Π·Ρ‹Π²Π°ΡŽΡ‚ Π½ΡƒΠ»Π΅Π²ΠΎΠΉ ΠΌΠ°Ρ‚Ρ€ΠΈΡ†Π΅ΠΉ ΡΠΎΠΎΡ‚Π²Π΅Ρ‚ΡΡ‚Π²ΡƒΡŽΡ‰Π΅Π³ΠΎ Ρ€Π°Π·ΠΌΠ΅Ρ€Π° ΠΈ ΠΎΠ±ΠΎΠ·Π½Π°Ρ‡Π°ΡŽΡ‚ Π±ΡƒΠΊΠ²ΠΎΠΉ \(\Theta\) ΠΈΠ»ΠΈ Ρ†ΠΈΡ„Ρ€ΠΎΠΉ 0.

Часто ΠΈΡΠΏΠΎΠ»ΡŒΠ·ΡƒΡŽΡ‚ ΠΌΠ°Ρ‚Ρ€ΠΈΡ†Ρ‹ ΠΈ Π΄Ρ€ΡƒΠ³ΠΈΡ… Π²ΠΈΠ΄ΠΎΠ², Π½Π°ΠΏΡ€ΠΈΠΌΠ΅Ρ€ Π²Π΅Ρ€Ρ…Π½ΠΈΠ΅ Ρ‚Ρ€Π΅ΡƒΠ³ΠΎΠ»ΡŒΠ½Ρ‹Π΅ ΠΌΠ°Ρ‚Ρ€ΠΈΡ†Ρ‹
$$ \begin{pmatrix} a_{11} & a_{12} & \cdots & a_{1n} \\ 0 & a_{22} & \cdots & a_{2n} \\ \vdots & \vdots & \ddots & \vdots \\ 0 & 0 & \cdots & a_{nn} \\ \end{pmatrix} $$
Ρƒ ΠΊΠΎΡ‚ΠΎΡ€Ρ‹Ρ… элСмСнты, располоТСнныС ΠΏΠΎΠ΄ Π³Π»Π°Π²Π½ΠΎΠΉ диагональю, Ρ€Π°Π²Π½Ρ‹ Π½ΡƒΠ»ΡŽ, ΠΈ Π½ΠΈΠΆΠ½ΠΈΠ΅ Ρ‚Ρ€Π΅ΡƒΠ³ΠΎΠ»ΡŒΠ½Ρ‹Π΅ ΠΌΠ°Ρ‚Ρ€ΠΈΡ†Ρ‹, Ρƒ ΠΊΠΎΡ‚ΠΎΡ€Ρ‹Ρ…, Π½Π°ΠΎΠ±ΠΎΡ€ΠΎΡ‚, элСмСнты Π½Π°Π΄ Π³Π»Π°Π²Π½ΠΎΠΉ диагональю Ρ€Π°Π²Π½Ρ‹ Π½ΡƒΠ»ΡŽ:
$$ \begin{pmatrix} a_{11} & 0 & \cdots & 0 \\ a_{21} & a_{22} & \cdots & 0 \\ \vdots & \vdots & \ddots & \vdots \\ a_{n1} & a_{n2} & \cdots & a_{nn} \\ \end{pmatrix} $$

Π‘Ρ‚ΡƒΠΏΠ΅Π½Ρ‡Π°Ρ‚ΠΎΠΉ ΠΌΠ°Ρ‚Ρ€ΠΈΡ†Π΅ΠΉ (ΠΌΠ°Ρ‚Ρ€ΠΈΡ†Π΅ΠΉ ступСнчатого Π²ΠΈΠ΄Π°) Π½Π°Π·Ρ‹Π²Π°ΡŽΡ‚ ΠΌΠ°Ρ‚Ρ€ΠΈΡ†Ρƒ Ρ€Π°Π·ΠΌΠ΅Ρ€Π° \(m \times n \), Ссли для любой Π΅Ρ‘ строки Π²Ρ‹ΠΏΠΎΠ»Π½Π΅Π½ΠΎ ΡΠ»Π΅Π΄ΡƒΡŽΡ‰Π΅Π΅ условиС: ΠΏΠΎΠ΄ ΠΏΠ΅Ρ€Π²Ρ‹ΠΌ слСва Π½Π΅Π½ΡƒΠ»Π΅Π²Ρ‹ΠΌ элСмСнтом строки ΠΈ ΠΏΡ€Π΅Π΄ΡˆΠ΅ΡΡ‚Π²ΡƒΡŽΡ‰ΠΈΠΌΠΈ Π΅ΠΌΡƒ Π½ΡƒΠ»Π΅Π²Ρ‹ΠΌΠΈ элСмСнтами строки всС элСмСнты ΠΌΠ°Ρ‚Ρ€ΠΈΡ†Ρ‹ Ρ€Π°Π²Π½Ρ‹ Π½ΡƒΠ»ΡŽ.
Π‘Π»Π΅Π΄ΡƒΡŽΡ‰ΠΈΠ΅ ΠΌΠ°Ρ‚Ρ€ΠΈΡ†Ρ‹ ΠΈΠΌΠ΅ΡŽΡ‚ ступСнчатый Π²ΠΈΠ΄:
\( \begin{pmatrix} 0 & 2 & 3 & 0 \\ 0 & 0 & -1 & 1 \\ 0 & 0 & 0 & 3 \\ \end{pmatrix} \) \( \begin{pmatrix} 3 & 1 & 3 & 3 \\ 0 & 0 & 2 & 1 \\ 0 & 0 & 0 & 0 \\ \end{pmatrix} \) \( \begin{pmatrix} 1 & 1 & 3 & 3 \\ 0 & 3 & 0 & 1 \\ 0 & 0 & 1 & -1 \\ \end{pmatrix} \)

Π›ΠΈΠ½Π΅ΠΉΠ½Ρ‹Π΅ ΠΎΠΏΠ΅Ρ€Π°Ρ†ΠΈΠΈ Π½Π°Π΄ ΠΌΠ°Ρ‚Ρ€ΠΈΡ†Π°ΠΌΠΈ

ΠžΠΏΡ€Π΅Π΄Π΅Π»Π΅Π½ΠΈΠ΅ 2.
Π”Π²Π΅ ΠΌΠ°Ρ‚Ρ€ΠΈΡ†Ρ‹ Π½Π°Π·Ρ‹Π²Π°ΡŽΡ‚ Ρ€Π°Π²Π½Ρ‹ΠΌΠΈ, Ссли ΠΎΠ½ΠΈ ΠΈΠΌΠ΅ΡŽΡ‚ ΠΎΠ΄ΠΈΠ½ ΠΈ Ρ‚ΠΎΡ‚ ΠΆΠ΅ Ρ€Π°Π·ΠΌΠ΅Ρ€ ΠΈ Ссли Ρƒ Π½ΠΈΡ… ΡΠΎΠ²ΠΏΠ°Π΄Π°ΡŽΡ‚ ΡΠΎΠΎΡ‚Π²Π΅Ρ‚ΡΡ‚Π²ΡƒΡŽΡ‰ΠΈΠ΅ элСмСнты.

ΠžΠΏΡ€Π΅Π΄Π΅Π»Π΅Π½ΠΈΠ΅ 3.
Π‘ΡƒΠΌΠΌΠΎΠΉ ΠΌΠ°Ρ‚Ρ€ΠΈΡ† \( A=(a_{ij}) \) ΠΈ \( B=(b_{ij}) \) Ρ€Π°Π·ΠΌΠ΅Ρ€Π° \(m \times n \) Π½Π°Π·Ρ‹Π²Π°ΡŽΡ‚ ΠΌΠ°Ρ‚Ρ€ΠΈΡ†Ρƒ \( C=(c_{ij}) \) Ρ‚ΠΎΠ³ΠΎ ΠΆΠ΅ Ρ€Π°Π·ΠΌΠ΅Ρ€Π° с элСмСнтами \( c_{ij} = a_{ij} + b_{ij}, \; i=\overline{1,m} , \; j=\overline{1,n} \)

Π’ ΠΏΠΎΠ΄Ρ€ΠΎΠ±Π½ΠΎΠΉ записи:
\( A+B = \begin{pmatrix} a_{11} & a_{12} & \cdots & a_{1n} \\ a_{21} & a_{22} & \cdots & a_{2n} \\ \vdots & \vdots & \ddots & \vdots \\ a_{m1} & a_{m2} & \cdots & a_{mn} \\ \end{pmatrix} + \) \( \begin{pmatrix} b_{11} & b_{12} & \cdots & b_{1n} \\ b_{21} & b_{22} & \cdots & b_{2n} \\ \vdots & \vdots & \ddots & \vdots \\ b_{m1} & b_{m2} & \cdots & b_{mn} \\ \end{pmatrix} = \) \( \begin{pmatrix} a_{11}+b_{11} & a_{12}+b_{12} & \cdots & a_{1n}+b_{1n} \\ a_{21}+b_{21} & a_{22}+b_{22} & \cdots & a_{2n}+b_{2n} \\ \vdots & \vdots & \ddots & \vdots \\ a_{m1}+b_{m1} & a_{m2}+b_{m2} & \cdots & a_{mn}+b_{mn} \\ \end{pmatrix} = C \)

Π‘ΡƒΠΌΠΌΠ° ΠΎΠΏΡ€Π΅Π΄Π΅Π»Π΅Π½Π° Ρ‚ΠΎΠ»ΡŒΠΊΠΎ для ΠΌΠ°Ρ‚Ρ€ΠΈΡ† ΠΎΠ΄Π½ΠΎΠ³ΠΎ Ρ€Π°Π·ΠΌΠ΅Ρ€Π°.

ΠžΠΏΡ€Π΅Π΄Π΅Π»Π΅Π½ΠΈΠ΅ 4.
ΠŸΡ€ΠΎΠΈΠ·Π²Π΅Π΄Π΅Π½ΠΈΠ΅ΠΌ ΠΌΠ°Ρ‚Ρ€ΠΈΡ†Ρ‹ \( A=(a_{ij}) \) Ρ€Π°Π·ΠΌΠ΅Ρ€Π° \(m \times n \) Π½Π° число \( k \in \mathbb{R}\) Π½Π°Π·Ρ‹Π²Π°ΡŽΡ‚ ΠΌΠ°Ρ‚Ρ€ΠΈΡ†Ρƒ \( C=(c_{ij}) \) Ρ€Π°Π·ΠΌΠ΅Ρ€Π° \(m \times n \) с элСмСнтами \( c_{ij} = k \cdot a_{ij} \).
ΠŸΠΎΠ΄Ρ€ΠΎΠ±Π½ΠΎ это ΠΏΡ€ΠΎΠΈΠ·Π²Π΅Π΄Π΅Π½ΠΈΠ΅ выглядит Ρ‚Π°ΠΊ:
\( k \cdot \begin{pmatrix} a_{11} & a_{12} & \cdots & a_{1n} \\ a_{21} & a_{22} & \cdots & a_{2n} \\ \vdots & \vdots & \ddots & \vdots \\ a_{m1} & a_{m2} & \cdots & a_{mn} \\ \end{pmatrix} = \) \( \begin{pmatrix} k \cdot a_{11} & k \cdot a_{12} & \cdots & k \cdot a_{1n} \\ k \cdot a_{21} & k \cdot a_{22} & \cdots & k \cdot a_{2n} \\ \vdots & \vdots & \ddots & \vdots \\ k \cdot a_{m1} & k \cdot a_{m2} & \cdots & k \cdot a_{mn} \\ \end{pmatrix} \)

Π—Π°ΠΌΠ΅Ρ‡Π°Π½ΠΈΠ΅. ΠžΠΏΠ΅Ρ€Π°Ρ†ΠΈΠΈ слоТСния ΠΈ умноТСния Π½Π° число для ΠΌΠ°Ρ‚Ρ€ΠΈΡ† Π°Π½Π°Π»ΠΎΠ³ΠΈΡ‡Π½Ρ‹ ΠΎΠ΄Π½ΠΎΠΈΠΌΠ΅Π½Π½Ρ‹ΠΌ опСрациям Π½Π°Π΄ Π²Π΅ΠΊΡ‚ΠΎΡ€Π°ΠΌΠΈ. {n+m}\).

Π­Π»Π΅ΠΌΠ΅Π½Ρ‚Π°Ρ€Π½Ρ‹Π΅ прСобразования ΠΌΠ°Ρ‚Ρ€ΠΈΡ†

Π‘Π»Π΅Π΄ΡƒΡŽΡ‰ΠΈΠ΅ Ρ‚Ρ€ΠΈ ΠΎΠΏΠ΅Ρ€Π°Ρ†ΠΈΠΈ Π½Π°Π·Ρ‹Π²Π°ΡŽΡ‚ элСмСнтарными прСобразованиями строк ΠΌΠ°Ρ‚Ρ€ΠΈΡ†Ρ‹ :

1. Π£ΠΌΠ½ΠΎΠΆΠ΅Π½ΠΈΠ΅ строки ΠΌΠ°Ρ‚Ρ€ΠΈΡ†Ρ‹ Π½Π° число Π½Π΅ Ρ€Π°Π²Π½ΠΎΠ΅ Π½ΡƒΠ»ΡŽ.

2. ΠŸΠ΅Ρ€Π΅ΡΡ‚Π°Π½ΠΎΠ²ΠΊΠ° Π΄Π²ΡƒΡ… строк ΠΌΠ°Ρ‚Ρ€ΠΈΡ†Ρ‹.

3. Π‘ΡƒΠΌΠΌΠΈΡ€ΠΎΠ²Π°Π½ΠΈΠ΅ ΠΎΠ΄Π½ΠΎΠΉ строки с Π΄Ρ€ΡƒΠ³ΠΎΠΉ строкой, ΡƒΠΌΠ½ΠΎΠΆΠ΅Π½Π½ΠΎΠΉ Π½Π° число.

АналогичныС ΠΎΠΏΠ΅Ρ€Π°Ρ†ΠΈΠΈ Π½Π°Π΄ столбцами ΠΌΠ°Ρ‚Ρ€ΠΈΡ†Ρ‹ Π½Π°Π·Ρ‹Π²Π°ΡŽΡ‚ элСмСнтарными прСобразованиями столбцов.

КаТдоС элСмСнтарноС ΠΏΡ€Π΅ΠΎΠ±Ρ€Π°Π·ΠΎΠ²Π°Π½ΠΈΠ΅ строк ΠΈΠ»ΠΈ столбцов ΠΌΠ°Ρ‚Ρ€ΠΈΡ†Ρ‹ ΠΈΠΌΠ΅Π΅Ρ‚ ΠΎΠ±Ρ€Π°Ρ‚Π½ΠΎΠ΅ элСмСнтарноС ΠΏΡ€Π΅ΠΎΠ±Ρ€Π°Π·ΠΎΠ²Π°Π½ΠΈΠ΅, ΠΊΠΎΡ‚ΠΎΡ€ΠΎΠ΅ ΠΏΡ€Π΅ΠΎΠ±Ρ€Π°Π·ΠΎΠ²Π°Π½Π½ΡƒΡŽ ΠΌΠ°Ρ‚Ρ€ΠΈΡ†Ρƒ ΠΏΡ€Π΅Π²Ρ€Π°Ρ‰Π°Π΅Ρ‚ Π² ΠΈΡΡ…ΠΎΠ΄Π½ΡƒΡŽ.

Π’Π΅ΠΎΡ€Π΅ΠΌΠ°. Π‘ ΠΏΠΎΠΌΠΎΡ‰ΡŒΡŽ элСмСнтарных ΠΏΡ€Π΅ΠΎΠ±Ρ€Π°Π·ΠΎΠ²Π°Π½ΠΈΠΉ строк Π»ΡŽΠ±ΡƒΡŽ ΠΌΠ°Ρ‚Ρ€ΠΈΡ†Ρƒ ΠΌΠΎΠΆΠ½ΠΎ привСсти ΠΊ ступСнчатому Π²ΠΈΠ΄Ρƒ.

ΠžΠΏΡ€Π΅Π΄Π΅Π»ΠΈΡ‚Π΅Π»ΠΈ

ΠžΠΏΡ€Π΅Π΄Π΅Π»ΠΈΡ‚Π΅Π»ΠΈ n-Π³ΠΎ порядка

Π’ Ρ‚Π΅ΠΎΡ€ΠΈΠΈ ΠΎΠΏΡ€Π΅Π΄Π΅Π»ΠΈΡ‚Π΅Π»Π΅ΠΉ n-Π³ΠΎ порядка ΠΈΡΠΏΠΎΠ»ΡŒΠ·ΡƒΡŽΡ‚ΡΡ понятия пСрСстановки, подстановки ΠΈ ΠΈΡ… чСтности. ВсякоС располоТСниС чисСл \( 1,\; 2,\; 3,\; …,\; n \) Π² ΠΎΠΏΡ€Π΅Π΄Π΅Π»Π΅Π½Π½ΠΎΠΌ порядкС Π½Π°Π·Ρ‹Π²Π°ΡŽΡ‚ пСрСстановкой ΠΈΠ· \(n\) чисСл.
Из \(n\) чисСл ΠΌΠΎΠΆΠ½ΠΎ ΠΎΠ±Ρ€Π°Π·ΠΎΠ²Π°Ρ‚ΡŒ \(n!\) Ρ€Π°Π·Π»ΠΈΡ‡Π½Ρ‹Ρ… пСрСстановок.
Π’ ΠΎΠ±Ρ‰Π΅ΠΌ случаС пСрСстановку Π·Π°ΠΏΠΈΡΡ‹Π²Π°ΡŽΡ‚ Π² Π²ΠΈΠ΄Π΅ ΠΌΠ°Ρ‚Ρ€ΠΈΡ†Ρ‹-строки \( \alpha = (\alpha_1,\; \alpha_2,\; …,\; \alpha_n ) \)
ΠŸΠ΅Ρ€Π΅ΡΡ‚Π°Π½ΠΎΠ²ΠΊΡƒ \( (1,\; 2,\; 3,\; …,\; n) \) Π½Π°Π·Ρ‹Π²Π°ΡŽΡ‚ Π½ΠΎΡ€ΠΌΠ°Π»ΡŒΠ½ΠΎΠΉ.

Π”Π²Π° числа \(\alpha_i\) ΠΈ \(\alpha_j\) Π² пСрСстановкС \( \alpha = (\alpha_1,\; …,\; \alpha_n ) \) ΠΎΠ±Ρ€Π°Π·ΡƒΡŽΡ‚ ΠΈΠ½Π²Π΅Ρ€ΡΠΈΡŽ, Ссли \(\alpha_j > \alpha_i \) Π½ΠΎ ΠΏΡ€ΠΈ этом \(\alpha_i\) стоит Π² пСрСстановкС ΠΏΡ€Π°Π²Π΅Π΅ \(\alpha_j\) (Ρ‚.Π΅. \(i>j\) ).
ΠžΠ±Ρ‰Π΅Π΅ количСство инвСрсий Π² пСрСстановкС \(\alpha \) ΠΎΠ±ΠΎΠ·Π½Π°Ρ‡Π°ΡŽΡ‚ \( |\alpha | \), ΠΈ Ссли это число Ρ‡Π΅Ρ‚Π½ΠΎΠ΅, Ρ‚ΠΎ пСрСстановку Π½Π°Π·Ρ‹Π²Π°ΡŽΡ‚ Ρ‡Π΅Ρ‚Π½ΠΎΠΉ, Π° Ссли ΠΎΠ½ΠΎ Π½Π΅Ρ‡Π΅Ρ‚Π½ΠΎΠ΅ β€” Π½Π΅Ρ‡Π΅Ρ‚Π½ΠΎΠΉ.

ВранспозициСй пСрСстановки Π½Π°Π·Ρ‹Π²Π°ΡŽΡ‚ Ρ‚Π°ΠΊΠΎΠ΅ Π΅Ρ‘ ΠΏΡ€Π΅ΠΎΠ±Ρ€Π°Π·ΠΎΠ²Π°Π½ΠΈΠ΅, ΠΏΡ€ΠΈ ΠΊΠΎΡ‚ΠΎΡ€ΠΎΠΌ Π² Π½Π΅ΠΉ ΠΌΠ΅Π½ΡΡŽΡ‚ΡΡ мСстами ΠΊΠ°ΠΊΠΈΠ΅-Π»ΠΈΠ±ΠΎ Π΄Π²Π° элСмСнта, Π° Π΄Ρ€ΡƒΠ³ΠΈΠ΅ ΠΎΡΡ‚Π°ΡŽΡ‚ΡΡ Π½Π° своих мСстах.

Π’Π΅ΠΎΡ€Π΅ΠΌΠ°. Π›ΡŽΠ±Π°Ρ транспозиция мСняСт Ρ‡Π΅Ρ‚Π½ΠΎΡΡ‚ΡŒ пСрСстановки.

Из Π΄Π²ΡƒΡ… пСрСстановок \( ( \alpha_1,\; …,\; \alpha_n ) \) ΠΈ \( ( \beta_1,\; …,\; \beta_n ) \) ΠΎΠ΄Π½ΠΈΡ… ΠΈ Ρ‚Π΅Ρ… ΠΆΠ΅ чисСл ΠΌΠΎΠΆΠ½ΠΎ ΡΠΎΡΡ‚Π°Π²ΠΈΡ‚ΡŒ Π½ΠΎΠ²Ρ‹ΠΉ ΠΎΠ±ΡŠΠ΅ΠΊΡ‚
$$ \sigma = \begin{pmatrix} \beta_1 & \beta_2 & \cdots & \beta_n \\ \alpha_1 & \alpha_2 & \cdots & \alpha_n \end{pmatrix} , \tag{1} $$
ΠΊΠΎΡ‚ΠΎΡ€Ρ‹ΠΉ Π½Π°Π·Ρ‹Π²Π°ΡŽΡ‚ подстановкой n-ΠΉ стСпСни.

ΠŸΠΎΠ΄ΡΡ‚Π°Π½ΠΎΠ²ΠΊΡƒ Π½Π°Π·Ρ‹Π²Π°ΡŽΡ‚ Ρ‡Π΅Ρ‚Π½ΠΎΠΉ, Ссли пСрСстановки, ΠΈΠ· ΠΊΠΎΡ‚ΠΎΡ€Ρ‹Ρ… ΠΎΠ½Π° состоит, ΠΈΠΌΠ΅ΡŽΡ‚ ΠΎΠ΄ΠΈΠ½Π°ΠΊΠΎΠ²ΡƒΡŽ Ρ‡Π΅Ρ‚Π½ΠΎΡΡ‚ΡŒ, ΠΈ Π½Π΅Ρ‡Π΅Ρ‚Π½ΠΎΠΉ Π² ΠΏΡ€ΠΎΡ‚ΠΈΠ²ΠΎΠΏΠΎΠ»ΠΎΠΆΠ½ΠΎΠΌ случаС.
Π§Π΅Ρ‚Π½ΠΎΡΡ‚ΡŒ подстановки \( (1) \) совпадаСт с Ρ‡Π΅Ρ‚Π½ΠΎΡΡ‚ΡŒΡŽ числа \( |\beta|+|\alpha| \) β€” ΠΎΠ±Ρ‰Π΅Π³ΠΎ количСства инвСрсий Π² строках подстановки, ΠΊΠΎΡ‚ΠΎΡ€ΠΎΠ΅ ΠΎΠ±ΠΎΠ·Π½Π°Ρ‡Π°ΡŽΡ‚ \( |\sigma| \).

ВранспозициСй подстановки Π½Π°Π·Ρ‹Π²Π°ΡŽΡ‚ Π»ΡŽΠ±ΡƒΡŽ пСрСстановку Π΅Ρ‘ столбцов. ΠŸΠΎΡΠΊΠΎΠ»ΡŒΠΊΡƒ транспозиция подстановки Π²Ρ‹Π·Ρ‹Π²Π°Π΅Ρ‚ транспозиции ΠΈ Π² ΠΎΠ±Ρ€Π°Π·ΡƒΡŽΡ‰ΠΈΡ… Π΅Ρ‘ пСрСстановках, Ρ‚ΠΎ, согласно ΠΏΡ€Π΅Π΄Ρ‹Π΄ΡƒΡ‰Π΅ΠΉ Ρ‚Π΅ΠΎΡ€Π΅ΠΌΠ΅, ΠΎΡ‡Π΅Π²ΠΈΠ΄Π½ΠΎ, Ρ‡Ρ‚ΠΎ транспозиция подстановки Π½Π΅ мСняСт Π΅Ρ‘ Ρ‡Π΅Ρ‚Π½ΠΎΡΡ‚ΡŒ.

КаТдая подстановка Π²ΠΈΠ΄Π° \( (1) \) Π·Π°Π΄Π°Π΅Ρ‚ Π²Π·Π°ΠΈΠΌΠ½ΠΎ ΠΎΠ΄Π½ΠΎΠ·Π½Π°Ρ‡Π½ΠΎΠ΅ ΠΎΡ‚ΠΎΠ±Ρ€Π°ΠΆΠ΅Π½ΠΈΠ΅ мноТСства чисСл \( 1,\; 2,\; 3,\; …,\; n \) Π½Π° сСбя, ΠΏΡ€ΠΈ ΠΊΠΎΡ‚ΠΎΡ€ΠΎΠΌ \( \beta_1 \) отобраТаСтся Π² \( \alpha_1 \; , \; \beta_2 \) β€” Π² \( \alpha_2\) ΠΈ Ρ‚.Π΄.
Π’ соотвСтствии с ΠΈΠ½Ρ‚Π΅Ρ€ΠΏΡ€Π΅Ρ‚Π°Ρ†ΠΈΠ΅ΠΉ подстановок ΠΊΠ°ΠΊ ΠΎΡ‚ΠΎΠ±Ρ€Π°ΠΆΠ΅Π½ΠΈΠΉ Π΄Π²Π΅ подстановки ΡΡ‡ΠΈΡ‚Π°ΡŽΡ‚ Ρ€Π°Π²Π½Ρ‹ΠΌΠΈ, Ссли ΠΎΠ½ΠΈ ΠΎΡ‚Π»ΠΈΡ‡Π°ΡŽΡ‚ΡΡ Ρ‚ΠΎΠ»ΡŒΠΊΠΎ порядком записи своих столбцов.
НапримСр, подстановки
\( \begin{pmatrix} 1 & 3 & 4 & 2 \\ 2 & 4 & 1 & 3 \\ \end{pmatrix} \;\; ΠΈ \;\; \begin{pmatrix} 1 & 2 & 3 & 4 \\ 2 & 3 & 4 & 1 \\ \end{pmatrix} \)
Ρ€Π°Π²Π½Ρ‹, Ρ‚Π°ΠΊ ΠΊΠ°ΠΊ вторая получаСтся ΠΈΠ· ΠΏΠ΅Ρ€Π²ΠΎΠΉ пСрСстановкой столбцов.

БоглашСниС ΠΎ равСнствС подстановок позволяСт Π·Π°ΠΏΠΈΡΠ°Ρ‚ΡŒ Π»ΡŽΠ±ΡƒΡŽ подстановку Ρ‚Π°ΠΊ, Ρ‡Ρ‚ΠΎΠ±Ρ‹ пСрвая строка являлась Π½ΠΎΡ€ΠΌΠ°Π»ΡŒΠ½ΠΎΠΉ пСрСстановкой. ΠŸΠΎΡΡ‚ΠΎΠΌΡƒ Ρ€Π°Π·Π»ΠΈΡ‡Π½Ρ‹Ρ… подстановок \(n\)-ΠΉ стСпСни имССтся Ρ€ΠΎΠ²Π½ΠΎ \(n!\)

ΠžΠΏΡ€Π΅Π΄Π΅Π»Π΅Π½ΠΈΠ΅. {|\sigma|} a_{1\alpha_1} a_{2\alpha_2} … a_{n\alpha_n} \tag{3} $$ которая бСрСтся ΠΏΠΎ всСвозмоТным подстановкам Π²ΠΈΠ΄Π°
\( \sigma = \begin{pmatrix} 1 & 2 & \cdots & n \\ \alpha_1 & \alpha_2 & \cdots & \alpha_n \end{pmatrix} \)

ΠžΠΏΡ€Π΅Π΄Π΅Π»ΠΈΡ‚Π΅Π»ΡŒ ΠΌΠ°Ρ‚Ρ€ΠΈΡ†Ρ‹ \(A\) часто Π½Π°Π·Ρ‹Π²Π°ΡŽΡ‚ просто ΠΎΠΏΡ€Π΅Π΄Π΅Π»ΠΈΡ‚Π΅Π»Π΅ΠΌ, ΠΈΠ»ΠΈ Π΄Π΅Ρ‚Π΅Ρ€ΠΌΠΈΠ½Π°Π½Ρ‚ΠΎΠΌ, ΠΈ ΠΎΠ±ΠΎΠ·Π½Π°Ρ‡Π°ΡŽΡ‚
\( \begin{vmatrix} a_{11} & a_{12} & \cdots & a_{1n} \\ a_{21} & a_{22} & \cdots & a_{2n} \\ \vdots & \vdots & \ddots & \vdots \\ a_{n1} & a_{n2} & \cdots & a_{nn} \\ \end{vmatrix} \)
ΠΈΠ»ΠΈ \( \det A\), называя \(A\) ΠΌΠ°Ρ‚Ρ€ΠΈΡ†Π΅ΠΉ этого опрСдСлитСля.

Бвойства ΠΎΠΏΡ€Π΅Π΄Π΅Π»ΠΈΡ‚Π΅Π»Π΅ΠΉ

ΠŸΠΎΡΠΊΠΎΠ»ΡŒΠΊΡƒ ΠΎΠΏΡ€Π΅Π΄Π΅Π»ΠΈΡ‚Π΅Π»ΠΈ ΡΠΎΠΎΡ‚Π²Π΅Ρ‚ΡΡ‚Π²ΡƒΡŽΡ‚ ΠΊΠ²Π°Π΄Ρ€Π°Ρ‚Π½Ρ‹ΠΌ ΠΌΠ°Ρ‚Ρ€ΠΈΡ†Π°ΠΌ, Π² ΠΈΡ… Ρ‚Π΅ΠΎΡ€ΠΈΡŽ Π»Π΅Π³ΠΊΠΎ пСрСносится матричная тСрминология (порядок, элСмСнты, строки, столбцы, диагональ, Π΄ΠΈΠ°Π³ΠΎΠ½Π°Π»ΡŒΠ½Ρ‹Π΅ элСмСнты, Π²ΠΈΠ΄Ρ‹ ΠΌΠ°Ρ‚Ρ€ΠΈΡ† ΠΈ ΠΎΠΏΡ€Π΅Π΄Π΅Π»ΠΈΡ‚Π΅Π»Π΅ΠΉ, транспонированиС, элСмСнтарныС прСобразования строк ΠΈ столбцов, Π»ΠΈΠ½Π΅ΠΉΠ½Ρ‹Π΅ ΠΊΠΎΠΌΠ±ΠΈΠ½Π°Ρ†ΠΈΠΈ строк ΠΈ столбцов ΠΈ Π΄Ρ€. T \right| \)

Бвойство 2. ΠŸΡ€ΠΈ пСрСстановкС Π΄Π²ΡƒΡ… строк (столбцов) ΠΎΠΏΡ€Π΅Π΄Π΅Π»ΠΈΡ‚Π΅Π»ΡŒ мСняСт свой Π·Π½Π°ΠΊ Π½Π° ΠΏΡ€ΠΎΡ‚ΠΈΠ²ΠΎΠΏΠΎΠ»ΠΎΠΆΠ½Ρ‹ΠΉ.

Бвойство 3. Если всС элСмСнты \(j\)-Π³ΠΎ столбца опрСдСлитСля прСдставлСны Π² Π²ΠΈΠ΄Π΅ суммы Π΄Π²ΡƒΡ… слагаСмых, Ρ‚ΠΎ ΠΎΠΏΡ€Π΅Π΄Π΅Π»ΠΈΡ‚Π΅Π»ΡŒ Ρ€Π°Π²Π΅Π½ суммС Π΄Π²ΡƒΡ… ΠΎΠΏΡ€Π΅Π΄Π΅Π»ΠΈΡ‚Π΅Π»Π΅ΠΉ, Ρƒ ΠΊΠΎΡ‚ΠΎΡ€Ρ‹Ρ… всС столбцы, ΠΊΡ€ΠΎΠΌΠ΅ \(j\)-Π³ΠΎ, Ρ‚Π°ΠΊΠΈΠ΅ ΠΆΠ΅, ΠΊΠ°ΠΊ ΠΈ Π² Π΄Π°Π½Π½ΠΎΠΌ ΠΎΠΏΡ€Π΅Π΄Π΅Π»ΠΈΡ‚Π΅Π»Π΅, Π° \(j\)-ΠΉ столбСц ΠΏΠ΅Ρ€Π²ΠΎΠ³ΠΎ опрСдСлитСля состоит ΠΈΠ· ΠΏΠ΅Ρ€Π²Ρ‹Ρ… слагаСмых \(j\)-Π³ΠΎ столбца Π΄Π°Π½Π½ΠΎΠ³ΠΎ опрСдСлитСля, Π° Π²Ρ‚ΠΎΡ€ΠΎΠ³ΠΎ β€” ΠΈΠ· Π²Ρ‚ΠΎΡ€Ρ‹Ρ… слагаСмых :
\( \begin{vmatrix} a_{11} & \cdots & \alpha_{1j} + \beta_{1j} & \cdots & a_{1n} \\ a_{21} & \cdots & \alpha_{2j} + \beta_{2j} & \cdots & a_{2n} \\ \cdots & \cdots & \cdots & \cdots & \cdots \\ a_{n1} & \cdots & \alpha_{nj} + \beta_{nj} & \cdots & a_{nn} \end{vmatrix} = \) \( \begin{vmatrix} a_{11} & \cdots & \alpha_{1j} & \cdots & a_{1n} \\ a_{21} & \cdots & \alpha_{2j} & \cdots & a_{2n} \\ \cdots & \cdots & \cdots & \cdots & \cdots \\ a_{n1} & \cdots & \alpha_{nj} & \cdots & a_{nn} \end{vmatrix} + \) \( \begin{vmatrix} a_{11} & \cdots & \beta_{1j} & \cdots & a_{1n} \\ a_{21} & \cdots & \beta_{2j} & \cdots & a_{2n} \\ \cdots & \cdots & \cdots & \cdots & \cdots \\ a_{n1} & \cdots & \beta_{nj} & \cdots & a_{nn} \end{vmatrix} \) Аналогично для строки.

Бвойство 4. ΠžΠ±Ρ‰ΠΈΠΉ ΠΌΠ½ΠΎΠΆΠΈΡ‚Π΅Π»ΡŒ элСмСнтов строки ΠΈΠ»ΠΈ столбца ΠΌΠΎΠΆΠ΅Ρ‚ Π±Ρ‹Ρ‚ΡŒ вынСсСн Π·Π° Π·Π½Π°ΠΊ опрСдСлитСля.
Для умноТСния опрСдСлитСля Π½Π° число достаточно ΡƒΠΌΠ½ΠΎΠΆΠΈΡ‚ΡŒ Π½Π° это число элСмСнты любой строки ΠΈΠ»ΠΈ любого столбца.

Бвойство 5. ΠžΠΏΡ€Π΅Π΄Π΅Π»ΠΈΡ‚Π΅Π»ΡŒ Ρ€Π°Π²Π΅Π½ Π½ΡƒΠ»ΡŽ, Ссли ΠΎΠ½ ΠΈΠΌΠ΅Π΅Ρ‚ :
1) Π½ΡƒΠ»Π΅Π²ΡƒΡŽ строку (столбСц)
2) хотя Π±Ρ‹ Π΄Π²Π΅ ΠΎΠ΄ΠΈΠ½Π°ΠΊΠΎΠ²Ρ‹Π΅ строки (столбца)
3) хотя Π±Ρ‹ Π΄Π²Π΅ строки (столбца), элСмСнты ΠΊΠΎΡ‚ΠΎΡ€Ρ‹Ρ… ΠΏΡ€ΠΎΠΏΠΎΡ€Ρ†ΠΈΠΎΠ½Π°Π»ΡŒΠ½Ρ‹
4) хотя Π±Ρ‹ ΠΎΠ΄Π½Ρƒ строку (столбСц), ΡΠ²Π»ΡΡŽΡ‰ΡƒΡŽΡΡ Π»ΠΈΠ½Π΅ΠΉΠ½ΠΎΠΉ ΠΊΠΎΠΌΠ±ΠΈΠ½Π°Ρ†ΠΈΠ΅ΠΉ Π΄Ρ€ΡƒΠ³ΠΈΡ… строк (столбцов).

Бвойство 6. ΠžΠΏΡ€Π΅Π΄Π΅Π»ΠΈΡ‚Π΅Π»ΡŒ Π½Π΅ измСнится, Ссли ΠΊ любой Π΅Π³ΠΎ строкС (столбцу) ΠΏΡ€ΠΈΠ±Π°Π²ΠΈΡ‚ΡŒ Π΄Ρ€ΡƒΠ³ΡƒΡŽ строку (столбцСц), ΡƒΠΌΠ½ΠΎΠΆΠ΅Π½Π½ΡƒΡŽ Π½Π° число.

Π’ ΠΌΠ°Ρ‚Ρ€ΠΈΡ†Π΅ \(A\) Π²Ρ‹Ρ‡Π΅Ρ€ΠΊΠ½Π΅ΠΌ \(i\)-ю строку ΠΈ \(j\)-ΠΉ столбСц, Π² ΠΊΠΎΡ‚ΠΎΡ€Ρ‹Ρ… стоит элСмСнт \(a_{ij}\). Из ΠΎΡΡ‚Π°Π²ΡˆΠΈΡ…ΡΡ элСмСнтов ΠΌΠΎΠΆΠ½ΠΎ ΡΠΎΡΡ‚Π°Π²ΠΈΡ‚ΡŒ Π½ΠΎΠ²ΡƒΡŽ ΠΊΠ²Π°Π΄Ρ€Π°Ρ‚Π½ΡƒΡŽ ΠΌΠ°Ρ‚Ρ€ΠΈΡ†Ρƒ (n-1)-Π³ΠΎ порядка, сдвинув строки ΠΈ столбцы послС вычСркивания.
ΠžΠΏΡ€Π΅Π΄Π΅Π»ΠΈΡ‚Π΅Π»ΡŒ построСнной ΠΌΠ°Ρ‚Ρ€ΠΈΡ†Ρ‹ ΠΎΠ±ΠΎΠ·Π½Π°Ρ‡Π°ΡŽΡ‚ Ρ‡Π΅Ρ€Π΅Π· \( M_{ij} \) ΠΈ Π½Π°Π·Ρ‹Π²Π°ΡŽΡ‚ ΠΌΠΈΠ½ΠΎΡ€ΠΎΠΌ (ΠΌΠ°Ρ‚Ρ€ΠΈΡ†Ρ‹ \(A\) ΠΈ Π΅Ρ‘ опрСдСлитСля \(\Delta\) ), ΡΠΎΠΎΡ‚Π²Π΅Ρ‚ΡΡ‚Π²ΡƒΡŽΡ‰ΠΈΠΌ элСмСнту \(a_{ij}\). {i+j} a_{ij} M_{ij} \tag{5} $$ ( Ρ€Π°Π·Π»ΠΎΠΆΠ΅Π½ΠΈΠ΅ ΠΏΠΎ \(j\)-ΠΌΡƒ столбцу )

РазлоТСния ΠΏΠΎ строкС (4) ΠΈ столбцу (5) Π΄Π°ΡŽΡ‚ ΠΏΡ€Π°Π²ΠΈΠ»Π°, Π² соотвСтствии с ΠΊΠΎΡ‚ΠΎΡ€Ρ‹ΠΌΠΈ ΠΎΠΏΡ€Π΅Π΄Π΅Π»ΠΈΡ‚Π΅Π»ΡŒ n-Π³ΠΎ порядка сводится ΠΊ n опрСдСлитСлям (n-1)-Π³ΠΎ порядка, раскладывая ΠΊΠΎΡ‚ΠΎΡ€Ρ‹Π΅ ΠΏΠΎΠ»ΡƒΡ‡ΠΈΠΌ n(n-1) ΠΎΠΏΡ€Π΅Π΄Π΅Π»ΠΈΡ‚Π΅Π»Π΅ΠΉ (n-2)-Π³ΠΎ порядка ΠΈ Ρ‚.Π΄.
Π­Ρ‚ΠΈ вычислСния ΠΏΠΎΠ»ΡƒΡ‡Π°ΡŽΡ‚ΡΡ Π³Ρ€ΠΎΠΌΠΎΠ·Π΄ΠΊΠΈΠΌΠΈ, ΠΎΠ΄Π½Π°ΠΊΠΎ процСсс упрощаСтся, Ссли срСди элСмСнтов опрСдСлитСля имССтся ΠΌΠ½ΠΎΠ³ΠΎ Π½ΡƒΠ»Π΅ΠΉ. ЦСлСсообразно Ρ€Π°ΡΠΊΠ»Π°Π΄Ρ‹Π²Π°Ρ‚ΡŒ ΠΎΠΏΡ€Π΅Π΄Π΅Π»ΠΈΡ‚Π΅Π»ΡŒ ΠΏΠΎ Ρ‚ΠΎΠΌΡƒ ряду (строкС, столбцу), Π² ΠΊΠΎΡ‚ΠΎΡ€ΠΎΠΌ большС Π½ΡƒΠ»Π΅ΠΉ.
Если ΠΆΠ΅ Π² этом смыслС Π½Π΅ΠΊΠΎΡ‚ΠΎΡ€Ρ‹Π΅ ряды ΠΎΠ΄ΠΈΠ½Π°ΠΊΠΎΠ²Ρ‹, Ρ‚ΠΎ ΡƒΠ΄ΠΎΠ±Π½Π΅Π΅ Π²Ρ‹Π±ΠΈΡ€Π°Ρ‚ΡŒ Ρ‚ΠΎΡ‚ ΠΈΠ· Π½ΠΈΡ…, Π² ΠΊΠΎΡ‚ΠΎΡ€ΠΎΠΌ элСмСнты ΠΈΠΌΠ΅ΡŽΡ‚ большиС значСния ΠΏΠΎ Π°Π±ΡΠΎΠ»ΡŽΡ‚Π½ΠΎΠΉ Π²Π΅Π»ΠΈΡ‡ΠΈΠ½Π΅, ΠΏΠΎΡΠΊΠΎΠ»ΡŒΠΊΡƒ это ΡƒΠΏΡ€ΠΎΡ‰Π°Π΅Ρ‚ Π²Ρ‹ΠΏΠΎΠ»Π½Π΅Π½ΠΈΠ΅ арифмСтичСских вычислСний.

Бвойство 8. ΠžΠΏΡ€Π΅Π΄Π΅Π»ΠΈΡ‚Π΅Π»ΡŒ Π²Π΅Ρ€Ρ…Π½Π΅ΠΉ (Π½ΠΈΠΆΠ½Π΅ΠΉ) Ρ‚Ρ€Π΅ΡƒΠ³ΠΎΠ»ΡŒΠ½ΠΎΠΉ ΠΌΠ°Ρ‚Ρ€ΠΈΡ†Ρ‹ Ρ€Π°Π²Π΅Π½ ΠΏΡ€ΠΎΠΈΠ·Π²Π΅Π΄Π΅Π½ΠΈΡŽ элСмСнтов Π΅Ρ‘ Π³Π»Π°Π²Π½ΠΎΠΉ Π΄ΠΈΠ°Π³ΠΎΠ½Π°Π»ΠΈ, Ρ‚.Π΅.
$$ \begin{vmatrix} a_{11} & a_{12} & \cdots & a_{1n} \\ 0 & a_{22} & \cdots & a_{2n} \\ \vdots & \vdots & \ddots & \vdots \\ 0 & 0 & \cdots & a_{nn} \\ \end{vmatrix} = $$ $$ \begin{vmatrix} a_{11} & 0 & \cdots & 0 \\ a_{21} & a_{22} & \cdots & 0 \\ \vdots & \vdots & \ddots & \vdots \\ a_{n1} & a_{n2} & \cdots & a_{nn} \\ \end{vmatrix} = $$ $$ a_{11}a_{22}. n a_{ii} $$

Π‘ ΠΏΠΎΠΌΠΎΡ‰ΡŒΡŽ элСмСнтарных ΠΏΡ€Π΅ΠΎΠ±Ρ€Π°Π·ΠΎΠ²Π°Π½ΠΈΠΉ строк любая ΠΌΠ°Ρ‚Ρ€ΠΈΡ†Π° приводится ΠΊ ступСнчатому Π²ΠΈΠ΄Ρƒ. ΠšΠ²Π°Π΄Ρ€Π°Ρ‚Π½Π°Ρ ΠΌΠ°Ρ‚Ρ€ΠΈΡ†Π° ступСнчатого Π²ΠΈΠ΄Π° являСтся частным случаСм Π²Π΅Ρ€Ρ…Π½Π΅ΠΉ Ρ‚Ρ€Π΅ΡƒΠ³ΠΎΠ»ΡŒΠ½ΠΎΠΉ ΠΌΠ°Ρ‚Ρ€ΠΈΡ†Ρ‹, Ρƒ ΠΊΠΎΡ‚ΠΎΡ€ΠΎΠΉ Π΄ΠΈΠ°Π³ΠΎΠ½Π°Π»ΡŒΠ½Ρ‹Π΅ элСмСнты, начиная с Π½Π΅ΠΊΠΎΡ‚ΠΎΡ€ΠΎΠ³ΠΎ, ΠΌΠΎΠ³ΡƒΡ‚ Π±Ρ‹Ρ‚ΡŒ Ρ€Π°Π²Π½Ρ‹ Π½ΡƒΠ»ΡŽ. ΠžΠΏΡ€Π΅Π΄Π΅Π»ΠΈΡ‚Π΅Π»ΡŒ Ρ‚Π°ΠΊΠΎΠΉ ΠΌΠ°Ρ‚Ρ€ΠΈΡ†Ρ‹ Π»Π΅Π³ΠΊΠΎ Π½Π°ΠΉΡ‚ΠΈ ΠΏΠΎ свойству 8. Π’ Π°Π»Π³ΠΎΡ€ΠΈΡ‚ΠΌΠ΅ привСдСния ΠΊ ступСнчатому Π²ΠΈΠ΄Ρƒ ΠΈΡΠΏΠΎΠ»ΡŒΠ·ΡƒΠ΅Ρ‚ΡΡ пСрСстановка строк, ΠΏΡ€ΠΈ ΠΊΠΎΡ‚ΠΎΡ€ΠΎΠΉ ΠΎΠΏΡ€Π΅Π΄Π΅Π»ΠΈΡ‚Π΅Π»ΡŒ ΠΌΠ°Ρ‚Ρ€ΠΈΡ†Ρ‹ мСняСт Π·Π½Π°ΠΊ. ИзмСнСниС Π·Π½Π°ΠΊΠ° ΠΌΠΎΠΆΠ½ΠΎ ΡƒΡ‡Π΅ΡΡ‚ΡŒ, Π½Π°ΠΏΡ€ΠΈΠΌΠ΅Ρ€, Π΄ΠΎΠΏΠΎΠ»Π½ΠΈΡ‚Π΅Π»ΡŒΠ½Ρ‹ΠΌ ΡƒΠΌΠ½ΠΎΠΆΠ΅Π½ΠΈΠ΅ΠΌ опрСдСлитСля ΠΈΠ»ΠΈ ΠΎΠ΄Π½ΠΎΠΉ ΠΈΠ· строк Π½Π° β€”1. Π‘Π»Π΅Π΄ΠΎΠ²Π°Ρ‚Π΅Π»ΡŒΠ½ΠΎ, ΠΊΠ²Π°Π΄Ρ€Π°Ρ‚Π½ΡƒΡŽ ΠΌΠ°Ρ‚Ρ€ΠΈΡ†Ρƒ всСгда ΠΌΠΎΠΆΠ½ΠΎ привСсти элСмСнтарными прСобразованиями строк ΠΊ Π²Π΅Ρ€Ρ…Π½Π΅ΠΌΡƒ Ρ‚Ρ€Π΅ΡƒΠ³ΠΎΠ»ΡŒΠ½ΠΎΠΌΡƒ Π²ΠΈΠ΄Ρƒ с сохранСниСм значСния Π΅Ρ‘ опрСдСлитСля.

Бвойство 9. ΠžΠΏΡ€Π΅Π΄Π΅Π»ΠΈΡ‚Π΅Π»ΡŒ произвСдСния Π΄Π²ΡƒΡ… ΠΊΠ²Π°Π΄Ρ€Π°Ρ‚Π½Ρ‹Ρ… ΠΌΠ°Ρ‚Ρ€ΠΈΡ† A, B Ρ€Π°Π²Π΅Π½ ΠΏΡ€ΠΎΠΈΠ·Π²Π΅Π΄Π΅Π½ΠΈΡŽ ΠΈΡ… ΠΎΠΏΡ€Π΅Π΄Π΅Π»ΠΈΡ‚Π΅Π»Π΅ΠΉ, Ρ‚.Π΅. \( |АВ| = |A||B| \).

Бвойство 10. ΠžΠΏΡ€Π΅Π΄Π΅Π»ΠΈΡ‚Π΅Π»ΡŒ ΠΎΠ±Ρ€Π°Ρ‚Π½ΠΎΠΉ ΠΌΠ°Ρ‚Ρ€ΠΈΡ†Ρ‹: \( \left| А^{-1} \right| = \frac{1}{|A|} \)

Бвойство 11. {1+3} a_{13} M_{13} = $$ $$ a_{11} \begin{vmatrix} a_{22} & a_{23} \\ a_{32} & a_{33} \end{vmatrix} — a_{12} \begin{vmatrix} a_{21} & a_{23} \\ a_{31} & a_{33} \end{vmatrix} + a_{13} \begin{vmatrix} a_{21} & a_{22} \\ a_{31} & a_{32} \end{vmatrix} = $$ $$ a_{11}a_{22}a_{33} — a_{11}a_{23}a_{32} — a_{12}a_{21}a_{33} + a_{12}a_{23}a_{31} + a_{13}a_{21}a_{32} — a_{13}a_{22}a_{31} $$

Π§Ρ‚ΠΎΠ±Ρ‹ Π·Π°ΠΏΠΎΠΌΠ½ΠΈΡ‚ΡŒ, ΠΊΠ°ΠΊΠΈΠ΅ произвСдСния элСмСнтов бСрутся со Π·Π½Π°ΠΊΠΎΠΌ \( «+» \), Π° ΠΊΠ°ΠΊΠΈΠ΅ со Π·Π½Π°ΠΊΠΎΠΌ \( «-» \) ΠΌΠΎΠΆΠ½ΠΎ ΠΈΡΠΏΠΎΠ»ΡŒΠ·ΠΎΠ²Π°Ρ‚ΡŒ ΡΠ»Π΅Π΄ΡƒΡŽΡ‰Π΅Π΅ ΠΏΡ€Π°Π²ΠΈΠ»ΠΎ Ρ‚Ρ€Π΅ΡƒΠ³ΠΎΠ»ΡŒΠ½ΠΈΠΊΠΎΠ².
ΠŸΡ€ΠΎΠΈΠ·Π²Π΅Π΄Π΅Π½ΠΈΡ элСмСнтов, стоящих Π½Π° Π·Π΅Π»Ρ‘Π½Ρ‹Ρ… Ρ‚ΠΎΡ‡ΠΊΠ°Ρ… ΡΠΊΠ»Π°Π΄Ρ‹Π²Π°ΡŽΡ‚ΡΡ, Π° Π½Π° синих — Π²Ρ‹Ρ‡ΠΈΡ‚Π°ΡŽΡ‚ΡΡ :

Π’Π°ΠΊΠΆΠ΅ для вычислСния опрСдСлитСля 3-Π³ΠΎ порядка сущСствуСт ΠΏΡ€Π°Π²ΠΈΠ»ΠΎ Π‘Π°Ρ€Ρ€ΡŽΡΠ°.
ΠŸΠ΅Ρ€Π²Ρ‹ΠΉ ΠΈ Π²Ρ‚ΠΎΡ€ΠΎΠΉ столбцы ΠΌΠ°Ρ‚Ρ€ΠΈΡ†Ρ‹ Π·Π°ΠΏΠΈΡΡ‹Π²Π°ΡŽΡ‚ΡΡ справа ΠΎΡ‚ основной ΠΌΠ°Ρ‚Ρ€ΠΈΡ†Ρ‹.
ΠŸΡ€ΠΎΠΈΠ·Π²Π΅Π΄Π΅Π½ΠΈΡ элСмСнтов, стоящих Π½Π° Π·Π΅Π»Ρ‘Π½Ρ‹Ρ… линиях ΡΠΊΠ»Π°Π΄Ρ‹Π²Π°ΡŽΡ‚ΡΡ, Π° Π½Π° синих — Π²Ρ‹Ρ‡ΠΈΡ‚Π°ΡŽΡ‚ΡΡ :

ΠžΠ±Ρ€Π°Ρ‚Π½Π°Ρ ΠΌΠ°Ρ‚Ρ€ΠΈΡ†Π° ΠΈ Π΅Ρ‘ свойства

ΠžΠΏΡ€Π΅Π΄Π΅Π»Π΅Π½ΠΈΠ΅. {-1}\), ΠΎΠ±Ρ€Π°Ρ‚Π½ΡƒΡŽ ΠΊ \(A\), фактичСски Π½Π°Π΄ΠΎ Ρ€Π΅ΡˆΠΈΡ‚ΡŒ ΠΌΠ°Ρ‚Ρ€ΠΈΡ‡Π½ΠΎΠ΅ ΡƒΡ€Π°Π²Π½Π΅Π½ΠΈΠ΅ \(AX=E\).
ΠžΡ‚ΠΌΠ΅Ρ‚ΠΈΠΌ, Ρ‡Ρ‚ΠΎ Ссли Π½Π°Π΄ ΠΌΠ°Ρ‚Ρ€ΠΈΡ†Π΅ΠΉ \(A\) выполняСтся ΠΊΠ°ΠΊΠΎΠ΅-Π»ΠΈΠ±ΠΎ элСмСнтарноС ΠΏΡ€Π΅ΠΎΠ±Ρ€Π°Π·ΠΎΠ²Π°Π½ΠΈΠ΅ строк, Ρ‚ΠΎ это ΠΆΠ΅ ΠΏΡ€Π΅ΠΎΠ±Ρ€Π°Π·ΠΎΠ²Π°Π½ΠΈΠ΅ осущСствляСтся ΠΈ Π½Π°Π΄ ΠΌΠ°Ρ‚Ρ€ΠΈΡ†Π΅ΠΉ \(AX\), ΠΏΠΎΡΠΊΠΎΠ»ΡŒΠΊΡƒ любоС элСмСнтарноС ΠΏΡ€Π΅ΠΎΠ±Ρ€Π°Π·ΠΎΠ²Π°Π½ΠΈΠ΅ строк ΠΌΠ°Ρ‚Ρ€ΠΈΡ†Ρ‹ эквивалСнтно ΡƒΠΌΠ½ΠΎΠΆΠ΅Π½ΠΈΡŽ Π΅Ρ‘ слСва Π½Π° ΡΠΎΠΎΡ‚Π²Π΅Ρ‚ΡΡ‚Π²ΡƒΡŽΡ‰ΡƒΡŽ ΠΌΠ°Ρ‚Ρ€ΠΈΡ†Ρƒ ΡΠΏΠ΅Ρ†ΠΈΠ°Π»ΡŒΠ½ΠΎΠ³ΠΎ Π²ΠΈΠ΄Π°. Π’Π°ΠΊΠΈΠΌ ΠΎΠ±Ρ€Π°Π·ΠΎΠΌ, Ссли Π² ΡƒΡ€Π°Π²Π½Π΅Π½ΠΈΠΈ \(AX=E\) Π½Π°Π΄ ΠΌΠ°Ρ‚Ρ€ΠΈΡ†Π°ΠΌΠΈ \(A\) ΠΈ \(E\) ΠΎΠ΄Π½ΠΎΠ²Ρ€Π΅ΠΌΠ΅Π½Π½ΠΎ Π²Ρ‹ΠΏΠΎΠ»Π½ΠΈΡ‚ΡŒ ΠΊΠ°ΠΊΠΎΠ΅-Π»ΠΈΠ±ΠΎ элСмСнтарноС ΠΏΡ€Π΅ΠΎΠ±Ρ€Π°Π·ΠΎΠ²Π°Π½ΠΈΠ΅ строк, Ρ‚.Π΅. Π΄ΠΎΠΌΠ½ΠΎΠΆΠΈΡ‚ΡŒ это равСнство слСва Π½Π° Π½Π΅ΠΊΠΎΡ‚ΠΎΡ€ΡƒΡŽ ΠΌΠ°Ρ‚Ρ€ΠΈΡ†Ρƒ ΡΠΏΠ΅Ρ†ΠΈΠ°Π»ΡŒΠ½ΠΎΠ³ΠΎ Π²ΠΈΠ΄Π°, Ρ‚ΠΎ Π² Ρ€Π΅Π·ΡƒΠ»ΡŒΡ‚Π°Ρ‚Π΅ получится Π½ΠΎΠ²ΠΎΠ΅ ΠΌΠ°Ρ‚Ρ€ΠΈΡ‡Π½ΠΎΠ΅ ΡƒΡ€Π°Π²Π½Π΅Π½ΠΈΠ΅ \(A_1X=B_1\). Оба эти ΠΌΠ°Ρ‚Ρ€ΠΈΡ‡Π½Ρ‹Π΅ уравнСния ΠΈΠΌΠ΅ΡŽΡ‚ ΠΎΠ΄Π½ΠΎ ΠΈ Ρ‚ΠΎ ΠΆΠ΅ Ρ€Π΅ΡˆΠ΅Π½ΠΈΠ΅, Ρ‚Π°ΠΊ ΠΊΠ°ΠΊ любоС элСмСнтарноС ΠΏΡ€Π΅ΠΎΠ±Ρ€Π°Π·ΠΎΠ²Π°Π½ΠΈΠ΅ строк ΠΈΠΌΠ΅Π΅Ρ‚ ΠΎΠ±Ρ€Π°Ρ‚Π½ΠΎΠ΅ элСмСнтарноС ΠΏΡ€Π΅ΠΎΠ±Ρ€Π°Π·ΠΎΠ²Π°Π½ΠΈΠ΅ строк.
ΠŸΠΎΡΠ»Π΅Π΄ΠΎΠ²Π°Ρ‚Π΅Π»ΡŒΠ½ΠΎΡΡ‚ΡŒ элСмСнтарных ΠΏΡ€Π΅ΠΎΠ±Ρ€Π°Π·ΠΎΠ²Π°Π½ΠΈΠΉ строк Π½Π°Π΄ΠΎ ΠΏΠΎΠ΄ΠΎΠ±Ρ€Π°Ρ‚ΡŒ Ρ‚Π°ΠΊ, Ρ‡Ρ‚ΠΎΠ±Ρ‹ Π½Π° s-ΠΌ шагС ΠΌΠ°Ρ‚Ρ€ΠΈΡ†Π° \(A\) ΠΏΡ€Π΅Π²Ρ€Π°Ρ‚ΠΈΠ»Π°ΡΡŒ Π² Π΅Π΄ΠΈΠ½ΠΈΡ‡Π½ΡƒΡŽ ΠΌΠ°Ρ‚Ρ€ΠΈΡ†Ρƒ.
Π’ Ρ€Π΅Π·ΡƒΠ»ΡŒΡ‚Π°Ρ‚Π΅ этих s шагов получаСтся ΡƒΡ€Π°Π²Π½Π΅Π½ΠΈΠ΅ \(A_sX=B_s\), Π³Π΄Π΅ \(A_s=E\), Ρ‚. {-1}\).

Π Π°Π½Π³ ΠΌΠ°Ρ‚Ρ€ΠΈΡ†Ρ‹

ΠžΠΏΡ€Π΅Π΄Π΅Π»Π΅Π½ΠΈΠ΅.
Π Π°Π½Π³ΠΎΠΌ ΠΌΠ°Ρ‚Ρ€ΠΈΡ†Ρ‹ Π½Π°Π·Ρ‹Π²Π°ΡŽΡ‚ число, ΠΊΠΎΡ‚ΠΎΡ€ΠΎΠ΅ Ρ€Π°Π²Π½ΠΎ ΠΌΠ°ΠΊΡΠΈΠΌΠ°Π»ΡŒΠ½ΠΎΠΌΡƒ порядку срСди Π΅Ρ‘ Π½Π΅Π½ΡƒΠ»Π΅Π²Ρ‹Ρ… ΠΌΠΈΠ½ΠΎΡ€ΠΎΠ².

Для Ρ€Π°Π½Π³Π° ΠΌΠ°Ρ‚Ρ€ΠΈΡ†Ρ‹ \(A\) ΠΈΡΠΏΠΎΠ»ΡŒΠ·ΡƒΡŽΡ‚ ΠΎΠ±ΠΎΠ·Π½Π°Ρ‡Π΅Π½ΠΈΠ΅ \(\text{rang}A\).

Если квадратная ΠΌΠ°Ρ‚Ρ€ΠΈΡ†Π° порядка n Π½Π΅Π²Ρ‹Ρ€ΠΎΠΆΠ΄Π΅Π½Π°, Ρ‚ΠΎ Π΅Ρ‘ Ρ€Π°Π½Π³ Ρ€Π°Π²Π΅Π½ Π΅Ρ‘ порядку n : Π½Π΅Π½ΡƒΠ»Π΅Π²Ρ‹ΠΌ являСтся СдинствСнный ΠΌΠΈΠ½ΠΎΡ€ максимального порядка n, ΡΠΎΠ²ΠΏΠ°Π΄Π°ΡŽΡ‰ΠΈΠΉ с ΠΎΠΏΡ€Π΅Π΄Π΅Π»ΠΈΡ‚Π΅Π»Π΅ΠΌ ΠΌΠ°Ρ‚Ρ€ΠΈΡ†Ρ‹.
Π’ частности, Ρ€Π°Π½Π³ Π΅Π΄ΠΈΠ½ΠΈΡ‡Π½ΠΎΠΉ ΠΌΠ°Ρ‚Ρ€ΠΈΡ†Ρ‹ \(E\) порядка n Ρ€Π°Π²Π΅Π½ n.

Если квадратная ΠΌΠ°Ρ‚Ρ€ΠΈΡ†Π° Π²Ρ‹Ρ€ΠΎΠΆΠ΄Π΅Π½Π°, Ρ‚ΠΎ Π΅Ρ‘ Ρ€Π°Π½Π³ мСньшС Π΅Ρ‘ порядка : СдинствСнный ΠΌΠΈΠ½ΠΎΡ€ максимального порядка, Ρ€Π°Π²Π½ΠΎΠ³ΠΎ порядку ΠΌΠ°Ρ‚Ρ€ΠΈΡ†Ρ‹, являСтся Π½ΡƒΠ»Π΅Π²Ρ‹ΠΌ, ΠΈ Π² этом случаС Π½Π΅Π½ΡƒΠ»Π΅Π²Ρ‹Π΅ ΠΌΠΈΠ½ΠΎΡ€Ρ‹ ΠΈΠΌΠ΅ΡŽΡ‚ мСньший порядок.
Π Π°Π½Π³ Π½ΡƒΠ»Π΅Π²ΠΎΠΉ ΠΌΠ°Ρ‚Ρ€ΠΈΡ†Ρ‹ ΠΏΠΎΠ»Π°Π³Π°ΡŽΡ‚ Ρ€Π°Π²Π½Ρ‹ΠΌ Π½ΡƒΠ»ΡŽ.

Π Π°Π½Π³ диагональной ΠΌΠ°Ρ‚Ρ€ΠΈΡ†Ρ‹ Ρ€Π°Π²Π΅Π½ количСству Π΅Ρ‘ Π½Π΅Π½ΡƒΠ»Π΅Π²Ρ‹Ρ… Π΄ΠΈΠ°Π³ΠΎΠ½Π°Π»ΡŒΠ½Ρ‹Ρ… элСмСнтов.

НСпосрСдствСнно ΠΈΠ· опрСдСлСния Ρ€Π°Π½Π³Π° ΠΌΠ°Ρ‚Ρ€ΠΈΡ†Ρ‹ слСдуСт, Ρ‡Ρ‚ΠΎ Ρ€Π°Π½Π³ ΠΈΠΌΠ΅Π΅Ρ‚ ΡΠ»Π΅Π΄ΡƒΡŽΡ‰Π΅Π΅ свойство, ΠΏΠΎΠ»Π½ΠΎΡΡ‚ΡŒΡŽ Π΅Π³ΠΎ Ρ…Π°Ρ€Π°ΠΊΡ‚Π΅Ρ€ΠΈΠ·ΡƒΡŽΡ‰Π΅Π΅. T = \text{rang} A \)

Π’Π΅ΠΎΡ€Π΅ΠΌΠ°. Π Π°Π½Π³ ΠΌΠ°Ρ‚Ρ€ΠΈΡ†Ρ‹ Π½Π΅ мСняСтся ΠΏΡ€ΠΈ элСмСнтарных прСобразованиях Π΅Ρ‘ строк ΠΈ столбцов.

Π’Π΅ΠΎΡ€Π΅ΠΌΠ° ΠΎ базисном ΠΌΠΈΠ½ΠΎΡ€Π΅

Π‘Ρ€Π΅Π΄ΠΈ ΠΌΠΈΠ½ΠΎΡ€ΠΎΠ² ΠΌΠ°Ρ‚Ρ€ΠΈΡ†Ρ‹ ΠΌΠΎΠ³ΡƒΡ‚ Π±Ρ‹Ρ‚ΡŒ ΠΊΠ°ΠΊ Ρ€Π°Π²Π½Ρ‹Π΅ Π½ΡƒΠ»ΡŽ, Ρ‚Π°ΠΊ ΠΈ ΠΎΡ‚Π»ΠΈΡ‡Π½Ρ‹Π΅ ΠΎΡ‚ нуля.

ΠžΠΏΡ€Π΅Π΄Π΅Π»Π΅Π½ΠΈΠ΅.
ΠœΠΈΠ½ΠΎΡ€ \(M\) ΠΌΠ°Ρ‚Ρ€ΠΈΡ†Ρ‹ \(M\) Π½Π°Π·Ρ‹Π²Π°ΡŽΡ‚ базисным, Ссли Π²Ρ‹ΠΏΠΎΠ»Π½Π΅Π½Ρ‹ Π΄Π²Π° условия:
1) ΠΎΠ½ Π½Π΅ Ρ€Π°Π²Π΅Π½ Π½ΡƒΠ»ΡŽ
2) Π΅Π³ΠΎ порядок Ρ€Π°Π²Π΅Π½ Ρ€Π°Π½Π³Ρƒ ΠΌΠ°Ρ‚Ρ€ΠΈΡ†Ρ‹ А

ΠœΠ°Ρ‚Ρ€ΠΈΡ†Π° \(A\) ΠΌΠΎΠΆΠ΅Ρ‚ ΠΈΠΌΠ΅Ρ‚ΡŒ нСсколько базисных ΠΌΠΈΠ½ΠΎΡ€ΠΎΠ². Π‘Ρ‚Ρ€ΠΎΠΊΠΈ ΠΈ столбцы ΠΌΠ°Ρ‚Ρ€ΠΈΡ†Ρ‹ \(A\), Π² ΠΊΠΎΡ‚ΠΎΡ€Ρ‹Ρ… располоТСн Π²Ρ‹Π±Ρ€Π°Π½Π½Ρ‹ΠΉ базисный ΠΌΠΈΠ½ΠΎΡ€, Π½Π°Π·Ρ‹Π²Π°ΡŽΡ‚ базисными.

Π’Π΅ΠΎΡ€Π΅ΠΌΠ° ΠΎ базисном ΠΌΠΈΠ½ΠΎΡ€Π΅. БазисныС строки (столбцы) ΠΌΠ°Ρ‚Ρ€ΠΈΡ†Ρ‹ \(A\), ΡΠΎΠΎΡ‚Π²Π΅Ρ‚ΡΡ‚Π²ΡƒΡŽΡ‰ΠΈΠ΅ Π»ΡŽΠ±ΠΎΠΌΡƒ Π΅Ρ‘ базисному ΠΌΠΈΠ½ΠΎΡ€Ρƒ \(M\), Π»ΠΈΠ½Π΅ΠΉΠ½ΠΎ нСзависимы. Π›ΡŽΠ±Ρ‹Π΅ строки (столбцы) ΠΌΠ°Ρ‚Ρ€ΠΈΡ†Ρ‹ \(A\), Π½Π΅ входящиС Π² \(M\), ΡΠ²Π»ΡΡŽΡ‚ΡΡ Π»ΠΈΠ½Π΅ΠΉΠ½Ρ‹ΠΌΠΈ комбинациями базисных строк (столбцов).

БлСдствиС. Для Ρ‚ΠΎΠ³ΠΎ Ρ‡Ρ‚ΠΎΠ±Ρ‹ квадратная ΠΌΠ°Ρ‚Ρ€ΠΈΡ†Π° Π±Ρ‹Π»Π° Π½Π΅Π²Ρ‹Ρ€ΠΎΠΆΠ΄Π΅Π½Π½ΠΎΠΉ, Π½Π΅ΠΎΠ±Ρ…ΠΎΠ΄ΠΈΠΌΠΎ ΠΈ достаточно, Ρ‡Ρ‚ΠΎΠ±Ρ‹ Π΅Ρ‘ строки (столбцы) Π±Ρ‹Π»ΠΈ Π»ΠΈΠ½Π΅ΠΉΠ½ΠΎ нСзависимы.

Π’Π΅ΠΎΡ€Π΅ΠΌΠ°. Π›ΠΈΠ½Π΅ΠΉΠ½ΠΎ нСзависимыС строки (столбцы) ΠΌΠ°Ρ‚Ρ€ΠΈΡ†Ρ‹, количСство ΠΊΠΎΡ‚ΠΎΡ€Ρ‹Ρ… Ρ€Π°Π²Π½ΠΎ Ρ€Π°Π½Π³Ρƒ ΠΌΠ°Ρ‚Ρ€ΠΈΡ†Ρ‹, ΡΠ²Π»ΡΡŽΡ‚ΡΡ базисными строками (столбцами).

Π’Π΅ΠΎΡ€Π΅ΠΌΠ°. Для любой ΠΌΠ°Ρ‚Ρ€ΠΈΡ†Ρ‹ Π΅Ρ‘ Ρ€Π°Π½Π³ Ρ€Π°Π²Π΅Π½ ΠΌΠ°ΠΊΡΠΈΠΌΠ°Π»ΡŒΠ½ΠΎΠΌΡƒ количСству Π΅Ρ‘ Π»ΠΈΠ½Π΅ΠΉΠ½ΠΎ нСзависимых строк (столбцов).

БлСдствиС. Для любой ΠΌΠ°Ρ‚Ρ€ΠΈΡ†Ρ‹ максимальноС число Π»ΠΈΠ½Π΅ΠΉΠ½ΠΎ нСзависимых строк Ρ€Π°Π²Π½ΠΎ ΠΌΠ°ΠΊΡΠΈΠΌΠ°Π»ΡŒΠ½ΠΎΠΌΡƒ числу Π»ΠΈΠ½Π΅ΠΉΠ½ΠΎ нСзависимых столбцов.

ВычислСниС Ρ€Π°Π½Π³Π° ΠΌΠ°Ρ‚Ρ€ΠΈΡ†Ρ‹

ΠœΠ΅Ρ‚ΠΎΠ΄ ΠΎΠΊΠ°ΠΉΠΌΠ»ΡΡŽΡ‰ΠΈΡ… ΠΌΠΈΠ½ΠΎΡ€ΠΎΠ²

ΠœΠΈΠ½ΠΎΡ€ \(M’\) ΠΌΠ°Ρ‚Ρ€ΠΈΡ†Ρ‹ \(A\) Π½Π°Π·Ρ‹Π²Π°ΡŽΡ‚ ΠΎΠΊΠ°ΠΉΠΌΠ»ΡΡŽΡ‰ΠΈΠΌ для ΠΌΠΈΠ½ΠΎΡ€Π° \(M\), Ссли ΠΎΠ½ получаСтся ΠΈΠ· послСднСго Π΄ΠΎΠ±Π°Π²Π»Π΅Π½ΠΈΠ΅ΠΌ ΠΎΠ΄Π½ΠΎΠΉ Π½ΠΎΠ²ΠΎΠΉ строки ΠΈ ΠΎΠ΄Π½ΠΎΠ³ΠΎ Π½ΠΎΠ²ΠΎΠ³ΠΎ столбца ΠΌΠ°Ρ‚Ρ€ΠΈΡ†Ρ‹ \(A\).
Ясно, Ρ‡Ρ‚ΠΎ порядок ΠΎΠΊΠ°ΠΉΠΌΠ»ΡΡŽΡ‰Π΅Π³ΠΎ ΠΌΠΈΠ½ΠΎΡ€Π° \(M’\) Π½Π° Π΅Π΄ΠΈΠ½ΠΈΡ†Ρƒ большС, Ρ‡Π΅ΠΌ порядок ΠΌΠΈΠ½ΠΎΡ€Π° \(M\).

ΠœΠ΅Ρ‚ΠΎΠ΄ ΠΎΠΊΠ°ΠΉΠΌΠ»ΡΡŽΡ‰ΠΈΡ… ΠΌΠΈΠ½ΠΎΡ€ΠΎΠ² позволяСт Π½Π°ΠΉΡ‚ΠΈ ΠΎΠ΄ΠΈΠ½ ΠΈΠ· базисных ΠΌΠΈΠ½ΠΎΡ€ΠΎΠ² ΠΌΠ°Ρ‚Ρ€ΠΈΡ†Ρ‹ ΠΈ состоит Π² ΡΠ»Π΅Π΄ΡƒΡŽΡ‰Π΅ΠΌ.
ВыбираСтся Π½Π΅Π½ΡƒΠ»Π΅Π²ΠΎΠΉ ΠΌΠΈΠ½ΠΎΡ€ ΠΏΠ΅Ρ€Π²ΠΎΠ³ΠΎ порядка (Π½Π΅Π½ΡƒΠ»Π΅Π²ΠΎΠΉ элСмСнт ΠΌΠ°Ρ‚Ρ€ΠΈΡ†Ρ‹). К ΠΎΡ‡Π΅Ρ€Π΅Π΄Π½ΠΎΠΌΡƒ Π½Π΅Π½ΡƒΠ»Π΅Π²ΠΎΠΌΡƒ ΠΌΠΈΠ½ΠΎΡ€Ρƒ ΠΏΠΎΡΠ»Π΅Π΄ΠΎΠ²Π°Ρ‚Π΅Π»ΡŒΠ½ΠΎ Π΄ΠΎΠ±Π°Π²Π»ΡΡŽΡ‚ΡΡ Ρ‚Π°ΠΊΠΈΠ΅ строка ΠΈ столбСц, Ρ‡Ρ‚ΠΎΠ±Ρ‹ Π½ΠΎΠ²Ρ‹ΠΉ ΠΎΠΊΠ°ΠΉΠΌΠ»ΡΡŽΡ‰ΠΈΠΉ ΠΌΠΈΠ½ΠΎΡ€ оказался Π½Π΅Π½ΡƒΠ»Π΅Π²Ρ‹ΠΌ. Если этого ΡΠ΄Π΅Π»Π°Ρ‚ΡŒ нСльзя, Ρ‚ΠΎ послСдний Π½Π΅Π½ΡƒΠ»Π΅Π²ΠΎΠΉ ΠΌΠΈΠ½ΠΎΡ€ являСтся базисным (Ρ‡Ρ‚ΠΎ ΡƒΡ‚Π²Π΅Ρ€ΠΆΠ΄Π°Π΅Ρ‚ ΡΠ»Π΅Π΄ΡƒΡŽΡ‰Π°Ρ Π½ΠΈΠΆΠ΅ Ρ‚Π΅ΠΎΡ€Π΅ΠΌΠ°). Π­Ρ‚ΠΎΡ‚ процСсс Ρ€Π°Π½ΠΎ ΠΈΠ»ΠΈ ΠΏΠΎΠ·Π΄Π½ΠΎ закончится ΠΈΠ·-Π·Π° ΠΎΠ³Ρ€Π°Π½ΠΈΡ‡Π΅Π½Π½Ρ‹Ρ… Ρ€Π°Π·ΠΌΠ΅Ρ€ΠΎΠ² ΠΌΠ°Ρ‚Ρ€ΠΈΡ†Ρ‹.

Π’Π΅ΠΎΡ€Π΅ΠΌΠ°. Если для Π½Π΅ΠΊΠΎΡ‚ΠΎΡ€ΠΎΠ³ΠΎ ΠΌΠΈΠ½ΠΎΡ€Π° ΠΌΠ°Ρ‚Ρ€ΠΈΡ†Ρ‹ всС ΠΎΠΊΠ°ΠΉΠΌΠ»ΡΡŽΡ‰ΠΈΠ΅ Π΅Π³ΠΎ ΠΌΠΈΠ½ΠΎΡ€Ρ‹ Ρ€Π°Π²Π½Ρ‹ Π½ΡƒΠ»ΡŽ, Ρ‚ΠΎ ΠΎΠ½ являСтся базисным.

ΠœΠ΅Ρ‚ΠΎΠ΄ элСмСнтарных ΠΏΡ€Π΅ΠΎΠ±Ρ€Π°Π·ΠΎΠ²Π°Π½ΠΈΠΉ

ΠŸΡ€ΠΈ элСмСнтарных прСобразованиях строк (столбцов) ΠΌΠ°Ρ‚Ρ€ΠΈΡ†Ρ‹ Π΅Ρ‘ Ρ€Π°Π½Π³ Π½Π΅ мСняСтся. Π‘ ΠΏΠΎΠΌΠΎΡ‰ΡŒΡŽ этих ΠΏΡ€Π΅ΠΎΠ±Ρ€Π°Π·ΠΎΠ²Π°Π½ΠΈΠΉ ΠΌΠΎΠΆΠ½ΠΎ Ρ‚Π°ΠΊ ΡƒΠΏΡ€ΠΎΡΡ‚ΠΈΡ‚ΡŒ ΠΌΠ°Ρ‚Ρ€ΠΈΡ†Ρƒ, Ρ‡Ρ‚ΠΎΠ±Ρ‹ Ρ€Π°Π½Π³ Π½ΠΎΠ²ΠΎΠΉ ΠΌΠ°Ρ‚Ρ€ΠΈΡ†Ρ‹ Π»Π΅Π³ΠΊΠΎ вычислялся.

НапримСр с ΠΏΠΎΠΌΠΎΡ‰ΡŒΡŽ элСмСнтарных ΠΏΡ€Π΅ΠΎΠ±Ρ€Π°Π·ΠΎΠ²Π°Π½ΠΈΠΉ строк Π»ΡŽΠ±ΡƒΡŽ ΠΌΠ°Ρ‚Ρ€ΠΈΡ†Ρƒ ΠΌΠΎΠΆΠ½ΠΎ привСсти ΠΊ ступСнчатому Π²ΠΈΠ΄Ρƒ. Π Π°Π½Π³ ΠΆΠ΅ ступСнчатой ΠΌΠ°Ρ‚Ρ€ΠΈΡ†Ρ‹ Ρ€Π°Π²Π΅Π½ количСству Π½Π΅Π½ΡƒΠ»Π΅Π²Ρ‹Ρ… строк. Базисным Π² Π½Π΅ΠΉ являСтся ΠΌΠΈΠ½ΠΎΡ€, располоТСнный Π½Π° пСрСсСчСнии Π½Π΅Π½ΡƒΠ»Π΅Π²Ρ‹Ρ… строк со столбцами, ΡΠΎΠΎΡ‚Π²Π΅Ρ‚ΡΡ‚Π²ΡƒΡŽΡ‰ΠΈΠΌΠΈ ΠΏΠ΅Ρ€Π²Ρ‹ΠΌ слСва Π½Π΅Π½ΡƒΠ»Π΅Π²Ρ‹ΠΌ элСмСнтам Π² ΠΊΠ°ΠΆΠ΄ΠΎΠΉ ΠΈΠ· строк. Π”Π΅ΠΉΡΡ‚Π²ΠΈΡ‚Π΅Π»ΡŒΠ½ΠΎ, этот ΠΌΠΈΠ½ΠΎΡ€ Π½Π΅Π½ΡƒΠ»Π΅Π²ΠΎΠΉ, Ρ‚Π°ΠΊ ΠΊΠ°ΠΊ ΡΠΎΠΎΡ‚Π²Π΅Ρ‚ΡΡ‚Π²ΡƒΡŽΡ‰Π°Ρ ΠΌΠ°Ρ‚Ρ€ΠΈΡ†Π° являСтся Π²Π΅Ρ€Ρ…Π½Π΅ΠΉ Ρ‚Ρ€Π΅ΡƒΠ³ΠΎΠ»ΡŒΠ½ΠΎΠΉ, Π° любоС Π΅Π³ΠΎ ΠΎΠΊΠ°ΠΉΠΌΠ»Π΅Π½ΠΈΠ΅ содСрТит Π½ΡƒΠ»Π΅Π²ΡƒΡŽ строку. ΠŸΠΎΡΡ‚ΠΎΠΌΡƒ ΠΏΡ€ΠΈΠ²Π΅Π΄Π΅Π½ΠΈΠ΅ ΠΌΠ°Ρ‚Ρ€ΠΈΡ†Ρ‹ ΠΊ ступСнчатому Π²ΠΈΠ΄Ρƒ с ΠΏΠΎΠΌΠΎΡ‰ΡŒΡŽ элСмСнтарных ΠΏΡ€Π΅ΠΎΠ±Ρ€Π°Π·ΠΎΠ²Π°Π½ΠΈΠΉ строк позволяСт Π²Ρ‹Ρ‡ΠΈΡΠ»ΠΈΡ‚ΡŒ Ρ€Π°Π½Π³ ΠΌΠ°Ρ‚Ρ€ΠΈΡ†Ρ‹.

ΠŸΡ€ΠΈΠ²Π΅Π΄Π΅Π½Π½Ρ‹Π΅ Π΄Π²Π° ΠΌΠ΅Ρ‚ΠΎΠ΄Π° сущСствСнно ΠΎΡ‚Π»ΠΈΡ‡Π°ΡŽΡ‚ΡΡ Π΄Ρ€ΡƒΠ³ ΠΎΡ‚ Π΄Ρ€ΡƒΠ³Π°.
ΠŸΡ€ΠΈ Π½Π°Ρ…ΠΎΠΆΠ΄Π΅Π½ΠΈΠΈ Ρ€Π°Π½Π³Π° ΠΊΠΎΠ½ΠΊΡ€Π΅Ρ‚Π½ΠΎΠΉ ΠΌΠ°Ρ‚Ρ€ΠΈΡ†Ρ‹ ΠΌΠ΅Ρ‚ΠΎΠ΄ΠΎΠΌ ΠΎΠΊΠ°ΠΉΠΌΠ»ΡΡŽΡ‰ΠΈΡ… ΠΌΠΈΠ½ΠΎΡ€ΠΎΠ² ΠΌΠΎΠΆΠ΅Ρ‚ ΠΏΠΎΡ‚Ρ€Π΅Π±ΠΎΠ²Π°Ρ‚ΡŒΡΡ большоС количСство вычислСний. Π­Ρ‚ΠΎ связано с Ρ‚Π΅ΠΌ, Ρ‡Ρ‚ΠΎ ΠΌΠ΅Ρ‚ΠΎΠ΄ Ρ‚Ρ€Π΅Π±ΡƒΠ΅Ρ‚ вычислСния ΠΎΠΏΡ€Π΅Π΄Π΅Π»ΠΈΡ‚Π΅Π»Π΅ΠΉ, порядок ΠΊΠΎΡ‚ΠΎΡ€Ρ‹Ρ… ΠΌΠΎΠΆΠ΅Ρ‚ возрасти Π΄ΠΎ минимального ΠΈΠ· Ρ€Π°Π·ΠΌΠ΅Ρ€ΠΎΠ² ΠΌΠ°Ρ‚Ρ€ΠΈΡ†Ρ‹. Однако Π² Ρ€Π΅Π·ΡƒΠ»ΡŒΡ‚Π°Ρ‚Π΅ Π±ΡƒΠ΄Π΅Ρ‚ Π½Π°ΠΉΠ΄Π΅Π½ Π½Π΅ Ρ‚ΠΎΠ»ΡŒΠΊΠΎ Ρ€Π°Π½Π³ ΠΌΠ°Ρ‚Ρ€ΠΈΡ†Ρ‹, Π½ΠΎ ΠΈ ΠΎΠ΄ΠΈΠ½ ΠΈΠ· Π΅Ρ‘ базисных ΠΌΠΈΠ½ΠΎΡ€ΠΎΠ².

ΠŸΡ€ΠΈ Π½Π°Ρ…ΠΎΠΆΠ΄Π΅Π½ΠΈΠΈ Ρ€Π°Π½Π³Π° ΠΌΠ°Ρ‚Ρ€ΠΈΡ†Ρ‹ ΠΌΠ΅Ρ‚ΠΎΠ΄ΠΎΠΌ элСмСнтарных ΠΏΡ€Π΅ΠΎΠ±Ρ€Π°Π·ΠΎΠ²Π°Π½ΠΈΠΉ трСбуСтся Π³ΠΎΡ€Π°Π·Π΄ΠΎ мСньшС вычислСний. ΠŸΡ€ΠΈΡ‡Π΅ΠΌ Ρ€Π°Π·Π½ΠΈΡ†Π° Π² ΠΎΠ±ΡŠΠ΅ΠΌΠ°Ρ… вычислСний возрастаСт с ростом Ρ€Π°Π·ΠΌΠ΅Ρ€ΠΎΠ² ΠΌΠ°Ρ‚Ρ€ΠΈΡ†Ρ‹ ΠΈ услоТнСниСм Π΅Ρ‘ Π²ΠΈΠ΄Π°. Но этот ΠΌΠ΅Ρ‚ΠΎΠ΄ позволяСт Π½Π°ΠΉΡ‚ΠΈ базисный ΠΌΠΈΠ½ΠΎΡ€ лишь для ΠΌΠ°Ρ‚Ρ€ΠΈΡ†Ρ‹ ступСнчатого Π²ΠΈΠ΄Π°, ΠΏΠΎΠ»ΡƒΡ‡Π΅Π½Π½ΠΎΠΉ Π² Ρ€Π΅Π·ΡƒΠ»ΡŒΡ‚Π°Ρ‚Π΅ элСмСнтарных ΠΏΡ€Π΅ΠΎΠ±Ρ€Π°Π·ΠΎΠ²Π°Π½ΠΈΠΉ. Π§Ρ‚ΠΎΠ±Ρ‹ Π½Π°ΠΉΡ‚ΠΈ базисный ΠΌΠΈΠ½ΠΎΡ€ исходной ΠΌΠ°Ρ‚Ρ€ΠΈΡ†Ρ‹, Π½ΡƒΠΆΠ½Ρ‹ Π΄ΠΎΠΏΠΎΠ»Π½ΠΈΡ‚Π΅Π»ΡŒΠ½Ρ‹Π΅ вычислСния с ΡƒΡ‡Π΅Ρ‚ΠΎΠΌ ΡƒΠΆΠ΅ извСстного Ρ€Π°Π½Π³Π° ΠΌΠ°Ρ‚Ρ€ΠΈΡ†Ρ‹.

ЛинСйная Π°Π»Π³Π΅Π±Ρ€Π° Π½Π° Python. [Π£Ρ€ΠΎΠΊ 3]. ДСйствия Π½Π°Π΄ ΠΌΠ°Ρ‚Ρ€ΠΈΡ†Π°ΠΌΠΈ

Π’Π΅ΠΌΠ° Ρ‚Ρ€Π΅Ρ‚ΡŒΠ΅Π³ΠΎ ΡƒΡ€ΠΎΠΊΠ°: дСйствия Π½Π°Π΄ ΠΌΠ°Ρ‚Ρ€ΠΈΡ†Π°ΠΌΠΈ. Π’ Ρ€Π°ΠΌΠΊΠ°Ρ… Π½Π΅Π΅ Π±ΡƒΠ΄ΡƒΡ‚ рассмотрСны ΡΠ»Π΅Π΄ΡƒΡŽΡ‰ΠΈΠ΅ вопросы: ΡƒΠΌΠ½ΠΎΠΆΠ΅Π½ΠΈΠ΅ ΠΌΠ°Ρ‚Ρ€ΠΈΡ†Ρ‹ Π½Π° число, слоТСниС ΠΈ ΡƒΠΌΠ½ΠΎΠΆΠ΅Π½ΠΈΠ΅ ΠΌΠ°Ρ‚Ρ€ΠΈΡ†.

ДСйствия Π½Π°Π΄ ΠΌΠ°Ρ‚Ρ€ΠΈΡ†Π°ΠΌΠΈ

Π£ΠΌΠ½ΠΎΠΆΠ΅Π½ΠΈΠ΅ ΠΌΠ°Ρ‚Ρ€ΠΈΡ†Ρ‹ Π½Π° число

ΠŸΡ€ΠΈ ΡƒΠΌΠ½ΠΎΠΆΠ΅Π½ΠΈΠΈ ΠΌΠ°Ρ‚Ρ€ΠΈΡ†Ρ‹ Π½Π° число, всС элСмСнты ΠΌΠ°Ρ‚Ρ€ΠΈΡ†Ρ‹ ΡƒΠΌΠ½ΠΎΠΆΠ°ΡŽΡ‚ΡΡ Π½Π° это число:

➣ ЧислСнный ΠΏΡ€ΠΈΠΌΠ΅Ρ€

➀ ΠŸΡ€ΠΈΠΌΠ΅Ρ€ Π½Π° Python

>>> A = np.matrix('1 2 3; 4 5 6')
>>> C = 3 * A
>>> print(C)
[[ 3 Β 6 9]
[12 15 18]]

Β 

Рассмотрим свойства ΠΎΠΏΠ΅Ρ€Π°Ρ†ΠΈΠΈ умноТСния ΠΌΠ°Ρ‚Ρ€ΠΈΡ†Ρ‹ Π½Π° число.

Бвойство 1. ΠŸΡ€ΠΎΠΈΠ·Π²Π΅Π΄Π΅Π½ΠΈΠ΅ Π΅Π΄ΠΈΠ½ΠΈΡ†Ρ‹ ΠΈ любой Π·Π°Π΄Π°Π½Π½ΠΎΠΉ ΠΌΠ°Ρ‚Ρ€ΠΈΡ†Ρ‹ Ρ€Π°Π²Π½ΠΎ Π·Π°Π΄Π°Π½Π½ΠΎΠΉ ΠΌΠ°Ρ‚Ρ€ΠΈΡ†Π΅:

➣ ЧислСнный ΠΏΡ€ΠΈΠΌΠ΅Ρ€

➀ ΠŸΡ€ΠΈΠΌΠ΅Ρ€ Π½Π° Python

>>> A = np. matrix('1 2; 3 4')
>>> L = 1 * A
>>> R = A
>>> print(L)
[[1 2]
[3 4]]

>>> print(R)
[[1 2]
[3 4]]

Β 

Бвойство 2. ΠŸΡ€ΠΎΠΈΠ·Π²Π΅Π΄Π΅Π½ΠΈΠ΅ нуля ΠΈ любой ΠΌΠ°Ρ‚Ρ€ΠΈΡ†Ρ‹ Ρ€Π°Π²Π½ΠΎ Π½ΡƒΠ»Π΅Π²ΠΎΠΉ ΠΌΠ°Ρ‚Ρ€ΠΈΡ†Π΅, Ρ€Π°Π·ΠΌΠ΅Ρ€Π½ΠΎΡΡ‚ΡŒ ΠΊΠΎΡ‚ΠΎΡ€ΠΎΠΉ Ρ€Π°Π²Π½Π° исходной ΠΌΠ°Ρ‚Ρ€ΠΈΡ†Ρ‹:

➣ ЧислСнный ΠΏΡ€ΠΈΠΌΠ΅Ρ€

➀ ΠŸΡ€ΠΈΠΌΠ΅Ρ€ Π½Π° Python

>>> A = np.matrix('1 2; 3 4')
>>> Z = np.matrix('0 0; 0 0')
>>> L = 0 * A
>>> R = Z

>>> print(L)
[[0 0]
[0 0]]

>>> print(R)
[[0 0]
[0 0]]

Β 

Бвойство 3. ΠŸΡ€ΠΎΠΈΠ·Π²Π΅Π΄Π΅Π½ΠΈΠ΅ ΠΌΠ°Ρ‚Ρ€ΠΈΡ†Ρ‹ Π½Π° сумму чисСл Ρ€Π°Π²Π½ΠΎ суммС ΠΏΡ€ΠΎΠΈΠ·Π²Π΅Π΄Π΅Π½ΠΈΠΉ ΠΌΠ°Ρ‚Ρ€ΠΈΡ†Ρ‹ Π½Π° ΠΊΠ°ΠΆΠ΄ΠΎΠ΅ ΠΈΠ· этих чисСл:

➣ ЧислСнный ΠΏΡ€ΠΈΠΌΠ΅Ρ€

➀ ΠŸΡ€ΠΈΠΌΠ΅Ρ€ Π½Π° Python

>>> A = np.matrix('1 2; 3 4')
>>> p = 2
>>> q = 3

>>> L = (p + q) * A
>>> R = p * A + q * A

>>> print(L)
[[ 5 10]
[15 20]]

>>> print(R)
[[ 5 10]
[15 20]]

Β 

Бвойство 4. ΠŸΡ€ΠΎΠΈΠ·Π²Π΅Π΄Π΅Π½ΠΈΠ΅ ΠΌΠ°Ρ‚Ρ€ΠΈΡ†Ρ‹ Π½Π° ΠΏΡ€ΠΎΠΈΠ·Π²Π΅Π΄Π΅Π½ΠΈΠ΅ Π΄Π²ΡƒΡ… чисСл Ρ€Π°Π²Π½ΠΎ ΠΏΡ€ΠΎΠΈΠ·Π²Π΅Π΄Π΅Π½ΠΈΡŽ Π²Ρ‚ΠΎΡ€ΠΎΠ³ΠΎ числа ΠΈ Π·Π°Π΄Π°Π½Π½ΠΎΠΉ ΠΌΠ°Ρ‚Ρ€ΠΈΡ†Ρ‹, ΡƒΠΌΠ½ΠΎΠΆΠ΅Π½Π½ΠΎΠΌΡƒ Π½Π° ΠΏΠ΅Ρ€Π²ΠΎΠ΅ число:

➣ ЧислСнный ΠΏΡ€ΠΈΠΌΠ΅Ρ€

βž€ΠŸΡ€ΠΈΠΌΠ΅Ρ€ Π½Π° Python

>>> A = np.matrix('1 2; 3 4')
>>> p = 2
>>> q = 3

>>> L = (p * q) * A
>>> R = p * (q * A)

>>> print(L)
[[ 6 12]
[18 24]]

>>> print(R)
[[ 6 12]
[18 24]]

Β 

Бвойство 5. ΠŸΡ€ΠΎΠΈΠ·Π²Π΅Π΄Π΅Π½ΠΈΠ΅ суммы ΠΌΠ°Ρ‚Ρ€ΠΈΡ† Π½Π° число Ρ€Π°Π²Π½ΠΎ суммС ΠΏΡ€ΠΎΠΈΠ·Π²Π΅Π΄Π΅Π½ΠΈΠΉ этих ΠΌΠ°Ρ‚Ρ€ΠΈΡ† Π½Π° Π·Π°Π΄Π°Π½Π½ΠΎΠ΅ число:

➣ ЧислСнный ΠΏΡ€ΠΈΠΌΠ΅Ρ€

➀ ΠŸΡ€ΠΈΠΌΠ΅Ρ€ Π½Π° Python

>>> A = np.matrix('1 2; 3 4')
>>> B = np.matrix('5 6; 7 8')
>>> k = 3

>>> L = k * (A + B)
>>> R = k * A + k * B

>>> print(L)
[[18 24]
[30 36]]

>>> print(R)
[[18 24]
[30 36]]

Β 

Π‘Π»ΠΎΠΆΠ΅Π½ΠΈΠ΅ ΠΌΠ°Ρ‚Ρ€ΠΈΡ†

Π‘ΠΊΠ»Π°Π΄Ρ‹Π²Π°Ρ‚ΡŒ ΠΌΠΎΠΆΠ½ΠΎ Ρ‚ΠΎΠ»ΡŒΠΊΠΎ ΠΌΠ°Ρ‚Ρ€ΠΈΡ†Ρ‹ ΠΎΠ΄ΠΈΠ½Π°ΠΊΠΎΠ²ΠΎΠΉ размСрности β€” Ρ‚ΠΎ Π΅ΡΡ‚ΡŒ ΠΌΠ°Ρ‚Ρ€ΠΈΡ†Ρ‹, Ρƒ ΠΊΠΎΡ‚ΠΎΡ€Ρ‹Ρ… совпадаСт количСство столбцов ΠΈ строк.

➣ ЧислСнный ΠΏΡ€ΠΈΠΌΠ΅Ρ€

➀ ΠŸΡ€ΠΈΠΌΠ΅Ρ€ Π½Π° Python

>>> A = np.matrix('1 6 3; 8 2 7')
>>> B = np.matrix('8 1 5; 6 9 12')
>>> C = A + B

>>> print(C)
[[ 9 Β 7 8]
[14 11 19]]

Β 

Рассмотрим свойства слоТСния ΠΌΠ°Ρ‚Ρ€ΠΈΡ†.

Бвойство 1. ΠšΠΎΠΌΠΌΡƒΡ‚Π°Ρ‚ΠΈΠ²Π½ΠΎΡΡ‚ΡŒ слоТСния. ΠžΡ‚ пСрСстановки ΠΌΠ°Ρ‚Ρ€ΠΈΡ† ΠΈΡ… сумма Π½Π΅ измСняСтся:

➣ ЧислСнный ΠΏΡ€ΠΈΠΌΠ΅Ρ€

➀ ΠŸΡ€ΠΈΠΌΠ΅Ρ€ Π½Π° Python

>>> A = np.matrix('1 2; 3 4')
>>> B = np.matrix('5 6; 7 8')

>>> L = A + B
>>> R = B + A

>>> print(L)
[[ 6 Β 8]
[10 12]]

>>> print(R)
[[ 6 Β 8]
[10 12]]

Β 

Бвойство 2. ΠΡΡΠΎΡ†ΠΈΠ°Ρ‚ΠΈΠ²Π½ΠΎΡΡ‚ΡŒ слоТСния. Π Π΅Π·ΡƒΠ»ΡŒΡ‚Π°Ρ‚ слоТСния Ρ‚Ρ€Π΅Ρ… ΠΈ Π±ΠΎΠ»Π΅Π΅ ΠΌΠ°Ρ‚Ρ€ΠΈΡ† Π½Π΅ зависит ΠΎΡ‚ порядка, Π² ΠΊΠΎΡ‚ΠΎΡ€ΠΎΠΌ эта опСрация Π±ΡƒΠ΄Π΅Ρ‚ Π²Ρ‹ΠΏΠΎΠ»Π½ΡΡ‚ΡŒΡΡ:

➣ ЧислСнный ΠΏΡ€ΠΈΠΌΠ΅Ρ€

➀ ΠŸΡ€ΠΈΠΌΠ΅Ρ€ Π½Π° Python

>>> A = np. matrix('1 2; 3 4')
>>> B = np.matrix('5 6; 7 8')
>>> C = np.matrix('1 7; 9 3')

>>> L = A + (B + C)
>>> R = (A + B) + C
>>> print(L)

[[ 7 15]
[19 15]]

>>> print(R)
[[ 7 15]
[19 15]]

Β 

Бвойство 3. Для любой ΠΌΠ°Ρ‚Ρ€ΠΈΡ†Ρ‹ сущСствуСт противополоТная Π΅ΠΉ , такая, Ρ‡Ρ‚ΠΎ ΠΈΡ… сумма являСтся Π½ΡƒΠ»Π΅Π²ΠΎΠΉ ΠΌΠ°Ρ‚Ρ€ΠΈΡ†Π΅ΠΉ :

➣ ЧислСнный ΠΏΡ€ΠΈΠΌΠ΅Ρ€

➀ ΠŸΡ€ΠΈΠΌΠ΅Ρ€ Π½Π° Python

>>> A = np.matrix('1 2; 3 4')
>>> Z = np.matrix('0 0; 0 0')

>>> L = A + (-1)*A

>>> print(L)
[[0 0]
[0 0]]

>>> print(Z)
[[0 0]
[0 0]]

Β 

Π£ΠΌΠ½ΠΎΠΆΠ΅Π½ΠΈΠ΅ ΠΌΠ°Ρ‚Ρ€ΠΈΡ†

Π£ΠΌΠ½ΠΎΠΆΠ΅Π½ΠΈΠ΅ ΠΌΠ°Ρ‚Ρ€ΠΈΡ† это ΡƒΠΆΠ΅ Π±ΠΎΠ»Π΅Π΅ слоТная опСрация, ΠΏΠΎ ΡΡ€Π°Π²Π½Π΅Π½ΠΈΡŽ с рассмотрСнными Π²Ρ‹ΡˆΠ΅. Π£ΠΌΠ½ΠΎΠΆΠ°Ρ‚ΡŒ ΠΌΠΎΠΆΠ½ΠΎ Ρ‚ΠΎΠ»ΡŒΠΊΠΎ ΠΌΠ°Ρ‚Ρ€ΠΈΡ†Ρ‹, ΠΎΡ‚Π²Π΅Ρ‡Π°ΡŽΡ‰ΠΈΠ΅ ΡΠ»Π΅Π΄ΡƒΡŽΡ‰Π΅ΠΌΡƒ Ρ‚Ρ€Π΅Π±ΠΎΠ²Π°Π½ΠΈΡŽ: количСство столбцов ΠΏΠ΅Ρ€Π²ΠΎΠΉ ΠΌΠ°Ρ‚Ρ€ΠΈΡ†Ρ‹ Π΄ΠΎΠ»ΠΆΠ½ΠΎ Π±Ρ‹Ρ‚ΡŒ Ρ€Π°Π²Π½ΠΎ числу строк Π²Ρ‚ΠΎΡ€ΠΎΠΉ ΠΌΠ°Ρ‚Ρ€ΠΈΡ†Ρ‹.

Для простоты запоминания этого ΠΏΡ€Π°Π²ΠΈΠ»Π° ΠΌΠΎΠΆΠ½ΠΎ ΠΈΡΠΏΠΎΠ»ΡŒΠ·ΠΎΠ²Π°Ρ‚ΡŒ Π΄ΠΈΠ°Π³Ρ€Π°ΠΌΠΌΡƒ умноТСния, ΠΏΡ€Π΅Π΄ΡΡ‚Π°Π²Π»Π΅Π½Π½ΡƒΡŽ Π½Π° рисункС 1.

Рисунок 1 β€” Π”ΠΈΠ°Π³Ρ€Π°ΠΌΠΌΠ° ΠΌΠ°Ρ‚Ρ€ΠΈΡ‡Π½ΠΎΠ³ΠΎ умноТСния

Рассмотрим ΡƒΠΌΠ½ΠΎΠΆΠ΅Π½ΠΈΠ΅ ΠΌΠ°Ρ‚Ρ€ΠΈΡ† Π½Π° ΠΏΡ€ΠΈΠΌΠ΅Ρ€Π΅.

➣ ЧислСнный ΠΏΡ€ΠΈΠΌΠ΅Ρ€

ΠšΠ°ΠΆΠ΄Ρ‹ΠΉ элСмСнт cij Π½ΠΎΠ²ΠΎΠΉ ΠΌΠ°Ρ‚Ρ€ΠΈΡ†Ρ‹ являСтся суммой ΠΏΡ€ΠΎΠΈΠ·Π²Π΅Π΄Π΅Π½ΠΈΠΉ элСмСнтов i-ΠΎΠΉ строки ΠΏΠ΅Ρ€Π²ΠΎΠΉ ΠΌΠ°Ρ‚Ρ€ΠΈΡ†Ρ‹ ΠΈ j-Π³ΠΎ столбца Π²Ρ‚ΠΎΡ€ΠΎΠΉ ΠΌΠ°Ρ‚Ρ€ΠΈΡ†Ρ‹. ΠœΠ°Ρ‚Π΅ΠΌΠ°Ρ‚ΠΈΡ‡Π΅ΡΠΊΠΈ это записываСтся Ρ‚Π°ΠΊ:

βž€ΠŸΡ€ΠΈΠΌΠ΅Ρ€ Π½Π° Python

РСшим Π·Π°Π΄Π°Ρ‡Ρƒ умноТСния ΠΌΠ°Ρ‚Ρ€ΠΈΡ† Π½Π° языкС Python. Для этого Π±ΡƒΠ΄Π΅ΠΌ ΠΈΡΠΏΠΎΠ»ΡŒΠ·ΠΎΠ²Π°Ρ‚ΡŒ Ρ„ΡƒΠ½ΠΊΡ†ΠΈΡŽ dot() ΠΈΠ· Π±ΠΈΠ±Π»ΠΈΠΎΡ‚Π΅ΠΊΠΈ Numpy:

>>> A = np.matrix('1 2 3; 4 5 6')
>>> B = np.matrix('7 8; 9 1; 2 3')

>>> C = A.dot(B)

>>> print(C)
[[31 19]
[85 55]]

Β 

НиТС прСдставлСны свойства произвСдСния ΠΌΠ°Ρ‚Ρ€ΠΈΡ†. ΠŸΡ€ΠΈΠΌΠ΅Ρ€Ρ‹ свойств Π±ΡƒΠ΄ΡƒΡ‚ ΠΏΠΎΠΊΠ°Π·Π°Π½Ρ‹ для ΠΊΠ²Π°Π΄Ρ€Π°Ρ‚Π½ΠΎΠΉ ΠΌΠ°Ρ‚Ρ€ΠΈΡ†Ρ‹.

Бвойство 1. ΠΡΡΠΎΡ†ΠΈΠ°Ρ‚ΠΈΠ²Π½ΠΎΡΡ‚ΡŒ умноТСния. Π Π΅Π·ΡƒΠ»ΡŒΡ‚Π°Ρ‚ умноТСния ΠΌΠ°Ρ‚Ρ€ΠΈΡ† Π½Π΅ зависит ΠΎΡ‚ порядка, Π² ΠΊΠΎΡ‚ΠΎΡ€ΠΎΠΌ Π±ΡƒΠ΄Π΅Ρ‚ Π²Ρ‹ΠΏΠΎΠ»Π½ΡΡ‚ΡŒΡΡ эта опСрация:

➣ ЧислСнный ΠΏΡ€ΠΈΠΌΠ΅Ρ€

➀ ΠŸΡ€ΠΈΠΌΠ΅Ρ€ Π½Π° Python

>>> A = np.matrix('1 2; 3 4')
>>> B = np.matrix('5 6; 7 8')
>>> C = np.matrix('2 4; 7 8')

>>> L = A.dot(B.dot(C))
>>> R = (A.dot(B)).dot(C)

>>> print(L)
[[192 252]
[436 572]]

>>> print(R)
[[192 252]
[436 572]]

Β 

Бвойство 2. Π”ΠΈΡΡ‚Ρ€ΠΈΠ±ΡƒΡ‚ΠΈΠ²Π½ΠΎΡΡ‚ΡŒ умноТСния. ΠŸΡ€ΠΎΠΈΠ·Π²Π΅Π΄Π΅Π½ΠΈΠ΅ ΠΌΠ°Ρ‚Ρ€ΠΈΡ†Ρ‹ Π½Π° сумму ΠΌΠ°Ρ‚Ρ€ΠΈΡ† Ρ€Π°Π²Π½ΠΎ суммС ΠΏΡ€ΠΎΠΈΠ·Π²Π΅Π΄Π΅Π½ΠΈΠΉ ΠΌΠ°Ρ‚Ρ€ΠΈΡ†:

➣ ЧислСнный ΠΏΡ€ΠΈΠΌΠ΅Ρ€

➀ ΠŸΡ€ΠΈΠΌΠ΅Ρ€ Π½Π° Python

>>> A = np.matrix('1 2; 3 4')
>>> B = np.matrix('5 6; 7 8')
>>> C = np.matrix('2 4; 7 8')

>>> L = A. dot(B + C)
>>> R = A.dot(B) + A.dot(C)

>>> print(L)
[[35 42]
[77 94]]
>>> print(R)
[[35 42]
[77 94]]

Β 

Бвойство 3. Π£ΠΌΠ½ΠΎΠΆΠ΅Π½ΠΈΠ΅ ΠΌΠ°Ρ‚Ρ€ΠΈΡ† Π² ΠΎΠ±Ρ‰Π΅ΠΌ Π²ΠΈΠ΄Π΅ Π½Π΅ ΠΊΠΎΠΌΠΌΡƒΡ‚Π°Ρ‚ΠΈΠ²Π½ΠΎ. Π­Ρ‚ΠΎ ΠΎΠ·Π½Π°Ρ‡Π°Π΅Ρ‚, Ρ‡Ρ‚ΠΎ для ΠΌΠ°Ρ‚Ρ€ΠΈΡ† Π½Π΅ выполняСтся ΠΏΡ€Π°Π²ΠΈΠ»ΠΎ нСзависимости произвСдСния ΠΎΡ‚ пСрСстановки ΠΌΠ½ΠΎΠΆΠΈΡ‚Π΅Π»Π΅ΠΉ:

➣ ЧислСнный ΠΏΡ€ΠΈΠΌΠ΅Ρ€

➀ ΠŸΡ€ΠΈΠΌΠ΅Ρ€ Π½Π° Python

>>> A = np.matrix('1 2; 3 4')
>>> B = np.matrix('5 6; 7 8')

>>> L = A.dot(B)
>>> R = B.dot(A)

>>> print(L)
[[19 22]
[43 50]]

>>> print(R)
[[23 34]
[31 46]]

Β 

Бвойство 4. ΠŸΡ€ΠΎΠΈΠ·Π²Π΅Π΄Π΅Π½ΠΈΠ΅ Π·Π°Π΄Π°Π½Π½ΠΎΠΉ ΠΌΠ°Ρ‚Ρ€ΠΈΡ†Ρ‹ Π½Π° Π΅Π΄ΠΈΠ½ΠΈΡ‡Π½ΡƒΡŽ Ρ€Π°Π²Π½ΠΎ исходной ΠΌΠ°Ρ‚Ρ€ΠΈΡ†Π΅:

➣ ЧислСнный ΠΏΡ€ΠΈΠΌΠ΅Ρ€

➀ ΠŸΡ€ΠΈΠΌΠ΅Ρ€ Π½Π° Python

>>> A = np. matrix('1 2; 3 4')
>>> E = np.matrix('1 0; 0 1')

>>> L = E.dot(A)
>>> R = A.dot(E)

>>> print(L)
[[1 2]
[3 4]]

>>> print(R)
[[1 2]
[3 4]]

>>> print(A)
[[1 2]
[3 4]]

Β 

Бвойство 5. ΠŸΡ€ΠΎΠΈΠ·Π²Π΅Π΄Π΅Π½ΠΈΠ΅ Π·Π°Π΄Π°Π½Π½ΠΎΠΉ ΠΌΠ°Ρ‚Ρ€ΠΈΡ†Ρ‹ Π½Π° Π½ΡƒΠ»Π΅Π²ΡƒΡŽ ΠΌΠ°Ρ‚Ρ€ΠΈΡ†Ρƒ Ρ€Π°Π²Π½ΠΎ Π½ΡƒΠ»Π΅Π²ΠΎΠΉ ΠΌΠ°Ρ‚Ρ€ΠΈΡ†Π΅:

➣ ЧислСнный ΠΏΡ€ΠΈΠΌΠ΅Ρ€

➀ ΠŸΡ€ΠΈΠΌΠ΅Ρ€ Π½Π° Python

>>> A = np.matrix('1 2; 3 4')
>>> Z = np.matrix('0 0; 0 0')

>>> L = Z.dot(A)
>>> R = A.dot(Z)

>>> print(L)
[[0 0]
[0 0]]

>>> print(R)
[[0 0]
[0 0]]

>>> print(Z)
[[0 0]
[0 0]]

Β 

P.S.

Π’Π²ΠΎΠ΄Π½Ρ‹Π΅ ΡƒΡ€ΠΎΠΊΠΈ ΠΏΠΎ β€œΠ›ΠΈΠ½Π΅ΠΉΠ½ΠΎΠΉ Π°Π»Π³Π΅Π±Ρ€Π΅ Π½Π° Python” Π²Ρ‹ ΠΌΠΎΠΆΠ΅Ρ‚Π΅ Π½Π°ΠΉΡ‚ΠΈ ΡΠΎΠΎΡ‚Π²Π΅Ρ‚ΡΡ‚Π²ΡƒΡŽΡ‰Π΅ΠΉ страницС нашСго сайта. ВсС ΡƒΡ€ΠΎΠΊΠΈ ΠΏΠΎ этой Ρ‚Π΅ΠΌΠ΅ собраны Π² ΠΊΠ½ΠΈΠ³Π΅ β€œΠ›ΠΈΠ½Π΅ΠΉΠ½Π°Ρ Π°Π»Π³Π΅Π±Ρ€Π° Π½Π° Python”.

Если Π²Π°ΠΌ интСрСсна Ρ‚Π΅ΠΌΠ° Π°Π½Π°Π»ΠΈΠ·Π° Π΄Π°Π½Π½Ρ‹Ρ…, Ρ‚ΠΎ ΠΌΡ‹ Ρ€Π΅ΠΊΠΎΠΌΠ΅Π½Π΄ΡƒΠ΅ΠΌ ΠΎΠ·Π½Π°ΠΊΠΎΠΌΠΈΡ‚ΡŒΡΡ с Π±ΠΈΠ±Π»ΠΈΠΎΡ‚Π΅ΠΊΠΎΠΉ Pandas.Β  Для Π½Π°Ρ‡Π°Π»Π° Π²Ρ‹ ΠΌΠΎΠΆΠ΅Ρ‚Π΅ ΠΏΠΎΠ·Π½Π°ΠΊΠΎΠΌΠΈΡ‚ΡŒΡΡ с вводными ΡƒΡ€ΠΎΠΊΠ°ΠΌΠΈ. ВсС ΡƒΡ€ΠΎΠΊΠΈ ΠΏΠΎ Π±ΠΈΠ±Π»ΠΈΠΎΡ‚Π΅ΠΊΠ΅ Pandas собраны Π² ΠΊΠ½ΠΈΠ³Π΅ β€œPandas. Π Π°Π±ΠΎΡ‚Π° с данными”.

ΠœΠ°Ρ‚Ρ€ΠΈΡ†Π° слоТСниС ΠΌΠ°Ρ‚Ρ€ΠΈΡ† — ЭнциклопСдия ΠΏΠΎ ΠΌΠ°ΡˆΠΈΠ½ΠΎΡΡ‚Ρ€ΠΎΠ΅Π½ΠΈΡŽ XXL

Π’ ΠΊΠΈΠ½Π΅ΠΌΠ°Ρ‚ΠΈΠΊΠ΅ ΠΌΠ΅Ρ…Π°Π½ΠΈΠ·ΠΌΠΎΠ² ΠΎΠΏΠ΅Ρ€Π°Ρ†ΠΈΠΈ слоТСния ΠΌΠ°Ρ‚Ρ€ΠΈΡ† ΠΈ умноТСния ΠΈΡ… Π½Π° скаляр находят ΠΏΡ€ΠΈΠΌΠ΅Π½Π΅Π½ΠΈΠ΅ Π² дСйствиях Π½Π°Π΄ ΠΌΠ°Ρ‚Ρ€ΠΈΡ†Π°ΠΌΠΈ-столбцами.  [c.631]

ΠœΠ°Ρ‚Ρ€ΠΈΡ†Π° [/(], называСмая глобальной ΠΌΠ°Ρ‚Ρ€ΠΈΡ†Π΅ΠΉ ТСсткости ΠΈΠ»ΠΈ просто ΠΌΠ°Ρ‚Ρ€ΠΈΡ†Π΅ΠΉ ТСсткости систСмы, получаСтся слоТСниСм Π»ΠΎΠΊΠ°Π»ΡŒΠ½Ρ‹Ρ… ΠΌΠ°Ρ‚Ρ€ΠΈΡ† ТСсткости [Π› ] ΠΏΠΎ ΡΠ»Π΅Π΄ΡƒΡŽΡ‰Π΅ΠΌΡƒ ΠΏΡ€Π°Π²ΠΈΠ»Ρƒ сначала ΠΊ Π½ΡƒΠ»Π΅Π²ΠΎΠΉ ΠΌΠ°Ρ‚Ρ€ΠΈΡ†Π΅ размСрности NxN добавляСтся ΠΌΠ°Ρ‚Ρ€ΠΈΡ†Π°, Π² Π»Π΅Π²ΠΎΠΌ Π²Π΅Ρ€Ρ…Π½Π΅ΠΌ ΡƒΠ³Π»Ρƒ ΠΊΠΎΡ‚ΠΎΡ€ΠΎΠΉ стоит локальная ΠΌΠ°Ρ‚Ρ€ΠΈΡ†Π° ТСсткости 1-Π³ΠΎ элСмСнта, ΠΊ ΠΏΠΎΠ»ΡƒΡ‡ΠΈΠ²ΡˆΠ΅ΠΉΡΡ ΠΌΠ°Ρ‚Ρ€ΠΈΡ†Π΅ добавляСтся ΠΌΠ°Ρ‚Ρ€ΠΈΡ†Π° Ρ€Π°Π·ΠΌΠ΅Ρ€Π° /V Ρ… /V, Π½Π΅Π½ΡƒΠ»Π΅Π²Ρ‹Π΅ элСмСнты ΠΊΠΎΡ‚ΠΎΡ€ΠΎΠΉ располоТСны Π½Π° пСрСсСчСнии 2-Π³ΠΎ ΠΈ 3-Π³ΠΎ столбцов ΠΈ 2-ΠΉ ΠΈ 3-ΠΉ строк ΠΈ Ρ€Π°Π²Π½Ρ‹ ΡΠΎΠΎΡ‚Π²Π΅Ρ‚ΡΡ‚Π²ΡƒΡŽΡ‰ΠΈΠΌ элСмСнтам локальной ΠΌΠ°Ρ‚Ρ€ΠΈΡ†Ρ‹ ТСсткости для 2-Π³ΠΎ элСмСнта ΠΈ Ρ‚. Π΄. Π½Π° -ΠΌ шагС добавляСтся ΠΌΠ°Ρ‚Ρ€ΠΈΡ†Π°, Π½Π΅Π½ΡƒΠ»Π΅Π²Ρ‹Π΅ элСмСнты ΠΊΠΎΡ‚ΠΎΡ€ΠΎΠΉ располоТСны Π½Π° пСрСсСчСнии ΠΊ ΠΈ ΠΊ- строк ΠΈ ΠΊ Π½ k- — столбцов ΠΈ Ρ€Π°Π²Π½Ρ‹ ΡΠΎΠΎΡ‚Π²Π΅Ρ‚ΡΡ‚Π²ΡƒΡŽΡ‰ΠΈΠΌ элСмСнтам локальной ΠΌΠ°Ρ‚Ρ€ΠΈΡ†Ρ‹ ТСсткости k-ro элСмСнта.  [c.134]


ВсС ΠΎΡΡ‚Π°Π»ΡŒΠ½Ρ‹Π΅ нСизвСстныС ΠΎΠΏΡ€Π΅Π΄Π΅Π»ΡΡŽΡ‚ΡΡ лишь ΠΏΡƒΡ‚Π΅ΠΌ умноТСния ΠΈ слоТСния ΠΌΠ°Ρ‚Ρ€ΠΈΡ†. Π’Π΅ΡΡŒ процСсс вычислСний ΠΏΡ€ΠΈ этом Π»Π΅Π³ΠΊΠΎ программируСтся с использованиСм стандартных ΠΏΡ€ΠΎΠ³Ρ€Π°ΠΌΠΌ слоТСния, умноТСния ΠΈ обращСния ΠΌΠ°Ρ‚Ρ€ΠΈΡ†.  [c.97]

НСпосрСдствСнным ΠΏΠ΅Ρ€Π΅ΠΌΠ½ΠΎΠΆΠ΅Π½ΠΈΠ΅ΠΌ ΠΈ слоТСниСм ΠΌΠ°Ρ‚Ρ€ΠΈΡ† (71.29) Π½Π΅Ρ‚Ρ€ΡƒΠ΄Π½ΠΎ ΡƒΠ±Π΅Π΄ΠΈΡ‚ΡŒΡΡ, Ρ‡Ρ‚ΠΎ ΠΎΠ½ΠΈ ΡƒΠ΄ΠΎΠ²Π»Π΅Ρ‚Π²ΠΎΡ€ΡΡŽΡ‚ ΡΠΎΠΎΡ‚Π½ΠΎΡˆΠ΅Π½ΠΈΡΠΌ (71.28), понимая, Ρ‡Ρ‚ΠΎ Π² ΠΈΡ… ΠΏΡ€Π°Π²ΠΎΠΉ части стоит Сдиничная ΠΌΠ°Ρ‚Ρ€ΠΈΡ†Π°. НапримСр, для ΠΈΠΌΠ΅Π΅ΠΌ  [c.387]

Π‘Π»ΠΎΠΆΠ΅Π½ΠΈΠ΅ ΠΈ Π²Ρ‹Ρ‡ΠΈΡ‚Π°Π½ΠΈΠ΅ ΠΌΠ°Ρ‚Ρ€ΠΈΡ†. Π”Π²Π΅ ΠΌΠ°Ρ‚Ρ€ΠΈΡ†Ρ‹ ΠΌΠΎΠΆΠ½ΠΎ ΡΠ»ΠΎΠΆΠΈΡ‚ΡŒ (Π²Ρ‹Ρ‡Π΅ΡΡ‚ΡŒ). Для этого Π½Π΅ΠΎΠ±Ρ…ΠΎΠ΄ΠΈΠΌΠΎ, Ρ‡Ρ‚ΠΎΠ±Ρ‹ порядки ΠΌΠ°Ρ‚Ρ€ΠΈΡ† Π±Ρ‹Π»ΠΈ ΠΎΠ΄ΠΈΠ½Π°ΠΊΠΎΠ²Ρ‹, ΠΈ Ρ‚ΠΎΠ³Π΄Π° ΡΠ»ΠΎΠΆΠΈΡ‚ΡŒ (Π²Ρ‹Ρ‡Π΅ΡΡ‚ΡŒ) ΡΠΎΠΎΡ‚Π²Π΅Ρ‚ΡΡ‚Π²ΡƒΡŽΡ‰ΠΈΠ΅ элСмСнты ΠΌΠ°Ρ‚Ρ€ΠΈΡ†.  [c.179]

Π’ Π°Π»Π³Π΅Π±Ρ€Π΅ ΠΌΠ°Ρ‚Ρ€ΠΈΡ† ΠΎΠΏΡ€Π΅Π΄Π΅Π»ΡΡŽΡ‚ΡΡ ΡΠ»Π΅Π΄ΡƒΡŽΡ‰ΠΈΠ΅ дСйствия Π½Π°Π΄ ΠΌΠ°Ρ‚Ρ€ΠΈΡ†Π°ΠΌΠΈ Π°) слоТСниС ΠΌΠ°Ρ‚Ρ€ΠΈΡ† Π±) ΡƒΠΌΠ½ΠΎΠΆΠ΅Π½ΠΈΠ΅ ΠΌΠ°Ρ‚Ρ€ΠΈΡ†Ρ‹ Π½Π° число Π²) ΡƒΠΌΠ½ΠΎΠΆΠ΅Π½ΠΈΠ΅ ΠΌΠ°Ρ‚Ρ€ΠΈΡ†. Π£ΠΊΠ°Π·Π°Π½Π½Ρ‹Π΅ дСйствия ΠΏΠΎΠ·Π²ΠΎΠ»ΡΡŽΡ‚ Π²Ρ‹Ρ‡ΠΈΡΠ»ΠΈΡ‚ΡŒ соотвСтствСнно сумму ΠΌΠ°Ρ‚Ρ€ΠΈΡ†, ΠΏΡ€ΠΎΠΈΠ·Π²Π΅Π΄Π΅Π½ΠΈΠ΅ ΠΌΠ°Ρ‚Ρ€ΠΈΡ†Ρ‹ Π½Π° число, ΠΏΡ€ΠΎΠΈΠ·Π²Π΅Π΄Π΅Π½ΠΈΠ΅ ΠΌΠ°Ρ‚Ρ€ΠΈΡ† ΠΈ, ΠΊΠ°ΠΊ слСдствиС, Ρ€Π°Π·Π½ΠΎΡΡ‚ΡŒ ΠΌΠ°Ρ‚Ρ€ΠΈΡ†.  [c.41]

ЛогичСскоС слоТСниС ΠΌΠ°Ρ‚Ρ€ΠΈΡ† рСализуСтся ΠΏΡƒΡ‚Π΅ΠΌ ΠΏΠΎΠΏΠ°Ρ€Π½ΠΎΠ³ΠΎ логичСского слоТСния ΠΎΠ΄Π½ΠΎΠΈΠΌΠ΅Π½Π½Ρ‹Ρ… элСмСнтов исходных ΠΌΠ°Ρ‚Ρ€ΠΈΡ†  [c.123]

Рассмотрим, ΠΊΠ°ΠΊ влияСт ΡƒΡ‡Π΅Ρ‚ диспСрсий Π½Π° Ρ€Π΅Π·ΡƒΠ»ΡŒΡ‚Π°Ρ‚Ρ‹ расчСта. Π€ΠΎΡ€ΠΌΠ°Π»ΡŒΠ½ΠΎ ΡƒΡ‡Π΅Ρ‚ диспСрсий выраТаСтся Π² слоТСнии ΠΌΠ°Ρ‚Ρ€ΠΈΡ†Ρ‹ [Π°] [Π°] с диагональной ΠΌΠ°Ρ‚Ρ€ΠΈΡ†Π΅ΠΉ [ )], ΠΈΠΌΠ΅ΡŽΡ‰Π΅ΠΉ ΠΏΠΎΠ»ΠΎΠΆΠΈΡ‚Π΅Π»ΡŒΠ½Ρ‹Π΅ Ρ‡Π»Π΅Π½Ρ‹. Π­Ρ‚ΠΎ способствуСт ΡƒΠ»ΡƒΡ‡ΡˆΠ΅Π½ΠΈΡŽ обусловлСнности ΠΌΠ°Ρ‚Ρ€ΠΈΡ†Ρ‹ [Π°] [Π°], Ρ‡Ρ‚ΠΎ особСнно Π²Π°ΠΆΠ½ΠΎ Π² случаях, ΠΊΠΎΠ³Π΄Π° послСдняя ΠΏΠ»ΠΎΡ…ΠΎ  [c.58]

Π‘Π»ΠΎΠΆΠ΅Π½ΠΈΠ΅ ΠΌΠ°Ρ‚Ρ€ΠΈΡ†. Π‘ΡƒΠΌΠΌΠΎΠΉ ΠΌΠ°Ρ‚Ρ€ΠΈΡ†  [c.95]

DN β€” присвоСниС Π·Π½Π°Ρ‡Π΅Π½ΠΈΠΉ Π΅Π΄ΠΈΠ½ΠΈΡ†Ρ‹ Π΄ΠΈΠ°Π³ΠΎΠ½Π°Π»ΡŒΠ½Ρ‹ΠΌ элСмСнтам Π΄Π²ΡƒΠΌΠ΅Ρ€Π½ΠΎΠ³ΠΎ массива А+Π’ β€” слоТСниС ΠΌΠ°Ρ‚Ρ€ΠΈΡ† А ΠΈ Π’  [c.163]

ΠΠ΅ΠΎΠ±Ρ…ΠΎΠ΄ΠΈΠΌΠΎΡΡ‚ΡŒ примСнСния динамичСского ΠΌΠ΅Ρ‚ΠΎΠ΄Π° сущСствСнно услоТняСт Ρ€Π΅ΡˆΠ΅Π½ΠΈΠ΅ нСконсСрвативных Π·Π°Π΄Π°Ρ‡ устойчивости. Π—Π΄Π΅ΡΡŒ трСбуСтся вСсьма эффСктивный ΠΌΠ΅Ρ‚ΠΎΠ΄ опрСдСлСния частот собствСнных ΠΊΠΎΠ»Π΅Π±Π°Π½ΠΈΠΉ. Π‘Ρ€Π΅Π΄ΠΈ Π΄Ρ€ΡƒΠ³ΠΈΡ… ΠΌΠ΅Ρ‚ΠΎΠ΄ΠΎΠ² Π² этом ΠΎΡ‚Π½ΠΎΡˆΠ΅Π½ΠΈΠΈ Π²ΡŒΡ‰Π΅Π»ΡΠ΅Ρ‚ΡΡ ΠœΠ“Π­. Он позволяСт ΠΏΠΎΠ»ΡƒΡ‡Π°Ρ‚ΡŒ Ρ‚ΠΎΡ‡Π½Ρ‹ΠΉ спСктр частот (устраняСт нСдостаток МКЭ), Π° Π² трансцСндСнтном частотном ΡƒΡ€Π°Π²Π½Π΅Π½ΠΈΠΈ ΠΎΡ‚ΡΡƒΡ‚ΡΡ‚Π²ΡƒΡŽΡ‚ Ρ‚ΠΎΡ‡ΠΊΠΈ Ρ€Π°Π·Ρ€Ρ‹Π²Π° 2-Π³ΠΎ Ρ€ΠΎΠ΄Π° (устраняСт нСдостаток ΠΌΠ΅Ρ‚ΠΎΠ΄Π° ΠΏΠ΅Ρ€Π΅ΠΌΠ΅Ρ‰Π΅Π½ΠΈΠΉ). Π”ΠΎΠΏΠΎΠ»Π½ΠΈΡ‚Π΅Π»ΡŒΠ½Ρ‹ΠΌΠΈ ΠΏΠΎΠ»ΠΎΠΆΠΈΡ‚Π΅Π»ΡŒΠ½Ρ‹ΠΌΠΈ Ρ„Π°ΠΊΡ‚ΠΎΡ€Π°ΠΌΠΈ ΡΠ²Π»ΡΡŽΡ‚ΡΡ простая Π»ΠΎΠ³ΠΈΠΊΠ° формирования динамичСской ΠΌΠ°Ρ‚Ρ€ΠΈΡ†Ρ‹ устойчивости, отсутствиС ΠΎΠΏΠ΅Ρ€Π°Ρ†ΠΈΠΉ умноТСния, обращСния ΠΈ слоТСния ΠΌΠ°Ρ‚Ρ€ΠΈΡ†, Ρ…ΠΎΡ€ΠΎΡˆΠ°Ρ ΡƒΡΡ‚ΠΎΠΉΡ‡ΠΈΠ²ΠΎΡΡ‚ΡŒ числСнных ΠΎΠΏΠ΅Ρ€Π°Ρ†ΠΈΠΉ ΠΏΡ€ΠΈ вычислСнии опрСдСлитСля ΠΈ Ρ‚.ΠΏ.  [c.196]

Π‘Π»ΠΎΠΆΠ΅Π½ΠΈΠ΅ ΠΌΠ°Ρ‚Ρ€ΠΈΡ†. Π‘ΡƒΠΌΠΌΠΎΠΉ ΠΌΠ°Ρ‚Ρ€ΠΈΡ† А Π° Π’ называСтся ΠΌΠ°Ρ‚Ρ€ΠΈΡ†Π° Π‘, элСмСнты ΠΊΠΎΡ‚ΠΎΡ€ΠΎΠΉ с,Ρƒ ΠΎΠΏΡ€Π΅Π΄Π΅Π»ΡΡŽΡ‚ΡΡ ΠΏΠΎ Ρ„ΠΎΡ€ΠΌΡƒΠ»Π΅  [c.91]


ΠŸΡ€ΠΈ слоТСнии ΠΌΠ°Ρ‚Ρ€ΠΈΡ† ТСсткости элСмСнтов с Ρ†Π΅Π»ΡŒΡŽ получСния глобальной ΠΌΠ°Ρ‚Ρ€ΠΈΡ†Ρ‹ ТСсткости Π½Π΅ΠΎΠ±Ρ…ΠΎΠ΄ΠΈΠΌΠΎ Π·Π°ΠΏΠΎΠΌΠΈΠ½Π°Ρ‚ΡŒ ΠΊΠ°ΠΊ ΠΌΠ°Ρ‚Ρ€ΠΈΡ†Ρ‹ ΠΎΡ‚Π΄Π΅Π»ΡŒΠ½Ρ‹Ρ… элСмСнтов, Ρ‚Π°ΠΊ ΠΈ ΠΌΠ°Ρ‚Ρ€ΠΈΡ†Ρƒ [/Π‘]. Π­Ρ‚ΠΎ Ρ‚Π°ΠΊΠΆΠ΅ ΠΏΠ΅Ρ€Π΅Π³Ρ€ΡƒΠΆΠ°Π΅Ρ‚ ΠΎΠΏΠ΅Ρ€Π°Ρ‚ΠΈΠ²Π½ΡƒΡŽ ΠΏΠ°ΠΌΡΡ‚ΡŒ.  [c.251]

Как ΠΈ Π² случаС ΠΌΠ°Ρ‚Ρ€ΠΈΡ†, слоТСнию ΠΏΠΎΠ²ΠΎΡ€ΠΎΡ‚ΠΎΠ² Π² случаС ΠΈΡ… Π°ΠΊΡ‚ΠΈΠ²Π½ΠΎΠ³ΠΎ прСдставлСния ΠΎΡ‚Π²Π΅Ρ‡Π°Π΅Ρ‚ ΠΏΡ€ΠΎΠΈΠ·Π²Π΅Π΄Π΅Π½ΠΈΠ΅ ΠΊΠ²Π°Ρ‚Π΅Ρ€Π½ΠΈΠΎΠ½ΠΎΠ² ΡΠΎΡΡ‚Π°Π²Π»ΡΡŽΡ‰ΠΈΡ… ΠΏΠΎΠ²ΠΎΡ€ΠΎΡ‚ΠΎΠ² Π² ΠΎΠ±Ρ€Π°Ρ‚Π½ΠΎΠΌ порядкС. ΠŸΡ€ΠΈ этом всС ΠΊΠ²Π°Ρ‚Π΅Ρ€Π½ΠΈΠΎΠ½Ρ‹ Π·Π°Π΄Π°Π½Ρ‹ Π² исходном базисС ΠΊ (/ = 1, 2, 3).  [c.46]

Π‘Π»ΠΎΠΆΠ΅Π½ΠΈΠ΅. ΠœΠ°Ρ‚Ρ€ΠΈΡ†Ρ‹ Π› ΠΈ Π’ ΠΌΠΎΠ³ΡƒΡ‚ Π±Ρ‹Ρ‚ΡŒ слоТСны, Ссли ΠΎΠ½ΠΈ ΠΈΠΌΠ΅ΡŽΡ‚ ΠΎΠ΄ΠΈΠ½Π°ΠΊΠΎΠ²ΠΎΠ΅ число строк ΠΈ столбцов.  [c.634]

Π’ Ρ‚Π°Π±Π». 7.1 ΠΏΡ€ΠΈΠ²Π΅Π΄Π΅Π½Π° ΡΠΊΠΎΡ€ΠΎΡΡ‚ΡŒ выполнСния ΠΎΠΏΠ΅Ρ€Π°Ρ†ΠΈΠΈ умноТСния ΠΌΠ°Ρ‚Ρ€ΠΈΡ†Ρ‹ Π½Π° Π²Π΅ΠΊΡ‚ΠΎΡ€. Π’ ΠΏΠ΅Ρ€Π²ΠΎΠΉ ΠΊΠΎΠ»ΠΎΠ½ΠΊΠ΅ прСдставлСны выраТСния для числа Ρ‚Π°ΠΊΡ‚ΠΎΠ²Ρ‹Ρ… Ρ†ΠΈΠΊΠ»ΠΎΠ², Π½Π΅ΠΎΠ±Ρ…ΠΎΠ΄ΠΈΠΌΡ‹Ρ… для Π·Π°Π²Π΅Ρ€ΡˆΠ΅Π½ΠΈΡ ΠΎΠ΄Π½ΠΎΠΉ ΠΎΠΏΠ΅Ρ€Π°Ρ†ΠΈΠΈ умноТСния. Π£ΠΌΠ½ΠΎΠΆΠ΅Π½ΠΈΠ΅ ΠΌΠ°Ρ‚Ρ€ΠΈΡ†Ρ‹ Ρ‚Π₯ΠΏ Π½Π° Π²Π΅ΠΊΡ‚ΠΎΡ€ /Π³Π₯1 Ρ‚Ρ€Π΅Π±ΡƒΠ΅Ρ‚ 2Ρ‚ΠΏ ΠΎΠΏΠ΅Ρ€Π°Ρ†ΠΈΠΉ слоТСния ΠΈ умноТСния. Если ΠΏΡ€Π΅Π΄ΠΏΠΎΠ»ΠΎΠΆΠΈΡ‚ΡŒ, Ρ‡Ρ‚ΠΎ Π±ΠΈΡ‚Ρ‹ Π΄Π°Π½Π½Ρ‹Ρ… проходят Π² систСмС с частотой 10 ΠœΠ“Ρ† (Π²Π΅Π»ΠΈΡ‡ΠΈΠ½Π° 0,1 мкс/Π±ΠΈΡ‚ являСтся достаточно обоснованной для ΡΡƒΡ‰Π΅ΡΡ‚Π²ΡƒΡŽΡ‰ΠΈΡ… элСктронных устройств), Ρ‚ΠΎ ΠΌΠΎΠΆΠ½ΠΎ Π²Ρ‹Ρ‡ΠΈΡΠ»ΠΈΡ‚ΡŒ ΡΠΊΠΎΡ€ΠΎΡΡ‚ΡŒ выполнСния ΠΎΠΏΠ΅Ρ€Π°Ρ†ΠΈΠΉ. ΠŸΡ€Π΅Π΄ΡΡ‚Π°Π²Π»Π΅Π½Ρ‹ Π΄Π²Π° случая. ΠŸΠ΅Ρ€Π²Ρ‹ΠΉ ΠΈΠ· Π½ΠΈΡ… соотвСтствуСт ΠΏ = Ρ‚ = 32, ΠΏΡ€ΠΈ /=16 (I эквивалСнтно точности вычислСний), Π° Π²Ρ‚ΠΎΡ€ΠΎΠΉ случай относится ΠΊ ΠΏ = Ρ‚=128, / = 32. Π’ Ρ‚Π°Π±Π». 7.2 прСдставлСны Π°Π½Π°Π»ΠΎΠ³ΠΈΡ‡Π½Ρ‹Π΅ Π΄Π°Π½Π½Ρ‹Π΅ для ΡƒΠΌΠ½ΠΎΠΆΠΈΡ‚Π΅Π»Π΅ΠΉ, Π²Ρ‹ΠΏΠΎΠ»Π½ΡΡŽΡ‰ΠΈΡ… ΡƒΠΌΠ½ΠΎΠΆΠ΅Π½ΠΈΠ΅ ΠΌΠ°Ρ‚Ρ€ΠΈΡ†Ρ‹ Π½Π° ΠΌΠ°Ρ‚Ρ€ΠΈΡ†Ρƒ с Ρ‚ΠΎΡ‡Π½ΠΎΡΡ‚ΡŒΡŽ I Ρ†ΠΈΡ„Ρ€. Π’ Ρ‚Ρ€Π΅Ρ‚ΡŒΠ΅ΠΌ столбцС ΠΏΠΎΠΊΠ°Π·Π°Π½Ρ‹ Ρ€Π΅Π·ΡƒΠ»ΡŒΡ‚Π°Ρ‚Ρ‹ для /=16, ΠΏ = Ρ‚ = ΠΊ = 32, Π° Ρ‡Π΅Ρ‚Π²Π΅Ρ€Ρ‚Ρ‹ΠΉ столбСц соотвСтствуСт / = 32, ΠΏ = Ρ‚ = ΠΊ= 28. Π’ΠΎ всСх случаях Ρ€Π΅Π·ΡƒΠ»ΡŒΡ‚Π°Ρ‚Ρ‹ Π΄Π°Π½Ρ‹ для ΠΎΠΏΠ΅Ρ€Π°Ρ†ΠΈΠΉ с фиксированной запятой, выполняСмых Π² ΠΎΠ΄Π½Ρƒ сСкунду. Ни ΠΎΠ΄ΠΈΠ½ Ρ†ΠΈΡ„Ρ€ΠΎΠ²ΠΎΠΉ процСссор (оптичСский ΠΈΠ»ΠΈ элСкт-  [c.207]

ΠŸΡ€Π΅Π΄ΠΏΠΎΠ»ΠΎΠΆΠ΅Π½ΠΈΡ, ΠΊΠΎΡ‚ΠΎΡ€Ρ‹Π΅ ΠΌΠΎΠΆΠ½ΠΎ ΡΠ΄Π΅Π»Π°Ρ‚ΡŒ ΠΎ симмСтрии, Π² точности ΡΠΎΠ²ΠΏΠ°Π΄Π°ΡŽΡ‚ с прСдполоТСниями, дСлавшимися Π² ΠΏΡ€Π΅Π΄Ρ‹Π΄ΡƒΡ‰ΠΈΡ… Ρ€Π°Π·Π΄Π΅Π»Π°Ρ…. Π‘Π»ΠΎΠΆΠ΅Π½ΠΈΠ΅ ΠΌΠ°Ρ‚Ρ€ΠΈΡ† S нСпосрСдствСнно Π΄Π°Π΅Ρ‚ Π½ΠΈΠΆΠ΅ΡΠ»Π΅Π΄ΡƒΡŽΡ‰ΠΈΠ΅ Ρ€Π΅Π·ΡƒΠ»ΡŒΡ‚Π°Ρ‚Ρ‹, прСдставлСнныС здСсь наряду с качСствСнной ΠΈΠ½Ρ‚Π΅Ρ€ΠΏΡ€Π΅Ρ‚Π°Ρ†ΠΈΠ΅ΠΉ. ОбъСм этой ΠΊΠ½ΠΈΠ³ΠΈ Π½Π΅ допускаСт ΠΏΠΎΠ΄Ρ€ΠΎΠ±Π½ΠΎΠ³ΠΎ обсуТдСния этого вопроса. ВсС эффСкты ΠΌΠΎΠΆΠ½ΠΎ вывСсти ΠΈΠ· ΠΎΠ±Ρ‰Π΅ΠΉ Ρ„ΠΎΡ€ΠΌΡƒΠ»Ρ‹ распространСния, Π΄Π°Π½Π½ΠΎΠΉ Π² Ρ€Π°Π·Π΄. 4.41.  [c.73]

ΠœΠ°Ρ‚Ρ€ΠΈΡ†Π° тСплопроводности элСмСнта получаСтся слоТСниСм ΠΌΠ°Ρ‚Ρ€ΠΈΡ† (8.15) ΠΈ (8.17)  [c.138]

ΠœΠ°Ρ‚Ρ€ΠΈΡ†Π° [Π›] получаСтся слоТСниСм ΠΌΠ°Ρ‚Ρ€ΠΈΡ† [Π‘] ΠΈ [/Π‘]  [c.208]

ΠŸΡ€ΠΈΠΌΠ΅Ρ€ 3.1. Π’ Ρ€Π΅Π·ΡƒΠ»ΡŒΡ‚Π°Ρ‚Π΅ слоТСния ΠΌΠ°Ρ‚Ρ€ΠΈΡ† [Π°]= 3 8 1 И[Π’Π¬ 2 4-1  [c. 155]

Π‘Π»ΠΎΠΆΠ΅Π½ΠΈΠ΅ ΠΌΠ°Ρ‚Ρ€ΠΈΡ† ΠΎΠ±Π»Π°Π΄Π°Π΅Ρ‚ свойством коммутативности (ΠΎΡ‚ пСрСстановки ΠΌΠ°Ρ‚Ρ€ΠΈΡ† ΠΈΡ… сумма Π½Π΅ мСняСтся) ΠΈ подчиняСтся ΡΠΎΡ‡Π΅Ρ‚Π°Ρ‚Π΅Π»ΡŒΠ½ΠΎΠΌΡƒ Π·Π°ΠΊΠΎΠ½Ρƒ  [c.155]

Π”Ρ€ΡƒΠ³ΠΎΠΉ способ нахоТдСния ΠΎΠ±Ρ€Π°Ρ‚Π½ΠΎΠΉ ΠΌΠ°Ρ‚Ρ€ΠΈΡ†Ρ‹ Π·Π°ΠΊΠ»ΡŽΡ‡Π°Π΅Ρ‚ΡΡ Π² ΠΏΠΎΡΠ»Π΅Π΄ΠΎΠ²Π°Ρ‚Π΅Π»ΡŒΠ½ΠΎΠΌ вычислСнии ΠΎΠ±Ρ€Π°Ρ‚Π½ΠΎΠΉ ΠΌΠ°Ρ‚Ρ€ΠΈΡ†Ρ‹ ΠΏΡƒΡ‚Π΅ΠΌ ΠΏΠΎΠ΄Π±ΠΎΡ€Π° ΠΏΡ€ΠΈ ΠΏΠΎΠΌΠΎΡ‰ΠΈ Π΅Π΄ΠΈΠ½ΠΈΡ‡Π½ΠΎΠΉ ΠΌΠ°Ρ‚Ρ€ΠΈΡ†Ρ‹. Записав рядом с Π·Π°Π΄Π°Π½Π½ΠΎΠΉ ΠΌΠ°Ρ‚Ρ€ΠΈΡ†Π΅ΠΉ Π΅Π΄ΠΈΠ½ΠΈΡ‡Π½ΡƒΡŽ ΠΌΠ°Ρ‚Ρ€ΠΈΡ†Ρƒ, ΠΏΡ€ΠΎΠΈΠ·Π²ΠΎΠ΄ΠΈΠΌ слоТСниС строк, ΠΏΡ€Π΅Π΄Π²Π°Ρ€ΠΈΡ‚Π΅Π»ΡŒΠ½ΠΎ ΠΏΠΎΠ΄ΠΎΠ±Ρ€Π°Π² ΠΌΠ½ΠΎΠΆΠΈΡ‚Π΅Π»ΠΈ Ρ‚Π°ΠΊ, Ρ‡Ρ‚ΠΎΠ±Ρ‹ Π½Π° мСстС Π·Π°Π΄Π°Π½Π½ΠΎΠΉ ΠΌΠ°Ρ‚Ρ€ΠΈΡ†Ρ‹ ΠΏΠΎΠ»ΡƒΡ‡ΠΈΡ‚ΡŒ Π΅Π΄ΠΈΠ½ΠΈΡ‡Π½ΡƒΡŽ. Π’ этом случаС Π½Π° мСстС Π΅Π΄ΠΈΠ½ΠΈΡ‡Π½ΠΎΠΉ ΠΌΠ°Ρ‚Ρ€ΠΈΡ†Ρ‹ Π±ΡƒΠ΄Π΅Ρ‚ искомая обратная ΠΌΠ°Ρ‚Ρ€ΠΈΡ†Π°.  [c.158]

Π’Π΅ΠΌ Π½Π΅ ΠΌΠ΅Π½Π΅Π΅ слоТСниС ΠΌΠ°Ρ‚Ρ€ΠΈΡ† удовлСтворяСт всСм свойствам слоТСния чисСл. НапримСр, ΠΈΠ· А- -Π’ = А- Π‘ слСдуСт, Ρ‡Ρ‚ΠΎ Π’ = Π‘.  [c.273]

ΠŸΡ€ΠΈ слоТСнии Π΄Π²ΡƒΡ… ΠΌΠ°Ρ‚Ρ€ΠΈΡ† ΠΎΠ΄ΠΈΠ½Π°ΠΊΠΎΠ²ΠΎΠ³ΠΎ Ρ€Π°Π·ΠΌΠ΅Ρ€Π° получаСтся ΠΌΠ°Ρ‚Ρ€ΠΈΡ†Π° Ρ‚ΠΎΠ³ΠΎ ΠΆΠ΅ Ρ€Π°Π·ΠΌΠ΅Ρ€Π°, ΠΊΠ°ΠΆΠ΄Ρ‹ΠΉ элСмСнт ΠΊΠΎΡ‚ΠΎΡ€ΠΎΠΉ Ρ€Π°Π²Π΅ΠΈ суммС ΡΠΎΠΎΡ‚Π²Π΅Ρ‚ΡΡ‚Π²ΡƒΡŽΡ‰ΠΈΡ… элСмСнтов слагаСмых ΠΌΠ°Ρ‚Ρ€ΠΈΡ†.  [c.104]

Над ΠΌΠ°Ρ‚Ρ€ΠΈΡ†Π°ΠΌΠΈ ΠΌΠΎΠΆΠ½ΠΎ Π²Ρ‹ΠΏΠΎΠ»Π½ΡΡ‚ΡŒ дСйствия транспонирования, слоТСния, умноТСния. ΠœΠ°Ρ‚Ρ€ΠΈΡ†Π° А, транспонированная ΠΏΠΎ ΠΎΡ‚Π½ΠΎΡˆΠ΅Π½ΠΈΡŽ ΠΊ ΠΌΠ°Ρ‚Ρ€ΠΈΡ†Π΅ А, образуСтся ΠΈΠ· ΠΌΠ°Ρ‚Ρ€ΠΈΡ†Ρ‹ А Π·Π°ΠΌΠ΅Π½ΠΎΠΉ ΠΊΠ°ΠΆΠ΄ΠΎΠΉ Π΅Π΅ строки Π½Π° столбСц Ρ‚ΠΎΠ³ΠΎ ΠΆΠ΅ Π½ΠΎΠΌΠ΅Ρ€Π°. НапримСр, ΠΏΡ€ΠΈ транспонировании ΠΌΠ°Ρ‚Ρ€ΠΈΡ†Ρ‹  [c.50]

ΠœΠ½ΠΎΠΆΠ΅ΡΡ‚Π²ΠΎ J n всСх Π»-ΠΌΠ΅Ρ€Π½Ρ‹Ρ… Π²Π΅ΠΊΡ‚ΠΎΡ€ΠΎΠ² Π½Π°Π·Ρ‹Π²Π°ΡŽΡ‚ Π»ΠΈΠ½Π΅ΠΉΠ½Ρ‹ΠΌ алгСбраичСским пространством, Ссли Π² Π½Π΅ΠΌ ΠΎΠΏΡ€Π΅Π΄Π΅Π»Π΅Π½Ρ‹ ΠΎΠΏΠ΅Ρ€Π°Ρ†ΠΈΠΈ слоТСния ΠΈ умноТСния Π½Π° скаляр Ρ‚ΠΎΡ‡Π½ΠΎ Ρ‚Π°ΠΊ ΠΆΠ΅, ΠΊΠ°ΠΊ для ΠΌΠ°Ρ‚Ρ€ΠΈΡ†. Число я называСтся Ρ€Π°Π·ΠΌΠ΅Ρ€Π½ΠΎΡΡ‚ΡŒΡŽ пространства Rn- Рассмотрим ΠΏΠΎΠΌΠΈΠΌΠΎ Π²Π΅ΠΊΡ‚ΠΎΡ€Π° Π° Π΄Ρ€ΡƒΠ³ΠΎΠΉ ΠΏ-ΠΌΠ΅Ρ€Π½Ρ‹ΠΉ Π²Π΅ΠΊΡ‚ΠΎΡ€  [c.19]

ΠŸΡ€ΠΈ вычислСнии ΠΌΠ°Ρ‚Ρ€ΠΈΡ†Ρ‹ плотности ΠΌΡ‹ Π±ΡƒΠ΄Π΅ΠΌ ΠΏΡ€Π΅Π΄ΠΏΠΎΠ»Π°Π³Π°Ρ‚ΡŒ, Ρ‡Ρ‚ΠΎ рассСянныС Π²ΠΎΠ»Π½Ρ‹ ΠΈΠΌΠ΅ΡŽΡ‚ бСспорядочныС Ρ„Π°Π·Ρ‹ ΠΈ ΠΏΡ€ΠΈ слоТСнии Π² срСднСм Π΄Π°ΡŽΡ‚ Π½ΡƒΠ»ΡŒ. Если ΠΈ Π³ ΠΈ Π³ Π»Π΅ΠΆΠ°Ρ‚ слСва ΠΎΡ‚ слоя, Ρ‚ΠΎ  [c.718]

Π’Π°ΠΊΠΎΠΌ ΠΎΠ±Ρ€Π°Π·ΠΎΠΌ, для выполнСния Π°Π»Π³ΠΎΡ€ΠΈΡ‚ΠΌΠ° (55) Ρ‚Ρ€Π΅Π±ΡƒΡŽΡ‚ΡΡ Π΄Π²Π° прямых ΠΈ ΠΎΠ΄Π½ΠΎ ΠΎΠ±Ρ€Π°Ρ‚Π½ΠΎΠ΅ ΠΏΡ€Π΅ΠΎΠ±Ρ€Π°Π·ΠΎΠ²Π°Π½ΠΈΠ΅ Π€/Ρ€ΡŒΠ΅, Π° Ρ‚Π°ΠΊΠΆΠ΅ прямоС ΡƒΠΌΠ½ΠΎΠΆΠ΅Π½ΠΈΠ΅ ΠΌΠ°Ρ‚Ρ€ΠΈΡ†Ρ‹ Π½Π° ΠΌΠ°Ρ‚Ρ€ΠΈΡ†Ρƒ. Если Π² качСствС Π΄ΠΈΠΆΡ€Π΅Ρ‚Π½ΠΎΠ³ΠΎ прСобразования Π€ΡƒΡ€ΡŒΠ΅ ΠΈΡΠΏΠΎΠ»ΡŒΠ·ΠΎΠ²Π°Ρ‚ΡŒ Π°Π»Π³ΠΎΡ€ΠΈΡ‚ΠΌ Π‘ΠŸΠ€, число ΠΎΠΏΠ΅Ρ€Π° Π΄Π°ΠΉ слоТСния составит 2N og2 , Π° число ΠΎΠΏΠ΅Ρ€Π°Ρ†ΠΈΠΉ умноТСния -.  [c.63]

ΠœΠ°Ρ‚Ρ€ΠΈΡ†Π° ТСсткости (13) конструкции образуСтся слоТСниСм ΠΌΠ°Ρ‚Ρ€ΠΈΡ† ТСсткости Π±Π°Π»ΠΎΠΊ ΠΈ Π½Π°Π½Π΅Π»Π΅ΠΉ = Од + Оп- Для ΠΏΠ°Π½Π΅Π»Π΅ΠΉ  [c.483]

ΠšΠ²Π°Ρ‚Π΅Ρ€Π½ΠΈΠΎΠ½Π½ΠΎΠ΅ слоТСниС ΠΏΠΎΠ²ΠΎΡ€ΠΎΡ‚ΠΎΠ². Как ΠΈ Π² случаС ΠΌΠ°Ρ‚Ρ€ΠΈΡ†, слоТСнию ΠΏΠΎΠ²ΠΎΡ€ΠΎΡ‚ΠΎΠ² ΠΎΡ‚Π²Π΅Ρ‡Π°Π΅Ρ‚ ΠΏΡ€ΠΎΠΈΠ·Π²Π΅Π΄Π΅Π½ΠΈΠ΅ ΠΊΠ²Π°Ρ‚Π΅Ρ€Π½ΠΈΠΎΠ½ΠΎΠ², ΠΏΡ€ΠΈ этом активная ΠΈ пассивная Ρ‚ΠΎΡ‡ΠΊΠΈ зрСния Π½Π° прСобразования ΠΈΠΌΠ΅ΡŽΡ‚ сущСствСнныС отличия.  [c.46]


Π›ΠΈΠ½Π΅ΠΉΠ½ΠΎ-алгСбраичСскиС ΠΎΠΏΠ΅Ρ€Π°Ρ†ΠΈΠΈ, рассмотрСнныС Π² ΠΏΡ€Π΅Π΄Ρ‹Π΄ΡƒΡ‰Π΅ΠΌ ΠΏΠ°Ρ€Π°Π³Ρ€Π°Ρ„Π΅, ΠΌΠΎΠΆΠ½ΠΎ отнСсти ΠΊ Ρ‚Ρ€Π΅ΠΌ Ρ‚ΠΈΠΏΠ°ΠΌ, исходя ΠΈΠ· ΠΏΡ€ΠΈΠ½Ρ†ΠΈΠΏΠΎΠ² ΠΎΡ€Π³Π°Π½ΠΈΠ·Π°Ρ†ΠΈΠΈ Ρ€Π΅Π°Π»ΠΈΠ·ΡƒΡŽΡ‰ΠΈΡ… ΠΈΡ… Π²Ρ‹Ρ‡ΠΈΡΠ»ΠΈΡ‚Π΅Π»ΡŒΠ½Ρ‹Ρ… процСссов. К Ρ‚ΠΈΠΏΡƒ ΠΊΠΎΡ€ΠΎΡ‚ΠΊΠΈΡ… отнСсСм ΠΎΠΏΠ΅Ρ€Π°Ρ†ΠΈΠΈ, для Ρ€Π΅Π°Π»ΠΈΠ·Π°Ρ†ΠΈΠΈ ΠΊΠΎΡ‚ΠΎΡ€Ρ‹Ρ… ΠΏΡ€ΠΈΠ½Ρ†ΠΈΠΏΠΈΠ°Π»ΡŒΠ½ΠΎ достаточСн ΠΎΠ΄Π½ΠΎΠΊΡ€Π°Ρ‚Π½Ρ‹ΠΉ ΠΎΠ±ΠΌΠ΅Π½ Ρ„Π°ΠΉΠ»ΠΎΠ², содСрТащих ΠΎΠΏΠ΅Ρ€Π°Π½Π΄Ρ‹ ΠΈ Ρ€Π΅Π·ΡƒΠ»ΡŒΡ‚Π°Ρ‚ слоТСниС ΠΌΠ°Ρ‚Ρ€ΠΈΡ† ΠΈ Π²Π΅ΠΊΡ‚ΠΎΡ€ΠΎΠ², ΡƒΠΌΠ½ΠΎΠΆΠ΅Π½ΠΈΠ΅ Π½Π° скаляр ΠΈ Ρ‚. ΠΏ. К Ρ‚ΠΈΠΏΡƒ Π΄Π»ΠΈΠ½Π½Ρ‹Ρ… отнСсСм ΠΎΠΏΠ΅Ρ€Π°Ρ†ΠΈΠΈ, для Ρ€Π΅Π°Π»ΠΈΠ·Π°Ρ†ΠΈΠΈ ΠΊΠΎΡ‚ΠΎΡ€Ρ‹Ρ… ΠΏΡ€ΠΈΠ½Ρ†ΠΈΠΏΠΈΠ°Π»ΡŒΠ½ΠΎ трСбуСтся ΠΌΠ½ΠΎΠ³ΠΎΠΊΡ€Π°Ρ‚Π½Ρ‹ΠΉ ΠΎΠ±ΠΌΠ΅Π½ ΠΎΠ΄Π½ΠΎΠ³ΠΎ Ρ„Π°ΠΉΠ»Π°, содСрТащСго ΠΎΠΏΠ΅Ρ€Π°Π½Π΄ ΠΈΠ»ΠΈ Ρ€Π΅Π·ΡƒΠ»ΡŒΡ‚Π°Ρ‚ транспонированиС, ΠΎΠ±Ρ€Π°Ρ‰Π΅Π½ΠΈΠ΅ ΠΌΠ°Ρ‚Ρ€ΠΈΡ†Ρ‹, ΡƒΠΌΠ½ΠΎΠΆΠ΅Π½ΠΈΠ΅ Π΄Π²ΡƒΡ… ΠΌΠ°Ρ‚Ρ€ΠΈΡ† ΠΈ Ρ‚. ΠΏ., Π² случаС ΠΊΠΎΠ³Π΄Π° ΠΎΠΏΠ΅Ρ€Π°Π½Π΄Ρ‹ ΠΈ Ρ€Π΅Π·ΡƒΠ»ΡŒΡ‚Π°Ρ‚ Π½Π΅ Ρ€Π°Π·ΠΌΠ΅Ρ‰Π°ΡŽΡ‚ΡΡ Ρ†Π΅Π»ΠΈΠΊΠΎΠΌ Π² ΠžΠ—Π£. К Ρ‚ΠΈΠΏΡƒ условнокоротких отнСсСм ΠΎΠΏΠ΅Ρ€Π°Ρ†ΠΈΠΈ, для Ρ€Π΅Π°Π»ΠΈΠ·Π°Ρ†ΠΈΠΈ ΠΊΠΎΡ‚ΠΎΡ€Ρ‹Ρ… ΠΏΡ€ΠΈ Π½Π΅ΠΊΠΎΡ‚ΠΎΡ€Ρ‹Ρ… Π΄ΠΎΠΏΠΎΠ»Π½ΠΈΡ‚Π΅Π»ΡŒΠ½Ρ‹Ρ… условиях достаточСн ΠΎΠ΄Π½ΠΎΠΊΡ€Π°Ρ‚Π½Ρ‹ΠΉ ΠΎΠ±ΠΌΠ΅Π½ Ρ„Π°ΠΉΠ»ΠΎΠ². Π’ основном, это вСсьма распространСнная Π² АБУ опСрация умноТСния ΠΌΠ°Ρ‚Ρ€ΠΈΡ†Ρ‹ Π½Π° Π²Π΅ΠΊΡ‚ΠΎΡ€, ΠΊΠΎΠ³Π΄Π° ΠΎΠΏΠ΅Ρ€Π°Π½Π΄Ρ‹ ΠΈ Ρ€Π΅Π·ΡƒΠ»ΡŒΡ‚Π°Ρ‚ Π½Π΅ Ρ€Π°Π·ΠΌΠ΅Ρ‰Π°ΡŽΡ‚ΡΡ Π² ΠžΠ—Π£. Π’ ΠΎΠ±Ρ‰Π΅ΠΌ случаС эта опСрация Π²Ρ‹ΠΏΠΎΠ»Π½ΠΈΠΌΠ° ΠΏΠΎ Π°Π»Π³ΠΎΡ€ΠΈΡ‚ΠΌΡƒ умноТСния Π΄Π²ΡƒΡ… ΠΌΠ°Ρ‚Ρ€ΠΈΡ†. Однако, Ссли ΠΌΠ°Ρ‚Ρ€ΠΈΡ†Π° упорядочСна Ρ‚Π°ΠΊ, Ρ‡Ρ‚ΠΎ ΡΡ‚Π°Ρ€ΡˆΠΈΠΉ индСкс Π΅Π΅ элСмСнтов являСтся индСксом, Ρ€Π°Π·Π»ΠΈΡ‡Π°ΡŽΡ‰ΠΈΠΌ элСмСнты Π²Π΅ΠΊΡ‚ΠΎΡ€Π°, Ρ‚ΠΎ эта опСрация рСализуСтся ΠΎΠ΄Π½ΠΎΠΊΡ€Π°Ρ‚Π½Ρ‹ΠΌ ΠΎΠ±ΠΌΠ΅Π½ΠΎΠΌ. Π’Π°ΠΊΠΈΠΌ ΠΎΠ±Ρ€Π°Π·ΠΎΠΌ, ΠΏΡ€ΠΈ Π΄ΠΎΠΏΠΎΠ»Π½ΠΈΡ‚Π΅Π»ΡŒΠ½ΠΎΠΌ условии β€” ΠΏΡ€ΠΈ совпадСнии упорядочСнностСй элСмСнтов ΠΌΠ°Ρ‚Ρ€ΠΈΡ†Ρ‹ ΠΈ Π²Π΅ΠΊΡ‚ΠΎΡ€Π° β€” эта опСрация являСтся ΠΊΠΎΡ€ΠΎΡ‚ΠΊΠΎΠΉ. Π‘Π΅Π· этого Π΄ΠΎΠΏΠΎΠ»Π½ΠΈΡ‚Π΅Π»ΡŒΠ½ΠΎΠ³ΠΎ условия опСрация являСтся Π΄Π»ΠΈΠ½Π½ΠΎΠΉ, Ρ‚Π°ΠΊ ΠΊΠ°ΠΊ Π² этом случаС ΠΎΠ½Π° выполняСтся Π»ΠΈΠ±ΠΎ ΠΊΠ°ΠΊ ΡƒΠΌΠ½ΠΎΠΆΠ΅Π½ΠΈΠ΅ Π΄Π²ΡƒΡ… ΠΌΠ°Ρ‚Ρ€ΠΈΡ†, Π»ΠΈΠ±ΠΎ (Ρ‡Ρ‚ΠΎ ΠΊΠΎΡ€ΠΎΡ‡Π΅) Π² Π΄Π²Π΅ стадии сначала выполняСтся транспонированиС ΠΌΠ°Ρ‚Ρ€ΠΈΡ†Ρ‹, Π·Π°Ρ‚Π΅ΠΌ собствСнно ΡƒΠΌΠ½ΠΎΠΆΠ΅Π½ΠΈΠ΅, Π½ΠΎ ΠΏΡ€ΠΈ ΠΎΠ΄Π½ΠΎΠΊΡ€Π°Ρ‚Π½ΠΎΠΌ ΠΎΠ±ΠΌΠ΅Π½Π΅.  [c. 77]

РассмотрСниС Π΄Π°Π½Π½ΠΎΠ³ΠΎ ΠΏΡ€ΠΈΠΌΠ΅Ρ€Π° Π±Ρ‹Π»ΠΎ Π²Ρ‹Π·Π²Π°Π½ΠΎ Π½Π΅ΠΎΠ±Ρ…ΠΎΠ΄ΠΈΠΌΠΎΡΡ‚ΡŒΡŽ ΠΎΠ±Π΅ΡΠΏΠ΅Ρ‡ΠΈΡ‚ΡŒ Π²Ρ‹ΡΠΎΠΊΡƒΡŽ ΡΠΊΠΎΡ€ΠΎΡΡ‚ΡŒ ΠΏΡ€ΠΈ Π²Ρ‹ΠΏΠΎΠ»Π½Π΅Π½ΠΈΠΈ ΠΎΠΏΠ΅Ρ€Π°Ρ†ΠΈΠΈ Π²Π½ΡƒΡ‚Ρ€Π΅Π½Π½Π΅Π³ΠΎ произвСдСния Π² Π»ΠΈΠ½Π΅ΠΉΠ½ΠΎΠΉ Π°Π»Π³Π΅Π±Ρ€Π΅ (Π½Π°ΠΏΡ€ΠΈΠΌΠ΅Ρ€, для умноТСния ΠΌΠ°Ρ‚Ρ€ΠΈΡ†Ρ‹ Π½Π° Π²Π΅ΠΊΡ‚ΠΎΡ€ ΠΈΠ»ΠΈ ΠΌΠ°Ρ‚Ρ€ΠΈΡ†Ρ‹ Π½Π° ΠΌΠ°Ρ‚Ρ€ΠΈΡ†Ρƒ), Π² ΠΏΡ€ΠΎΡ‚ΠΈΠ²Π½ΠΎΠΌ случаС эти ΠΎΠΏΠ΅Ρ€Π°Ρ†ΠΈΠΈ становятся бСссыслСнными. ΠžΠΏΠ΅Ρ€Π°Ρ†ΠΈΠΈ Π²Π½ΡƒΡ‚Ρ€Π΅Π½Π½Π΅Π³ΠΎ произвСдСния Π²ΠΊΠ»ΡŽΡ‡Π°ΡŽΡ‚ ΡƒΠΌΠ½ΠΎΠΆΠ΅Π½ΠΈΠ΅ Π΄Π²ΡƒΡ… чисСл ΠΈ слоТСниС Ρ€Π΅Π·ΡƒΠ»ΡŒΡ‚Π°Ρ‚Π° с Ρ‚Ρ€Π΅Ρ‚ΡŒΠΈΠΌ числом. НапримСр, 2-разрядный ΡƒΠΌΠ½ΠΎΠΆΠΈΡ‚Π΅Π»ΡŒ-сумматор ΡƒΠΌΠ½ΠΎΠΆΠ°Π΅Ρ‚ Π΄Π²Π° 2-Ρ€Π°Π·-рядных числа М Ρ† Π«, прибавляСт Ρ€Π΅Π·ΡƒΠ»ΡŒΡ‚Π°Ρ‚ ΠΊ 5-разрядному Π²Ρ…ΠΎΠ΄Π½ΠΎΠΌΡƒ числу X ΠΈ Π²Ρ‹Π²ΠΎΠ΄ΠΈΡ‚ Ρ€Π΅Π·ΡƒΠ»ΡŒΡ‚Π°Ρ‚Ρ‹ Π² Π²ΠΈΠ΄Π΅ 5-разрядного числа Π£. Π’ синхронизированном Ρ€Π΅ΠΆΠΈΠΌΠ΅ Ρ€Π°Π±ΠΎΡ‚Ρ‹ Π²Ρ‹Ρ…ΠΎΠ΄Π½ΠΎΠΉ сигнал Π£ ΠΌΠΎΠ³ Π±Ρ‹ ΠΏΠΎΠ΄Π°Π²Π°Ρ‚ΡŒΡΡ ΠΏΠΎ Ρ†Π΅ΠΏΠΈ ΠΎΠ±Ρ€Π°Ρ‚Π½ΠΎΠΉ связи Π½Π° Π²Ρ…ΠΎΠ΄ X для Ρ‚ΠΎΠ³ΠΎ, Ρ‡Ρ‚ΠΎΠ±Ρ‹ Π΄ΠΎΡΡ‚ΠΈΡ‡ΡŒ эффСкта ΠΌΠ½ΠΎΠ³ΠΎΠΊΡ€Π°Ρ‚Π½ΠΎΠ³ΠΎ накоплСния Ρ€Π΅Π·ΡƒΠ»ΡŒΡ‚Π°Ρ‚Π° (Ссли имССтся Π²ΠΎΠ·ΠΌΠΎΠΆΠ½ΠΎΡΡ‚ΡŒ накоплСния Π΄ΠΎ Ρ‚Ρ€Π΅Ρ… ΠΏΡ€ΠΎΠΈΠ·Π²Π΅Π΄Π΅Π½ΠΈΠΉ ΠΈ ΠΏΡ€ΠΈ этом Π½Π΅ Π²ΠΎΠ·Π½ΠΈΠΊΠ°Π΅Ρ‚ ΠΏΠ΅Ρ€Π΅ΠΏΠΎΠ»Π½Π΅Π½ΠΈΠ΅).  [c.155]

БыстродСйствиС оптичСских ΠΌΠ°Ρ‚Ρ€ΠΈΡ‡Π½Ρ‹Ρ… ΡƒΠΌΠ½ΠΎΠΆΠΈΡ‚Π΅Π»Π΅ΠΉ Π±Ρ‹Π»ΠΎ ΡƒΠΆΠ΅ описано Π² Ρ‚Π°Π±Π». 7.1 ΠΈ 7.2. ΠŸΡ€ΠΈ вычислСнии ΠΎΡ‚Π½ΠΎΡˆΠ΅Π½ΠΈΡ ΠŸΡΠ°Π»Ρ‚ΠΈΡΠ° числа Π² Ρ‚Π°Π±Π». 7.1 ΠΈ 7.2 ΠΈΠΌΠ΅ΡŽΡ‚ коэффициСнт запаса 2, ΠΏΠΎΡΠΊΠΎΠ»ΡŒΠΊΡƒ Π±Ρ‹Π»ΠΈ ΡƒΡ‡Ρ‚Π΅Π½Ρ‹ ΠΎΠΏΠ΅Ρ€Π°Ρ†ΠΈΠΈ умноТСния ΠΈ слоТСния. Для ΡƒΠΌΠ½ΠΎΠΆΠΈΡ‚Π΅Π»Π΅ΠΉ ΠΌΠ°Ρ‚Ρ€ΠΈΡ† Π½Π° Π²Π΅ΠΊΡ‚ΠΎΡ€Ρ‹ ΡΠΎΠΎΡ‚Π½ΠΎΡˆΠ΅Π½ΠΈΠ΅ ΠŸΡΠ°Π»Ρ‚ΠΈΡΠ° ΠΏΠΎΠΊΠ°Π·Π°Π½ΠΎ Π² Ρ‚Π°Π±Π». 7.3. Π£ΠΌΠ½ΠΎΠΆΠΈΡ‚Π΅Π»ΠΈ ΠΌΠ°Ρ‚Ρ€ΠΈΡ†Ρ‹ Π½Π° ΠΌΠ°Ρ‚Ρ€ΠΈΡ†Ρ‹ ΠΏΠΎΠΊΠ°Π·Π°Π½Ρ‹ Π² Ρ‚Π°Π±Π». 7.4. ВычислСния ΠΏΡ€ΠΎΠ²Π΅Π΄Π΅Π½Ρ‹ для Ρ‚Π΅Ρ… ΠΆΠ΅ случаСв, Ρ‡Ρ‚ΠΎ ΠΈ Π² Ρ‚Π°Π±Π». 7.1 ΠΈ 7.2. Π’Ρ‚ΠΎΡ€ΠΎΠΉ столбСц Ρ‚Π°Π±Π». 7.3 ΠΏΡ€Π΅Π΄ΠΏΠΎΠ»Π°Π³Π°Π΅Ρ‚ Π·Π½Π°Ρ‡Π΅Π½ΠΈΠ΅ / = 16, ΠΏ=Ρ‚ = 32, Π² Ρ‚ΠΎ врСмя ΠΊΠ°ΠΊ для Ρ‚Ρ€Π΅Ρ‚ΡŒΠ΅Π³ΠΎ столбца /=32, ΠΏ = Ρ‚= 28. Π’ Ρ‚Π°Π±Π». 7.4 Π²Ρ‚ΠΎΡ€ΠΎΠΉ столбСц соотвСтствуСт значСниям /=16, n = m = = 32, Π° Ρ‚Ρ€Π΅Ρ‚ΠΈΠΉ столбСц относится ΠΊ / = 32, ΠΏ=Ρ‚ = k=l28.  [c.210]


Π”ΠΎΠ±Π°Π²Π»Π΅Π½ΠΈΠ΅ ΠΌΠ°Ρ‚Ρ€ΠΈΡ†Ρ‹

Как ΠΈ Π² ΠΎΠ±Ρ‹Ρ‡Π½ΠΎΠΉ Π°Π»Π³Π΅Π±Ρ€Π΅, матричная Π°Π»Π³Π΅Π±Ρ€Π° ΠΈΠΌΠ΅Π΅Ρ‚ Ρ‚Π°ΠΊΠΈΠ΅ ΠΎΠΏΠ΅Ρ€Π°Ρ†ΠΈΠΈ, ΠΊΠ°ΠΊ слоТСниС ΠΈ Π²Ρ‹Ρ‡ΠΈΡ‚Π°Π½ΠΈΠ΅.

Как ΡΠΊΠ»Π°Π΄Ρ‹Π²Π°Ρ‚ΡŒ ΠΈ Π²Ρ‹Ρ‡ΠΈΡ‚Π°Ρ‚ΡŒ ΠΌΠ°Ρ‚Ρ€ΠΈΡ†Ρ‹

Π”Π²Π΅ ΠΌΠ°Ρ‚Ρ€ΠΈΡ†Ρ‹ ΠΌΠΎΠΆΠ½ΠΎ ΡΠΊΠ»Π°Π΄Ρ‹Π²Π°Ρ‚ΡŒ ΠΈΠ»ΠΈ Π²Ρ‹Ρ‡ΠΈΡ‚Π°Ρ‚ΡŒ Ρ‚ΠΎΠ»ΡŒΠΊΠΎ Π² Ρ‚ΠΎΠΌ случаС, Ссли ΠΎΠ½ΠΈ ΠΈΠΌΠ΅ΡŽΡ‚ ΠΎΠ΄ΠΈΠ½Π°ΠΊΠΎΠ²Ρ‹Π΅ ΠΈΠ·ΠΌΠ΅Ρ€Π΅Π½ΠΈΠ΅; Ρ‚ΠΎ Π΅ΡΡ‚ΡŒ Π² Π½ΠΈΡ… Π΄ΠΎΠ»ΠΆΠ½ΠΎ Π±Ρ‹Ρ‚ΡŒ ΠΎΠ΄ΠΈΠ½Π°ΠΊΠΎΠ²ΠΎΠ΅ количСство строк ΠΈ столбцов.

Π‘Π»ΠΎΠΆΠ΅Π½ΠΈΠ΅ ΠΈΠ»ΠΈ Π²Ρ‹Ρ‡ΠΈΡ‚Π°Π½ΠΈΠ΅ осущСствляСтся ΠΏΡƒΡ‚Π΅ΠΌ слоТСния ΠΈΠ»ΠΈ вычитания. ΡΠΎΠΎΡ‚Π²Π΅Ρ‚ΡΡ‚Π²ΡƒΡŽΡ‰ΠΈΠ΅ элСмСнты. НапримСр, рассмотрим ΠΌΠ°Ρ‚Ρ€ΠΈΡ†Ρƒ A ΠΈ ΠΌΠ°Ρ‚Ρ€ΠΈΡ†Π° B .

ОбС ΠΌΠ°Ρ‚Ρ€ΠΈΡ†Ρ‹ ΠΈΠΌΠ΅ΡŽΡ‚ ΠΎΠ΄ΠΈΠ½Π°ΠΊΠΎΠ²ΠΎΠ΅ количСство строк ΠΈ столбцов (2 строки ΠΈ 3 столбца), поэтому ΠΈΡ… ΠΌΠΎΠΆΠ½ΠΎ ΡΠΊΠ»Π°Π΄Ρ‹Π²Π°Ρ‚ΡŒ ΠΈ Π²Ρ‹Ρ‡ΠΈΡ‚Π°Ρ‚ΡŒ. Π’Π°ΠΊΠΈΠΌ ΠΎΠ±Ρ€Π°Π·ΠΎΠΌ,

A + B =
1 + 5 2 + 6 3 + 7
7 + 3 8 + 4 9 + 5

А,

A B =
1–5 2–6 3–7
7 — 3 8–4 9–5

И, Π½Π°ΠΊΠΎΠ½Π΅Ρ†, ΠΎΠ±Ρ€Π°Ρ‚ΠΈΡ‚Π΅ Π²Π½ΠΈΠΌΠ°Π½ΠΈΠ΅, Ρ‡Ρ‚ΠΎ порядок добавлСния ΠΌΠ°Ρ‚Ρ€ΠΈΡ† Π½Π΅ Π²Π°ΠΆΠ½Ρ‹ΠΉ; Ρ‚Π°ΠΊΠΈΠΌ ΠΎΠ±Ρ€Π°Π·ΠΎΠΌ, A + B = Π’ + А .

ΠŸΡ€ΠΎΠ²Π΅Ρ€ΡŒΡ‚Π΅ своС ΠΏΠΎΠ½ΠΈΠΌΠ°Π½ΠΈΠ΅

ΠŸΡ€ΠΎΠ±Π»Π΅ΠΌΠ° 1

Рассмотрим прСдставлСнныС Π½ΠΈΠΆΠ΅ ΠΌΠ°Ρ‚Ρ€ΠΈΡ†Ρ‹ — A , B , C ΠΈ D

КакиС ΠΈΠ· ΡΠ»Π΅Π΄ΡƒΡŽΡ‰ΠΈΡ… ΡƒΡ‚Π²Π΅Ρ€ΠΆΠ΄Π΅Π½ΠΈΠΉ Π²Π΅Ρ€Π½Ρ‹?

I. A + B = C
II. B + C = D
III. B C = D

(A) Ρ‚ΠΎΠ»ΡŒΠΊΠΎ я Волько
(B) II Волько
(C) III
(D) I ΠΈ II
(E) I ΠΈ III

РСшСниС

ΠŸΡ€Π°Π²ΠΈΠ»ΡŒΠ½Ρ‹ΠΉ ΠΎΡ‚Π²Π΅Ρ‚ — (C), ΠΊΠ°ΠΊ ΠΏΠΎΠΊΠ°Π·Π°Π½ΠΎ Π½ΠΈΠΆΠ΅.


B C = = D

ΠžΠ±Ρ€Π°Ρ‚ΠΈΡ‚Π΅ Π²Π½ΠΈΠΌΠ°Π½ΠΈΠ΅, Ρ‡Ρ‚ΠΎ ΠΌΠ°Ρ‚Ρ€ΠΈΡ†Ρ‹ A ΠΈ B нСльзя Π΄ΠΎΠ±Π°Π²ΠΈΡ‚ΡŒ, ΠΏΠΎΡ‚ΠΎΠΌΡƒ Ρ‡Ρ‚ΠΎ B ΠΈΠΌΠ΅Π΅Ρ‚ большС столбцов, Ρ‡Π΅ΠΌ A . ΠœΠ°Ρ‚Ρ€ΠΈΡ†Ρ‹ ΠΌΠΎΠΆΠ½ΠΎ ΡΠΊΠ»Π°Π΄Ρ‹Π²Π°Ρ‚ΡŒ ΠΈΠ»ΠΈ Π²Ρ‹Ρ‡ΠΈΡ‚Π°Ρ‚ΡŒ Ρ‚ΠΎΠ»ΡŒΠΊΠΎ Π² Ρ‚ΠΎΠΌ случаС, Ссли ΠΎΠ½ΠΈ ΠΈΠΌΠ΅ΡŽΡ‚ ΠΎΠ΄ΠΈΠ½Π°ΠΊΠΎΠ²Ρ‹Π΅ количСство строк ΠΈ Ρ‚Π°ΠΊΠΎΠ΅ ΠΆΠ΅ количСство столбцов.

ΠœΠ°Ρ‚Ρ€ΠΈΡ†Ρ‹ слоТСния ΠΈ вычитания — ChiliMath

Π’ этом ΡƒΡ€ΠΎΠΊΠ΅ я ΠΏΠΎΠ΄Π³ΠΎΡ‚ΠΎΠ²ΠΈΠ» сСмь (7) Ρ€Π°Π±ΠΎΡ‡ΠΈΡ… ΠΏΡ€ΠΈΠΌΠ΅Ρ€ΠΎΠ², Ρ‡Ρ‚ΠΎΠ±Ρ‹ ΠΏΡ€ΠΎΠΈΠ»Π»ΡŽΡΡ‚Ρ€ΠΈΡ€ΠΎΠ²Π°Ρ‚ΡŒ Π±Π°Π·ΠΎΠ²Ρ‹ΠΉ ΠΏΠΎΠ΄Ρ…ΠΎΠ΄ ΠΊ Ρ‚ΠΎΠΌΡƒ, ΠΊΠ°ΠΊ Π»Π΅Π³ΠΊΠΎ ΡΠΊΠ»Π°Π΄Ρ‹Π²Π°Ρ‚ΡŒ ΠΈΠ»ΠΈ Π²Ρ‹Ρ‡ΠΈΡ‚Π°Ρ‚ΡŒ ΠΌΠ°Ρ‚Ρ€ΠΈΡ†Ρ‹.

Если Π²Ρ‹ Π·Π½Π°Π΅Ρ‚Π΅, ΠΊΠ°ΠΊ ΡΠΊΠ»Π°Π΄Ρ‹Π²Π°Ρ‚ΡŒ ΠΈ Π²Ρ‹Ρ‡ΠΈΡ‚Π°Ρ‚ΡŒ Π΄Π΅ΠΉΡΡ‚Π²ΠΈΡ‚Π΅Π»ΡŒΠ½Ρ‹Π΅ числа, эта Ρ‚Π΅ΠΌΠ° Π΄ΠΎΠ»ΠΆΠ½Π° Π±Ρ‹Ρ‚ΡŒ Π΄Π΅ΠΉΡΡ‚Π²ΠΈΡ‚Π΅Π»ΡŒΠ½ΠΎ Π»Π΅Π³ΠΊΠΎΠΉ. ЕдинствСнноС, Ρ‡Ρ‚ΠΎ трСбуСтся для «лСгального» выполнСния ΠΎΠΏΠ΅Ρ€Π°Ρ†ΠΈΠΉ слоТСния ΠΈΠ»ΠΈ вычитания Π² Β«ΠΌΠΈΡ€Π΅Β» ΠΌΠ°Ρ‚Ρ€ΠΈΡ†, — это ΡƒΠ±Π΅Π΄ΠΈΡ‚ΡŒΡΡ, Ρ‡Ρ‚ΠΎ Π΄Π°Π½Π½Ρ‹Π΅ ΠΌΠ°Ρ‚Ρ€ΠΈΡ†Ρ‹ Π΄ΠΎΠ»ΠΆΠ½Ρ‹ ΠΈΠΌΠ΅Ρ‚ΡŒ ΠΎΠ΄ΠΈΠ½Π°ΠΊΠΎΠ²Ρ‹ΠΉ Ρ€Π°Π·ΠΌΠ΅Ρ€ ΠΈΠ»ΠΈ Ρ€Π°Π·ΠΌΠ΅Ρ€Π½ΠΎΡΡ‚ΡŒ.

Π§Ρ‚ΠΎ ΠΎΠ·Π½Π°Ρ‡Π°Π΅Ρ‚, Ρ‡Ρ‚ΠΎ данная ΠΌΠ°Ρ‚Ρ€ΠΈΡ†Π° ΠΈΠΌΠ΅Π΅Ρ‚ ΠΎΠ΄ΠΈΠ½Π°ΠΊΠΎΠ²Ρ‹ΠΉ Ρ€Π°Π·ΠΌΠ΅Ρ€ ΠΈΠ»ΠΈ Ρ€Π°Π·ΠΌΠ΅Ρ€Π½ΠΎΡΡ‚ΡŒ?

ΠŸΡ€Π΅Π΄ΠΏΠΎΠ»ΠΎΠΆΠΈΠΌ, Π½Π°ΠΌ Π΄Π°Π½Ρ‹ ΠΌΠ°Ρ‚Ρ€ΠΈΡ†Ρ‹ A ΠΈ B. Они ΠΈΠΌΠ΅ΡŽΡ‚ ΠΎΠ΄ΠΈΠ½Π°ΠΊΠΎΠ²Ρ‹ΠΉ Ρ€Π°Π·ΠΌΠ΅Ρ€ ΠΈΠ»ΠΈ Ρ€Π°Π·ΠΌΠ΅Ρ€, ΠΏΠΎΡ‚ΠΎΠΌΡƒ Ρ‡Ρ‚ΠΎ Ρƒ Π½ΠΈΡ… ΠΎΠ΄ΠΈΠ½Π°ΠΊΠΎΠ²ΠΎΠ΅ количСство строк ΠΈ столбцов.

ΠœΡ‹ ΠΌΠΎΠΆΠ΅ΠΌ ΠΎΠΏΠΈΡΠ°Ρ‚ΡŒ Ρ€Π°Π·ΠΌΠ΅Ρ€ ΠΈΠ»ΠΈ Ρ€Π°Π·ΠΌΠ΅Ρ€Π½ΠΎΡΡ‚ΡŒ ΠΌΠ°Ρ‚Ρ€ΠΈΡ†Ρ‹, ΠΈΡΠΏΠΎΠ»ΡŒΠ·ΡƒΡ ΡΠ»Π΅Π΄ΡƒΡŽΡ‰ΠΈΠΉ стандартный Ρ„ΠΎΡ€ΠΌΠ°Ρ‚:

количСство строк x количСство столбцов

ΠŸΠΎΠ·Π²ΠΎΠ»ΡŒΡ‚Π΅ ΠΌΠ½Π΅ ΠΏΠΎΠΊΠ°Π·Π°Ρ‚ΡŒ Π²Π°ΠΌ нСсколько примСров…

ПослСдняя ΠΌΠ°Ρ‚Ρ€ΠΈΡ†Π° Ρ€Π°Π·ΠΌΠ΅Ρ€ΠΎΠΌ 5 x 5 Ρ‚Π°ΠΊΠΆΠ΅ считаСтся Β«ΠΊΠ²Π°Π΄Ρ€Π°Ρ‚Π½ΠΎΠΉ ΠΌΠ°Ρ‚Ρ€ΠΈΡ†Π΅ΠΉΒ», ΠΏΠΎΡ‚ΠΎΠΌΡƒ Ρ‡Ρ‚ΠΎ количСство строк ΠΈ количСство столбцов Ρ€Π°Π²Π½Ρ‹. Π’Π°ΠΆΠ½ΠΎ Π·Π½Π°Ρ‚ΡŒ, Ρ‡Ρ‚ΠΎ для Ρ‚ΠΎΠ³ΠΎ, Ρ‡Ρ‚ΠΎΠ±Ρ‹ любая заданная ΠΌΠ°Ρ‚Ρ€ΠΈΡ†Π° ΠΈΠΌΠ΅Π»Π° ΠΎΠ±Ρ€Π°Ρ‚Π½ΡƒΡŽ, ΠΎΠ½Π° Π΄ΠΎΠ»ΠΆΠ½Π° Π±Ρ‹Ρ‚ΡŒ ΠΊΠ²Π°Π΄Ρ€Π°Ρ‚Π½ΠΎΠΉ ΠΌΠ°Ρ‚Ρ€ΠΈΡ†Π΅ΠΉ. Π― Π½Π΅ Π³ΠΎΠ²ΠΎΡ€ΡŽ, Ρ‡Ρ‚ΠΎ всС ΠΊΠ²Π°Π΄Ρ€Π°Ρ‚Π½Ρ‹Π΅ ΠΌΠ°Ρ‚Ρ€ΠΈΡ†Ρ‹ ΠΈΠΌΠ΅ΡŽΡ‚ ΠΎΠ±Ρ€Π°Ρ‚Π½Ρ‹Π΅, Π½ΠΎ ΠΏΠ΅Ρ€Π²ΠΎΠ΅ Ρ‚Ρ€Π΅Π±ΠΎΠ²Π°Π½ΠΈΠ΅ ΠΊ ΠΌΠ°Ρ‚Ρ€ΠΈΡ†Π΅, Ρ‡Ρ‚ΠΎΠ±Ρ‹ ΠΈΠΌΠ΅Ρ‚ΡŒ ΠΎΠ±Ρ€Π°Ρ‚Π½ΡƒΡŽ ΠΌΠ°Ρ‚Ρ€ΠΈΡ†Ρƒ, — это Ρ‚ΠΎ, Ρ‡Ρ‚ΠΎ сначала ΠΎΠ½Π° Π΄ΠΎΠ»ΠΆΠ½Π° Π±Ρ‹Ρ‚ΡŒ ΠΊΠ²Π°Π΄Ρ€Π°Ρ‚Π½ΠΎΠΉ ΠΌΠ°Ρ‚Ρ€ΠΈΡ†Π΅ΠΉ.

ΠžΠ·Π½Π°ΠΊΠΎΠΌΡŒΡ‚Π΅ΡΡŒ с ΠΌΠΎΠΈΠΌ ΠΎΡ‚Π΄Π΅Π»ΡŒΠ½Ρ‹ΠΌ руководством ΠΎ Ρ‚ΠΎΠΌ, ΠΊΠ°ΠΊ Π½Π°ΠΉΡ‚ΠΈ ΠΎΠ±Ρ€Π°Ρ‚Π½ΡƒΡŽ ΠΌΠ°Ρ‚Ρ€ΠΈΡ†Ρƒ 2 Γ— 2.

Π― Π΄ΠΎΠ»ΠΆΠ΅Π½ ΠΏΠΎΠ΄Ρ‡Π΅Ρ€ΠΊΠ½ΡƒΡ‚ΡŒ, Ρ‡Ρ‚ΠΎ для Ρ‚ΠΎΠ³ΠΎ, Ρ‡Ρ‚ΠΎΠ±Ρ‹ ΡΠ»ΠΎΠΆΠΈΡ‚ΡŒ ΠΈΠ»ΠΈ Π²Ρ‹Ρ‡Π΅ΡΡ‚ΡŒ Π΄Π²Π΅ Π·Π°Π΄Π°Π½Π½Ρ‹Π΅ ΠΌΠ°Ρ‚Ρ€ΠΈΡ†Ρ‹, ΠΎΠ½ΠΈ Π΄ΠΎΠ»ΠΆΠ½Ρ‹ ΠΈΠΌΠ΅Ρ‚ΡŒ ΠΎΠ΄ΠΈΠ½Π°ΠΊΠΎΠ²Ρ‹ΠΉ Ρ€Π°Π·ΠΌΠ΅Ρ€ ΠΈΠ»ΠΈ Ρ€Π°Π·ΠΌΠ΅Ρ€. Π’ ΠΏΡ€ΠΎΡ‚ΠΈΠ²Π½ΠΎΠΌ случаС ΠΌΡ‹ Π·Π°ΠΊΠ»ΡŽΡ‡Π°Π΅ΠΌ, Ρ‡Ρ‚ΠΎ сумма (слоТСниС) ΠΈΠ»ΠΈ Ρ€Π°Π·Π½ΠΎΡΡ‚ΡŒ (Π²Ρ‹Ρ‡ΠΈΡ‚Π°Π½ΠΈΠ΅) Π΄Π²ΡƒΡ… ΠΌΠ°Ρ‚Ρ€ΠΈΡ†, ΠΈΠΌΠ΅ΡŽΡ‰ΠΈΡ… Ρ€Π°Π·Π½Ρ‹Π΅ Ρ€Π°Π·ΠΌΠ΅Ρ€Ρ‹ ΠΈΠ»ΠΈ Ρ€Π°Π·ΠΌΠ΅Ρ€Ρ‹, Π½Π΅ ΠΎΠΏΡ€Π΅Π΄Π΅Π»Π΅Π½Π°!

Π’Π΅ΠΏΠ΅Ρ€ΡŒ Π΄Π°Π²Π°ΠΉΡ‚Π΅ посмотрим Π½Π° ΠΎΠ±Ρ‰Π΅Π΅ ΠΏΡ€Π°Π²ΠΈΠ»ΠΎ Ρ‚ΠΎΠ³ΠΎ, ΠΊΠ°ΠΊ ΡΠΊΠ»Π°Π΄Ρ‹Π²Π°Ρ‚ΡŒ ΠΈ Π²Ρ‹Ρ‡ΠΈΡ‚Π°Ρ‚ΡŒ ΠΌΠ°Ρ‚Ρ€ΠΈΡ†Ρ‹, Ρ€Π°Π·ΠΌΠ΅Ρ€Ρ‹ ΠΈΠ»ΠΈ Ρ€Π°Π·ΠΌΠ΅Ρ€Ρ‹ ΠΊΠΎΡ‚ΠΎΡ€Ρ‹Ρ… ΡΠΎΠ²ΠΏΠ°Π΄Π°ΡŽΡ‚.


ΠŸΡ€Π°Π²ΠΈΠ»Π° слоТСния ΠΈ вычитания ΠΌΠ°Ρ‚Ρ€ΠΈΡ† ΠΎΠ΄ΠΈΠ½Π°ΠΊΠΎΠ²ΠΎΠ³ΠΎ Ρ€Π°Π·ΠΌΠ΅Ρ€Π° ΠΈΠ»ΠΈ размСрности

ΠŸΡ€Π΅Π΄ΠΏΠΎΠ»ΠΎΠΆΠΈΠΌ, Ρ‡Ρ‚ΠΎ ΠΌΠ°Ρ‚Ρ€ΠΈΡ†Ρ‹ A ΠΈ B ΠΈΠΌΠ΅ΡŽΡ‚ Π΄Π²Π΅ строки ΠΈ Π΄Π²Π° столбца (2 Γ— 2) с Π½Π΅ΠΊΠΎΡ‚ΠΎΡ€Ρ‹ΠΌΠΈ ΠΏΡ€ΠΎΠΈΠ·Π²ΠΎΠ»ΡŒΠ½Ρ‹ΠΌΠΈ элСмСнтами ΠΈΠ»ΠΈ записями…

Β«Π€ΠΎΡ€ΠΌΡƒΠ»Ρ‹Β» для слоТСния ΠΈ вычитания ΠΌΠ°Ρ‚Ρ€ΠΈΡ† ΠΏΠΎΠΊΠ°Π·Π°Π½Ρ‹ ниТС…

  • Π”ΠΎΠ±Π°Π²ΠΈΡ‚ΡŒ ΠΌΠ°Ρ‚Ρ€ΠΈΡ†Ρ‹ , Π΄ΠΎΠ±Π°Π²ΠΈΠ² ΠΈΡ… ΡΠΎΠΎΡ‚Π²Π΅Ρ‚ΡΡ‚Π²ΡƒΡŽΡ‰ΠΈΠ΅ записи
  • Π’Ρ‹Ρ‡Π΅ΡΡ‚ΡŒ ΠΌΠ°Ρ‚Ρ€ΠΈΡ†Ρ‹ ΠΏΡƒΡ‚Π΅ΠΌ вычитания ΠΈΡ… ΡΠΎΠΎΡ‚Π²Π΅Ρ‚ΡΡ‚Π²ΡƒΡŽΡ‰ΠΈΡ… записСй

Π”Π°Π²Π°ΠΉΡ‚Π΅ ΠΏΠΎΡ€Π°Π±ΠΎΡ‚Π°Π΅ΠΌ Π½Π°Π΄ Π½Π΅ΠΊΠΎΡ‚ΠΎΡ€Ρ‹ΠΌΠΈ ΠΏΡ€ΠΎΠ±Π»Π΅ΠΌΠ°ΠΌΠΈ.


ΠŸΡ€ΠΈΠΌΠ΅Ρ€Ρ‹ слоТСния ΠΈ вычитания ΠΌΠ°Ρ‚Ρ€ΠΈΡ†

ΠŸΡ€ΠΈΠΌΠ΅Ρ€ 1 : Π’Ρ‹ΠΏΠΎΠ»Π½ΠΈΡ‚Π΅ ΡƒΠΊΠ°Π·Π°Π½Π½ΡƒΡŽ ΠΎΠΏΠ΅Ρ€Π°Ρ†ΠΈΡŽ для A + C.

ΠžΠ±Ρ€Π°Ρ‚ΠΈΡ‚Π΅ Π²Π½ΠΈΠΌΠ°Π½ΠΈΠ΅, Ρ‡Ρ‚ΠΎ ΠΌΠ°Ρ‚Ρ€ΠΈΡ†Ρ‹ A ΠΈ C ΠΈΠΌΠ΅ΡŽΡ‚ ΠΎΠ΄ΠΈΠ½Π°ΠΊΠΎΠ²Ρ‹ΠΉ Β«Ρ€Π°Π·ΠΌΠ΅Ρ€Β» ΠΈΠ»ΠΈ Β«Ρ€Π°Π·ΠΌΠ΅Ρ€Π½ΠΎΡΡ‚ΡŒΒ», ΠΏΠΎΡ‚ΠΎΠΌΡƒ Ρ‡Ρ‚ΠΎ ΠΈΡ… количСство строк ΠΈ столбцов ΠΎΠ΄ΠΈΠ½Π°ΠΊΠΎΠ²ΠΎ. Оба ΠΌΠΎΠ³ΡƒΡ‚ Π±Ρ‹Ρ‚ΡŒ описаны ΠΊΠ°ΠΊ ΠΌΠ°Ρ‚Ρ€ΠΈΡ†Π° 3 x 3 . Π­Ρ‚ΠΎ Π³ΠΎΠ²ΠΎΡ€ΠΈΡ‚ ΠΌΠ½Π΅ ΠΎ Ρ‚ΠΎΠΌ, Ρ‡Ρ‚ΠΎ Π½Π°ΠΉΡ‚ΠΈ ΠΈΡ… сумму — это Π½ΠΎΡ€ΠΌΠ°Π»ΡŒΠ½ΠΎ.

Π”ΠΎΠ±Π°Π²Π»ΡŽ ΡΠΎΠΎΡ‚Π²Π΅Ρ‚ΡΡ‚Π²ΡƒΡŽΡ‰ΠΈΠ΅ записи ΠΈ ΡƒΠΏΡ€ΠΎΡ‰Ρƒ.

Π’ΠΎΡ‚ ΠΊΠ°ΠΊ это просто!


ΠŸΡ€ΠΈΠΌΠ΅Ρ€ 2 : Π’Ρ‹ΠΏΠΎΠ»Π½ΠΈΡ‚Π΅ ΡƒΠΊΠ°Π·Π°Π½Π½ΡƒΡŽ ΠΎΠΏΠ΅Ρ€Π°Ρ†ΠΈΡŽ для B + F.

ΠžΠ±Ρ€Π°Ρ‚ΠΈΡ‚Π΅ Π²Π½ΠΈΠΌΠ°Π½ΠΈΠ΅, Ρ‡Ρ‚ΠΎ ΠΌΠ°Ρ‚Ρ€ΠΈΡ†Π° B ΠΈΠΌΠ΅Π΅Ρ‚ Ρ€Π°Π·ΠΌΠ΅Ρ€Π½ΠΎΡΡ‚ΡŒ 2 Γ— 3 , Π° ΠΌΠ°Ρ‚Ρ€ΠΈΡ†Π° F ΠΈΠΌΠ΅Π΅Ρ‚ Ρ€Π°Π·ΠΌΠ΅Ρ€Π½ΠΎΡΡ‚ΡŒ 2 Γ— 2 .

ΠŸΠΎΡΠΊΠΎΠ»ΡŒΠΊΡƒ количСство строк ΠΈ столбцов Π½Π΅ совпадаСт, сумма ΠΌΠ°Ρ‚Ρ€ΠΈΡ† B ΠΈ F Π½Π΅ сущСствуСт ΠΈΠ»ΠΈ Π½Π΅ ΠΎΠΏΡ€Π΅Π΄Π΅Π»Π΅Π½Π°. . Π― ΠΎΡΡ‚Π°Π½ΠΎΠ²Π»ΡŽΡΡŒ здСсь. Π­Ρ‚ΠΎ наш ΠΎΡ‚Π²Π΅Ρ‚, Ρ…ΠΎΡ‚ΠΈΡ‚Π΅ Π²Π΅Ρ€ΡŒΡ‚Π΅, Ρ…ΠΎΡ‚ΠΈΡ‚Π΅ Π½Π΅Ρ‚.


ΠŸΡ€ΠΈΠΌΠ΅Ρ€ 3 : Π’Ρ‹ΠΏΠΎΠ»Π½ΠΈΡ‚Π΅ ΡƒΠΊΠ°Π·Π°Π½Π½ΡƒΡŽ ΠΎΠΏΠ΅Ρ€Π°Ρ†ΠΈΡŽ для E-B.

ПослСдниС Π΄Π²Π° ΠΏΡ€ΠΈΠΌΠ΅Ρ€Π° ΠΏΠΎΠΊΠ°Π·Π°Π»ΠΈ Π²Π°ΠΌ, ΠΊΠ°ΠΊ ΡΠΊΠ»Π°Π΄Ρ‹Π²Π°Ρ‚ΡŒ ΠΌΠ°Ρ‚Ρ€ΠΈΡ†Ρ‹. На этот Ρ€Π°Π· ΠΌΡ‹ ΠΏΠΎΠ³ΠΎΠ²ΠΎΡ€ΠΈΠΌ ΠΎ Π²Ρ‹Ρ‡ΠΈΡ‚Π°Π½ΠΈΠΈ ΠΌΠ°Ρ‚Ρ€ΠΈΡ†.ΠŸΠΎΠΌΠ½ΠΈΡ‚Π΅, Ρ‡Ρ‚ΠΎ процСсс слоТСния ΠΈ вычитания ΠΌΠ°Ρ‚Ρ€ΠΈΡ† ΠΎΡ‡Π΅Π½ΡŒ ΠΏΠΎΡ…ΠΎΠΆ. Если Π²Ρ‹ Π·Π°Π±Ρ‹Π»ΠΈ, просмотритС ΠΏΡ€ΠΈΠ²Π΅Π΄Π΅Π½Π½ΡƒΡŽ Π²Ρ‹ΡˆΠ΅ Β«Ρ„ΠΎΡ€ΠΌΡƒΠ»ΡƒΒ».

Π’ этом ΠΏΡ€ΠΈΠΌΠ΅Ρ€Π΅ Π½Π°ΠΌ Π½ΡƒΠΆΠ½ΠΎ Π½Π°ΠΉΡ‚ΠΈ Ρ€Π°Π·Π½ΠΈΡ†Ρƒ ΠΌΠ΅ΠΆΠ΄Ρƒ ΠΌΠ°Ρ‚Ρ€ΠΈΡ†Π΅ΠΉ E ΠΈ ΠΌΠ°Ρ‚Ρ€ΠΈΡ†Π΅ΠΉ B.

Однако каТСтся, Ρ‡Ρ‚ΠΎ это Π½Π΅Π²ΠΎΠ·ΠΌΠΎΠΆΠ½ΠΎ, ΠΏΠΎΡΠΊΠΎΠ»ΡŒΠΊΡƒ ΠΎΠ½ΠΈ ΠΈΠΌΠ΅ΡŽΡ‚ Ρ€Π°Π·Π»ΠΈΡ‡Π½Ρ‹Ρ… Ρ€Π°Π·ΠΌΠ΅Ρ€ΠΎΠ² ΠΈΠ»ΠΈ Ρ€Π°Π·ΠΌΠ΅Ρ€ΠΎΠ². ΠœΠ°Ρ‚Ρ€ΠΈΡ†Π° E ΠΈΠΌΠ΅Π΅Ρ‚ Ρ€Π°Π·ΠΌΠ΅Ρ€ 3 Γ— 2, Π° ΠΌΠ°Ρ‚Ρ€ΠΈΡ†Π° B — 2 Γ— 3.

ΠŸΠΎΡΠΊΠΎΠ»ΡŒΠΊΡƒ я Π½Π΅ ΠΌΠΎΠ³Ρƒ Π²Ρ‹ΠΏΠΎΠ»Π½ΠΈΡ‚ΡŒ Π²Ρ‹Ρ‡ΠΈΡ‚Π°Π½ΠΈΠ΅ ΠΏΠΎ Π²Ρ…ΠΎΠ΄Π°ΠΌ ΠΈΠ·-Π·Π° Ρ‚ΠΎΠ³ΠΎ, Ρ‡Ρ‚ΠΎ записи Π΄Π²ΡƒΡ… ΠΌΠ°Ρ‚Ρ€ΠΈΡ† Π½Π΅ ΠΈΠΌΠ΅ΡŽΡ‚ прямого соотвСтствия, я Π΄ΠΎΠ»ΠΆΠ΅Π½ Π·Π°ΡΠ²ΠΈΡ‚ΡŒ, Ρ‡Ρ‚ΠΎ НЕ Π²ΠΎΠ·ΠΌΠΎΠΆΠ½ΠΎ, Π½Π°ΠΉΡ‚ΠΈ ΠΈΡ… Ρ€Π°Π·Π»ΠΈΡ‡ΠΈΠ΅.Π‘Π»Π΅Π΄ΠΎΠ²Π°Ρ‚Π΅Π»ΡŒΠ½ΠΎ, наш ΠΎΡ‚Π²Π΅Ρ‚ — undefined .

Π­Ρ‚ΠΎ Π½Π΅ вопрос с ΠΏΠΎΠ΄Π²ΠΎΡ…ΠΎΠΌ. УчитСля ΠΈΠ½ΠΎΠ³Π΄Π° Β«Π΄ΠΎΠ±Π°Π²Π»ΡΡŽΡ‚Β» это Π² смСсь, Ρ‡Ρ‚ΠΎΠ±Ρ‹ ΠΏΡ€ΠΎΠ²Π΅Ρ€ΠΈΡ‚ΡŒ, ΠΏΠΎΠ½ΠΈΠΌΠ°Π΅Ρ‚Π΅ Π»ΠΈ Π²Ρ‹ ΠΊΠΎΠ½Ρ†Π΅ΠΏΡ†ΠΈΡŽ, согласно ΠΊΠΎΡ‚ΠΎΡ€ΠΎΠΉ ΠΌΠΎΠΆΠ½ΠΎ ΡΠΊΠ»Π°Π΄Ρ‹Π²Π°Ρ‚ΡŒ ΠΈΠ»ΠΈ Π²Ρ‹Ρ‡ΠΈΡ‚Π°Ρ‚ΡŒ Ρ‚ΠΎΠ»ΡŒΠΊΠΎ ΠΌΠ°Ρ‚Ρ€ΠΈΡ†Ρ‹ с ΠΎΠ΄ΠΈΠ½Π°ΠΊΠΎΠ²Ρ‹ΠΌΠΈ Ρ€Π°Π·ΠΌΠ΅Ρ€Π°ΠΌΠΈ ΠΈΠ»ΠΈ Ρ€Π°Π·ΠΌΠ΅Ρ€Π°ΠΌΠΈ. НС Ρ€Π°ΡΡΡ‚Ρ€Π°ΠΈΠ²Π°ΠΉΡ‚Π΅ΡΡŒ, я сам ΠΏΠΎΠΏΠ°Π» Π² эту Β«Π»ΠΎΠ²ΡƒΡˆΠΊΡƒΒ». НадСюсь, Ρ‚Π΅ΠΏΠ΅Ρ€ΡŒ, ΠΊΠΎΠ³Π΄Π° Π²Ρ‹ Π·Π½Π°Π΅Ρ‚Π΅, Π²Ρ‹ Π±ΡƒΠ΄Π΅Ρ‚Π΅ остороТны Π² ΡΠ»Π΅Π΄ΡƒΡŽΡ‰ΠΈΠΉ Ρ€Π°Π·, ΠΊΠΎΠ³Π΄Π° ΡΡ‚ΠΎΠ»ΠΊΠ½Π΅Ρ‚Π΅ΡΡŒ с Ρ‚Π°ΠΊΠΎΠΉ ΠΏΡ€ΠΎΠ±Π»Π΅ΠΌΠΎΠΉ.


ΠŸΡ€ΠΈΠΌΠ΅Ρ€ 4 : Π’Ρ‹ΠΏΠΎΠ»Π½ΠΈΡ‚Π΅ ΡƒΠΊΠ°Π·Π°Π½Π½ΡƒΡŽ ΠΎΠΏΠ΅Ρ€Π°Ρ†ΠΈΡŽ для F-D.

ΠŸΡ€ΠΈ быстром просмотрС я Π²ΠΈΠΆΡƒ, Ρ‡Ρ‚ΠΎ ΠΌΠΎΠΆΠ½ΠΎ Π½Π°ΠΉΡ‚ΠΈ Ρ€Π°Π·Π½ΠΈΡ†Ρƒ ΠΌΠ΅ΠΆΠ΄Ρƒ ΠΌΠ°Ρ‚Ρ€ΠΈΡ†Π°ΠΌΠΈ F ΠΈ D, ΠΏΠΎΡ‚ΠΎΠΌΡƒ Ρ‡Ρ‚ΠΎ ΠΎΠ±Π΅ ΠΈΠΌΠ΅ΡŽΡ‚ ΠΎΠ΄ΠΈΠ½Π°ΠΊΠΎΠ²ΠΎΠ΅ количСство строк ΠΈ столбцов.Π‘ΠΎΠ»ΡŒΡˆΠΎΠΉ!

Для Π½Π°Ρ‡Π°Π»Π° я Π²Ρ‹Ρ‡Ρ‚Ρƒ ΡΠΎΠΎΡ‚Π²Π΅Ρ‚ΡΡ‚Π²ΡƒΡŽΡ‰ΠΈΠ΅ записи F ΠΈ D. МоС СдинствСнноС прСдостСрСТСниС — Π±ΡƒΠ΄ΡŒΡ‚Π΅ ΠΎΡ‡Π΅Π½ΡŒ остороТны ΠΏΡ€ΠΈ Π²Ρ‹Ρ‡ΠΈΡ‚Π°Π½ΠΈΠΈ Π΄Π΅ΠΉΡΡ‚Π²ΠΈΡ‚Π΅Π»ΡŒΠ½Ρ‹Ρ… чисСл. ΠžΠ±Ρ‹Ρ‡Π½ΠΎ здСсь Π²ΠΎΠ·Π½ΠΈΠΊΠ°ΡŽΡ‚ ΠΎΠ±Ρ‰ΠΈΠ΅ ошибки. ΠŸΠΎΠΌΠ½ΠΈΡ‚Π΅, Ρ‡Ρ‚ΠΎ Π΄Π²Π° сосСдних ΠΎΡ‚Ρ€ΠΈΡ†Π°Ρ‚Π΅Π»ΡŒΠ½Ρ‹Ρ… Π·Π½Π°ΠΊΠ° ΠΎΠΊΠ°Π·Ρ‹Π²Π°ΡŽΡ‚ΡΡ ΠΏΠΎΠ»ΠΎΠΆΠΈΡ‚Π΅Π»ΡŒΠ½Ρ‹ΠΌΠΈ.

НСплохо, ΠΏΡ€Π°Π²Π΄Π°?


ΠŸΡ€ΠΈΠΌΠ΅Ρ€ 5 : Π’Ρ‹ΠΏΠΎΠ»Π½ΠΈΡ‚Π΅ ΡƒΠΊΠ°Π·Π°Π½Π½ΡƒΡŽ ΠΎΠΏΠ΅Ρ€Π°Ρ†ΠΈΡŽ для C-A.

Π”Π²Π΅ Π·Π°Π΄Π°Π½Π½Ρ‹Π΅ ΠΌΠ°Ρ‚Ρ€ΠΈΡ†Ρ‹ C ΠΈ A ΠΈΠΌΠ΅ΡŽΡ‚ ΠΎΠ΄ΠΈΠ½Π°ΠΊΠΎΠ²Ρ‹Π΅ Ρ€Π°Π·ΠΌΠ΅Ρ€Ρ‹ ΠΈΠ»ΠΈ Ρ€Π°Π·ΠΌΠ΅Ρ€Ρ‹ (ΠΎΠ±Π΅ ΠΌΠ°Ρ‚Ρ€ΠΈΡ†Ρ‹ 3 Γ— 3). Π­Ρ‚ΠΎ позволяСт Π½Π°ΠΌ Π²Ρ‹ΠΏΠΎΠ»Π½ΡΡ‚ΡŒ ΠΎΠΏΠ΅Ρ€Π°Ρ†ΠΈΡŽ вычитания.

Вычитая ΠΏΠΎ Π²Ρ…ΠΎΠ΄Ρƒ, я получил…


ΠŸΡ€ΠΈΠΌΠ΅Ρ€ 6 : Π’Ρ‹ΠΏΠΎΠ»Π½ΠΈΡ‚Π΅ ΡƒΠΊΠ°Π·Π°Π½Π½ΡƒΡŽ ΠΎΠΏΠ΅Ρ€Π°Ρ†ΠΈΡŽ для (A + C) + (C-A).

Π­Ρ‚ΠΎ ΠΎΡ‚Π»ΠΈΡ‡Π½Ρ‹ΠΉ ΠΏΡ€ΠΈΠΌΠ΅Ρ€ «многоступСнчатой» Π·Π°Π΄Π°Ρ‡ΠΈ, которая Π²ΠΊΠ»ΡŽΡ‡Π°Π΅Ρ‚ Π² сСбя слоТСниС ΠΈ Π²Ρ‹Ρ‡ΠΈΡ‚Π°Π½ΠΈΠ΅ ΠΌΠ°Ρ‚Ρ€ΠΈΡ†. ЦСль состоит Π² Ρ‚ΠΎΠΌ, Ρ‡Ρ‚ΠΎΠ±Ρ‹ Π²Ρ‹ΠΏΠΎΠ»Π½ΠΈΡ‚ΡŒ ΡƒΠΊΠ°Π·Π°Π½Π½ΡƒΡŽ ΠΎΠΏΠ΅Ρ€Π°Ρ†ΠΈΡŽ Π½Π°Π΄ ΠΊΠ°ΠΆΠ΄ΠΎΠΉ ΠΊΡ€ΡƒΠ³Π»ΠΎΠΉ скобкой, Π° Π·Π°Ρ‚Π΅ΠΌ Π΄ΠΎΠ±Π°Π²ΠΈΡ‚ΡŒ ΠΈΡ… вмСстС.

Π§Ρ‚ΠΎΠ±Ρ‹ ΠΏΡ€ΠΎΠΏΡƒΡΡ‚ΠΈΡ‚ΡŒ Π½Π΅ΠΊΠΎΡ‚ΠΎΡ€Ρ‹Π΅ шаги, просмотритС, ΠΊΠ°ΠΊ ΠΌΡ‹ Ρ€Π΅ΡˆΠΈΠ»ΠΈ для (A + C) Π² ΠΏΡ€ΠΈΠΌΠ΅Ρ€Π΅ 1 ΠΈ C-A Π² ΠΏΡ€ΠΈΠΌΠ΅Ρ€Π΅ 5.

Пока Ρƒ нас Π΅ΡΡ‚ΡŒ эти частичныС отвСты…

Π˜Ρ‚Π°ΠΊ, послСдний шаг — ΡΠ»ΠΎΠΆΠΈΡ‚ΡŒ ΠΈΡ… вмСстС, Ρ‡Ρ‚ΠΎΠ±Ρ‹ ΠΏΠΎΠ»ΡƒΡ‡ΠΈΡ‚ΡŒ Ρ‚Ρ€Π΅Π±ΡƒΠ΅ΠΌΡ‹ΠΉ ΠΎΡ‚Π²Π΅Ρ‚.

Как Π²ΠΈΠ΄ΠΈΡ‚Π΅, слоТСниС ΠΈ Π²Ρ‹Ρ‡ΠΈΡ‚Π°Π½ΠΈΠ΅ ΠΌΠ°Ρ‚Ρ€ΠΈΡ† ΠΎΡ‡Π΅Π½ΡŒ просто. Π― надСюсь, Ρ‡Ρ‚ΠΎ Π²Ρ‹ ΠΏΡ€ΠΈΠΎΠ±Ρ€Π΅Π»ΠΈ Π½Π΅ΠΊΠΎΡ‚ΠΎΡ€ΡƒΡŽ ΡƒΠ²Π΅Ρ€Π΅Π½Π½ΠΎΡΡ‚ΡŒ ΠΈ ΡƒΠ·Π½Π°Π»ΠΈ, ΠΊΠ°ΠΊ с этим ΡΠΏΡ€Π°Π²ΠΈΡ‚ΡŒΡΡ.


ΠŸΡ€ΠΈΠΌΠ΅Ρ€ 7 : Π’Ρ‹ΠΏΠΎΠ»Π½ΠΈΡ‚Π΅ ΡƒΠΊΠ°Π·Π°Π½Π½ΡƒΡŽ ΠΎΠΏΠ΅Ρ€Π°Ρ†ΠΈΡŽ для (A + C) + (C-A).

Π­Ρ‚ΠΎ Ρ‚Π° ΠΆΠ΅ ΠΏΡ€ΠΎΠ±Π»Π΅ΠΌΠ°, Ρ‡Ρ‚ΠΎ ΠΈ Π² ΠΏΡ€ΠΈΠΌΠ΅Ρ€Π΅ 6. Но я Ρ…ΠΎΡ‡Ρƒ Ρ€Π΅ΡˆΠΈΡ‚ΡŒ Π΅Π΅ Π½Π΅ΠΌΠ½ΠΎΠ³ΠΎ ΠΈΠ½Π°Ρ‡Π΅, Ρ‡Ρ‚ΠΎΠ±Ρ‹ ΠΏΡ€ΠΎΠ΄Π΅ΠΌΠΎΠ½ΡΡ‚Ρ€ΠΈΡ€ΠΎΠ²Π°Ρ‚ΡŒ Ρ‚ΠΎΡ‚ Ρ„Π°ΠΊΡ‚, Ρ‡Ρ‚ΠΎ Π΅ΡΡ‚ΡŒ Π΄Ρ€ΡƒΠ³ΠΈΠ΅ способы Ρ€Π΅ΡˆΠ΅Π½ΠΈΡ ΠΎΠΏΡ€Π΅Π΄Π΅Π»Π΅Π½Π½ΠΎΠΉ ΠΏΡ€ΠΎΠ±Π»Π΅ΠΌΡ‹. Π₯отя ΠΌΠ΅Ρ‚ΠΎΠ΄, ΠΏΡ€ΠΈΠΌΠ΅Π½Π΅Π½Π½Ρ‹ΠΉ Π² ΠΏΡ€ΠΈΠΌΠ΅Ρ€Π΅ 6, Π²ΠΏΠΎΠ»Π½Π΅ ΠΏΡ€ΠΈΠ΅ΠΌΠ»Π΅ΠΌ, этот Β«Π°Π»ΡŒΡ‚Π΅Ρ€Π½Π°Ρ‚ΠΈΠ²Π½Ρ‹ΠΉΒ» ΠΏΠΎΠ΄Ρ…ΠΎΠ΄ ΠΈΠΌΠ΅Π΅Ρ‚ Π³ΠΎΡ€Π°Π·Π΄ΠΎ большС смысла, ΠΏΠΎΡΠΊΠΎΠ»ΡŒΠΊΡƒ ΠΎΠ½ ΠΎΡ‡Π΅Π½ΡŒ прост.

ΠŸΠΎΠ΅Ρ…Π°Π»ΠΈβ€¦

Если Π²Ρ‹ рассматриваСтС Π²Ρ‹Ρ€Π°ΠΆΠ΅Π½ΠΈΠ΅ (A + C) + (C-A) ΠΊΠ°ΠΊ , ΠΎΠ±ΡŠΠ΅Π΄ΠΈΠ½ΡΡŽΡ‰Π΅Π΅ ΠΏΠΎΡ…ΠΎΠΆΠΈΠ΅ ΠΈΠ»ΠΈ ΠΏΠΎΡ…ΠΎΠΆΠΈΠ΅ Ρ‚Π΅Ρ€ΠΌΠΈΠ½Ρ‹, ΠΏΡ€ΠΎΠ±Π»Π΅ΠΌΠ° Ρ‚ΠΈΠΏΠ° , Ρ‚ΠΎ ΠΈΠΌΠ΅Π΅Ρ‚ смысл быстро ΡƒΠΏΡ€ΠΎΡΡ‚ΠΈΡ‚ΡŒ ΠΈΡΡ…ΠΎΠ΄Π½ΡƒΡŽ Π·Π°Π΄Π°Ρ‡Ρƒ, Π΄Π°ΠΆΠ΅ Π½Π΅ занимаясь слоТСниСм ΠΈ Π²Ρ‹Ρ‡ΠΈΡ‚Π°Π½ΠΈΠ΅ΠΌ ΠΌΠ°Ρ‚Ρ€ΠΈΡ†.

ΠžΠ±Ρ€Π°Ρ‚ΠΈΡ‚Π΅ Π²Π½ΠΈΠΌΠ°Π½ΠΈΠ΅, Ρ‡Ρ‚ΠΎ я ΠΌΠΎΠ³Ρƒ ΠΎΠ±ΡŠΠ΅Π΄ΠΈΠ½ΠΈΡ‚ΡŒ C-Ρ‚Π΅Ρ€ΠΌΠΈΠ½Ρ‹ ΠΊΠ°ΠΊ 2C.

Π’Π΅ΠΏΠ΅Ρ€ΡŒ, Ρ‡Π»Π΅Π½Ρ‹ A Π΄ΠΎΠ»ΠΆΠ½Ρ‹ ΡΠΎΠΊΡ€Π°Ρ‰Π°Ρ‚ΡŒΡΡ, ΠΏΠΎΡ‚ΠΎΠΌΡƒ Ρ‡Ρ‚ΠΎ ΠΎΠ½ΠΈ ΠΈΠΌΠ΅ΡŽΡ‚ ΠΏΡ€ΠΎΡ‚ΠΈΠ²ΠΎΠΏΠΎΠ»ΠΎΠΆΠ½Ρ‹Π΅ Π·Π½Π°ΠΊΠΈ.

Наша исходная Π·Π°Π΄Π°Ρ‡Π° сводится ΠΊ 2C, Ρ‡Ρ‚ΠΎ составляСт Π²Π΄Π²ΠΎΠ΅ ΠΈΠ»ΠΈ Π²Π΄Π²ΠΎΠ΅ большС ΠΌΠ°Ρ‚Ρ€ΠΈΡ†Ρ‹ C .

Π­Ρ‚ΠΎ ΠΎΠ·Π½Π°Ρ‡Π°Π΅Ρ‚, Ρ‡Ρ‚ΠΎ я ΡΠΎΠ±ΠΈΡ€Π°ΡŽΡΡŒ ΡƒΠΌΠ½ΠΎΠΆΠΈΡ‚ΡŒ ΠΊΠ°ΠΆΠ΄Ρ‹ΠΉ элСмСнт ΠΌΠ°Ρ‚Ρ€ΠΈΡ†Ρ‹ C Π½Π° 2. На самом Π΄Π΅Π»Π΅ это Ρ‚Π΅ΠΌΠ° ΠΌΠΎΠ΅Π³ΠΎ Π΄Ρ€ΡƒΠ³ΠΎΠ³ΠΎ ΡƒΡ€ΠΎΠΊΠ° ΠΏΠΎ Π°Π»Π³Π΅Π±Ρ€Π΅, посвящСнного скалярному ΡƒΠΌΠ½ΠΎΠΆΠ΅Π½ΠΈΡŽ ΠΌΠ°Ρ‚Ρ€ΠΈΡ†Ρ‹.

Π‘

, Ρ‚ΠΎ 2C Ρ€Π΅ΡˆΠ°Π΅Ρ‚ΡΡ с ΠΏΠΎΠΌΠΎΡ‰ΡŒΡŽβ€¦

ΠžΠΊΠΎΠ½Ρ‡Π°Ρ‚Π΅Π»ΡŒΠ½Ρ‹ΠΉ ΠΎΡ‚Π²Π΅Ρ‚, ΠΏΠΎΠ»ΡƒΡ‡Π΅Π½Π½Ρ‹ΠΉ с ΠΏΠΎΠΌΠΎΡ‰ΡŒΡŽ этого ΠΌΠ΅Ρ‚ΠΎΠ΄Π°, Ρ‚ΠΎΡ‡Π½ΠΎ Ρ‚Π°ΠΊΠΎΠΉ ΠΆΠ΅, ΠΊΠ°ΠΊ Π² ΠΏΡ€ΠΈΠΌΠ΅Ρ€Π΅ 6. Π›Π΅Π³ΠΊΠΎ, ΠΏΡ€Π°Π²Π΄Π°?


ΠŸΡ€Π°ΠΊΡ‚ΠΈΠΊΠ° с Ρ€Π°Π±ΠΎΡ‡ΠΈΠΌΠΈ листами

Π’ΠΎΠ·ΠΌΠΎΠΆΠ½ΠΎ, вас заинтСрСсуСт:

БкалярноС ΡƒΠΌΠ½ΠΎΠΆΠ΅Π½ΠΈΠ΅

Π£ΠΌΠ½ΠΎΠΆΠ΅Π½ΠΈΠ΅ ΠΌΠ°Ρ‚Ρ€ΠΈΡ†

ΠœΠ°Ρ‚Ρ€ΠΈΡ†Ρ‹ слоТСния ΠΈ вычитания

А ΠΌΠ°Ρ‚Ρ€ΠΈΡ†Π° ΠΌΠΎΠ³ΡƒΡ‚ Π±Ρ‹Ρ‚ΡŒ Π΄ΠΎΠ±Π°Π²Π»Π΅Π½Ρ‹ (ΠΈΠ»ΠΈ Π²Ρ‹Ρ‡Ρ‚Π΅Π½Ρ‹ ΠΈΠ·) Π΄Ρ€ΡƒΠ³ΠΎΠΉ ΠΌΠ°Ρ‚Ρ€ΠΈΡ†Ρ‹ Ρ‚ΠΎΠ»ΡŒΠΊΠΎ Π² Ρ‚ΠΎΠΌ случаС, Ссли Π΄Π²Π΅ ΠΌΠ°Ρ‚Ρ€ΠΈΡ†Ρ‹ ΠΈΠΌΠ΅ΡŽΡ‚ ΠΎΠ΄ΠΈΠ½Π°ΠΊΠΎΠ²Ρ‹Π΅ Π“Π°Π±Π°Ρ€ΠΈΡ‚Π½Ρ‹Π΅ Ρ€Π°Π·ΠΌΠ΅Ρ€Ρ‹ .

Π§Ρ‚ΠΎΠ±Ρ‹ Π΄ΠΎΠ±Π°Π²ΠΈΡ‚ΡŒ Π΄Π²Π΅ ΠΌΠ°Ρ‚Ρ€ΠΈΡ†Ρ‹, просто Π΄ΠΎΠ±Π°Π²ΡŒΡ‚Π΅ ΡΠΎΠΎΡ‚Π²Π΅Ρ‚ΡΡ‚Π²ΡƒΡŽΡ‰ΠΈΠ΅ записи ΠΈ помСститС эту сумму Π² ΡΠΎΠΎΡ‚Π²Π΅Ρ‚ΡΡ‚Π²ΡƒΡŽΡ‰ΡƒΡŽ ΠΏΠΎΠ·ΠΈΡ†ΠΈΡŽ Π² ΠΌΠ°Ρ‚Ρ€ΠΈΡ†Π΅, которая получится.

ΠŸΡ€ΠΈΠΌΠ΅Ρ€ 1:

Π”ΠΎΠ±Π°Π²ΡŒΡ‚Π΅ ΠΌΠ°Ρ‚Ρ€ΠΈΡ†Ρ‹.

[ 1 5 — 4 3 ] + [ 2 — 1 4 — 1 ]

Π‘Π½Π°Ρ‡Π°Π»Π° ΠΎΠ±Ρ€Π°Ρ‚ΠΈΡ‚Π΅ Π²Π½ΠΈΠΌΠ°Π½ΠΈΠ΅, Ρ‡Ρ‚ΠΎ ΠΎΠ±Π° слагаСмых 2 Γ— 2 ΠΌΠ°Ρ‚Ρ€ΠΈΡ†Ρ‹, Ρ‡Ρ‚ΠΎΠ±Ρ‹ ΠΌΡ‹ ΠΌΠΎΠ³Π»ΠΈ ΠΈΡ… Π΄ΠΎΠ±Π°Π²ΠΈΡ‚ΡŒ.

[ 1 5 — 4 3 ] + [ 2 — 1 4 — 1 ] Π·Π½Π°ΠΊ Ρ€Π°Π²Π½ΠΎ [ 1 + 2 5 + ( — 1 ) — 4 + 4 3 + ( — 1 ) ]

Π·Π½Π°ΠΊ Ρ€Π°Π²Π½ΠΎ [ 3 4 0 2 ]

Π’Ρ‹Ρ‡ΠΈΡ‚Π°Π½ΠΈΠ΅ с ΠΏΠΎΠΌΠΎΡ‰ΡŒΡŽ ΠΌΠ°Ρ‚Ρ€ΠΈΡ† Ρ‚Π°ΠΊ ΠΆΠ΅ просто.

ΠŸΡ€ΠΈΠΌΠ΅Ρ€ 2:

Π’Ρ‹Ρ‡Π΅ΡΡ‚ΡŒ.

[ 4 5 6 2 3 4 ] — [ 2 4 6 1 2 3 ]

Π’Ρ‹Ρ‡Ρ‚ΠΈΡ‚Π΅ ΡΠΎΠΎΡ‚Π²Π΅Ρ‚ΡΡ‚Π²ΡƒΡŽΡ‰ΠΈΠ΅ записи.

[ 4 5 6 2 3 4 ] — [ 2 4 6 1 2 3 ] Π·Π½Π°ΠΊ Ρ€Π°Π²Π½ΠΎ [ 4 — 2 5 — 4 6 — 6 2 — 1 3 — 2 4 — 3 ]

Π·Π½Π°ΠΊ Ρ€Π°Π²Π½ΠΎ [ 2 1 0 1 1 1 ]

Как ΡΠΊΠ»Π°Π΄Ρ‹Π²Π°Ρ‚ΡŒ ΠΈ Π²Ρ‹Ρ‡ΠΈΡ‚Π°Ρ‚ΡŒ ΠΌΠ°Ρ‚Ρ€ΠΈΡ†Ρ‹

Π‘Π»ΠΎΠΆΠ΅Π½ΠΈΠ΅ ΠΈ Π²Ρ‹Ρ‡ΠΈΡ‚Π°Π½ΠΈΠ΅ ΠΌΠ°Ρ‚Ρ€ΠΈΡ†

На этом этапС нашСго курса Π»ΠΈΠ½Π΅ΠΉΠ½ΠΎΠΉ Π°Π»Π³Π΅Π±Ρ€Ρ‹ ΠΌΡ‹ ΠΌΠ½ΠΎΠ³ΠΎ Ρ€Π°Π±ΠΎΡ‚Π°Π»ΠΈ с ΠΌΠ°Ρ‚Ρ€ΠΈΡ†Π°ΠΌΠΈ, ΠΌΡ‹ выполняли ΠΎΠΏΠ΅Ρ€Π°Ρ†ΠΈΠΈ со строками для Ρ€Π΅ΡˆΠ΅Π½ΠΈΡ систСм Π»ΠΈΠ½Π΅ΠΉΠ½Ρ‹Ρ… ΡƒΡ€Π°Π²Π½Π΅Π½ΠΈΠΉ, прСдставляя Π»ΠΈΠ½Π΅ΠΉΠ½ΡƒΡŽ систСму Π² Π²ΠΈΠ΄Π΅ ΠΌΠ°Ρ‚Ρ€ΠΈΡ†Ρ‹ ΠΈ ΠΈΡΠΏΠΎΠ»ΡŒΠ·ΡƒΡ ΠΈΡ… Π² Π½Π°ΡˆΠΈΡ… интСрСсах посрСдством сокращСния строк ΠΈ ΡΡˆΠ΅Π»ΠΎΠ½ΠΈΡ€ΠΎΠ²Π°Π½Π½Ρ‹Ρ… Ρ„ΠΎΡ€ΠΌ. ΠœΡ‹ Ρ‚Π°ΠΊΠΆΠ΅ Π½Π°ΡƒΡ‡ΠΈΠ»ΠΈΡΡŒ ΠΏΠΎΠ½ΠΈΠΌΠ°Ρ‚ΡŒ Π½Π΅ Ρ‚ΠΎΠ»ΡŒΠΊΠΎ обозначСния ΠΌΠ°Ρ‚Ρ€ΠΈΡ†, Π½ΠΎ ΠΈ ΠΈΡ… элСмСнты, ΠΈΡ… Ρ€Π°Π·ΠΌΠ΅Ρ€Ρ‹, ΠΈ Ρ‚Π΅ΠΏΠ΅Ρ€ΡŒ ΠΏΠΎΡ€Π° ΠΏΠΎΠ΄ΡƒΠΌΠ°Ρ‚ΡŒ ΠΎ Π½ΠΈΡ… Π² Ρ†Π΅Π»ΠΎΠΌ ΠΈ ΡƒΠ·Π½Π°Ρ‚ΡŒ, ΠΊΠ°ΠΊΠΈΠ΅ ΠΎΠΏΠ΅Ρ€Π°Ρ†ΠΈΠΈ ΠΌΠΎΠ³ΡƒΡ‚ Π±Ρ‹Ρ‚ΡŒ Π²Ρ‹ΠΏΠΎΠ»Π½Π΅Π½Ρ‹ с Π½ΠΈΠΌΠΈ (ΠΎΠ±Ρ€Π°Ρ‚ΠΈΡ‚Π΅ Π²Π½ΠΈΠΌΠ°Π½ΠΈΠ΅: «с Π½ΠΈΠΌΠΈΒ», НЕ Β«Π² ΠΈΡ…»).

На этом ΡƒΡ€ΠΎΠΊΠ΅ ΠΌΡ‹ познакомимся с ΠΏΠ΅Ρ€Π²Ρ‹ΠΌΠΈ двумя ΠΈΠ· Ρ‡Π΅Ρ‚Ρ‹Ρ€Π΅Ρ… основных ΠΎΠΏΠ΅Ρ€Π°Ρ†ΠΈΠΉ, ΠΊΠΎΡ‚ΠΎΡ€Ρ‹Π΅ ΠΌΠΎΠΆΠ½ΠΎ Π²Ρ‹ΠΏΠΎΠ»Π½ΠΈΡ‚ΡŒ с ΠΏΠ°Ρ€ΠΎΠΉ (ΠΈΠ»ΠΈ Π±ΠΎΠ»Π΅Π΅) ΠΌΠ°Ρ‚Ρ€ΠΈΡ†: слоТСниС, Π²Ρ‹Ρ‡ΠΈΡ‚Π°Π½ΠΈΠ΅, скалярноС ΡƒΠΌΠ½ΠΎΠΆΠ΅Π½ΠΈΠ΅ ΠΈ ΡƒΠΌΠ½ΠΎΠΆΠ΅Π½ΠΈΠ΅. ΠœΡ‹ Π·Π½Π°Π΅ΠΌ эти Π±Π°Π·ΠΎΠ²Ρ‹Π΅ ΠΎΠΏΠ΅Ρ€Π°Ρ†ΠΈΠΈ ΠΈΠ· Π°Ρ€ΠΈΡ„ΠΌΠ΅Ρ‚ΠΈΠΊΠΈ, поэтому Π½Π°ΠΌ просто Π½ΡƒΠΆΠ½ΠΎ ΡƒΠ·Π½Π°Ρ‚ΡŒ, ΠΊΠ°ΠΊ ΠΎΠ½ΠΈ Ρ€Π°Π±ΠΎΡ‚Π°ΡŽΡ‚ со всСй структурой ΠΌΠ°Ρ‚Ρ€ΠΈΡ† ΠΈ с Ρ€Π΅Π·ΡƒΠ»ΡŒΡ‚Π°Ρ‚Π°ΠΌΠΈ, ΠΊΠΎΡ‚ΠΎΡ€Ρ‹Π΅ ΠΎΠ½ΠΈ производят.ΠžΠ±Ρ€Π°Ρ‚ΠΈΡ‚Π΅ Π²Π½ΠΈΠΌΠ°Π½ΠΈΠ΅, Ρ‡Ρ‚ΠΎ ΠΌΡ‹ Π½Π΅ упомянули Π΄Π΅Π»Π΅Π½ΠΈΠ΅ ΠΌΠ°Ρ‚Ρ€ΠΈΡ†, ΠΏΠΎΡ‚ΠΎΠΌΡƒ Ρ‡Ρ‚ΠΎ дСлСния ΠΌΠ΅ΠΆΠ΄Ρƒ ΠΌΠ°Ρ‚Ρ€ΠΈΡ†Π°ΠΌΠΈ Π½Π΅ сущСствуСт (ΠΈΠ»ΠΈ, ΠΏΠΎ ΠΊΡ€Π°ΠΉΠ½Π΅ΠΉ ΠΌΠ΅Ρ€Π΅, процСсс Π΅Ρ‰Π΅ Π½Π΅ ΠΎΠΏΡ€Π΅Π΄Π΅Π»Π΅Π½), СдинствСнноС Π΄Π΅Π»Π΅Π½ΠΈΠ΅, ΠΊΠΎΡ‚ΠΎΡ€ΠΎΠ΅ ΠΌΡ‹ ΠΌΠΎΠΆΠ΅ΠΌ ΡΠ΄Π΅Π»Π°Ρ‚ΡŒ, — это Ρ€Π°Π·Π΄Π΅Π»ΠΈΡ‚ΡŒ элСмСнты Π² ΠΌΠ°Ρ‚Ρ€ΠΈΡ†Π΅ Π½Π° скаляр, Π½ΠΎ Ρ€Π°Π·Π΄Π΅Π»ΠΈΡ‚ΡŒ ΠΌΠ°Ρ‚Ρ€ΠΈΡ†Ρƒ Π½Π° Π΄Ρ€ΡƒΠ³ΡƒΡŽ ΠΌΠ°Ρ‚Ρ€ΠΈΡ†Ρƒ Π½Π΅Π²ΠΎΠ·ΠΌΠΎΠΆΠ½ΠΎ.

Π‘Π»ΠΎΠΆΠ΅Π½ΠΈΠ΅ ΠΈ Π²Ρ‹Ρ‡ΠΈΡ‚Π°Π½ΠΈΠ΅ ΠΌΠ°Ρ‚Ρ€ΠΈΡ†

ΠœΡ‹ Π½Π°Ρ‡ΠΈΠ½Π°Π΅ΠΌ с Ρ€Π°Π±ΠΎΡ‚Ρ‹ с двумя основными опСрациями, извСстными Π½Π°ΠΌ с Ρ‚Π΅Ρ… ΠΏΠΎΡ€, ΠΊΠ°ΠΊ ΠΌΡ‹ Π²ΠΏΠ΅Ρ€Π²Ρ‹Π΅ познакомились с ΠΌΠ°Ρ‚Π΅ΠΌΠ°Ρ‚ΠΈΠΊΠΎΠΉ Π² β€‹β€‹Π½Π°Ρ‡Π°Π»ΡŒΠ½ΠΎΠΉ школС: слоТСниС ΠΈ Π²Ρ‹Ρ‡ΠΈΡ‚Π°Π½ΠΈΠ΅ (ΠΈΠ»ΠΈ Ρ‡Ρ‚ΠΎ-Ρ‚ΠΎ «плюс» ΠΈ «минус»). Π§Ρ‚ΠΎΠ±Ρ‹ ΠΏΡ€ΠΎΡ€Π°Π±ΠΎΡ‚Π°Ρ‚ΡŒ эти ΠΎΠΏΠ΅Ρ€Π°Ρ†ΠΈΠΈ Π² этом ΡƒΡ€ΠΎΠΊΠ΅, Π΄Π°Π²Π°ΠΉΡ‚Π΅ ΠΎΠΏΡ€Π΅Π΄Π΅Π»ΠΈΠΌ Π½Π°Π±ΠΎΡ€ ΠΈΠ· Π΄Π²ΡƒΡ… ΠΌΠ°Ρ‚Ρ€ΠΈΡ† для слоТСния ΠΈ вычитания. Π˜Ρ‚Π°ΠΊ, опрСдСляСм ΠΌΠ°Ρ‚Ρ€ΠΈΡ†Ρ‹ A ΠΈ B ΡΠ»Π΅Π΄ΡƒΡŽΡ‰ΠΈΠΌ ΠΎΠ±Ρ€Π°Π·ΠΎΠΌ:

Π£Ρ€Π°Π²Π½Π΅Π½ΠΈΠ΅ 1: ΠΌΠ°Ρ‚Ρ€ΠΈΡ†Ρ‹ A ΠΈ B

Π’Π΅ΠΏΠ΅Ρ€ΡŒ с Π½ΠΈΠΌΠΈ ΠΌΡ‹ настроСны для Ρ€Π°Π±ΠΎΡ‚Ρ‹ Ρ‡Π΅Ρ€Π΅Π· ΠΎΠΏΠ΅Ρ€Π°Ρ†ΠΈΠΈ слоТСния ΠΈ вычитания ΠΌΠ°Ρ‚Ρ€ΠΈΡ†. К ΠΎΠΏΠ΅Ρ€Π°Ρ†ΠΈΠΈ Π±ΡƒΠ΄ΡƒΡ‚ ΠΏΡ€ΠΈΠΌΠ΅Π½ΡΡ‚ΡŒΡΡ нСсколько ΠΏΡ€Π°Π²ΠΈΠ», Π½ΠΎ ΠΌΡ‹ ΠΏΠΎΠ³ΠΎΠ²ΠΎΡ€ΠΈΠΌ ΠΎ Π½ΠΈΡ… Π² ΡΠ»Π΅Π΄ΡƒΡŽΡ‰Π΅ΠΌ Ρ€Π°Π·Π΄Π΅Π»Π΅, Π° ΠΏΠΎΠΊΠ° Π΄Π°Π²Π°ΠΉΡ‚Π΅ быстро пройдСмся ΠΏΠΎ процСссу, Π° Π·Π°Ρ‚Π΅ΠΌ ΠΌΡ‹ смоТСм Π²Π΅Ρ€Π½ΡƒΡ‚ΡŒΡΡ ΠΈ ΠΏΡ€ΠΎΠ²Π΅Ρ€ΠΈΡ‚ΡŒ, насколько ΠΎΠΏΠ΅Ρ€Π°Ρ†ΠΈΠΈ Π±Ρ‹Π»ΠΈ Π²ΠΎΠ·ΠΌΠΎΠΆΠ½Ρ‹.НачнСм с добавлСния A ΠΈ B:

Π£Ρ€Π°Π²Π½Π΅Π½ΠΈΠ΅ 2: слоТСниС ΠΌΠ°Ρ‚Ρ€ΠΈΡ† A ΠΈ B

ΠœΡ‹ Π·Π°ΠΊΠΎΠ΄ΠΈΡ€ΠΎΠ²Π°Π»ΠΈ ΠΊΠ°ΠΆΠ΄Ρ‹ΠΉ элСмСнт Π² ΠΌΠ°Ρ‚Ρ€ΠΈΡ†Π°Ρ… Ρ†Π²Π΅Ρ‚ΠΎΠΌ, Ρ‡Ρ‚ΠΎΠ±Ρ‹ Π²Ρ‹ ΠΌΠΎΠ³Π»ΠΈ Π²ΠΈΠ΄Π΅Ρ‚ΡŒ, Ρ‡Ρ‚ΠΎ Π±Ρ‹Π»ΠΎ сдСлано для ΠΊΠ°ΠΆΠ΄ΠΎΠ³ΠΎ элСмСнта. ΠžΠ±Ρ€Π°Ρ‚ΠΈΡ‚Π΅ Π²Π½ΠΈΠΌΠ°Π½ΠΈΠ΅ Π½Π° Ρ‚ΠΎ, ΠΊΠ°ΠΊ слоТСниС Π΄Π²ΡƒΡ… ΠΌΠ°Ρ‚Ρ€ΠΈΡ† дСйствуСт Π² случаС ΠΎΡ‚Π΄Π΅Π»ΡŒΠ½Ρ‹Ρ… элСмСнтов, трСбуя, Ρ‡Ρ‚ΠΎΠ±Ρ‹ Ρƒ вас Π±Ρ‹Π»ΠΈ Ρ‚ΠΎΡ‡Π½ΠΎ Ρ‚Π°ΠΊΠΈΠ΅ ΠΆΠ΅ элСмСнты Π² ΠΌΠ°Ρ‚Ρ€ΠΈΡ†Π°Ρ…, ΡƒΡ‡Π°ΡΡ‚Π²ΡƒΡŽΡ‰ΠΈΡ… Π² ΠΎΠΏΠ΅Ρ€Π°Ρ†ΠΈΠΈ, ΠΈ Ρ‡Ρ‚ΠΎΠ±Ρ‹ ΠΎΠ½ΠΈ Ρ€Π°ΡΠΏΡ€Π΅Π΄Π΅Π»ΡΠ»ΠΈΡΡŒ ΠΎΠ΄ΠΈΠ½Π°ΠΊΠΎΠ²Ρ‹ΠΌ ΠΎΠ±Ρ€Π°Π·ΠΎΠΌ.

ΠœΡ‹ вСрнСмся ΠΊ этой Ρ‚Π΅ΠΌΠ΅ Π² ΡΠ»Π΅Π΄ΡƒΡŽΡ‰Π΅ΠΌ Ρ€Π°Π·Π΄Π΅Π»Π΅, Π½ΠΎ ΠΏΠΎΠΊΠ° Π²Ρ‹ Π΄ΠΎΠ»ΠΆΠ½Ρ‹ ΠΏΠΎΠΌΠ½ΠΈΡ‚ΡŒ, Ρ‡Ρ‚ΠΎ ΠΏΡ€ΠΈ Π΄ΠΎΠ±Π°Π²Π»Π΅Π½ΠΈΠΈ ΠΌΠ°Ρ‚Ρ€ΠΈΡ† Π²Ρ‹ Π±ΡƒΠ΄Π΅Ρ‚Π΅ Π΄ΠΎΠ±Π°Π²Π»ΡΡ‚ΡŒ элСмСнты ΠΏΠ΅Ρ€Π²ΠΎΠΉ ΠΌΠ°Ρ‚Ρ€ΠΈΡ†Ρ‹ ΠΊ ΡΠΎΠΎΡ‚Π²Π΅Ρ‚ΡΡ‚Π²ΡƒΡŽΡ‰Π΅ΠΉ (Ρ‚ΠΎΠΉ, которая находится Π² Β«Ρ‚ΠΎΠΌ ΠΆΠ΅ мСстС», Ρ‚ΠΎΠΉ ΠΆΠ΅ строкС ΠΈ столбцС число) Π²ΠΎ Π²Ρ‚ΠΎΡ€ΠΎΠΉ ΠΌΠ°Ρ‚Ρ€ΠΈΡ†Π΅.

Π’Π΅ΠΏΠ΅Ρ€ΡŒ Π΄Π°Π²Π°ΠΉΡ‚Π΅ посмотрим Π½Π° ΠΌΠ°Ρ‚Ρ€ΠΈΡ‡Π½ΠΎΠ΅ Π²Ρ‹Ρ‡ΠΈΡ‚Π°Π½ΠΈΠ΅:

Π£Ρ€Π°Π²Π½Π΅Π½ΠΈΠ΅ 3: Π²Ρ‹Ρ‡ΠΈΡ‚Π°Π½ΠΈΠ΅ ΠΌΠ°Ρ‚Ρ€ΠΈΡ†Ρ‹ B ΠΈΠ· A

Глядя Π½Π° ΠΏΡ€ΠΈΠ²Π΅Π΄Π΅Π½Π½ΠΎΠ΅ Π²Ρ‹ΡˆΠ΅ Π²Ρ‹Ρ‡ΠΈΡ‚Π°Π½ΠΈΠ΅, ΠΌΡ‹ ΠΌΠΎΠΆΠ΅ΠΌ Π»Π΅Π³ΠΊΠΎ Π·Π°ΠΊΠ»ΡŽΡ‡ΠΈΡ‚ΡŒ, Ρ‡Ρ‚ΠΎ процСсс ΠΎΡ‡Π΅Π½ΡŒ ΠΏΠΎΡ…ΠΎΠΆ Π½Π° процСсс слоТСния, ΠΏΠΎΡΠΊΠΎΠ»ΡŒΠΊΡƒ Π²Ρ‹Ρ‡ΠΈΡ‚Π°Π½ΠΈΠ΅ вычисляСтся ΠΏΠΎ ΠΎΡ‚Π΄Π΅Π»ΡŒΠ½ΠΎΡΡ‚ΠΈ для ΠΊΠ°ΠΆΠ΄ΠΎΠ³ΠΎ ΠΈΠ· элСмСнтов ΠΌΠ°Ρ‚Ρ€ΠΈΡ†, Ρ‚Π°ΠΊΠΈΠΌ ΠΎΠ±Ρ€Π°Π·ΠΎΠΌ давая Ρ€Π΅Π·ΡƒΠ»ΡŒΡ‚Π°Ρ‚ для ΠΊΠ°ΠΆΠ΄ΠΎΠ³ΠΎ ΡΠΎΠΎΡ‚Π²Π΅Ρ‚ΡΡ‚Π²ΡƒΡŽΡ‰Π΅Π³ΠΎ пространства элСмСнтов ΠΈ Π² Ρ€Π΅Π·ΡƒΠ»ΡŒΡ‚Π°Ρ‚Π΅ получаСтся ΠΌΠ°Ρ‚Ρ€ΠΈΡ†Π° с Ρ‚Π°ΠΊΠΈΠΌ ΠΆΠ΅ количСством элСмСнтов Π² Ρ‚ΠΎΠΉ ΠΆΠ΅ ΠΎΡ€Π³Π°Π½ΠΈΠ·Π°Ρ†ΠΈΠΈ.

ΠŸΡ€ΠΎΡ‰Π΅ говоря, слоТСниС ΠΈ Π²Ρ‹Ρ‡ΠΈΡ‚Π°Π½ΠΈΠ΅ ΠΌΠ°Ρ‚Ρ€ΠΈΡ† Ρ€Π°Π±ΠΎΡ‚Π°ΡŽΡ‚ ΠΎΠ΄ΠΈΠ½Π°ΠΊΠΎΠ²ΠΎ, СдинствСнная Ρ€Π°Π·Π½ΠΈΡ†Π° ΠΌΠ΅ΠΆΠ΄Ρƒ Π½ΠΈΠΌΠΈ — это арифмСтичСский Π·Π½Π°ΠΊ, ΠΊΠΎΡ‚ΠΎΡ€Ρ‹ΠΉ Π²Ρ‹ ΠΈΡΠΏΠΎΠ»ΡŒΠ·ΡƒΠ΅Ρ‚Π΅ для Ρ€Π°Π±ΠΎΡ‚Ρ‹: Π·Π½Π°ΠΊ плюс для слоТСния ΠΈ Π·Π½Π°ΠΊ минус для вычитания.

ΠšΡ€Π°ΠΉΠ½Π΅ Π²Π°ΠΆΠ½ΠΎ, Ρ‡Ρ‚ΠΎΠ±Ρ‹ Π²Ρ‹ поняли этот Π²Ρ‹Π²ΠΎΠ΄, ΠΏΠΎΡΠΊΠΎΠ»ΡŒΠΊΡƒ это ΠΌΠΎΠΆΠ΅Ρ‚ ΠΏΠΎΠΌΠΎΡ‡ΡŒ Π²Π°ΠΌ Π½Π΅ Π·Π°ΠΏΡƒΡ‚Π°Ρ‚ΡŒΡΡ ΠΏΡ€ΠΈ Ρ€Π°Π±ΠΎΡ‚Π΅ с Ρ€Π°Π·Π»ΠΈΡ‡Π½ΠΎΠΉ Π»ΠΈΡ‚Π΅Ρ€Π°Ρ‚ΡƒΡ€ΠΎΠΉ ΠΏΠΎ Π΄Π°Π½Π½ΠΎΠΉ Ρ‚Π΅ΠΌΠ΅. Π”Π΅Π»ΠΎ Π² Ρ‚ΠΎΠΌ, Ρ‡Ρ‚ΠΎ Π½Π΅ΠΊΠΎΡ‚ΠΎΡ€Ρ‹Π΅ ΠΌΠ°Ρ‚Π΅ΠΌΠ°Ρ‚ΠΈΠΊΠΈ, ΠΊΠ½ΠΈΠ³ΠΈ ΠΈΠ»ΠΈ ΡƒΡ‡Π΅Π±Π½Ρ‹Π΅ ΠΌΠ°Ρ‚Π΅Ρ€ΠΈΠ°Π»Ρ‹ Π² Ρ†Π΅Π»ΠΎΠΌ ΠΌΠΎΠ³ΡƒΡ‚ ΠΎΡ‚Π½ΠΎΡΠΈΡ‚ΡŒΡΡ ΠΊ опСрациям слоТСния ΠΈ вычитания ΠΊΠ°ΠΊ ΠΊ «слоТСнию» Π² Ρ†Π΅Π»ΠΎΠΌ. Π­Ρ‚Π° конкрСтная ссылка исходит ΠΈΠ· Ρ‚ΠΎΠ³ΠΎ Ρ„Π°ΠΊΡ‚Π°, Ρ‡Ρ‚ΠΎ Π²Ρ‹Ρ‡ΠΈΡ‚Π°Π½ΠΈΠ΅ эквивалСнтно слоТСнию чисСл, Π² ΠΊΠΎΡ‚ΠΎΡ€Ρ‹Ρ… хотя Π±Ρ‹ ΠΎΠ΄Π½ΠΎ ΠΈΠ· Π½ΠΈΡ… ΠΎΡ‚Ρ€ΠΈΡ†Π°Ρ‚Π΅Π»ΡŒΠ½ΠΎ. Если Ρƒ вас Π΅ΡΡ‚ΡŒ ΠΊΠ°ΠΊΠΈΠ΅-Π»ΠΈΠ±ΠΎ вопросы ΠΏΠΎ этому ΠΏΠΎΠ²ΠΎΠ΄Ρƒ, ΠΏΠΎΠΌΠ½ΠΈΡ‚Π΅, Ρ‡Ρ‚ΠΎ Π²Ρ‹Ρ‡ΠΈΡ‚Π°Π½ΠΈΠ΅ Π² Π°Ρ€ΠΈΡ„ΠΌΠ΅Ρ‚ΠΈΠΊΠ΅ Ρ‚Π°ΠΊΠΆΠ΅ называСтся Β«Ρ€Π°Π·Π½ΠΈΡ†Π΅ΠΉΒ», ΠΈ всС ΠΏΠΎΡ‚ΠΎΠΌΡƒ, Ρ‡Ρ‚ΠΎ ΠΊΠΎΠ³Π΄Π° Π²Ρ‹ складываСтС числа с Ρ€Π°Π·Π½Ρ‹ΠΌΠΈ Π·Π½Π°ΠΊΠ°ΠΌΠΈ, Ρ€Π΅Π·ΡƒΠ»ΡŒΡ‚Π°Ρ‚ΠΎΠΌ становится Ρ€Π°Π·Π½ΠΈΡ†Π° ΠΌΠ΅ΠΆΠ΄Ρƒ Π½ΠΈΠΌΠΈ.ΠœΡ‹ рассмотрСли эту Ρ‚Π΅ΠΌΡƒ Π² нашСм ΡƒΡ€ΠΎΠΊΠ΅ ΠΏΠΎ Π²Ρ‹Ρ‡ΠΈΡ‚Π°Π½ΠΈΡŽ Ρ†Π΅Π»Ρ‹Ρ… чисСл, ΠΊΠΎΡ‚ΠΎΡ€Ρ‹ΠΉ, хотя ΠΈ Π½Π΅ являСтся Ρ‡Π°ΡΡ‚ΡŒΡŽ этого курса Π»ΠΈΠ½Π΅ΠΉΠ½ΠΎΠΉ Π°Π»Π³Π΅Π±Ρ€Ρ‹, Π²Ρ‹ ΠΌΠΎΠΆΠ΅Ρ‚Π΅ ΠΏΡ€ΠΎΠ²Π΅Ρ€ΠΈΡ‚ΡŒ Π² любоС врСмя Π½Π° Π²Π΅Π±-сайтС StudyPug. Π’Ρ‹ Ρ‚Π°ΠΊΠΆΠ΅ ΠΌΠΎΠΆΠ΅Ρ‚Π΅ ΠΏΡ€ΠΎΠ²Π΅Ρ€ΠΈΡ‚ΡŒ ΡΠ»Π΅Π΄ΡƒΡŽΡ‰ΡƒΡŽ ΡΡ‚Π°Ρ‚ΡŒΡŽ ΠΎ слоТСнии ΠΈ Π²Ρ‹Ρ‡ΠΈΡ‚Π°Π½ΠΈΠΈ Ρ†Π΅Π»Ρ‹Ρ… чисСл, которая содСрТит Π΄ΠΎΠΏΠΎΠ»Π½ΠΈΡ‚Π΅Π»ΡŒΠ½Ρ‹Π΅ упраТнСния ΠΈ Π½Π΅ΠΊΠΎΡ‚ΠΎΡ€Ρ‹Π΅ графичСскиС изобраТСния.

Π’ΠΊΡ€Π°Ρ‚Ρ†Π΅: Как ΡΠΊΠ»Π°Π΄Ρ‹Π²Π°Ρ‚ΡŒ ΠΌΠ°Ρ‚Ρ€ΠΈΡ†Ρ‹? ΠœΡ‹ добавляСм ΠΊΠ°ΠΆΠ΄Ρ‹ΠΉ ΡΠΎΠΎΡ‚Π²Π΅Ρ‚ΡΡ‚Π²ΡƒΡŽΡ‰ΠΈΠΉ элСмСнт Π² задСйствованныС ΠΌΠ°Ρ‚Ρ€ΠΈΡ†Ρ‹, Ρ‡Ρ‚ΠΎΠ±Ρ‹ ΡΠΎΠ·Π΄Π°Ρ‚ΡŒ Π½ΠΎΠ²ΡƒΡŽ ΠΌΠ°Ρ‚Ρ€ΠΈΡ†Ρƒ, Π² ΠΊΠΎΡ‚ΠΎΡ€ΠΎΠΉ Ρ‚Π°ΠΊΠΈΠ΅ элСмСнты Π±ΡƒΠ΄ΡƒΡ‚ Π·Π°Π½ΠΈΠΌΠ°Ρ‚ΡŒ Ρ‚ΠΎ ΠΆΠ΅ мСсто, Ρ‡Ρ‚ΠΎ ΠΈ ΠΈΡ… ΠΏΡ€Π΅Π΄ΡˆΠ΅ΡΡ‚Π²Π΅Π½Π½ΠΈΠΊΠΈ.Как Π²Ρ‹Ρ‡Π΅ΡΡ‚ΡŒ ΠΌΠ°Ρ‚Ρ€ΠΈΡ†Ρ‹? Π’ΠΎΡ‡Π½ΠΎ Ρ‚Π°ΠΊ ΠΆΠ΅, ΠΊΠ°ΠΊ ΠΌΡ‹ вычисляСм слоТСниС, Π·Π° ΠΈΡΠΊΠ»ΡŽΡ‡Π΅Π½ΠΈΠ΅ΠΌ Ρ‚ΠΎΠ³ΠΎ, Ρ‡Ρ‚ΠΎ ΠΌΡ‹ ΠΈΡΠΏΠΎΠ»ΡŒΠ·ΡƒΠ΅ΠΌ Π·Π½Π°ΠΊ минус для Ρ€Π°Π±ΠΎΡ‚Ρ‹ вмСсто Π·Π½Π°ΠΊΠ° плюс.

ΠŸΡ€Π°Π²ΠΈΠ»Π° слоТСния ΠΈ вычитания ΠΌΠ°Ρ‚Ρ€ΠΈΡ†

ΠœΠ°Ρ‚Ρ€ΠΈΡ‡Π½ΠΎΠ΅ слоТСниС ΠΈ Π²Ρ‹Ρ‡ΠΈΡ‚Π°Π½ΠΈΠ΅ — это Π΄Π²Π΅ ΠΎΠΏΠ΅Ρ€Π°Ρ†ΠΈΠΈ, ΠΊΠΎΡ‚ΠΎΡ€Ρ‹Π΅, хотя ΠΈ просты для Ρ€Π΅ΡˆΠ΅Π½ΠΈΡ (ΠΊΠ°ΠΊ ΠΏΠΎΠΊΠ°Π·Π°Π½ΠΎ Π²Ρ‹ΡˆΠ΅), Ρƒ вас всС ΠΆΠ΅ Π΅ΡΡ‚ΡŒ нСсколько ΠΏΡ€Π°Π²ΠΈΠ», ΠΊΠΎΡ‚ΠΎΡ€Ρ‹ΠΌ Π½ΡƒΠΆΠ½ΠΎ ΡΠ»Π΅Π΄ΠΎΠ²Π°Ρ‚ΡŒ, Ρ‡Ρ‚ΠΎΠ±Ρ‹ Ρ€Π°Π±ΠΎΡ‚Π°Ρ‚ΡŒ.

НапримСр, ΠΊΠΎΠ³Π΄Π° ΠΌΡ‹ добавляли ΠΌΠ°Ρ‚Ρ€ΠΈΡ†Ρƒ A ΠΊ ΠΌΠ°Ρ‚Ρ€ΠΈΡ†Π΅ B Π²Ρ‹ΡˆΠ΅, сначала ΠΎΠ±Ρ€Π°Ρ‚ΠΈΡ‚Π΅ Π²Π½ΠΈΠΌΠ°Π½ΠΈΠ΅, Ρ‡Ρ‚ΠΎ A ΠΈ B — ΠΎΡ‡Π΅Π½ΡŒ ΠΏΠΎΡ…ΠΎΠΆΠΈΠ΅ ΠΌΠ°Ρ‚Ρ€ΠΈΡ†Ρ‹, ΠΎΠ½ΠΈ ΠΈΠΌΠ΅ΡŽΡ‚ ΠΎΠ΄ΠΈΠ½Π°ΠΊΠΎΠ²ΠΎΠ΅ количСство строк ΠΈ столбцов каТдая ΠΈ опСрация слоТСния примСняСтся ΠΊ ΠΊΠ°ΠΆΠ΄ΠΎΠΌΡƒ Β«ΡΠΎΠΎΡ‚Π²Π΅Ρ‚ΡΡ‚Π²ΡƒΡŽΡ‰Π΅ΠΌΡƒ Ρ‡Π»Π΅Π½ΡƒΒ» Π² Π½ΠΈΡ….Π­Ρ‚ΠΎ ΠΏΡ€ΠΈΠ²ΠΎΠ΄ΠΈΡ‚ ΠΊ основному ΠΏΡ€Π°Π²ΠΈΠ»Ρƒ слоТСния ΠΈ вычитания ΠΌΠ°Ρ‚Ρ€ΠΈΡ†, ΠΊΠΎΡ‚ΠΎΡ€ΠΎΠ΅ соотвСтствуСт Ρ€Π°Π·ΠΌΠ΅Ρ€Π°ΠΌ ΠΌΠ°Ρ‚Ρ€ΠΈΡ†, с ΠΊΠΎΡ‚ΠΎΡ€Ρ‹ΠΌΠΈ Ρ€Π°Π±ΠΎΡ‚Π°ΡŽΡ‚ Π΄Ρ€ΡƒΠ³ с Π΄Ρ€ΡƒΠ³ΠΎΠΌ.

Π’Π°ΠΊΠΆΠ΅ ΠΎΠ±Ρ€Π°Ρ‚ΠΈΡ‚Π΅ Π²Π½ΠΈΠΌΠ°Π½ΠΈΠ΅, Ρ‡Ρ‚ΠΎ Π²Ρ‹Ρ‡ΠΈΡ‚Π°Π½ΠΈΠ΅ ΠΌΠ°Ρ‚Ρ€ΠΈΡ† происходит Π² ΠΎΡ‡Π΅Π½ΡŒ ΠΏΠΎΡ…ΠΎΠΆΠ΅ΠΌ Ρ€Π΅ΠΆΠΈΠΌΠ΅, Π½Π° самом Π΄Π΅Π»Π΅ СдинствСнноС ΠΈΠ·ΠΌΠ΅Π½Π΅Π½ΠΈΠ΅ состоит Π² Ρ‚ΠΎΠΌ, Ρ‡Ρ‚ΠΎ вмСсто Π·Π½Π°ΠΊΠ° слоТСния ΠΌΠ΅ΠΆΠ΄Ρƒ ΠΌΠ°Ρ‚Ρ€ΠΈΡ†Π°ΠΌΠΈ ΠΈ ΡΠΎΠΎΡ‚Π²Π΅Ρ‚ΡΡ‚Π²ΡƒΡŽΡ‰ΠΈΠΌΠΈ элСмСнтами Π² ΠΊΠ°ΠΆΠ΄ΠΎΠΌ ΠΌΠ°Ρ‚Ρ€ΠΈΡ‡Π½ΠΎΠΌ пространствС Ρƒ вас Π΅ΡΡ‚ΡŒ Π·Π½Π°ΠΊ вычитания.

Π‘ ΡƒΡ‡Π΅Ρ‚ΠΎΠΌ всСго этого ΠΏΡ€Π°Π²ΠΈΠ»Π° слоТСния ΠΈ вычитания ΠΌΠ°Ρ‚Ρ€ΠΈΡ† ΡΠ»Π΅Π΄ΡƒΡŽΡ‰ΠΈΠ΅:

  1. Π’Ρ‹ ΠΌΠΎΠΆΠ΅Ρ‚Π΅ ΡΠΊΠ»Π°Π΄Ρ‹Π²Π°Ρ‚ΡŒ ΠΈΠ»ΠΈ Π²Ρ‹Ρ‡ΠΈΡ‚Π°Ρ‚ΡŒ Ρ‚ΠΎΠ»ΡŒΠΊΠΎ ΠΌΠ°Ρ‚Ρ€ΠΈΡ†Ρ‹ с ΠΎΠ΄ΠΈΠ½Π°ΠΊΠΎΠ²Ρ‹ΠΌΠΈ Ρ€Π°Π·ΠΌΠ΅Ρ€Π°ΠΌΠΈ.
    1. Если Π·Π°Π΄ΡƒΠΌΠ°Ρ‚ΡŒΡΡ, это происходит ΠΈΠ·-Π·Π° Ρ‚ΠΎΠ³ΠΎ, Ρ‡Ρ‚ΠΎ Π΄ΠΎΠ±Π°Π²Π»Π΅Π½ΠΈΠ΅ ΠΌΠ°Ρ‚Ρ€ΠΈΡ† с Ρ€Π°Π·Π½Ρ‹ΠΌΠΈ Ρ€Π°Π·ΠΌΠ΅Ρ€Π°ΠΌΠΈ создаСт ΠΏΡ€ΠΎΠ±Π»Π΅ΠΌΡƒ, ΠΏΠΎΡ‚ΠΎΠΌΡƒ Ρ‡Ρ‚ΠΎ Π½Π΅ всС элСмСнты Π² ΠΊΠ°ΠΆΠ΄ΠΎΠΉ ΠΌΠ°Ρ‚Ρ€ΠΈΡ†Π΅ Π±ΡƒΠ΄ΡƒΡ‚ ΠΈΠΌΠ΅Ρ‚ΡŒ ΡΠΎΠΎΡ‚Π²Π΅Ρ‚ΡΡ‚Π²ΡƒΡŽΡ‰ΠΈΠΉ элСмСнт для Ρ€Π°Π±ΠΎΡ‚Ρ‹, Ρ‡Ρ‚ΠΎ Π΄Π΅Π»Π°Π΅Ρ‚ ΠΎΠΏΠ΅Ρ€Π°Ρ†ΠΈΡŽ Π½Π΅Π²ΠΎΠ·ΠΌΠΎΠΆΠ½ΠΎΠΉ.
  2. Π‘Π»ΠΎΠΆΠ΅Π½ΠΈΠ΅ ΠΌΠ°Ρ‚Ρ€ΠΈΡ† являСтся ΠΊΠΎΠΌΠΌΡƒΡ‚Π°Ρ‚ΠΈΠ²Π½Ρ‹ΠΌ, Π½ΠΎ Π²Ρ‹Ρ‡ΠΈΡ‚Π°Π½ΠΈΠ΅ — Π½Π΅Ρ‚, Ссли сначала Π²Ρ‹ Π½Π΅ ΠΏΡ€ΠΈΠΌΠ΅Π½ΠΈΡ‚Π΅ Π·Π½Π°ΠΊ минуса ΠΊ ΠΌΠ°Ρ‚Ρ€ΠΈΡ†Π΅ справа, Ρ‚Π°ΠΊΠΈΠΌ ΠΎΠ±Ρ€Π°Π·ΠΎΠΌ сначала прСобразуя ΠΎΠΏΠ΅Ρ€Π°Ρ†ΠΈΡŽ Π² слоТСниС.
    1. Бвойство коммутативности ΠΎΠ·Π½Π°Ρ‡Π°Π΅Ρ‚, Ρ‡Ρ‚ΠΎ ΠΏΡ€ΠΈ Π΄ΠΎΠ±Π°Π²Π»Π΅Π½ΠΈΠΈ Π΄Π²ΡƒΡ… ΠΌΠ°Ρ‚Ρ€ΠΈΡ† Π²Ρ‹ ΠΌΠΎΠΆΠ΅Ρ‚Π΅ ΠΈΠ·ΠΌΠ΅Π½ΠΈΡ‚ΡŒ порядок ΠΈΡ… добавлСния, ΠΈ это Π½Π΅ повлияСт Π½Π° Ρ€Π΅Π·ΡƒΠ»ΡŒΡ‚Π°Ρ‚.
    2. Π­Ρ‚ΠΎ Π½Π΅ Ρ‚Π°ΠΊ просто ΠΏΡ€ΠΈ Π²Ρ‹Ρ‡ΠΈΡ‚Π°Π½ΠΈΠΈ. Π’ ΡƒΡ€Π°Π²Π½Π΅Π½ΠΈΠΈ 3 Ρƒ нас Π΅ΡΡ‚ΡŒ ΠΏΡ€ΠΈΠΌΠ΅Ρ€ вычитания ΠΌΠ°Ρ‚Ρ€ΠΈΡ†, ΠΎΠ±Ρ€Π°Ρ‚ΠΈΡ‚Π΅ Π²Π½ΠΈΠΌΠ°Π½ΠΈΠ΅, Ρ‡Ρ‚ΠΎ Ссли Π²Ρ‹ просто ΠΈΠ·ΠΌΠ΅Π½ΠΈΡ‚Π΅ порядок ΠΌΠ°Ρ‚Ρ€ΠΈΡ†, Π½Π΅ пСрСмСщая Π·Π½Π°ΠΊ минус, Ρ€Π΅Π·ΡƒΠ»ΡŒΡ‚Π°Ρ‚ Π½Π΅ Π±ΡƒΠ΄Π΅Ρ‚ Ρ‚Π°ΠΊΠΈΠΌ ΠΆΠ΅, Ρ‚ΠΎΠ³Π΄Π° ΠΊΠ°ΠΊ Π²Ρ‹Ρ‡ΠΈΡ‚Π°Π½ΠΈΠ΅ ΠΌΠ°Ρ‚Ρ€ΠΈΡ† ΠΌΠΎΠΆΠ΅Ρ‚ Π±Ρ‹Ρ‚ΡŒ ΠΊΠΎΠΌΠΌΡƒΡ‚Π°Ρ‚ΠΈΠ²Π½Ρ‹ΠΌ? Π’Ρ‹ Π΄ΠΎΠ»ΠΆΠ½Ρ‹ ΠΏΡ€ΠΈΠΌΠ΅Π½ΠΈΡ‚ΡŒ ΠΎΡ‚Ρ€ΠΈΡ†Π°Ρ‚Π΅Π»ΡŒΠ½Ρ‹ΠΉ Π·Π½Π°ΠΊ ΠΊ ΠΌΠ°Ρ‚Ρ€ΠΈΡ†Π΅, которая вычитаСтся ΠΏΠ΅Ρ€Π²ΠΎΠΉ! ΠŸΡ€ΠΎΠ²Π΅Ρ€ΡŒΡ‚Π΅ процСсс шаг Π·Π° шагом Π½ΠΈΠΆΠ΅: Π£Ρ€Π°Π²Π½Π΅Π½ΠΈΠ΅ 4: УсловиС, Ρ‡Ρ‚ΠΎΠ±Ρ‹ ΡΠ΄Π΅Π»Π°Ρ‚ΡŒ Π²Ρ‹Ρ‡ΠΈΡ‚Π°Π½ΠΈΠ΅ ΠΌΠ°Ρ‚Ρ€ΠΈΡ†Ρ‹ ΠΊΠΎΠΌΠΌΡƒΡ‚Π°Ρ‚ΠΈΠ²Π½Ρ‹ΠΌ = сначала ΠΏΡ€Π΅ΠΎΠ±Ρ€Π°Π·ΠΎΠ²Π°Ρ‚ΡŒ Π΅Π³ΠΎ Π² слоТСниС! ΠžΠ±Ρ€Π°Ρ‚ΠΈΡ‚Π΅ Π²Π½ΠΈΠΌΠ°Π½ΠΈΠ΅, ΠΊΠ°ΠΊ для Ρ‚ΠΎΠ³ΠΎ, Ρ‡Ρ‚ΠΎΠ±Ρ‹ ΠΈΠΌΠ΅Ρ‚ΡŒ Π²ΠΎΠ·ΠΌΠΎΠΆΠ½ΠΎΡΡ‚ΡŒ ΠΈΠ·ΠΌΠ΅Π½ΡΡ‚ΡŒ порядок ΠΌΠ°Ρ‚Ρ€ΠΈΡ†, Π½Π΅ влияя Π½Π° Ρ€Π΅Π·ΡƒΠ»ΡŒΡ‚Π°Ρ‚, ΠΌΡ‹ Π΄ΠΎΠ»ΠΆΠ½Ρ‹ ΠΏΡ€Π΅ΠΎΠ±Ρ€Π°Π·ΠΎΠ²Π°Ρ‚ΡŒ Π²Ρ‹Ρ‡ΠΈΡ‚Π°Π½ΠΈΠ΅ Π² слоТСниС, Ρ‡Ρ‚ΠΎ просто ΠΎΠ·Π½Π°Ρ‡Π°Π΅Ρ‚, Ρ‡Ρ‚ΠΎ Π²Ρ‹ Π΄ΠΎΠ»ΠΆΠ½Ρ‹ сначала ΠΏΡ€ΠΈΠΌΠ΅Π½ΠΈΡ‚ΡŒ ΠΎΡ‚Ρ€ΠΈΡ†Π°Ρ‚Π΅Π»ΡŒΠ½Ρ‹ΠΉ Π·Π½Π°ΠΊ ΠΊΠΎ Π²Ρ‚ΠΎΡ€ΠΎΠΉ ΠΌΠ°Ρ‚Ρ€ΠΈΡ†Π΅, Ρ‚Π΅ΠΌ самым дСлая всС ΠΎΡ‚Ρ€ΠΈΡ†Π°Ρ‚Π΅Π»ΡŒΠ½Ρ‹Π΅ числа Π΅Π³ΠΎ элСмСнтов (ΠΊΠΎΡ‚ΠΎΡ€Ρ‹Π΅ ΠΏΡ€ΠΎΠΈΠ·Π²Π΅Π΄ΡƒΡ‚ ΠΎΠΏΠ΅Ρ€Π°Ρ†ΠΈΡŽ вычитания, Π½Π΅ бСспокоясь ΠΎ порядкС элСмСнтов).Π­Ρ‚ΠΎ СдинствСнный способ ΠΈΠ·ΠΌΠ΅Π½ΠΈΡ‚ΡŒ порядок ΠΌΠ°Ρ‚Ρ€ΠΈΡ† ΠΏΡ€ΠΈ Π²Ρ‹Ρ‡ΠΈΡ‚Π°Π½ΠΈΠΈ, поэтому ΠΏΠΎΠΌΠ½ΠΈΡ‚Π΅ ΠΎΠ± этом процСссС ΠΏΡ€ΠΈ Ρ€Π°Π±ΠΎΡ‚Π΅ с Π΅Ρ‰Π΅ Π±ΠΎΠ»Π΅Π΅ слоТными ΠΏΡ€ΠΎΠ±Π»Π΅ΠΌΠ°ΠΌΠΈ: Π½Π°ΠΏΡ€ΠΈΠΌΠ΅Ρ€, Π² случаях, ΠΊΠΎΠ³Π΄Π° Ρƒ вас ΠΌΠΎΠΆΠ΅Ρ‚ Π±Ρ‹Ρ‚ΡŒ Π±ΠΎΠ»Π΅Π΅ Π΄Π²ΡƒΡ… ΠΌΠ°Ρ‚Ρ€ΠΈΡ† ΠΈ Π²Ρ‹ ΡΠ²ΡΠ·Ρ‹Π²Π°Π΅Ρ‚Π΅ΡΡŒ Π² Π³Ρ€ΡƒΠΏΠΏΠ°Ρ…. ΠŸΠΎΠ΄Ρ€ΠΎΠ±Π½Π΅Π΅ ΠΎΠ± ассоциации Π² ΡΠ»Π΅Π΄ΡƒΡŽΡ‰Π΅ΠΌ ΠΏΡƒΠ½ΠΊΡ‚Π΅.
  3. ΠœΠ°Ρ‚Ρ€ΠΈΡ‡Π½ΠΎΠ΅ слоТСниС ΠΈ Π²Ρ‹Ρ‡ΠΈΡ‚Π°Π½ΠΈΠ΅ ассоциативны.
    1. Бвойство ассоциативности ΠΈΡΠΏΠΎΠ»ΡŒΠ·ΡƒΠ΅Ρ‚ΡΡ Π² Π·Π°Π΄Π°Ρ‡Π°Ρ…, Π²ΠΊΠ»ΡŽΡ‡Π°ΡŽΡ‰ΠΈΡ… Π±ΠΎΠ»Π΅Π΅ Π΄Π²ΡƒΡ… ΠΌΠ°Ρ‚Ρ€ΠΈΡ†, ΠΊΠΎΡ‚ΠΎΡ€Ρ‹Π΅ ΡΠΊΠ»Π°Π΄Ρ‹Π²Π°ΡŽΡ‚ΡΡ ΠΈΠ»ΠΈ Π²Ρ‹Ρ‡ΠΈΡ‚Π°ΡŽΡ‚ΡΡ Π΄Ρ€ΡƒΠ³ ΠΈΠ· Π΄Ρ€ΡƒΠ³Π°.Π­Ρ‚ΠΎ свойство позволяСт Π²Π°ΠΌ сначала Ρ€Π°Π±ΠΎΡ‚Π°Ρ‚ΡŒ с ΠΎΠΏΠ΅Ρ€Π°Ρ†ΠΈΠ΅ΠΉ ΠΌΠ΅ΠΆΠ΄Ρƒ двумя ΠΌΠ°Ρ‚Ρ€ΠΈΡ†Π°ΠΌΠΈ, Π° Π·Π°Ρ‚Π΅ΠΌ ΠΈΡΠΏΠΎΠ»ΡŒΠ·ΠΎΠ²Π°Ρ‚ΡŒ ΠΌΠ°Ρ‚Ρ€ΠΈΡ†Ρƒ, ΠΏΠΎΠ»ΡƒΡ‡Π΅Π½Π½ΡƒΡŽ Π² Ρ€Π΅Π·ΡƒΠ»ΡŒΡ‚Π°Ρ‚Π΅ Ρ‚Π°ΠΊΠΎΠΉ ΠΎΠΏΠ΅Ρ€Π°Ρ†ΠΈΠΈ, для выполнСния Π²Ρ‚ΠΎΡ€ΠΎΠΉ Ρ‚Π΅ΠΊΡƒΡ‰Π΅ΠΉ ΠΎΠΏΠ΅Ρ€Π°Ρ†ΠΈΠΈ. ΠŸΠΎΠ²Ρ‚ΠΎΡ€ΡΠΉΡ‚Π΅ Ρ‚Π°ΠΊΠΎΠΉ процСсс, ΠΏΠΎΠΊΠ° Π½Π΅ Π²Ρ‹ΠΏΠΎΠ»Π½ΠΈΡ‚Π΅ всС Π½Π΅ΠΎΠ±Ρ…ΠΎΠ΄ΠΈΠΌΡ‹Π΅ ΠΎΠΏΠ΅Ρ€Π°Ρ†ΠΈΠΈ, Π·Π°Ρ‚Ρ€Π°Π³ΠΈΠ²Π°ΡŽΡ‰ΠΈΠ΅ всС ΠΌΠ°Ρ‚Ρ€ΠΈΡ†Ρ‹ Π² Π·Π°Π΄Π°Ρ‡Π΅.
  4. ΠŸΡ€ΠΈ слоТСнии ΠΈΠ»ΠΈ Π²Ρ‹Ρ‡ΠΈΡ‚Π°Π½ΠΈΠΈ ΠΌΠ°Ρ‚Ρ€ΠΈΡ† Ρ€Π΅Π·ΡƒΠ»ΡŒΡ‚ΠΈΡ€ΡƒΡŽΡ‰Π°Ρ ΠΌΠ°Ρ‚Ρ€ΠΈΡ†Π° Π±ΡƒΠ΄Π΅Ρ‚ ΠΈΠΌΠ΅Ρ‚ΡŒ Ρ‚Π΅ ΠΆΠ΅ Ρ€Π°Π·ΠΌΠ΅Ρ€Ρ‹, Ρ‡Ρ‚ΠΎ ΠΈ ΠΌΠ°Ρ‚Ρ€ΠΈΡ†Π°, которая Π΅Π΅ ΠΏΡ€ΠΎΠΈΠ·Π²Π΅Π»Π°.
    1. Π­Ρ‚ΠΎ ΠΈΠΌΠ΅Π΅Ρ‚ смысл, ΠΏΠΎΡΠΊΠΎΠ»ΡŒΠΊΡƒ ΠΊΠ°ΠΆΠ΄Ρ‹ΠΉ элСмСнт Π² ΠΏΠ΅Ρ€Π²ΠΎΠΉ ΠΌΠ°Ρ‚Ρ€ΠΈΡ†Π΅ Π±ΡƒΠ΄Π΅Ρ‚ Ρ€Π°Π±ΠΎΡ‚Π°Ρ‚ΡŒ с ΡΠΎΠΎΡ‚Π²Π΅Ρ‚ΡΡ‚Π²ΡƒΡŽΡ‰ΠΈΠΌ элСмСнтом Π²Ρ‚ΠΎΡ€ΠΎΠΉ ΠΌΠ°Ρ‚Ρ€ΠΈΡ†Ρ‹, Ρ‚Π°ΠΊΠΈΠΌ ΠΎΠ±Ρ€Π°Π·ΠΎΠΌ производя Ρ€Π΅Π·ΡƒΠ»ΡŒΡ‚Π°Ρ‚ для ΠΊΠ°ΠΆΠ΄ΠΎΠ³ΠΎ пространства элСмСнтов.Π—Π°Ρ‚Π΅ΠΌ Ρ‚Π°ΠΊΠΈΠ΅ Ρ€Π΅Π·ΡƒΠ»ΡŒΡ‚Π°Ρ‚Ρ‹ станут элСмСнтами Π½ΠΎΠ²ΠΎΠΉ ΠΌΠ°Ρ‚Ρ€ΠΈΡ†Ρ‹.

ΠŸΡ€ΠΈΠΌΠ΅Ρ€Ρ‹ слоТСния ΠΈ вычитания

Π”Π°Π»Π΅Π΅ ΠΌΡ‹ ΠΏΠΎΠΊΠ°ΠΆΠ΅ΠΌ ΠΏΡ€ΠΈΠΌΠ΅Ρ€Ρ‹ слоТСния ΠΈ вычитания ΠΌΠ°Ρ‚Ρ€ΠΈΡ†. Π’ ΠΏΠ΅Ρ€Π²ΠΎΠΌ ΠΏΡ€ΠΈΠΌΠ΅Ρ€Π΅ упраТнСния ΠΌΡ‹ сосрСдоточимся Π½Π° дСмонстрации условий, Π² ΠΊΠΎΡ‚ΠΎΡ€Ρ‹Ρ… Ρ‚Π°ΠΊΠΈΠ΅ ΠΎΠΏΠ΅Ρ€Π°Ρ†ΠΈΠΈ Π²ΠΎΠ·ΠΌΠΎΠΆΠ½Ρ‹. Π—Π°Ρ‚Π΅ΠΌ ΠΌΡ‹ Ρ€Π΅ΡˆΠ°Π΅ΠΌ нСсколько ΠΏΡ€ΠΈΠΌΠ΅Ρ€ΠΎΠ² ΠΌΠ°Ρ‚Ρ€ΠΈΡ‡Π½Ρ‹Ρ… Π·Π°Π΄Π°Ρ‡ с ΠΏΠΎΡˆΠ°Π³ΠΎΠ²Ρ‹ΠΌ объяснСниСм ΠΎΠΏΠ΅Ρ€Π°Ρ†ΠΈΠΉ. Π’ ΠΊΠΎΠ½Ρ†Π΅ этого Ρ€Π°Π·Π΄Π΅Π»Π° ΠΌΡ‹ Ρ€Π΅ΡˆΠΈΠΌ ΠΌΠ°Ρ‚Ρ€ΠΈΡ‡Π½Ρ‹Π΅ уравнСния, ΠΊΠΎΡ‚ΠΎΡ€Ρ‹Π΅ Π²ΠΊΠ»ΡŽΡ‡Π°ΡŽΡ‚ Π»ΠΈΠ±ΠΎ слоТСниС, Π»ΠΈΠ±ΠΎ Π²Ρ‹Ρ‡ΠΈΡ‚Π°Π½ΠΈΠ΅ ΠΌΠ°Ρ‚Ρ€ΠΈΡ†.

ΠŸΡ€ΠΈΠΌΠ΅Ρ€ 1

МоТСм Π»ΠΈ ΠΌΡ‹ ΡΠ»ΠΎΠΆΠΈΡ‚ΡŒ ΠΈΠ»ΠΈ Π²Ρ‹Ρ‡Π΅ΡΡ‚ΡŒ ΡΠ»Π΅Π΄ΡƒΡŽΡ‰ΠΈΠ΅ ΠΌΠ°Ρ‚Ρ€ΠΈΡ†Ρ‹?

  • Π‘Π»ΡƒΡ‡Π°ΠΉ 1: Π£Ρ€Π°Π²Π½Π΅Π½ΠΈΠ΅ 5: ΠœΠΎΠΆΠ΅Ρ‚Π΅ Π»ΠΈ Π²Ρ‹ ΡΠ»ΠΎΠΆΠΈΡ‚ΡŒ эти Π΄Π²Π΅ ΠΌΠ°Ρ‚Ρ€ΠΈΡ†Ρ‹ вмСстС? (Π”Π΅Π»ΠΎ 1) ΠžΠ±Ρ€Π°Ρ‚ΠΈΡ‚Π΅ Π²Π½ΠΈΠΌΠ°Π½ΠΈΠ΅, Ρ‡Ρ‚ΠΎ Ρ€Π°Π·ΠΌΠ΅Ρ€Ρ‹ ΠΌΠ°Ρ‚Ρ€ΠΈΡ†, ΡƒΡ‡Π°ΡΡ‚Π²ΡƒΡŽΡ‰ΠΈΡ… Π² слоТСнии, Π½Π΅ ΡΠΎΠ²ΠΏΠ°Π΄Π°ΡŽΡ‚. ΠŸΠ΅Ρ€Π²Π°Ρ ΠΌΠ°Ρ‚Ρ€ΠΈΡ†Π° ΠΈΠΌΠ΅Π΅Ρ‚ Ρ‚Ρ€ΠΈ строки ΠΈ Π΄Π²Π° столбца, Ρ‚Π°ΠΊΠΈΠΌ ΠΎΠ±Ρ€Π°Π·ΠΎΠΌ, это ΠΌΠ°Ρ‚Ρ€ΠΈΡ†Π° 3×2, Π° вторая содСрТит Π΄Π²Π΅ строки ΠΈ Ρ‚Ρ€ΠΈ столбца, Ρ‡Ρ‚ΠΎ Π΄Π΅Π»Π°Π΅Ρ‚ Π΅Π΅ Ρ€Π°Π·ΠΌΠ΅Ρ€ Ρ€Π°Π²Π½Ρ‹ΠΌ 2×3. НаличиС Ρ‚Π°ΠΊΠΈΡ… Ρ€Π°Π·Π»ΠΈΡ‡ΠΈΠΉ ΠΎΠ·Π½Π°Ρ‡Π°Π΅Ρ‚, Ρ‡Ρ‚ΠΎ, хотя ΠΎΠ½ΠΈ ΠΈΠΌΠ΅ΡŽΡ‚ ΠΎΠ΄ΠΈΠ½Π°ΠΊΠΎΠ²ΠΎΠ΅ количСство элСмСнтов, ΠΈΡ… ΠΏΠΎΠ·ΠΈΡ†ΠΈΠΈ Π½Π΅ ΡΠΎΠΎΡ‚Π²Π΅Ρ‚ΡΡ‚Π²ΡƒΡŽΡ‚ Π΄Ρ€ΡƒΠ³ Π΄Ρ€ΡƒΠ³Ρƒ, ΠΈ поэтому ΠΏΠΎ ΠΏΡ€Π°Π²ΠΈΠ»Π°ΠΌ слоТСния ΠΌΠ°Ρ‚Ρ€ΠΈΡ† слоТСниС Π½Π΅Π²ΠΎΠ·ΠΌΠΎΠΆΠ½ΠΎ.
  • Π‘Π»ΡƒΡ‡Π°ΠΉ 2: Π£Ρ€Π°Π²Π½Π΅Π½ΠΈΠ΅ 6: ΠœΠΎΠΆΠ΅Ρ‚Π΅ Π»ΠΈ Π²Ρ‹ ΡΠ»ΠΎΠΆΠΈΡ‚ΡŒ эти Π΄Π²Π΅ ΠΌΠ°Ρ‚Ρ€ΠΈΡ†Ρ‹ вмСстС? (случай 2) Учитывая, Ρ‡Ρ‚ΠΎ Π² этом случаС ΠΌΡ‹ Π±ΡƒΠ΄Π΅ΠΌ Π΄ΠΎΠ±Π°Π²Π»ΡΡ‚ΡŒ ΠΌΠ°Ρ‚Ρ€ΠΈΡ†Ρ‹ 2×2 (явно; ΠΎΠ±Π΅ ΠΌΠ°Ρ‚Ρ€ΠΈΡ†Ρ‹ ΠΈΠΌΠ΅ΡŽΡ‚ ΠΎΠ΄ΠΈΠ½Π°ΠΊΠΎΠ²ΡƒΡŽ Ρ€Π°Π·ΠΌΠ΅Ρ€Π½ΠΎΡΡ‚ΡŒ 2×2), Ρ‚ΠΎΠ³Π΄Π° Π΄Π°, Π΄ΠΎΠ±Π°Π²Π»Π΅Π½ΠΈΠ΅ этих Π΄Π²ΡƒΡ… ΠΌΠ°Ρ‚Ρ€ΠΈΡ† Π²ΠΎΠ·ΠΌΠΎΠΆΠ½ΠΎ ΠΏΡƒΡ‚Π΅ΠΌ добавлСния ΠΊΠ°ΠΆΠ΄ΠΎΠ³ΠΎ ΠΈΠ· элСмСнтов Π² ΠΏΠ΅Ρ€Π²ΠΎΠΉ ΠΌΠ°Ρ‚Ρ€ΠΈΡ†Π΅ с ΡΠΎΠΎΡ‚Π²Π΅Ρ‚ΡΡ‚Π²ΡƒΡŽΡ‰ΠΈΠΌ Π΅ΠΌΡƒ элСмСнтом ΠΈΠ· вторая ΠΌΠ°Ρ‚Ρ€ΠΈΡ†Π°. Π Π΅Π·ΡƒΠ»ΡŒΡ‚ΠΈΡ€ΡƒΡŽΡ‰Π°Ρ ΠΌΠ°Ρ‚Ρ€ΠΈΡ†Π° Ρ‚Π°ΠΊΠΆΠ΅ Π±ΡƒΠ΄Π΅Ρ‚ ΠΈΠΌΠ΅Ρ‚ΡŒ Ρ€Π°Π·ΠΌΠ΅Ρ€Ρ‹ 2×2.
  • Π‘Π»ΡƒΡ‡Π°ΠΉ 3: Π£Ρ€Π°Π²Π½Π΅Π½ΠΈΠ΅ 7: ΠœΠΎΠΆΠ΅Ρ‚Π΅ Π»ΠΈ Π²Ρ‹ ΡΠ»ΠΎΠΆΠΈΡ‚ΡŒ эти Π΄Π²Π΅ ΠΌΠ°Ρ‚Ρ€ΠΈΡ†Ρ‹ вмСстС? (случай 3) Как Π²Ρ‹, Π²ΠΎΠ·ΠΌΠΎΠΆΠ½ΠΎ, ΡƒΠΆΠ΅ ΠΏΠΎΠ΄ΡƒΠΌΠ°Π»ΠΈ, Π΄ΠΎΠ±Π°Π²ΠΈΡ‚ΡŒ 2 ΠΌΠ°Ρ‚Ρ€ΠΈΡ†Ρ‹, ΠΏΠΎΠ΄ΠΎΠ±Π½Ρ‹Π΅ ΠΏΡ€ΠΈΠ²Π΅Π΄Π΅Π½Π½Ρ‹ΠΌ Π²Ρ‹ΡˆΠ΅, Π½Π΅Π²ΠΎΠ·ΠΌΠΎΠΆΠ½ΠΎ.Π Π°Π·ΠΌΠ΅Ρ€Ρ‹ этих ΠΌΠ°Ρ‚Ρ€ΠΈΡ† Π½Π΅ ΡΠΎΠ²ΠΏΠ°Π΄Π°ΡŽΡ‚ (ΠΎΠ΄Π½Π° прСдставляСт собой ΠΌΠ°Ρ‚Ρ€ΠΈΡ†Ρƒ 3×3, Π° другая — 2×2), поэтому это Π΄ΠΎΠ±Π°Π²Π»Π΅Π½ΠΈΠ΅ Π½Π΅Π²ΠΎΠ·ΠΌΠΎΠΆΠ½ΠΎ.

ΠŸΡ€ΠΈΠΌΠ΅Ρ€ 2

Π‘ΠΊΠ»Π°Π΄Ρ‹Π²Π°Π΅ΠΌ ΡΠ»Π΅Π΄ΡƒΡŽΡ‰ΠΈΠ΅ Π΄Π²Π΅ ΠΌΠ°Ρ‚Ρ€ΠΈΡ†Ρ‹:

Π£Ρ€Π°Π²Π½Π΅Π½ΠΈΠ΅ 8: слоТСниС Π΄Π²ΡƒΡ… ΠΌΠ°Ρ‚Ρ€ΠΈΡ†

ΠŸΡ€ΠΈ Π΄ΠΎΠ±Π°Π²Π»Π΅Π½ΠΈΠΈ Π΄Π²ΡƒΡ… ΠΌΠ°Ρ‚Ρ€ΠΈΡ† ΠΊΠ°ΠΆΠ΄Ρ‹ΠΉ элСмСнт добавляСтся ΠΊ ΡΠΎΠΎΡ‚Π²Π΅Ρ‚ΡΡ‚Π²ΡƒΡŽΡ‰Π΅ΠΌΡƒ элСмСнту Π² Π΄Ρ€ΡƒΠ³ΠΎΠΉ ΠΌΠ°Ρ‚Ρ€ΠΈΡ†Π΅. ΠŸΡ€ΠΎΡ†Π΅ΡΡ прост ΠΈ ΠΏΠΎΠΊΠ°Π·Π°Π½ Π½ΠΈΠΆΠ΅:

Π£Ρ€Π°Π²Π½Π΅Π½ΠΈΠ΅ 9: РСшСниС для слоТСния Π΄Π²ΡƒΡ… ΠΌΠ°Ρ‚Ρ€ΠΈΡ†

Π’Ρ‹ΠΏΠΎΠ»Π½ΠΈΡ‚Π΅ Ρ‚Π΅ ΠΆΠ΅ дСйствия Π² ΡΠ»Π΅Π΄ΡƒΡŽΡ‰ΠΈΡ… Π΄Π²ΡƒΡ… упраТнСниях.

ΠŸΡ€ΠΈΠΌΠ΅Ρ€ 3

Π‘ΠΊΠ»Π°Π΄Ρ‹Π²Π°Π΅ΠΌ ΡΠ»Π΅Π΄ΡƒΡŽΡ‰ΠΈΠ΅ Π΄Π²Π΅ ΠΌΠ°Ρ‚Ρ€ΠΈΡ†Ρ‹:

Π£Ρ€Π°Π²Π½Π΅Π½ΠΈΠ΅ 10: слоТСниС Π΄Π²ΡƒΡ… ΠΌΠ°Ρ‚Ρ€ΠΈΡ†

Π˜Ρ‚Π°ΠΊ, ΠΌΡ‹ добавляСм ΠΊΠ°ΠΆΠ΄Ρ‹ΠΉ элСмСнт Π² ΠΌΠ°Ρ‚Ρ€ΠΈΡ†Π°Ρ… ΠΊ ΡΠΎΠΎΡ‚Π²Π΅Ρ‚ΡΡ‚Π²ΡƒΡŽΡ‰Π΅ΠΌΡƒ элСмСнту Π² Π΄Ρ€ΡƒΠ³ΠΎΠΉ ΠΌΠ°Ρ‚Ρ€ΠΈΡ†Π΅. ΠžΠ±Ρ€Π°Ρ‚ΠΈΡ‚Π΅ Π²Π½ΠΈΠΌΠ°Π½ΠΈΠ΅, Ρ‡Ρ‚ΠΎ, ΠΊΠ°ΠΊ ΡƒΠΏΠΎΠΌΠΈΠ½Π°Π»ΠΎΡΡŒ Ρ€Π°Π½Π΅Π΅, ΠΌΠ°Ρ‚Ρ€ΠΈΡ†Ρ‹ Π΄ΠΎΠ»ΠΆΠ½Ρ‹ ΠΈΠΌΠ΅Ρ‚ΡŒ ΠΎΠ΄ΠΈΠ½Π°ΠΊΠΎΠ²Ρ‹Π΅ Ρ€Π°Π·ΠΌΠ΅Ρ€Ρ‹, Ρ‡Ρ‚ΠΎΠ±Ρ‹ Π²ΠΎΠ·Π½ΠΈΠΊΠ»ΠΎ поэлСмСнтноС соотвСтствиС:

Π£Ρ€Π°Π²Π½Π΅Π½ΠΈΠ΅ 11: РСшСниС для слоТСния Π΄Π²ΡƒΡ… ΠΌΠ°Ρ‚Ρ€ΠΈΡ†

ΠŸΡ€ΠΈΠΌΠ΅Ρ€ 4

Π‘ΠΊΠ»Π°Π΄Ρ‹Π²Π°Π΅ΠΌ ΡΠ»Π΅Π΄ΡƒΡŽΡ‰ΠΈΠ΅ Π΄Π²Π΅ ΠΌΠ°Ρ‚Ρ€ΠΈΡ†Ρ‹:

Π£Ρ€Π°Π²Π½Π΅Π½ΠΈΠ΅ 12: слоТСниС Ρ‚Ρ€Π΅Ρ… ΠΌΠ°Ρ‚Ρ€ΠΈΡ†

ΠŸΡ€ΠΈ Π΄ΠΎΠ±Π°Π²Π»Π΅Π½ΠΈΠΈ Π±ΠΎΠ»Π΅Π΅ Π΄Π²ΡƒΡ… ΠΌΠ°Ρ‚Ρ€ΠΈΡ† Π²Ρ‹ ΠΌΠΎΠΆΠ΅Ρ‚Π΅ ΡΠ²ΡΠ·Π°Ρ‚ΡŒ Π΄Π²Π΅, Π²Ρ‹ΠΏΠΎΠ»Π½ΠΈΠ² сначала ΠΈΡ… слоТСниС, Π° Π·Π°Ρ‚Π΅ΠΌ Π΄ΠΎΠ±Π°Π²ΠΈΠ² ΠΏΠΎΠ»ΡƒΡ‡Π΅Π½Π½ΡƒΡŽ ΠΌΠ°Ρ‚Ρ€ΠΈΡ†Ρƒ ΠΊ ΠΎΡΡ‚Π°Π²ΡˆΠ΅ΠΉΡΡ ΠΌΠ°Ρ‚Ρ€ΠΈΡ†Π΅.Π­Ρ‚ΠΎ Ρ‚ΠΎ, Ρ‡Ρ‚ΠΎ ΠΌΡ‹ Π½Π°Π·Ρ‹Π²Π°Π΅ΠΌ ассоциативным свойством слоТСния ΠΌΠ°Ρ‚Ρ€ΠΈΡ†, это ΠΈ Π΄Ρ€ΡƒΠ³ΠΈΠ΅ свойства слоТСния ΠΌΠ°Ρ‚Ρ€ΠΈΡ† Π±ΡƒΠ΄ΡƒΡ‚ ΠΎΠ±ΡŠΡΡΠ½Π΅Π½Ρ‹ Π² ΠΏΠΎΡΠ»Π΅Π΄ΡƒΡŽΡ‰ΠΈΡ… ΡƒΡ€ΠΎΠΊΠ°Ρ…, Π° ΠΏΠΎΠΊΠ° ΠΏΡ€ΠΎΠ²Π΅Ρ€ΡŒΡ‚Π΅ процСсс Ρ€Π΅ΡˆΠ΅Π½ΠΈΡ этой ΠΏΡ€ΠΎΠ±Π»Π΅ΠΌΡ‹ Π½ΠΈΠΆΠ΅:

Π£Ρ€Π°Π²Π½Π΅Π½ΠΈΠ΅ 13: РСшСниС для слоТСния Ρ‚Ρ€Π΅Ρ… ΠΌΠ°Ρ‚Ρ€ΠΈΡ†

Π˜Ρ‚Π°ΠΊ, ΠΊΠ°ΠΊ Π²Ρ‹ ΠΌΠΎΠΆΠ΅Ρ‚Π΅ Π²ΠΈΠ΄Π΅Ρ‚ΡŒ, ΠΏΠ΅Ρ€Π²Ρ‹ΠΌ шагом Π±Ρ‹Π»ΠΎ Π΄ΠΎΠ±Π°Π²Π»Π΅Π½ΠΈΠ΅ ΠΌΠ°Ρ‚Ρ€ΠΈΡ† ΠΎΠ΄ΠΈΠ½ ΠΈ Π΄Π²Π°, Ρ‡Ρ‚ΠΎ ΠΏΡ€ΠΈΠ²Π΅Π»ΠΎ ΠΊ Π½ΠΎΠ²ΠΎΠΉ ΠΌΠ°Ρ‚Ρ€ΠΈΡ†Π΅, которая ΠΏΠΎΠ·ΠΆΠ΅ добавляСтся ΠΊ Ρ‚Ρ€Π΅Ρ‚ΡŒΠ΅ΠΉ. Если Π΄ΠΎΠ±Π°Π²Π»Π΅Π½ΠΈΠ΅ ΠΌΠ°Ρ‚Ρ€ΠΈΡ† содСрТит большС Ρ‚Ρ€Π΅Ρ… ΠΌΠ°Ρ‚Ρ€ΠΈΡ†, ΠΎΠ΄ΠΈΠ½ ΠΈ Ρ‚ΠΎΡ‚ ΠΆΠ΅ процСсс связывания ΠΌΠΎΠΆΠ½ΠΎ ΠΏΠΎΠ²Ρ‚ΠΎΡ€ΡΡ‚ΡŒ ΡΡ‚ΠΎΠ»ΡŒΠΊΠΎ Ρ€Π°Π·, сколько Π½Π΅ΠΎΠ±Ρ…ΠΎΠ΄ΠΈΠΌΠΎ, Ρ‡Ρ‚ΠΎΠ±Ρ‹ ΡƒΠΏΡ€ΠΎΡΡ‚ΠΈΡ‚ΡŒ Π·Π°Π΄Π°Ρ‡Ρƒ, ΠΏΠΎΠΊΠ° Ρƒ вас Π½Π΅ получится простая опСрация с двумя ΠΌΠ°Ρ‚Ρ€ΠΈΡ†Π°ΠΌΠΈ.

ΠŸΡ€ΠΈΠΌΠ΅Ρ€ 5

Π’Ρ‹Ρ‡Ρ‚ΠΈΡ‚Π΅ ΡΠ»Π΅Π΄ΡƒΡŽΡ‰ΠΈΠ΅ ΠΌΠ°Ρ‚Ρ€ΠΈΡ†Ρ‹:

Π£Ρ€Π°Π²Π½Π΅Π½ΠΈΠ΅ 14: Π’Ρ‹Ρ‡ΠΈΡ‚Π°Π½ΠΈΠ΅ Π΄Π²ΡƒΡ… ΠΌΠ°Ρ‚Ρ€ΠΈΡ†

Π•Ρ‰Π΅ Ρ€Π°Π· ΠΎΠ±Ρ€Π°Ρ‚ΠΈΡ‚Π΅ Π²Π½ΠΈΠΌΠ°Π½ΠΈΠ΅, Ρ‡Ρ‚ΠΎ ΠΌΠ°Ρ‚Ρ€ΠΈΡ†Ρ‹, ΡƒΡ‡Π°ΡΡ‚Π²ΡƒΡŽΡ‰ΠΈΠ΅ Π² Π²Ρ‹Ρ‡ΠΈΡ‚Π°Π½ΠΈΠΈ, Π΄ΠΎΠ»ΠΆΠ½Ρ‹ ΠΈΠΌΠ΅Ρ‚ΡŒ ΠΎΠ΄ΠΈΠ½Π°ΠΊΠΎΠ²Ρ‹Π΅ Ρ€Π°Π·ΠΌΠ΅Ρ€Ρ‹, Ρ‡Ρ‚ΠΎΠ±Ρ‹ соблюдалось поэлСмСнтноС соотвСтствиС.

Π£Ρ€Π°Π²Π½Π΅Π½ΠΈΠ΅ 15: РСшСниС для вычитания Π΄Π²ΡƒΡ… ΠΌΠ°Ρ‚Ρ€ΠΈΡ†

ΠŸΡ€ΠΈΠΌΠ΅Ρ€ 6

Π’Ρ‹Ρ‡Ρ‚ΠΈΡ‚Π΅ ΡΠ»Π΅Π΄ΡƒΡŽΡ‰ΠΈΠ΅ ΠΌΠ°Ρ‚Ρ€ΠΈΡ†Ρ‹:

Π£Ρ€Π°Π²Π½Π΅Π½ΠΈΠ΅ 16: Π’Ρ‹Ρ‡ΠΈΡ‚Π°Π½ΠΈΠ΅ Π΄Π²ΡƒΡ… ΠΌΠ°Ρ‚Ρ€ΠΈΡ†

НСваТно, насколько Π²Π΅Π»ΠΈΠΊΠΈ ΠΌΠ°Ρ‚Ρ€ΠΈΡ†Ρ‹, Ссли ΠΎΠ½ΠΈ ΠΈΠΌΠ΅ΡŽΡ‚ ΠΎΠ΄ΠΈΠ½Π°ΠΊΠΎΠ²Ρ‹Π΅ Ρ€Π°Π·ΠΌΠ΅Ρ€Ρ‹, Π²Ρ‹ ΠΌΠΎΠΆΠ΅Ρ‚Π΅ Π²Ρ‹ΠΏΠΎΠ»Π½ΠΈΡ‚ΡŒ Π²Ρ‹Ρ‡ΠΈΡ‚Π°Π½ΠΈΠ΅.

Π£Ρ€Π°Π²Π½Π΅Π½ΠΈΠ΅ 17: РСшСниС для вычитания Π΄Π²ΡƒΡ… ΠΌΠ°Ρ‚Ρ€ΠΈΡ†

ΠŸΡ€ΠΈΠΌΠ΅Ρ€ 7

Π’Ρ‹Ρ‡Ρ‚ΠΈΡ‚Π΅ ΡΠ»Π΅Π΄ΡƒΡŽΡ‰ΠΈΠ΅ ΠΌΠ°Ρ‚Ρ€ΠΈΡ†Ρ‹:

Π£Ρ€Π°Π²Π½Π΅Π½ΠΈΠ΅ 18: Π’Ρ‹Ρ‡ΠΈΡ‚Π°Π½ΠΈΠ΅ Ρ‚Ρ€Π΅Ρ… ΠΌΠ°Ρ‚Ρ€ΠΈΡ†

ΠžΠ±Ρ€Π°Ρ‚ΠΈΡ‚Π΅ Π²Π½ΠΈΠΌΠ°Π½ΠΈΠ΅, Ρ‡Ρ‚ΠΎ для вычитания ΠΌΠ°Ρ‚Ρ€ΠΈΡ† с Π±ΠΎΠ»Π΅Π΅ Ρ‡Π΅ΠΌ двумя ΠΌΠ°Ρ‚Ρ€ΠΈΡ†Π°ΠΌΠΈ ΠΌΡ‹ ΠΌΠΎΠΆΠ΅ΠΌ снова ΠΈΡΠΏΠΎΠ»ΡŒΠ·ΠΎΠ²Π°Ρ‚ΡŒ свойство ассоциативности. ΠŸΠΎΠΌΠ½ΠΈΡ‚Π΅, Ρ‡Ρ‚ΠΎ Π²Ρ‹Ρ‡ΠΈΡ‚Π°Π½ΠΈΠ΅ ΠΌΠ°Ρ‚Ρ€ΠΈΡ† Π½Π΅ ΠΊΠΎΠΌΠΌΡƒΡ‚Π°Ρ‚ΠΈΠ²Π½ΠΎ (Π²Ρ‹ Π½Π΅ ΠΌΠΎΠΆΠ΅Ρ‚Π΅ ΠΈΠ·ΠΌΠ΅Π½ΠΈΡ‚ΡŒ порядок ΠΌΠ°Ρ‚Ρ€ΠΈΡ† Π² ΠΎΠΏΠ΅Ρ€Π°Ρ†ΠΈΠΈ ΠΈ ΠΏΠΎΠ»ΡƒΡ‡ΠΈΡ‚ΡŒ Ρ‚ΠΎΡ‚ ΠΆΠ΅ Ρ€Π΅Π·ΡƒΠ»ΡŒΡ‚Π°Ρ‚).

Π£Ρ€Π°Π²Π½Π΅Π½ΠΈΠ΅ 19: РСшСниС вычитания Ρ‚Ρ€Π΅Ρ… ΠΌΠ°Ρ‚Ρ€ΠΈΡ†

ΠŸΡ€ΠΈΠΌΠ΅Ρ€ 8

Π Π΅ΡˆΠΈΡ‚Π΅ ΡΠ»Π΅Π΄ΡƒΡŽΡ‰Π΅Π΅ ΡƒΡ€Π°Π²Π½Π΅Π½ΠΈΠ΅, содСрТащСС Π΄ΠΎΠ±Π°Π²Π»Π΅Π½ΠΈΠ΅ постоянной ΠΌΠ°Ρ‚Ρ€ΠΈΡ†Ρ‹ ΠΊ ΠΏΠ΅Ρ€Π΅ΠΌΠ΅Π½Π½ΠΎΠΉ.

Π£Ρ€Π°Π²Π½Π΅Π½ΠΈΠ΅ 20: НайдитС A Π² ΠΌΠ°Ρ‚Ρ€ΠΈΡ‡Π½ΠΎΠΌ ΡƒΡ€Π°Π²Π½Π΅Π½ΠΈΠΈ (Ρ‡Π°ΡΡ‚ΡŒ 1)

ΠœΡ‹ Π½Π°Ρ‡ΠΈΠ½Π°Π΅ΠΌ с вычитания ΠΌΠ°Ρ‚Ρ€ΠΈΡ†Ρ‹ Π² Π»Π΅Π²ΠΎΠΉ части ΠΊ ΠΎΠ±Π΅ΠΈΠΌ частям уравнСния, Ρ‡Ρ‚ΠΎΠ±Ρ‹ ΠΌΡ‹ ΠΌΠΎΠ³Π»ΠΈ Π»Π΅Π³ΠΊΠΎ Π½Π°ΠΉΡ‚ΠΈ A, Π²Ρ‹ΠΏΠΎΠ»Π½ΠΈΠ² Π²Ρ‹Ρ‡ΠΈΡ‚Π°Π½ΠΈΠ΅ ΠΌΠ°Ρ‚Ρ€ΠΈΡ†Ρ‹:

Π£Ρ€Π°Π²Π½Π΅Π½ΠΈΠ΅ 21: НайдитС A Π² ΠΌΠ°Ρ‚Ρ€ΠΈΡ‡Π½ΠΎΠΌ ΡƒΡ€Π°Π²Π½Π΅Π½ΠΈΠΈ (Ρ‡Π°ΡΡ‚ΡŒ 2)

ΠŸΡ€ΠΈΠΌΠ΅Ρ€ 9

Π Π΅ΡˆΠΈΡ‚Π΅ ΡΠ»Π΅Π΄ΡƒΡŽΡ‰ΠΈΠ΅ ΠΌΠ°Ρ‚Ρ€ΠΈΡ‡Π½Ρ‹Π΅ уравнСния

Π£Ρ€Π°Π²Π½Π΅Π½ΠΈΠ΅ 22: НайдитС X Π² ΠΌΠ°Ρ‚Ρ€ΠΈΡ‡Π½ΠΎΠΌ ΡƒΡ€Π°Π²Π½Π΅Π½ΠΈΠΈ (Ρ‡Π°ΡΡ‚ΡŒ 1)

Π’ этом случаС самый простой способ Π½Π°ΠΉΡ‚ΠΈ X — это ΠΏΠ΅Ρ€Π΅Π΄Π°Ρ‚ΡŒ Π΅Π³ΠΎ Π² ΠΏΡ€Π°Π²ΡƒΡŽ Ρ‡Π°ΡΡ‚ΡŒ, Π΄ΠΎΠ±Π°Π²ΠΈΠ² Π΅Π³ΠΎ ΠΊ ΠΎΠ±Π΅ΠΈΠΌ частям уравнСния, Ρ‡Ρ‚ΠΎΠ±Ρ‹ ΠΎΠ½ΠΎ стало ΠΏΠΎΠ»ΠΎΠΆΠΈΡ‚Π΅Π»ΡŒΠ½Ρ‹ΠΌ.ΠœΡ‹ Ρ‚Π°ΠΊΠΆΠ΅ Π²Ρ‹Ρ‡ΠΈΡ‚Π°Π΅ΠΌ ΠΌΠ°Ρ‚Ρ€ΠΈΡ†Ρƒ, Π½Π°ΠΉΠ΄Π΅Π½Π½ΡƒΡŽ Π² ΠΏΡ€Π°Π²ΠΎΠΉ части уравнСния, с ΠΎΠ±Π΅ΠΈΡ… сторон, ΠΈ, Ρ‚Π°ΠΊΠΈΠΌ ΠΎΠ±Ρ€Π°Π·ΠΎΠΌ, ΠΏΠΎΠ»ΡƒΡ‡Π°Π΅ΠΌ Π²Ρ‹Ρ‡ΠΈΡ‚Π°Π½ΠΈΠ΅ ΠΌΠ°Ρ‚Ρ€ΠΈΡ†Ρ‹, ΠΎΠΏΡ€Π΅Π΄Π΅Π»ΡΡŽΡ‰Π΅Π΅ X.

Π£Ρ€Π°Π²Π½Π΅Π½ΠΈΠ΅ 23: НайдитС X Π² ΠΌΠ°Ρ‚Ρ€ΠΈΡ‡Π½ΠΎΠΌ ΡƒΡ€Π°Π²Π½Π΅Π½ΠΈΠΈ (Ρ‡Π°ΡΡ‚ΡŒ 2)

РСшая Π²Ρ‹Ρ‡ΠΈΡ‚Π°Π½ΠΈΠ΅ ΠΌΠ°Ρ‚Ρ€ΠΈΡ†, Π½Π°Ρ…ΠΎΠ΄ΠΈΠΌ X.

ΠŸΡ€ΠΈΠΌΠ΅Ρ€ 10

НайдитС значСния элСмСнтов a, b, c, d, e ΠΈ f, Ρ€Π΅ΡˆΠΈΠ² ΡΠ»Π΅Π΄ΡƒΡŽΡ‰Π΅Π΅ ΠΌΠ°Ρ‚Ρ€ΠΈΡ‡Π½ΠΎΠ΅ ΡƒΡ€Π°Π²Π½Π΅Π½ΠΈΠ΅ Π½Π°ΠΈΠ±ΠΎΠ»Π΅Π΅ эффСктивным способом.

Π£Ρ€Π°Π²Π½Π΅Π½ΠΈΠ΅ 24: НахоТдСниС Π·Π½Π°Ρ‡Π΅Π½ΠΈΠΉ элСмСнтов Π² самой Π»Π΅Π²ΠΎΠΉ ΠΌΠ°Ρ‚Ρ€ΠΈΡ†Π΅

Π’ этой Π·Π°Π΄Π°Ρ‡Π΅ ΠΌΡ‹ ΠΌΠΎΠΆΠ΅ΠΌ ΠΏΡ€Π΅ΠΎΠ±Ρ€Π°Π·ΠΎΠ²Π°Ρ‚ΡŒ элСмСнт Π·Π° элСмСнтом случай, ΡƒΡ€Π°Π²Π½Π΅Π½ΠΈΠ΅, ΠΊΠΎΡ‚ΠΎΡ€ΠΎΠ΅ ΠΏΠΎΠ·Π²ΠΎΠ»ΠΈΡ‚ Π½Π°ΠΌ ΠΏΠΎΠ»ΡƒΡ‡ΠΈΡ‚ΡŒ Π·Π½Π°Ρ‡Π΅Π½ΠΈΠ΅ ΠΊΠ°ΠΆΠ΄ΠΎΠ³ΠΎ ΠΈΠ· элСмСнтов Π² ΠΏΠ΅Ρ€Π²ΠΎΠΉ ΠΌΠ°Ρ‚Ρ€ΠΈΡ†Π΅.

Π”Ρ€ΡƒΠ³ΠΈΠΌΠΈ словами, Π²Ρ‹ Π·Π½Π°Π΅Ρ‚Π΅, Ρ‡Ρ‚ΠΎ элСмСнт ΠΈΠ· ΠΏΠ΅Ρ€Π²ΠΎΠΉ строки ΠΈ ΠΏΠ΅Ρ€Π²ΠΎΠ³ΠΎ столбца (Π²Π²Π΅Ρ€Ρ…Ρƒ слСва) Π² ΠΏΠ΅Ρ€Π²ΠΎΠΉ ΠΌΠ°Ρ‚Ρ€ΠΈΡ†Π΅ Π±ΡƒΠ΄Π΅Ρ‚ Π΄ΠΎΠ±Π°Π²Π»Π΅Π½ ΠΊ элСмСнту Π² ΠΏΠ΅Ρ€Π²ΠΎΠΉ строкС ΠΈ ΠΏΠ΅Ρ€Π²ΠΎΠΌ столбцС Π²Ρ‚ΠΎΡ€ΠΎΠΉ ΠΌΠ°Ρ‚Ρ€ΠΈΡ†Ρ‹, Ρ‡Ρ‚ΠΎΠ±Ρ‹ ΠΏΠΎΠ»ΡƒΡ‡ΠΈΡ‚ΡŒ элСмСнт Π² этом пространствС. Π² ΠΏΠΎΠ»ΡƒΡ‡ΠΈΠ²ΡˆΠ΅ΠΉΡΡ ΠΌΠ°Ρ‚Ρ€ΠΈΡ†Π΅ справа. Π”Ρ€ΡƒΠ³ΠΈΠΌΠΈ словами, ΠΌΡ‹ ΠΌΠΎΠΆΠ΅ΠΌ Π»Π΅Π³ΠΊΠΎ Π·Π°ΠΏΠΈΡΠ°Ρ‚ΡŒ ΡΠ»Π΅Π΄ΡƒΡŽΡ‰ΠΈΠ΅ уравнСния для ΠΊΠ°ΠΆΠ΄ΠΎΠ³ΠΎ пространства элСмСнтов:

Π£Ρ€Π°Π²Π½Π΅Π½ΠΈΠ΅ 25: Набор ΡƒΡ€Π°Π²Π½Π΅Π½ΠΈΠΉ для ΠΊΠ°ΠΆΠ΄ΠΎΠΉ ΠΏΠ΅Ρ€Π΅ΠΌΠ΅Π½Π½ΠΎΠΉ, ΠΊΠΎΡ‚ΠΎΡ€ΡƒΡŽ Π½Π΅ΠΎΠ±Ρ…ΠΎΠ΄ΠΈΠΌΠΎ Ρ€Π΅ΡˆΠΈΡ‚ΡŒ

И ΠΈΠ· этого ΠΌΡ‹ ΠΌΠΎΠΆΠ΅ΠΌ Ρ€Π΅ΡˆΠΈΡ‚ΡŒ Π·Π½Π°Ρ‡Π΅Π½ΠΈΠ΅ ΠΊΠ°ΠΆΠ΄ΠΎΠΉ ΠΏΠ΅Ρ€Π΅ΠΌΠ΅Π½Π½ΠΎΠΉ:

Π£Ρ€Π°Π²Π½Π΅Π½ΠΈΠ΅ 25: РСшСниС ΠΏΠ΅Ρ€Π΅ΠΌΠ΅Π½Π½Ρ‹Ρ… ΠΎΡ‚ a Π΄ΠΎ f

ПослС Ρ‚ΠΎΠ³ΠΎ, ΠΊΠ°ΠΊ ΠΌΡ‹ научимся ΡΠΊΠ»Π°Π΄Ρ‹Π²Π°Ρ‚ΡŒ ΠΈ Π²Ρ‹Ρ‡ΠΈΡ‚Π°Ρ‚ΡŒ ΠΌΠ°Ρ‚Ρ€ΠΈΡ†Ρ‹, ΠΏΡ€ΠΈΡˆΠ»ΠΎ врСмя ΠΈΠ·ΡƒΡ‡ΠΈΡ‚ΡŒ Π΄Ρ€ΡƒΠ³ΠΈΠ΅ ΠΎΠΏΠ΅Ρ€Π°Ρ†ΠΈΠΈ: скалярноС ΡƒΠΌΠ½ΠΎΠΆΠ΅Π½ΠΈΠ΅ ΠΌΠ°Ρ‚Ρ€ΠΈΡ† ΠΈ ΡƒΠΌΠ½ΠΎΠΆΠ΅Π½ΠΈΠ΅ Π΄Π²ΡƒΡ… ΠΈΠ»ΠΈ Π±ΠΎΠ»Π΅Π΅ ΠΌΠ°Ρ‚Ρ€ΠΈΡ† Π±ΡƒΠ΄ΡƒΡ‚ ΠΎΠ±ΡŠΡΡΠ½Π΅Π½Ρ‹ Π² ΡΠ»Π΅Π΄ΡƒΡŽΡ‰ΠΈΡ… Π΄Π²ΡƒΡ… ΡƒΡ€ΠΎΠΊΠ°Ρ….

ΠœΡ‹ Π·Π°Π²Π΅Ρ€ΡˆΠ°Π΅ΠΌ наш ΡƒΡ€ΠΎΠΊ Π½Π° сСгодня нСсколькими ΠΏΠΎΠ»Π΅Π·Π½Ρ‹ΠΌΠΈ ссылками: пСрвая содСрТит ΠΊΡ€Π°Ρ‚ΠΊΠΈΠΉ список основных ΠΏΡ€Π°Π²ΠΈΠ» слоТСния ΠΌΠ°Ρ‚Ρ€ΠΈΡ† ΠΈ Π΄ΠΎΠΏΠΎΠ»Π½ΠΈΡ‚Π΅Π»ΡŒΠ½ΡƒΡŽ Π»Π΅ΠΊΡ†ΠΈΡŽ ΠΏΠΎ ΠΌΠ°Ρ‚Ρ€ΠΈΡ‡Π½Ρ‹ΠΌ опСрациям.

И это всС, Π΄ΠΎ встрСчи Π½Π° нашСм ΡΠ»Π΅Π΄ΡƒΡŽΡ‰Π΅ΠΌ ΡƒΡ€ΠΎΠΊΠ΅!

6.2 — ΠžΠΏΠ΅Ρ€Π°Ρ†ΠΈΠΈ с ΠΌΠ°Ρ‚Ρ€ΠΈΡ†Π°ΠΌΠΈ

6.2 — ΠžΠΏΠ΅Ρ€Π°Ρ†ΠΈΠΈ с ΠΌΠ°Ρ‚Ρ€ΠΈΡ†Π°ΠΌΠΈ

РавСнство

Π”Π²Π΅ ΠΌΠ°Ρ‚Ρ€ΠΈΡ†Ρ‹ Ρ€Π°Π²Π½Ρ‹ Ρ‚ΠΎΠ³Π΄Π° ΠΈ Ρ‚ΠΎΠ»ΡŒΠΊΠΎ Ρ‚ΠΎΠ³Π΄Π°, ΠΊΠΎΠ³Π΄Π°

  • ΠŸΠΎΡ€ΡΠ΄ΠΎΠΊ ΠΌΠ°Ρ‚Ρ€ΠΈΡ† ΠΎΠ΄ΠΈΠ½Π°ΠΊΠΎΠ²Ρ‹ΠΉ
  • Π‘ΠΎΠΎΡ‚Π²Π΅Ρ‚ΡΡ‚Π²ΡƒΡŽΡ‰ΠΈΠ΅ элСмСнты ΠΌΠ°Ρ‚Ρ€ΠΈΡ† ΠΎΠ΄ΠΈΠ½Π°ΠΊΠΎΠ²Ρ‹Π΅

Π”ΠΎΠΏΠΎΠ»Π½Π΅Π½ΠΈΠ΅

  • ΠŸΠΎΡ€ΡΠ΄ΠΎΠΊ ΠΌΠ°Ρ‚Ρ€ΠΈΡ† Π΄ΠΎΠ»ΠΆΠ΅Π½ Π±Ρ‹Ρ‚ΡŒ Ρ‚Π°ΠΊΠΈΠΌ ΠΆΠ΅
  • Π‘Π»ΠΎΠΆΠΈΡ‚Π΅ ΡΠΎΠΎΡ‚Π²Π΅Ρ‚ΡΡ‚Π²ΡƒΡŽΡ‰ΠΈΠ΅ элСмСнты вмСстС
  • ΠœΠ°Ρ‚Ρ€ΠΈΡ‡Π½ΠΎΠ΅ слоТСниС ΠΊΠΎΠΌΠΌΡƒΡ‚Π°Ρ‚ΠΈΠ²Π½ΠΎ
  • ΠœΠ°Ρ‚Ρ€ΠΈΡ†Π° слоТСния ассоциативна

Π’Ρ‹Ρ‡ΠΈΡ‚Π°Π½ΠΈΠ΅

  • ΠŸΠΎΡ€ΡΠ΄ΠΎΠΊ ΠΌΠ°Ρ‚Ρ€ΠΈΡ† Π΄ΠΎΠ»ΠΆΠ΅Π½ Π±Ρ‹Ρ‚ΡŒ ΠΎΠ΄ΠΈΠ½Π°ΠΊΠΎΠ²Ρ‹ΠΌ
  • Π’Ρ‹Ρ‡Π΅ΡΡ‚ΡŒ ΡΠΎΠΎΡ‚Π²Π΅Ρ‚ΡΡ‚Π²ΡƒΡŽΡ‰ΠΈΠ΅ элСмСнты
  • ΠœΠ°Ρ‚Ρ€ΠΈΡ‡Π½ΠΎΠ΅ Π²Ρ‹Ρ‡ΠΈΡ‚Π°Π½ΠΈΠ΅ Π½Π΅ ΠΊΠΎΠΌΠΌΡƒΡ‚Π°Ρ‚ΠΈΠ²Π½ΠΎ (ΠΊΠ°ΠΊ ΠΈ Π²Ρ‹Ρ‡ΠΈΡ‚Π°Π½ΠΈΠ΅ Π΄Π΅ΠΉΡΡ‚Π²ΠΈΡ‚Π΅Π»ΡŒΠ½Ρ‹Ρ… чисСл)
  • ΠœΠ°Ρ‚Ρ€ΠΈΡ‡Π½ΠΎΠ΅ Π²Ρ‹Ρ‡ΠΈΡ‚Π°Π½ΠΈΠ΅ Π½Π΅ ассоциативно (ΠΊΠ°ΠΊ ΠΈ Π²Ρ‹Ρ‡ΠΈΡ‚Π°Π½ΠΈΠ΅ Π΄Π΅ΠΉΡΡ‚Π²ΠΈΡ‚Π΅Π»ΡŒΠ½Ρ‹Ρ… чисСл)

БкалярноС ΡƒΠΌΠ½ΠΎΠΆΠ΅Π½ΠΈΠ΅

Бкаляр — это число, Π° Π½Π΅ ΠΌΠ°Ρ‚Ρ€ΠΈΡ†Π°.

  • ΠœΠ°Ρ‚Ρ€ΠΈΡ†Π° ΠΌΠΎΠΆΠ΅Ρ‚ Π±Ρ‹Ρ‚ΡŒ любого порядка
  • Π£ΠΌΠ½ΠΎΠΆΠΈΡ‚ΡŒ всС элСмСнты Π² ΠΌΠ°Ρ‚Ρ€ΠΈΡ†Π΅ Π½Π° скаляр
  • БкалярноС ΡƒΠΌΠ½ΠΎΠΆΠ΅Π½ΠΈΠ΅ ΠΊΠΎΠΌΠΌΡƒΡ‚Π°Ρ‚ΠΈΠ²Π½ΠΎ
  • БкалярноС ΡƒΠΌΠ½ΠΎΠΆΠ΅Π½ΠΈΠ΅ ассоциативно

НулСвая ΠΌΠ°Ρ‚Ρ€ΠΈΡ†Π°

  • ΠœΠ°Ρ‚Ρ€ΠΈΡ†Π° любого Π·Π°ΠΊΠ°Π·Π°
  • Бостоит ΠΈΠ· всСх Π½ΡƒΠ»Π΅ΠΉ
  • ΠžΠ±ΠΎΠ·Π½Π°Ρ‡Π°Π΅Ρ‚ΡΡ Π·Π°Π³Π»Π°Π²Π½ΠΎΠΉ Π±ΡƒΠΊΠ²Ρ‹ O
  • Аддитивный ΠΈΠ΄Π΅Π½Ρ‚ΠΈΡ„ΠΈΠΊΠ°Ρ‚ΠΎΡ€ для ΠΌΠ°Ρ‚Ρ€ΠΈΡ†
  • Π›ΡŽΠ±Π°Ρ ΠΌΠ°Ρ‚Ρ€ΠΈΡ†Π° плюс нулСвая ΠΌΠ°Ρ‚Ρ€ΠΈΡ†Π° являСтся исходной ΠΌΠ°Ρ‚Ρ€ΠΈΡ†Π΅ΠΉ

Π£ΠΌΠ½ΠΎΠΆΠ΅Π½ΠΈΠ΅ ΠΌΠ°Ρ‚Ρ€ΠΈΡ†

A m Γ— n Γ— B n Γ— p = C m Γ— p

  • ΠšΠΎΠ»ΠΈΡ‡Π΅ΡΡ‚Π²ΠΎ столбцов Π² ΠΏΠ΅Ρ€Π²ΠΎΠΉ ΠΌΠ°Ρ‚Ρ€ΠΈΡ†Π΅ Π΄ΠΎΠ»ΠΆΠ½ΠΎ Π±Ρ‹Ρ‚ΡŒ Ρ€Π°Π²Π½ΠΎ количСство строк Π²ΠΎ Π²Ρ‚ΠΎΡ€ΠΎΠΉ ΠΌΠ°Ρ‚Ρ€ΠΈΡ†Π΅.Π’ΠΎ Π΅ΡΡ‚ΡŒ Π²Π½ΡƒΡ‚Ρ€Π΅Π½Π½ΠΈΠ΅ Ρ€Π°Π·ΠΌΠ΅Ρ€Ρ‹ Π΄ΠΎΠ»ΠΆΠ½Ρ‹ Π±Ρ‹Ρ‚ΡŒ ΠΎΠ΄ΠΈΠ½Π°ΠΊΠΎΠ²Ρ‹ΠΌΠΈ.
  • ΠŸΠΎΡ€ΡΠ΄ΠΎΠΊ произвСдСния — это количСство строк Π² ΠΏΠ΅Ρ€Π²ΠΎΠΉ ΠΌΠ°Ρ‚Ρ€ΠΈΡ†Π΅ Π½Π° количСство столбцов Π² вторая ΠΌΠ°Ρ‚Ρ€ΠΈΡ†Π°. Π’ΠΎ Π΅ΡΡ‚ΡŒ Π³Π°Π±Π°Ρ€ΠΈΡ‚Ρ‹ издСлия — это Π³Π°Π±Π°Ρ€ΠΈΡ‚Ρ‹ внСшниС.
  • ΠŸΠΎΡΠΊΠΎΠ»ΡŒΠΊΡƒ количСство столбцов Π² ΠΏΠ΅Ρ€Π²ΠΎΠΉ ΠΌΠ°Ρ‚Ρ€ΠΈΡ†Π΅ Ρ€Π°Π²Π½ΠΎ количСству строк Π²ΠΎ Π²Ρ‚ΠΎΡ€ΠΎΠΉ ΠΌΠ°Ρ‚Ρ€ΠΈΡ†Ρƒ, Π²Ρ‹ ΠΌΠΎΠΆΠ΅Ρ‚Π΅ ΠΎΠ±ΡŠΠ΅Π΄ΠΈΠ½ΠΈΡ‚ΡŒ записи.
  • ΠšΠ°ΠΆΠ΄Ρ‹ΠΉ элСмСнт Π² строкС i ΠΈΠ· ΠΏΠ΅Ρ€Π²ΠΎΠΉ ΠΌΠ°Ρ‚Ρ€ΠΈΡ†Ρ‹ ΠΎΠ±ΡŠΠ΅Π΄ΠΈΠ½ΡΠ΅Ρ‚ΡΡ Π² ΠΏΠ°Ρ€Ρƒ с элСмСнтом Π² столбцС j ΠΈΠ· вторая ΠΌΠ°Ρ‚Ρ€ΠΈΡ†Π°.
  • Π­Π»Π΅ΠΌΠ΅Π½Ρ‚ Π² строкС i , столбцС j ΠΏΡ€ΠΎΠ΄ΡƒΠΊΡ‚Π° формируСтся ΠΏΡƒΡ‚Π΅ΠΌ умноТСния этих ΠΏΠ°Ρ€Π½Ρ‹Ρ… элСмСнтов. ΠΈ суммируя ΠΈΡ….
  • ΠšΠ°ΠΆΠ΄Ρ‹ΠΉ элСмСнт Π² ΠΏΡ€ΠΎΠ΄ΡƒΠΊΡ‚Π΅ прСдставляСт собой сумму ΠΏΡ€ΠΎΠΈΠ·Π²Π΅Π΄Π΅Π½ΠΈΠΉ элСмСнтов ΠΈΠ· строка i ΠΏΠ΅Ρ€Π²ΠΎΠΉ ΠΌΠ°Ρ‚Ρ€ΠΈΡ†Ρ‹ ΠΈ столбСц j Π²Ρ‚ΠΎΡ€ΠΎΠΉ ΠΌΠ°Ρ‚Ρ€ΠΈΡ†Ρ‹.
  • Π‘ΡƒΠ΄Π΅Ρ‚ n ΠΏΡ€ΠΎΠ΄ΡƒΠΊΡ‚ΠΎΠ², ΠΊΠΎΡ‚ΠΎΡ€Ρ‹Π΅ ΡΡƒΠΌΠΌΠΈΡ€ΡƒΡŽΡ‚ΡΡ для ΠΊΠ°ΠΆΠ΄ΠΎΠ³ΠΎ элСмСнта Π² ΠΏΡ€ΠΎΠ΄ΡƒΠΊΡ‚Π΅.

Π‘ΠΌ. ΠŸΠΎΠ»Π½Ρ‹ΠΉ ΠΏΡ€ΠΈΠΌΠ΅Ρ€ умноТСния ΠΌΠ°Ρ‚Ρ€ΠΈΡ†.

Π£ΠΌΠ½ΠΎΠΆΠ΅Π½ΠΈΠ΅ ΠΌΠ°Ρ‚Ρ€ΠΈΡ† Π½Π΅ΠΊΠΎΠΌΠΌΡƒΡ‚Π°Ρ‚ΠΈΠ²Π½ΠΎΠ΅

  • Π£ΠΌΠ½ΠΎΠΆΠ΅Π½ΠΈΠ΅ Π΄Π΅ΠΉΡΡ‚Π²ΠΈΡ‚Π΅Π»ΡŒΠ½Ρ‹Ρ… чисСл.
  • Π’Π½ΡƒΡ‚Ρ€Π΅Π½Π½ΠΈΠ΅ Ρ€Π°Π·ΠΌΠ΅Ρ€Ρ‹ ΠΌΠΎΠ³ΡƒΡ‚ Π½Π΅ ΡΠΎΠ²ΠΏΠ°Π΄Π°Ρ‚ΡŒ ΠΏΡ€ΠΈ ΠΈΠ·ΠΌΠ΅Π½Π΅Π½ΠΈΠΈ порядка ΠΌΠ°Ρ‚Ρ€ΠΈΡ†.

НС просто ΠΏΠ΅Ρ€Π΅ΠΌΠ½ΠΎΠΆΠ°ΠΉΡ‚Π΅ ΡΠΎΠΎΡ‚Π²Π΅Ρ‚ΡΡ‚Π²ΡƒΡŽΡ‰ΠΈΠ΅ элСмСнты вмСстС

  • ΠŸΠΎΡΠΊΠΎΠ»ΡŒΠΊΡƒ порядок (Ρ€Π°Π·ΠΌΠ΅Ρ€Ρ‹) ΠΌΠ°Ρ‚Ρ€ΠΈΡ† Π½Π΅ ΠΎΠ±ΡΠ·Π°Ρ‚Π΅Π»ΡŒΠ½ΠΎ Π΄ΠΎΠ»ΠΆΠ½Ρ‹ Π±Ρ‹Ρ‚ΡŒ ΠΎΠ΄ΠΈΠ½Π°ΠΊΠΎΠ²Ρ‹ΠΌΠΈ, ΠΌΠΎΠΆΠ΅Ρ‚ Π½Π΅ Π±Ρ‹Ρ‚ΡŒ ΡΠΎΠΎΡ‚Π²Π΅Ρ‚ΡΡ‚Π²ΡƒΡŽΡ‰ΠΈΠ΅ элСмСнты ΡƒΠΌΠ½ΠΎΠΆΠ°ΡŽΡ‚ΡΡ вмСстС.
  • Π£ΠΌΠ½ΠΎΠΆΡŒΡ‚Π΅ строки ΠΏΠ΅Ρ€Π²ΠΎΠ³ΠΎ столбца Π½Π° столбцы Π²Ρ‚ΠΎΡ€ΠΎΠ³ΠΎ ΠΈ слоТитС.

НСт ΠΌΠ°Ρ‚Ρ€ΠΈΡ‡Π½ΠΎΠ³ΠΎ дСлСния

  • НС сущСствуСт ΠΎΠΏΡ€Π΅Π΄Π΅Π»Π΅Π½Π½ΠΎΠ³ΠΎ процСсса дСлСния ΠΌΠ°Ρ‚Ρ€ΠΈΡ†Ρ‹ Π½Π° Π΄Ρ€ΡƒΠ³ΡƒΡŽ ΠΌΠ°Ρ‚Ρ€ΠΈΡ†Ρƒ.
  • ΠœΠ°Ρ‚Ρ€ΠΈΡ†Π° ΠΌΠΎΠΆΠ΅Ρ‚ Π±Ρ‹Ρ‚ΡŒ Ρ€Π°Π·Π΄Π΅Π»Π΅Π½Π° скаляром.

Π˜Π΄Π΅Π½Ρ‚ΠΈΡ„ΠΈΠΊΠ°Ρ†ΠΈΠΎΠ½Π½Π°Ρ ΠΌΠ°Ρ‚Ρ€ΠΈΡ†Π°

  • ΠšΠ²Π°Π΄Ρ€Π°Ρ‚Π½Π°Ρ ΠΌΠ°Ρ‚Ρ€ΠΈΡ†Π°
  • На Π³Π»Π°Π²Π½ΠΎΠΉ Π΄ΠΈΠ°Π³ΠΎΠ½Π°Π»ΠΈ
  • Π½ΡƒΠ»ΠΈ Π²Π΅Π·Π΄Π΅
  • ΠžΠ±ΠΎΠ·Π½Π°Ρ‡Π°Π΅Ρ‚ΡΡ I. Если ΡƒΠΊΠ°Π·Π°Π½ Π½ΠΈΠΆΠ½ΠΈΠΉ индСкс, это порядок Π΅Π΄ΠΈΠ½ΠΈΡ‡Π½ΠΎΠΉ ΠΌΠ°Ρ‚Ρ€ΠΈΡ†Ρ‹.
  • I — ΠΌΡƒΠ»ΡŒΡ‚ΠΈΠΏΠ»ΠΈΠΊΠ°Ρ‚ΠΈΠ²Π½ΠΎΠ΅ тоТдСство для ΠΌΠ°Ρ‚Ρ€ΠΈΡ†
  • Π›ΡŽΠ±Π°Ρ ΠΌΠ°Ρ‚Ρ€ΠΈΡ†Π°, умноТСнная Π½Π° Π΅Π΄ΠΈΠ½ΠΈΡ‡Π½ΡƒΡŽ ΠΌΠ°Ρ‚Ρ€ΠΈΡ†Ρƒ, являСтся исходной ΠΌΠ°Ρ‚Ρ€ΠΈΡ†Π΅ΠΉ.
  • Π£ΠΌΠ½ΠΎΠΆΠ΅Π½ΠΈΠ΅ Π½Π° Π΅Π΄ΠΈΠ½ΠΈΡ‡Π½ΡƒΡŽ ΠΌΠ°Ρ‚Ρ€ΠΈΡ†Ρƒ ΠΊΠΎΠΌΠΌΡƒΡ‚Π°Ρ‚ΠΈΠ²Π½ΠΎ, хотя порядок Π΅Π΄ΠΈΠ½ΠΈΡ†Ρ‹ ΠΌΠΎΠΆΠ΅Ρ‚ ΠΈΠ·ΠΌΠ΅Π½Π΅Π½ΠΈΠ΅

Π˜Π΄Π΅Π½Ρ‚ΠΈΡ„ΠΈΠΊΠ°Ρ†ΠΈΠΎΠ½Π½Π°Ρ ΠΌΠ°Ρ‚Ρ€ΠΈΡ†Π° Ρ€Π°Π·ΠΌΠ΅Ρ€Π° 2

Π˜Π΄Π΅Π½Ρ‚ΠΈΡ„ΠΈΠΊΠ°Ρ†ΠΈΠΎΠ½Π½Π°Ρ ΠΌΠ°Ρ‚Ρ€ΠΈΡ†Π° Ρ€Π°Π·ΠΌΠ΅Ρ€Π° 3

I 3 = 1 0 0
0 1 0
0 0 1

Бвойства ΠΌΠ°Ρ‚Ρ€ΠΈΡ†

Π˜ΠΌΡƒΡ‰Π΅ΡΡ‚Π²ΠΎ ΠŸΡ€ΠΈΠΌΠ΅Ρ€
ΠšΠΎΠΌΠΌΡƒΡ‚Π°Ρ‚ΠΈΠ²Π½ΠΎΡΡ‚ΡŒ слоТСния А + Π’ = Π’ + А
ΠΡΡΠΎΡ†ΠΈΠ°Ρ‚ΠΈΠ²Π½ΠΎΡΡ‚ΡŒ добавлСния А + (Π’ + Π‘) = (А + Π’) + Π‘
ΠΡΡΠΎΡ†ΠΈΠ°Ρ‚ΠΈΠ²Π½ΠΎΡΡ‚ΡŒ скалярного умноТСния (ΠΊΠ΄) А = с (дА)
Бкалярная идСнтификация 1А = А (1) = А
Π Π°ΡΠΏΡ€Π΅Π΄Π΅Π»ΠΈΡ‚Π΅Π»ΡŒΠ½Ρ‹ΠΉ c (A + B) = cA + cB
Π Π°ΡΠΏΡ€Π΅Π΄Π΅Π»ΠΈΡ‚Π΅Π»ΡŒΠ½Ρ‹ΠΉ (с + Π³) А = сА + дА
Π˜Π΄Π΅Π½Ρ‚ΠΈΡ„ΠΈΠΊΠ°Ρ‚ΠΎΡ€ Π΄ΠΎΠ±Π°Π²ΠΊΠΈ А + О = О + А = А
ΠΡΡΠΎΡ†ΠΈΠ°Ρ‚ΠΈΠ²Π½ΠΎΡΡ‚ΡŒ умноТСния А (Π’Π‘) = (АВ) Π‘
Π Π°ΡΠΏΡ€Π΅Π΄Π΅Π»ΠΈΡ‚Π΅Π»ΡŒΠ½Ρ‹ΠΉ Π»Π΅Π²Ρ‹ΠΉ А (Π’ + Π‘) = АВ + АБ
ΠŸΡ€Π°Π²Ρ‹ΠΉ Ρ€Π°ΡΠΏΡ€Π΅Π΄Π΅Π»ΠΈΡ‚Π΅Π»ΡŒ (А + Π’) Π‘ = АБ + Π’Π‘
Бкалярная Π°ΡΡΠΎΡ†ΠΈΠ°Ρ‚ΠΈΠ²Π½ΠΎΡΡ‚ΡŒ / ΠΊΠΎΠΌΠΌΡƒΡ‚Π°Ρ‚ΠΈΠ²Π½ΠΎΡΡ‚ΡŒ c (AB) = (cA) B = A (cB) = (AB) c
ΠœΡƒΠ»ΡŒΡ‚ΠΈΠΏΠ»ΠΈΠΊΠ°Ρ‚ΠΈΠ²Π½Π°Ρ ΠΈΠ΄Π΅Π½Ρ‚ΠΈΡ‡Π½ΠΎΡΡ‚ΡŒ IA = AI = A

Бвойства вСщСствСнных чисСл, Π½Π΅ ΡΠ²Π»ΡΡŽΡ‰ΠΈΠ΅ΡΡ свойствами ΠΌΠ°Ρ‚Ρ€ΠΈΡ†

ΠšΠΎΠΌΠΌΡƒΡ‚Π°Ρ‚ΠΈΠ²Π½ΠΎΡΡ‚ΡŒ умноТСния

  • Π’Ρ‹ Π½Π΅ ΠΌΠΎΠΆΠ΅Ρ‚Π΅ ΠΈΠ·ΠΌΠ΅Π½ΠΈΡ‚ΡŒ порядок Π·Π°Π΄Π°Ρ‡ΠΈ умноТСния ΠΈ ΠΎΠΆΠΈΠ΄Π°Ρ‚ΡŒ ΠΏΠΎΠ»ΡƒΡ‡ΠΈΡ‚ΡŒ Ρ‚ΠΎ ΠΆΠ΅ самоС ΠΏΡ€ΠΎΠ΄ΡƒΠΊΡ‚.AB β‰  BA
  • Π’Ρ‹ Π΄ΠΎΠ»ΠΆΠ½Ρ‹ Π±Ρ‹Ρ‚ΡŒ остороТны ΠΏΡ€ΠΈ Ρ€Π°Π·Π»ΠΎΠΆΠ΅Π½ΠΈΠΈ ΠΎΠ±Ρ‰ΠΈΡ… ΠΌΠ½ΠΎΠΆΠΈΡ‚Π΅Π»Π΅ΠΉ, Ρ‡Ρ‚ΠΎΠ±Ρ‹ ΡƒΠ±Π΅Π΄ΠΈΡ‚ΡŒΡΡ, Ρ‡Ρ‚ΠΎ ΠΎΠ½ΠΈ ΠΎΠ΄Π½ΠΎ ΠΈ Ρ‚ΠΎΠΆΠ΅ боковая сторона. AX + BX = (A + B) X ΠΈ XA + XB = X (A + B), Π½ΠΎ AX + XB Π½Π΅ учитываСтся.

Бвойство Π½ΡƒΠ»Π΅Π²ΠΎΠ³ΠΎ ΠΏΡ€ΠΎΠ΄ΡƒΠΊΡ‚Π°

  • Π’ΠΎΡ‚ Ρ„Π°ΠΊΡ‚, Ρ‡Ρ‚ΠΎ ΠΏΡ€ΠΎΠΈΠ·Π²Π΅Π΄Π΅Π½ΠΈΠ΅ Π΄Π²ΡƒΡ… ΠΌΠ°Ρ‚Ρ€ΠΈΡ† являСтся Π½ΡƒΠ»Π΅Π²ΠΎΠΉ ΠΌΠ°Ρ‚Ρ€ΠΈΡ†Π΅ΠΉ, Π½Π΅ ΠΎΠ·Π½Π°Ρ‡Π°Π΅Ρ‚, Ρ‡Ρ‚ΠΎ ΠΎΠ΄Π½Π° ΠΈΠ· ΠΈΠΌ Π±Ρ‹Π»Π° нулСвая ΠΌΠ°Ρ‚Ρ€ΠΈΡ†Π°.

ΠœΡƒΠ»ΡŒΡ‚ΠΈΠΏΠ»ΠΈΠΊΠ°Ρ‚ΠΈΠ²Π½ΠΎΠ΅ свойство равСнства

  • Если A = B, Ρ‚ΠΎ AC = BC. Π­Ρ‚ΠΎ свойство всС Π΅Ρ‰Π΅ Π²Π΅Ρ€Π½ΠΎ, Π½ΠΎ ΠΎΠ±Ρ€Π°Ρ‚Π½ΠΎΠ΅ Π½Π΅ ΠΎΠ±ΡΠ·Π°Ρ‚Π΅Π»ΡŒΠ½ΠΎ Π²Π΅Ρ€Π½ΠΎ.Π’ΠΎ, Ρ‡Ρ‚ΠΎ AC = BC, Π½Π΅ ΠΎΠ·Π½Π°Ρ‡Π°Π΅Ρ‚, Ρ‡Ρ‚ΠΎ A = B.
  • ΠŸΠΎΡΠΊΠΎΠ»ΡŒΠΊΡƒ ΡƒΠΌΠ½ΠΎΠΆΠ΅Π½ΠΈΠ΅ ΠΌΠ°Ρ‚Ρ€ΠΈΡ† Π½Π΅ ΠΊΠΎΠΌΠΌΡƒΡ‚Π°Ρ‚ΠΈΠ²Π½ΠΎ, Π²Ρ‹ Π΄ΠΎΠ»ΠΆΠ½Ρ‹ ΡƒΠΌΠ½ΠΎΠΆΠ΅Π½ΠΈΠ΅ Π΄ΠΎ ΠΈΠ»ΠΈ послС умноТСния с ΠΎΠ±Π΅ΠΈΡ… сторон уравнСния. Π’ΠΎ Π΅ΡΡ‚ΡŒ, Ссли A = B, Ρ‚ΠΎΠ³Π΄Π° AC = BC ΠΈΠ»ΠΈ CA = CB, Π½ΠΎ AC β‰  CB.

НСт ΠΌΠ°Ρ‚Ρ€ΠΈΡ‡Π½ΠΎΠ³ΠΎ дСлСния

  • Надо ΡƒΠΌΠ½ΠΎΠΆΠΈΡ‚ΡŒ Π½Π° ΠΎΠ±Ρ€Π°Ρ‚Π½ΡƒΡŽ ΠΌΠ°Ρ‚Ρ€ΠΈΡ†Ρƒ

ΠžΡ†Π΅Π½ΠΊΠ° Ρ„ΡƒΠ½ΠΊΡ†ΠΈΠΈ с ΠΏΠΎΠΌΠΎΡ‰ΡŒΡŽ ΠΌΠ°Ρ‚Ρ€ΠΈΡ†Ρ‹

Рассмотрим Ρ„ΡƒΠ½ΠΊΡ†ΠΈΡŽ f (x) = x 2 — 4x + 3 ΠΈ ΠΌΠ°Ρ‚Ρ€ΠΈΡ†Ρƒ A

ΠŸΠ΅Ρ€Π²ΠΎΠ½Π°Ρ‡Π°Π»ΡŒΠ½Π°Ρ ΠΏΠΎΠΏΡ‹Ρ‚ΠΊΠ° ΠΎΡ†Π΅Π½ΠΈΡ‚ΡŒ f (A) Π±ΡƒΠ΄Π΅Ρ‚ Π·Π°ΠΊΠ»ΡŽΡ‡Π°Ρ‚ΡŒΡΡ Π² Π·Π°ΠΌΠ΅Π½Π΅ ΠΊΠ°ΠΆΠ΄ΠΎΠ³ΠΎ x с A, Ρ‡Ρ‚ΠΎΠ±Ρ‹ ΠΏΠΎΠ»ΡƒΡ‡ΠΈΡ‚ΡŒ f (A) = A 2 — 4A + 3.Π•ΡΡ‚ΡŒ ΠΎΠ΄Π½ΠΎ нСбольшоС ΠΏΡ€ΠΎΠ±Π»Π΅ΠΌΠ°, ΠΎΠ΄Π½Π°ΠΊΠΎ. ΠšΠΎΠ½ΡΡ‚Π°Π½Ρ‚Π° 3 Ρ€Π°Π²Π½Π° Π½Π΅ ΠΌΠ°Ρ‚Ρ€ΠΈΡ†Π°, ΠΈ Π²Ρ‹ Π½Π΅ ΠΌΠΎΠΆΠ΅Ρ‚Π΅ Π΄ΠΎΠ±Π°Π²ΠΈΡ‚ΡŒ ΠΌΠ°Ρ‚Ρ€ΠΈΡ†Ρ‹ ΠΈ скаляры вмСстС. Π˜Ρ‚Π°ΠΊ, ΡƒΠΌΠ½ΠΎΠΆΠ°Π΅ΠΌ константа ΠΌΠ°Ρ‚Ρ€ΠΈΡ†Π΅ΠΉ идСнтичности.

f (A) = A 2 — 4A + 3I.

ΠžΡ†Π΅Π½ΠΈΡ‚Π΅ ΠΊΠ°ΠΆΠ΄Ρ‹ΠΉ Ρ‡Π»Π΅Π½ Ρ„ΡƒΠ½ΠΊΡ†ΠΈΠΈ, Π° Π·Π°Ρ‚Π΅ΠΌ слоТитС ΠΈΡ….

А 2 = 1 2 * 1 2 = 7 10
3 4 3 4 15 22
-4 А = -4 1 2 = -4 -8
3 4 -12 -16
3I = 3 1 0 = 3 0
0 1 0 3
f (А) = 7 10 + -4 -8 + 3 0 = 6 2
15 22 -12 -16 0 3 3 9

Π€Π°ΠΊΡ‚ΠΎΡ€ΠΈΠ½Π³ΠΎΠ²Ρ‹Π΅ выраТСния

ΠŸΠΎΠΊΠ°Π·Π°Π½Ρ‹ Π½Π΅ΠΊΠΎΡ‚ΠΎΡ€Ρ‹Π΅ ΠΏΡ€ΠΈΠΌΠ΅Ρ€Ρ‹ Ρ„Π°ΠΊΡ‚ΠΎΡ€ΠΈΠ½Π³Π°.Π£ΠΏΡ€ΠΎΡ‰Π°ΠΉΡ‚Π΅ ΠΈ Ρ€Π΅ΡˆΠ°ΠΉΡ‚Π΅ ΠΊΠ°ΠΊ ΠΎΠ±Ρ‹Ρ‡Π½ΠΎ, Π½ΠΎ ΠΏΠΎΠΌΠ½ΠΈΡ‚Π΅ эта ΠΌΠ°Ρ‚Ρ€ΠΈΡ†Π° ΡƒΠΌΠ½ΠΎΠΆΠ΅Π½ΠΈΠ΅ Π½Π΅ ΠΊΠΎΠΌΠΌΡƒΡ‚Π°Ρ‚ΠΈΠ²Π½ΠΎ ΠΈ Π½Π΅Ρ‚ ΠΌΠ°Ρ‚Ρ€ΠΈΡ‡Π½ΠΎΠ³ΠΎ дСлСния.

2X + 3X = 5X

AX + BX = (A + B) X

Π₯А + Π₯Π’ = Π₯ (А + Π’)

AX + 5X = (A + 5I) X

AX + XB Π½Π΅ ΡƒΡ‡ΠΈΡ‚Ρ‹Π²Π°Π΅Ρ‚

РСшСниС ΡƒΡ€Π°Π²Π½Π΅Π½ΠΈΠΉ

БистСму Π»ΠΈΠ½Π΅ΠΉΠ½Ρ‹Ρ… ΡƒΡ€Π°Π²Π½Π΅Π½ΠΈΠΉ ΠΌΠΎΠΆΠ½ΠΎ Π·Π°ΠΏΠΈΡΠ°Ρ‚ΡŒ ΠΊΠ°ΠΊ AX = B, Π³Π΄Π΅ A — коэффициСнт ΠΌΠ°Ρ‚Ρ€ΠΈΡ†Π°, X — Π²Π΅ΠΊΡ‚ΠΎΡ€-столбСц, содСрТащий ΠΏΠ΅Ρ€Π΅ΠΌΠ΅Π½Π½Ρ‹Π΅, Π° B — правая боковая сторона. Π’ ΡΠ»Π΅Π΄ΡƒΡŽΡ‰Π΅ΠΌ Ρ€Π°Π·Π΄Π΅Π»Π΅ ΠΌΡ‹ ΡƒΠ·Π½Π°Π΅ΠΌ, ΠΊΠ°ΠΊ Ρ€Π΅ΡˆΠΈΡ‚ΡŒ это ΡƒΡ€Π°Π²Π½Π΅Π½ΠΈΠ΅.

Если сущСствуСт Π±ΠΎΠ»Π΅Π΅ ΠΎΠ΄Π½ΠΎΠΉ систСмы Π»ΠΈΠ½Π΅ΠΉΠ½Ρ‹Ρ… ΡƒΡ€Π°Π²Π½Π΅Π½ΠΈΠΉ с ΠΎΠ΄Π½ΠΈΠΌ ΠΈ Ρ‚Π΅ΠΌ ΠΆΠ΅ коэффициСнтом ΠΌΠ°Ρ‚Ρ€ΠΈΡ†Π°, Ρ‚ΠΎ Π²Ρ‹ ΠΌΠΎΠΆΠ΅Ρ‚Π΅ Ρ€Π°ΡΡˆΠΈΡ€ΠΈΡ‚ΡŒ ΠΌΠ°Ρ‚Ρ€ΠΈΡ†Ρƒ B, Ρ‡Ρ‚ΠΎΠ±Ρ‹ ΠΎΠ½Π° содСрТала Π±ΠΎΠ»Π΅Π΅ ΠΎΠ΄Π½ΠΎΠ³ΠΎ столбца. Π‘Ρ‚Π°Π²ΠΈΡ‚ΡŒ ΠΊΠ°ΠΆΠ΄ΡƒΡŽ ΠΏΡ€Π°Π²ΡƒΡŽ Ρ‡Π°ΡΡ‚ΡŒ Π² свой столбСц.

Π£ΠΌΠ½ΠΎΠΆΠ΅Π½ΠΈΠ΅ ΠΌΠ°Ρ‚Ρ€ΠΈΡ†

Π£ΠΌΠ½ΠΎΠΆΠ΅Π½ΠΈΠ΅ ΠΌΠ°Ρ‚Ρ€ΠΈΡ† Π²ΠΊΠ»ΡŽΡ‡Π°Π΅Ρ‚ суммированиС произвСдСния. Π­Ρ‚ΠΎ умСстно Ρ‚Π°ΠΌ, Π³Π΄Π΅ Π²Ρ‹ Π½ΡƒΠΆΠ½ΠΎ ΡƒΠΌΠ½ΠΎΠΆΠ°ΠΉΡ‚Π΅ Π²Π΅Ρ‰ΠΈ вмСстС, Π° Π·Π°Ρ‚Π΅ΠΌ Π”ΠΎΠ±Π°Π²Π»ΡΡ‚ΡŒ. НапримСр, ΡƒΠΌΠ½ΠΎΠΆΠ΅Π½ΠΈΠ΅ количСства Π΅Π΄ΠΈΠ½ΠΈΡ† Π½Π° ΡΡ‚ΠΎΠΈΠΌΠΎΡΡ‚ΡŒ Π΅Π΄ΠΈΠ½ΠΈΡ†Ρ‹ Π±ΡƒΠ΄Π΅Ρ‚ Π΄Π°Ρ‚ΡŒ ΠΎΠ±Ρ‰ΡƒΡŽ Π‘Ρ‚ΠΎΠΈΠΌΠΎΡΡ‚ΡŒ.

Π•Π΄ΠΈΠ½ΠΈΡ†Ρ‹ ΠΏΠΎ ΠΏΡ€ΠΎΠ΄ΡƒΠΊΡ‚Ρƒ Π½Π°ΠΉΠ΄Π΅Π½Ρ‹ ΠΏΡƒΡ‚Π΅ΠΌ выполнСния ΠΌΠΎΠ΄ΡƒΠ»ΡŒΠ½ΠΎΠ³ΠΎ Π°Π½Π°Π»ΠΈΠ·Π° Π½Π° ΠΌΠ°Ρ‚Ρ€ΠΈΡ†Ρ‹.Π­Ρ‚ΠΈΠΊΠ΅Ρ‚ΠΊΠΈ для ΠΏΡ€ΠΎΠ΄ΡƒΠΊΡ‚Π° — это этикСтки строк ΠΏΠ΅Ρ€Π²ΠΎΠ³ΠΎ ΠΌΠ°Ρ‚Ρ€ΠΈΡ†Π° ΠΈ ΠΌΠ΅Ρ‚ΠΊΠΈ столбцов вторая ΠΌΠ°Ρ‚Ρ€ΠΈΡ†Π°.

ΠœΠ°Ρ‚Ρ€ΠΈΡ†Ρ‹ слоТСния ΠΈ вычитания

Π”ΠΎΠ±Π°Π²Π»Π΅Π½ΠΈΠ΅ ΠΈ ΠΌΠ°Ρ‚Ρ€ΠΈΡ†Ρ‹ вычитания


ΠœΠ°Ρ‚Ρ€ΠΈΡ†Π° слоТСна справСдливо простой, ΠΈ дСлаСтся ΠΏΠΎ Π²Ρ…ΠΎΠ΄Ρƒ.

  • Π”ΠΎΠ±Π°Π²ΡŒΡ‚Π΅ ΡΠ»Π΅Π΄ΡƒΡŽΡ‰ΠΈΠ΅ ΠΌΠ°Ρ‚Ρ€ΠΈΡ†Ρ‹:

    МнС Π½ΡƒΠΆΠ½ΠΎ Π΄ΠΎΠ±Π°Π²ΠΈΡ‚ΡŒ ΠΏΠ°Ρ€Ρ‹ записСй, Π° Π·Π°Ρ‚Π΅ΠΌ упроститС для ΠΎΠΊΠΎΠ½Ρ‡Π°Ρ‚Π΅Π»ΡŒΠ½ΠΎΠ³ΠΎ ΠΎΡ‚Π²Π΅Ρ‚Π°:

    Π˜Ρ‚Π°ΠΊ, ΠΎΡ‚Π²Π΅Ρ‚:


Π”ΠΎ сих ΠΏΠΎΡ€ Π²Ρ‹ Π±Ρ‹Π»ΠΈ Π²ΠΎΠ·ΠΌΠΎΠΆΠ½ΠΎΡΡ‚ΡŒ Π΄ΠΎΠ±Π°Π²Π»ΡΡ‚ΡŒ Π»ΡŽΠ±Ρ‹Π΅ Π΄Π²Π΅ Π²Π΅Ρ‰ΠΈ, ΠΊΠΎΡ‚ΠΎΡ€Ρ‹Π΅ Π²Π°ΠΌ нравятся: числа, ΠΏΠ΅Ρ€Π΅ΠΌΠ΅Π½Π½Ρ‹Π΅, уравнСния, ΠΈ Ρ‚Π°ΠΊ Π΄Π°Π»Π΅Π΅.Но слоТСниС Π½Π΅ всСгда Ρ€Π°Π±ΠΎΡ‚Π°Π΅Ρ‚ с ΠΌΠ°Ρ‚Ρ€ΠΈΡ†Π°ΠΌΠΈ.

  • Π’Ρ‹ΠΏΠΎΠ»Π½ΠΈΡ‚Π΅ ΡƒΠΊΠ°Π·Π°Π½Π½Ρ‹Π΅ ΠΎΠΏΠ΅Ρ€Π°Ρ†ΠΈΠΈ, ΠΈΠ»ΠΈ ΠΎΠ±ΡŠΡΡΠ½ΠΈΡ‚Π΅, ΠΏΠΎΡ‡Π΅ΠΌΡƒ это Π½Π΅Π²ΠΎΠ·ΠΌΠΎΠΆΠ½ΠΎ.

    ΠŸΠΎΡΠΊΠΎΠ»ΡŒΠΊΡƒ ΠΌΠ°Ρ‚Ρ€ΠΈΡ†Ρ‹ ΡΠΊΠ»Π°Π΄Ρ‹Π²Π°ΡŽΡ‚ΡΡ ΠΏΠΎ ΡƒΠΌΠΎΠ»Ρ‡Π°Π½ΠΈΡŽ, я Π΄ΠΎΠ»ΠΆΠ΅Π½ Π΄ΠΎΠ±Π°Π²ΠΈΡ‚ΡŒ 1 ΠΈ 4, 2 ΠΈ 5, 0 ΠΈ 7, ΠΈ 3 ΠΈ 8. Но Ρ‡Ρ‚ΠΎ ΠΌΠ½Π΅ Π΄ΠΎΠ±Π°Π²ΠΈΡ‚ΡŒ ΠΊ 6 Π° ΠΊ 9? Π’ ΠΏΠ΅Ρ€Π²ΠΎΠΉ ΠΌΠ°Ρ‚Ρ€ΠΈΡ†Π΅ Π½Π΅Ρ‚ ΡΠΎΠΎΡ‚Π²Π΅Ρ‚ΡΡ‚Π²ΡƒΡŽΡ‰ΠΈΡ… записСй, ΠΊΠΎΡ‚ΠΎΡ€Ρ‹Π΅ ΠΌΠΎΠΆΠ½ΠΎ Π΄ΠΎΠ±Π°Π²ΠΈΡ‚ΡŒ ΠΊ этим элСмСнтам Π²Ρ‚ΠΎΡ€ΠΎΠΉ ΠΌΠ°Ρ‚Ρ€ΠΈΡ†Ρ‹.Π˜Ρ‚Π°ΠΊ, ΠΎΡ‚Π²Π΅Ρ‚:

      Π― Π½Π΅ ΠΌΠΎΠ³Ρƒ Π΄ΠΎΠ±Π°Π²ΠΈΡ‚ΡŒ эти ΠΌΠ°Ρ‚Ρ€ΠΈΡ†Ρ‹, ΠΏΠΎΡ‚ΠΎΠΌΡƒ Ρ‡Ρ‚ΠΎ ΠΎΠ½ΠΈ Π½Π΅ ΠΎΠ΄Π½ΠΎΠ³ΠΎ Ρ€Π°Π·ΠΌΠ΅Ρ€Π° .

Π­Ρ‚ΠΎ всСгда Ρ‚Π°ΠΊ: Π§Ρ‚ΠΎΠ±Ρ‹ ΠΌΠΎΠΆΠ½ΠΎ Π±Ρ‹Π»ΠΎ Π΄ΠΎΠ±Π°Π²ΠΈΡ‚ΡŒ Π΄Π²Π΅ ΠΌΠ°Ρ‚Ρ€ΠΈΡ†Ρ‹, ΠΎΠ½ΠΈ Π΄ΠΎΠ»ΠΆΠ½Ρ‹ Π±Ρ‹Ρ‚ΡŒ ΠΎΠ΄ΠΈΠ½Π°ΠΊΠΎΠ²ΠΎΠ³ΠΎ Ρ€Π°Π·ΠΌΠ΅Ρ€Π°. Если ΠΎΠ½ΠΈ Ρ€Π°Π·Π½ΠΎΠ³ΠΎ Ρ€Π°Π·ΠΌΠ΅Ρ€Π° (Ссли Ρƒ Π½ΠΈΡ… Ρ€Π°Π·Π½Ρ‹Π΅ Β«Ρ€Π°Π·ΠΌΠ΅Ρ€Ρ‹Β»), Ρ‚ΠΎΠ³Π΄Π° слоТСниС «Π½Π΅ ΠΎΠΏΡ€Π΅Π΄Π΅Π»Π΅Π½ΠΎ» (Π½Π΅ Π΄Π΅Π»Π°Π΅Ρ‚ матСматичСских смысл).


Π’Ρ‹Ρ‡ΠΈΡ‚Π°Ρ‚Π΅Π»ΡŒΠ½Ρ‹Π΅ Ρ€Π°Π±ΠΎΡ‚Ρ‹ Π½Π°Ρ‡Π°Π»ΡŒΠ½Ρ‹Π΅, Ρ‚ΠΎΠΆΠ΅.АвторскиС ΠΏΡ€Π°Π²Π° Π­Π»ΠΈΠ·Π°Π±Π΅Ρ‚ Π‘Ρ‚Π°ΠΏΠ΅Π»ΡŒ 2003-2011 ВсС ΠΏΡ€Π°Π²Π° Π·Π°Ρ‰ΠΈΡ‰Π΅Π½Ρ‹

  • Учитывая ΡΠ»Π΅Π΄ΡƒΡŽΡ‰Π΅Π΅ ΠΌΠ°Ρ‚Ρ€ΠΈΡ†Ρ‹, Π½Π°ΠΉΡ‚ΠΈ A B ΠΈ A C , ΠΈΠ»ΠΈ ΠΎΠ±ΡŠΡΡΠ½ΠΈΡ‚ΡŒ ΠΏΠΎΡ‡Π΅ΠΌΡƒ нСльзя.

    А ΠΈ B ΠΎΠ΄ΠΈΠ½Π°ΠΊΠΎΠ²ΠΎΠ³ΠΎ Ρ€Π°Π·ΠΌΠ΅Ρ€Π°, ΠΏΠΎ 2 3 ΠΌΠ°Ρ‚Ρ€ΠΈΡ†Ρ‹, Ρ‚Π°ΠΊ Ρ‡Ρ‚ΠΎ я ΠΌΠΎΠ³Ρƒ Π²Ρ‹Ρ‡Π΅ΡΡ‚ΡŒ, Ρ€Π°Π±ΠΎΡ‡ΠΈΠΉ Π½Π°Ρ‡Π°Π»ΡŒΠ½Ρ‹ΠΉ:

    Однако A ΠΈ C Π½Π΅ ΠΎΠ΄ΠΈΠ½Π°ΠΊΠΎΠ²ΠΎΠ³ΠΎ Ρ€Π°Π·ΠΌΠ΅Ρ€Π°, Ρ‚Π°ΠΊ ΠΊΠ°ΠΊ A это 2 3 ΠΈ C Ρ€Π°Π²Π½ΠΎ 2 2.Π’Π°ΠΊΠΈΠΌ ΠΎΠ±Ρ€Π°Π·ΠΎΠΌ, это Π²Ρ‹Ρ‡ΠΈΡ‚Π°Π½ΠΈΠ΅ Π½Π΅ ΠΎΠΏΡ€Π΅Π΄Π΅Π»Π΅Π½ΠΎ.

      А Π‘ Π½Π΅ ΠΎΠΏΡ€Π΅Π΄Π΅Π»Π΅Π½ΠΎ, Ρ‚.ΠΊ. A ΠΈ C Π½Π΅ ΠΎΠ΄ΠΈΠ½Π°ΠΊΠΎΠ²ΠΎΠ³ΠΎ Ρ€Π°Π·ΠΌΠ΅Ρ€Π°.


ΠœΠ°Ρ‚Ρ€ΠΈΡ‡Π½ΠΎΠ΅ слоТСниС ΠΈ Π²Ρ‹Ρ‡ΠΈΡ‚Π°Π½ΠΈΠ΅, Π³Π΄Π΅ ΠΎΠΏΡ€Π΅Π΄Π΅Π»Π΅Π½ΠΎ (Ρ‚ΠΎ Π΅ΡΡ‚ΡŒ, Π³Π΄Π΅ ΠΌΠ°Ρ‚Ρ€ΠΈΡ†Ρ‹ ΠΈΠΌΠ΅ΡŽΡ‚ ΠΎΠ΄ΠΈΠ½Π°ΠΊΠΎΠ²Ρ‹ΠΉ Ρ€Π°Π·ΠΌΠ΅Ρ€, поэтому Π΄ΠΎΠ±Π°Π²Π»Π΅Π½ΠΈΠ΅ ΠΈ Π²Ρ‹Ρ‡ΠΈΡ‚Π°Π½ΠΈΠ΅ ΠΈΠΌΠ΅Π΅Ρ‚ смысл), ΠΌΠΎΠΆΠ½ΠΎ ΠΏΡ€Π΅Π²Ρ€Π°Ρ‚ΠΈΡ‚ΡŒ Π² домашниС задания.

  • НайдитС значСния x ΠΈ y Π΄Π°Π½ΠΎ ΡΠ»Π΅Π΄ΡƒΡŽΡ‰Π΅Π΅ ΡƒΡ€Π°Π²Π½Π΅Π½ΠΈΠ΅:

    Π‘Π½Π°Ρ‡Π°Π»Π° я ΡƒΠΏΡ€ΠΎΡ‰Ρƒ Π»Π΅Π²ΡƒΡŽ Ρ‡Π°ΡΡ‚ΡŒ Π½Π΅ΠΌΠ½ΠΎΠ³ΠΎ, Π΄ΠΎΠ±Π°Π²ΠΈΠ² пошагово:

    ΠŸΠΎΡΠΊΠΎΠ»ΡŒΠΊΡƒ ΠΌΠ°Ρ‚Ρ€ΠΈΡ‡Π½ΠΎΠ΅ равСнство Ρ€Π°Π±ΠΎΡ‚Π°Π΅Ρ‚ ΠΏΠΎ Π²Ρ…ΠΎΠ΄Π°ΠΌ, Π― ΠΌΠΎΠ³Ρƒ ΡΡ€Π°Π²Π½ΠΈΠ²Π°Ρ‚ΡŒ записи ΠΈ ΡΠΎΡΡ‚Π°Π²Π»ΡΡ‚ΡŒ простыС уравнСния, ΠΊΠΎΡ‚ΠΎΡ€Ρ‹Π΅ я ΠΌΠΎΠ³Ρƒ Ρ€Π΅ΡˆΠΈΡ‚ΡŒ.Π’ этом случаС 1,2-записи скаТитС ΠΌΠ½Π΅, Ρ‡Ρ‚ΠΎ x + 6 = 7, Π° 2,1-элСмСнты скаТи ΠΌΠ½Π΅, Ρ‡Ρ‚ΠΎ 2 Π³ 3 = 5. РСшая, я ΠΏΠΎΠ»ΡƒΡ‡Π°ΡŽ:

      Ρ… + 6 = 7
      x = 1

      2 y 3 = 5
      2 y = 2
      y = 1

Π’Π΅Ρ€Ρ… | Π’Π΅Ρ€Π½ΡƒΡ‚ΡŒΡΡ ΠΊ индСксу

Π¦ΠΈΡ‚ΠΈΡ€ΡƒΠΉΡ‚Π΅ эту ΡΡ‚Π°Ρ‚ΡŒΡŽ ΠΊΠ°ΠΊ:

Π‘Ρ‚Π°ΠΏΠ΅Π»ΡŒ, Π•Π»ΠΈΠ·Π°Π²Π΅Ρ‚Π°.Β«Π‘Π»ΠΎΠΆΠ΅Π½ΠΈΠ΅ ΠΈ Π²Ρ‹Ρ‡ΠΈΡ‚Π°Π½ΠΈΠ΅ ΠΌΠ°Ρ‚Ρ€ΠΈΡ†Β». ΠŸΡƒΡ€ΠΏΡƒΡ€Π½Π°Ρ ΠœΠ°Ρ‚Π΅ΠΌΠ°Ρ‚ΠΈΠΊΠ° . Доступно ΠΏΠΎ Π½ΠΎΠΌΠ΅Ρ€Ρƒ
https://www.purplemath.com/modules/mtrxadd.htm . Π”Π°Ρ‚Π° обращСния [Π”Π°Ρ‚Π°] [ΠœΠ΅ΡΡΡ†] 2016 Π³.


Π‘Π»ΠΎΠΆΠ΅Π½ΠΈΠ΅ ΠΈ Π²Ρ‹Ρ‡ΠΈΡ‚Π°Π½ΠΈΠ΅ ΠΌΠ°Ρ‚Ρ€ΠΈΡ† ΠΈ ΡƒΠΌΠ½ΠΎΠΆΠ΅Π½ΠΈΠ΅ ΠΌΠ°Ρ‚Ρ€ΠΈΡ†Ρ‹ Π½Π° константу

ΠŸΡ€ΠΈ ΡƒΠΌΠ½ΠΎΠΆΠ΅Π½ΠΈΠΈ ΠΌΠ°Ρ‚Ρ€ΠΈΡ†Ρ‹ Π½Π° скаляр (константа ΠΈΠ»ΠΈ число), Π° Ρ‚Π°ΠΊΠΆΠ΅ ΠΏΡ€ΠΈ слоТСнии ΠΈ Π²Ρ‹Ρ‡ΠΈΡ‚Π°Π½ΠΈΠΈ ΠΌΠ°Ρ‚Ρ€ΠΈΡ† ΠΎΠΏΠ΅Ρ€Π°Ρ†ΠΈΠΈ Π²Ρ‹ΠΏΠΎΠ»Π½ΡΡŽΡ‚ΡΡ построчно.Π”Π°Π²Π°ΠΉΡ‚Π΅ рассмотрим ΠΊΠ°ΠΆΠ΄ΡƒΡŽ ΠΎΠΏΠ΅Ρ€Π°Ρ†ΠΈΡŽ ΠΎΡ‚Π΄Π΅Π»ΡŒΠ½ΠΎ, Ρ‡Ρ‚ΠΎΠ±Ρ‹ ΡƒΠ²ΠΈΠ΄Π΅Ρ‚ΡŒ, ΠΊΠ°ΠΊ это Ρ€Π°Π±ΠΎΡ‚Π°Π΅Ρ‚.

Π‘ΠΎΠ΄Π΅Ρ€ΠΆΠ°Π½ΠΈΠ΅

  1. Π”ΠΎΠ±Π°Π²Π»Π΅Π½ΠΈΠ΅ ΠΌΠ°Ρ‚Ρ€ΠΈΡ†
  2. ΠœΠ°Ρ‚Ρ€ΠΈΡ†Ρ‹ вычитания
  3. Π£ΠΌΠ½ΠΎΠΆΠ΅Π½ΠΈΠ΅ ΠΌΠ°Ρ‚Ρ€ΠΈΡ†Ρ‹ Π½Π° константу (скалярноС ΡƒΠΌΠ½ΠΎΠΆΠ΅Π½ΠΈΠ΅)
  4. Π‘ΠΎΡ‡Π΅Ρ‚Π°Π½ΠΈΠ΅ слоТСния, вычитания ΠΈ скалярного умноТСния

Ρ€Π΅ΠΊΠ»Π°ΠΌΠ°

Π”ΠΎΠ±Π°Π²Π»Π΅Π½ΠΈΠ΅ ΠΌΠ°Ρ‚Ρ€ΠΈΡ†

Π§Ρ‚ΠΎΠ±Ρ‹ Π΄ΠΎΠ±Π°Π²ΠΈΡ‚ΡŒ Π΄Π²Π΅ ΠΌΠ°Ρ‚Ρ€ΠΈΡ†Ρ‹, Π΄ΠΎΠ±Π°Π²ΡŒΡ‚Π΅ ΡΠΎΠΎΡ‚Π²Π΅Ρ‚ΡΡ‚Π²ΡƒΡŽΡ‰ΠΈΠ΅ записи, ΠΊΠ°ΠΊ ΠΏΠΎΠΊΠ°Π·Π°Π½ΠΎ Π½ΠΈΠΆΠ΅. ΠžΠ±Ρ€Π°Ρ‚ΠΈΡ‚Π΅ Π²Π½ΠΈΠΌΠ°Π½ΠΈΠ΅, Ρ‡Ρ‚ΠΎ Π²Π°ΠΌ Π½ΡƒΠΆΠ½ΠΎ, Ρ‡Ρ‚ΠΎΠ±Ρ‹ ΠΌΠ°Ρ‚Ρ€ΠΈΡ†Ρ‹ Π±Ρ‹Π»ΠΈ ΠΎΠ΄ΠΈΠ½Π°ΠΊΠΎΠ²ΠΎΠ³ΠΎ Ρ€Π°Π·ΠΌΠ΅Ρ€Π°, Ρ‡Ρ‚ΠΎΠ±Ρ‹ это ΠΈΠΌΠ΅Π»ΠΎ смысл.

Если ΠΌΠ°Ρ‚Ρ€ΠΈΡ†Ρ‹ Ρ€Π°Π·Π½ΠΎΠ³ΠΎ Ρ€Π°Π·ΠΌΠ΅Ρ€Π°, слоТСниС Π½Π΅ ΠΎΠΏΡ€Π΅Π΄Π΅Π»Π΅Π½ΠΎ.

ΠœΠ°Ρ‚Ρ€ΠΈΡ†Ρ‹ вычитания

Π’Ρ‹Ρ‡ΠΈΡ‚Π°Π½ΠΈΠ΅ ΠΌΠ°Ρ‚Ρ€ΠΈΡ† Ρ€Π°Π±ΠΎΡ‚Π°Π΅Ρ‚ Ρ‚ΠΎΡ‡Π½ΠΎ Ρ‚Π°ΠΊ ΠΆΠ΅. Π’Ρ‹ ΠΌΠΎΠΆΠ΅Ρ‚Π΅ Π²Ρ‹Ρ‡Π΅ΡΡ‚ΡŒ запись Π·Π° записью.

Как ΠΈ Π² случаС слоТСния, это Π½Π΅ Π±Ρ‹Π»ΠΎ Π±Ρ‹ ΠΎΠΏΡ€Π΅Π΄Π΅Π»Π΅Π½ΠΎ, Ссли Π±Ρ‹ ΠΌΠ°Ρ‚Ρ€ΠΈΡ†Ρ‹ Π±Ρ‹Π»ΠΈ Ρ€Π°Π·Π½Ρ‹Ρ… Ρ€Π°Π·ΠΌΠ΅Ρ€ΠΎΠ². Π’ этой ситуации ваш ΠΎΡ‚Π²Π΅Ρ‚ Π±ΡƒΠ΄Π΅Ρ‚ просто Β«Π½Π΅ ΠΎΠΏΡ€Π΅Π΄Π΅Π»Π΅Π½Β».

Π£ΠΌΠ½ΠΎΠΆΠ΅Π½ΠΈΠ΅ ΠΌΠ°Ρ‚Ρ€ΠΈΡ†Ρ‹ Π½Π° константу (скалярноС ΡƒΠΌΠ½ΠΎΠΆΠ΅Π½ΠΈΠ΅)

Π£ΠΌΠ½ΠΎΠΆΠ΅Π½ΠΈΠ΅ ΠΌΠ°Ρ‚Ρ€ΠΈΡ†Ρ‹ Π½Π° константу ΠΈΠ»ΠΈ число (ΠΈΠ½ΠΎΠ³Π΄Π° Π½Π°Π·Ρ‹Π²Π°Π΅ΠΌΠΎΠ΅ скаляром) всСгда опрСдСляСтся нСзависимо ΠΎΡ‚ Ρ€Π°Π·ΠΌΠ΅Ρ€Π° ΠΌΠ°Ρ‚Ρ€ΠΈΡ†Ρ‹.Π’Π°ΠΌ просто Π½ΡƒΠΆΠ½ΠΎ ΡƒΠ±Π΅Π΄ΠΈΡ‚ΡŒΡΡ, Ρ‡Ρ‚ΠΎ каТдая запись Π² ΠΌΠ°Ρ‚Ρ€ΠΈΡ†Π΅ ΡƒΠΌΠ½ΠΎΠΆΠ΅Π½Π° Π½Π° число.

ОбъСдинСниС ΠΎΠΏΠ΅Ρ€Π°Ρ†ΠΈΠΉ

Π’ Π½Π΅ΠΊΠΎΡ‚ΠΎΡ€Ρ‹Ρ… вопросах вас ΠΌΠΎΠ³ΡƒΡ‚ ΠΏΠΎΠΏΡ€ΠΎΡΠΈΡ‚ΡŒ ΠΎΠ±ΡŠΠ΅Π΄ΠΈΠ½ΠΈΡ‚ΡŒ слоТСниС, Π²Ρ‹Ρ‡ΠΈΡ‚Π°Π½ΠΈΠ΅ ΠΈ ΡƒΠΌΠ½ΠΎΠΆΠ΅Π½ΠΈΠ΅ Π½Π° константу. Π—Π΄Π΅ΡΡŒ ΠΌΡ‹ рассмотрим нСсколько ΠΏΡ€ΠΈΠΌΠ΅Ρ€ΠΎΠ², Ρ‡Ρ‚ΠΎΠ±Ρ‹ ΡƒΠ±Π΅Π΄ΠΈΡ‚ΡŒΡΡ, Ρ‡Ρ‚ΠΎ Π²Ρ‹ Π·Π½Π°Π΅Ρ‚Π΅, ΠΊΠ°ΠΊ это ΡΠ΄Π΅Π»Π°Ρ‚ΡŒ.

ΠŸΡ€ΠΈΠΌΠ΅Ρ€

НайдитС \ (- 2A + B \) для:

\ (A = \ left [\ begin {array} {cc} -4 & 1 \\ 2 & -2 \\ \ end {array} \ right] \) ΠΈ \ (B = \ left [\ begin {array} {cc} 9 & -4 \\ 0 & 8 \\ \ end {array} \ right] \)

НС Π·Π°Π±ΡƒΠ΄ΡŒΡ‚Π΅ ΡƒΠΌΠ½ΠΎΠΆΠΈΡ‚ΡŒ ΠΊΠ°ΠΆΠ΄ΡƒΡŽ запись Π½Π° константу ΠΈ Ρ€Π°Π±ΠΎΡ‚Π°Ρ‚ΡŒ с записью ΠΏΡ€ΠΈ слоТСнии.

\ (\ begin {align} -2A + B & = 2 \ left [\ begin {array} {cc} -4 & 1 \\ 2 & -2 \\ \ end {array} \ right] + \ left [\ begin {array} {cc} 9 & -4 \\ 0 & 8 \\ \ end {array} \ right] \\ & = \ left [\ begin {array} {cc} -2 \ times -4 & -2 \ times 1 \\ -2 \ times 2 & -2 \ times -2 \\ \ end {array} \ right] + \ left [\ begin {array} {cc} 9 & -4 \\ 0 & 8 \\ \ end {array} \ right] \\ & = \ left [\ begin {array} {cc} 8 & -2 \\ -4 & 4 \\ \ end {array} \ right] + \ left [\ begin { массив} {cc} 9 & -4 \\ 0 & 8 \\ \ end {array} \ right] \\ & = \ left [\ begin {array} {cc} 8 + 9 & -2 + (-4) \\ -4 + 0 & 4 + 8 \\ \ end {array} \ right] \\ & = \ boxed {\ left [\ begin {array} {cc} 17 & -6 \\ -4 & 12 \\ \ end {array} \ right]} \ end {align} \)

Π­Ρ‚ΠΎ Ρ‚Π°ΠΊΠΆΠ΅ Ρ€Π°Π±ΠΎΡ‚Π°Π΅Ρ‚, ΠΊΠΎΠ³Π΄Π° Ρƒ вас Π±ΠΎΠ»Π΅Π΅ Π΄Π²ΡƒΡ… ΠΌΠ°Ρ‚Ρ€ΠΈΡ†, ΠΊΠ°ΠΊ ΠΏΠΎΠΊΠ°Π·Π°Π½ΠΎ Π² ΡΠ»Π΅Π΄ΡƒΡŽΡ‰Π΅ΠΌ ΠΏΡ€ΠΈΠΌΠ΅Ρ€Π΅.

ΠŸΡ€ΠΈΠΌΠ΅Ρ€

НайдитС \ (A — 3B + 2C \) для:

\ (A = \ left [\ begin {array} {cc} 1 & 1 \\ 0 & 0 \\ \ end {array} \ right] \), \ (B = \ left [\ begin {array} {cc} 2 & 1 \\ 1 & 4 \\ \ end {array} \ right] \) ΠΈ \ (C = \ left [\ begin {array} { cc} 5 & 2 \\ 3 & 0 \\ \ end {array} \ right] \)

\ (\ begin {align} A — 3B + 2C & = \ left [\ begin {array} {cc} 1 & 1 \\ 0 & 0 \\ \ end {array} \ right] — 3 \ left [\ begin {массив} {cc} 2 & 1 \\ 1 & 4 \\ \ end {array} \ right] + 2 \ left [\ begin {array} {cc} 5 & 2 \\ 3 & 0 \\ \ end { массив} \ right] \\ & = \ left [\ begin {array} {cc} 1 & 1 \\ 0 & 0 \\ \ end {array} \ right] — \ left [\ begin {array} {cc} 3 \ times2 ΠΈ 3 \ times1 \\ 3 \ times1 & 3 \ times4 \\ \ end {array} \ right] + \ left [\ begin {array} {cc} 2 \ times5 & 2 \ times2 \\ 2 \ times3 & 2 \ times0 \\ \ end {массив} \ right] \\ & = \ left [\ begin {array} {cc} 1 & 1 \\ 0 & 0 \\ \ end {array} \ right] — \ left [\ begin {array} {cc} 6 & 3 \\ 3 & 12 \\ \ end {array} \ right] + \ left [\ begin {array} {cc} 10 & 4 \\ 6 & 0 \\ \ ΠΊΠΎΠ½Π΅Ρ† {массив} \ right] \\ & = \ left [\ begin {array} {cc} 1 — 6 + 10 & 1 — 3 + 4 \\ 0 — 3 + 6 & 0 — 12 + 0 \\ \ end {массив} \ right] \\ & = \ boxed {\ left [\ begin {array} {cc} 5 & 2 \\ 3 & — 12 \\ \ end {array} \ right]} \ end {align} \ )

Π‘Π²ΠΎΠ΄ΠΊΠ°

Π—Π°ΠΏΠΎΠΌΠ½ΠΈΡ‚Π΅ ΡΠ»Π΅Π΄ΡƒΡŽΡ‰Π΅Π΅ для ΠΎΠΏΠ΅Ρ€Π°Ρ†ΠΈΠΉ с ΠΌΠ°Ρ‚Ρ€ΠΈΡ†Π°ΠΌΠΈ:

  • Для слоТСния ΠΈΠ»ΠΈ вычитания ΠΏΠ΅Ρ€Π΅Ρ…ΠΎΠ΄ΠΈΡ‚Π΅ Π²Π²ΠΎΠ΄ Π·Π° Π²Π²ΠΎΠ΄ΠΎΠΌ.

Π”ΠΎΠ±Π°Π²ΠΈΡ‚ΡŒ ΠΊΠΎΠΌΠΌΠ΅Π½Ρ‚Π°Ρ€ΠΈΠΉ

Π’Π°Ρˆ адрСс email Π½Π΅ Π±ΡƒΠ΄Π΅Ρ‚ ΠΎΠΏΡƒΠ±Π»ΠΈΠΊΠΎΠ²Π°Π½. ΠžΠ±ΡΠ·Π°Ρ‚Π΅Π»ΡŒΠ½Ρ‹Π΅ поля ΠΏΠΎΠΌΠ΅Ρ‡Π΅Π½Ρ‹ *